Download as pdf or txt
Download as pdf or txt
You are on page 1of 253

3

A Workbook of
GEOGRAPHY
For Civil Services Prelims Examination
A WORKBOOK OF
GEOGRAPHY
For Civil Services Prelims Examination

WORKBOOK

Forum Learning Centre


DELHI PATNA HYDERABAD
2nd Floor, IAPL House, 2nd Floor, AG Palace, East 1st Floor, SM Plaza, RTC
19, Pusa Road, Opp. Metro Boring Canal Road Patna - X Rd, Indira Park Road,
Pillar 95-96, Karol Bagh - 800001 Jawahar Nagar, Telangana -
110005 500020

https://blog.forumias.com | www.academy.forumias.com | 9821711605


THE WORKBOOKS
Dear Aspirants,

On the basis of the success of the previous series of workbooks as most of the UPSC CSE Prelims
2022 questions came from our workbook, we are glad to inform you that we have come up with the
2nd series of Workbooks to aid your Prelims preparations. The Workbooks are a series of question
banks of MCQs aimed at covering the entire syllabus holistically. How to use the workbook?
The workbook of each subject, consist of a set of units intended to cover the entire subject. Each
unit consist of 2 parts:
1. Previous Year Questions: This part contains all the PYQs asked by the Commission in that
specific Unit.
2. Practise Questions: This section contains MCQs for your practise, which are divided into
sub-sections. This will help the students to practise questions on specific sub-topics.
The combination of both these sections will help our students to check the questions asked by the
commission and at the same time practise similar question. You can also use these workbooks to
solve questions as and when you prepare small portions of the syllabus.
No preparation is complete unless you have understood the concept and applied them in solving
questions. Solving questions is the only way to validate your knowledge. Therefore, the workbooks
would be the perfect friend which will test your conceptual clarity and challenge your knowledge
base.
Happy Learning and Happy Testing!!!
– Your team @ForumIAS

No part of this publication may be reproduced or transmitted, in any form or by any means,
electronic, mechanical, photocopying, recording or otherwise, or stored in any retrieval system of
any nature without the written permission of the copyright holder and the publisher, application
for which shall be made to the publisher.
©Forum (Flaviant Network Pvt. Ltd.)

This book/material is sold subject to the conditions that it shall not, by way of trade or otherwise,
be lent, re-sold, hired out or otherwise circulated without the publisher’s prior consent in any
form of binding or cover other than that in which it is published and without a similar condition
including this condition being imposed on the subsequent purchaser.

We are continuously working on improving our quality. If you are unhappy or happy with the
quality of this study material, want to report an error, or want to leave us feedback, you can email
us at ravi@forumias.com

© 2023 2nd Print Edition | For feedback email us : ravi@forumias.com


INDEX
UNIT 1. GEOMORPHOLOGY .....1 – 69
1.1. Previous Years Questions .....1
1.2. The Earth’s crust & Formation of Earth .....4
1.3. Volcanism & Earthquakes .....12
1.4. Weathering .....15
1.5. Mass Movements and Groundwater .....16
1.6. Landforms .....17
1.7. Running Water .....17
1.8. Glaciers .....18
1.9. Deserts .....19
1.10. Limestone .....19
1.11. Lakes, Coastal landforms, Islands and Miscellaneous .....20
SOLUTIONS .....24

UNIT 2. OCEANOGRAPHY .....70 – 88


2.1. Previous Years Questions .....70
2.2. Movement, relief, temperature and salinity of Ocean Water .....72
2.3. Others .....74
SOLUTIONS .....77

UNIT 3. CLIMATOLOGY .....89 – 132


3.1. Previous Years Questions .....89
3.2. Weather .....91
3.3. Climate Phenomena .....100
3.4. Different forms of Vegetation (World) .....102
SOLUTIONS .....106

UNIT 4. INDIA PHYSICAL ENVIRONMENT .....133 – 196


4.1. Previous Years Questions .....133
4.2. Location and Physiography .....137
4.3. Drainage .....146
4.4. Indian Climate .....149
4.5. Soils .....153
4.6. Natural Vegetation .....154
SOLUTIONS .....158

UNIT 5. ECONOMIC AND HUMAN GEOGRAPHY .....197 – 247


5.1. Previous Years Questions .....197
5.2. World Economic and Human Geography .....200
5.3. Water Resources and Irrigation .....203
5.4. Major Crops and Cropping Pattern .....206
5.5. Industry and Mineral and Energy Resources .....208
5.6. Population .....210
5.7. India Human and Economic Geography .....212
SOLUTIONS .....216
GEOGRAPHY

GEOGRAPHY
GEOMORPHOLOGY
(b) if both A and R are true but R is not the correct
1.1. Previous Years Questions explanation of A
1. Consider the following statements: (c) if A is true but R is false
[UPSC CSE Pre. 2005] (d) if A is false but R is true
1. Total land area of Earth is approximately 1475 5. Which of the following pairs is/are correctly
lakh square kilometres. matched? [UPSC CSE Pre 2008]
2. Ratio of land area to water area of Earth is
approximately 1:4. Theory/Law Associated Scientist
3. Maximum percentage of Earth’s water is in the 1. Continental Drift Edwin Hubble
Pacific Ocean.
2. Expansion of Alfred Wegener
Which of the statements given above is/are correct? Universe
(a) 1 and 3
(b) 2 and 3 3. Photoelectric Effect Albert Einstein
(c) 1 only Select the correct answer using the code given
(d) 3 only below:
(a) 2 and 3 only
2. Which one of the following is the correct sequence
(b) 3 only
of the given Continents in the decreasing order of
their percentage of Earth’s land? (c) 2 only
[UPSC CSE Pre. 2005] (d) 1 only
(a) North America - Africa - South America – 6. Which of the following is/are the geologic features
Europe that can be seen off a Rocky Coast?
(b) Africa - North America - South America - [UPSC CSE 2008]
Europe 1. Wave-cut terrace
(c) North America - Africa - Europe - South 2. Sea Arches and Stacks
America
3. Barrier Bars
(d) Africa - North America - Europe - South
America 4. Lagoons
Select the correct answer using the code given below:
3. Consider the following statements:
(a) 1 and 2 only
[UPSC CSE Pre. 2005]
(b) 3 and 4 only
1. The axis of the Earth’s magnetic field is inclined (c) 1, 3 and 4 only
at 23 ½° to the geographic axis of the earth. (d) 1, 2, 3 and 4
2. The Earth’s magnetic pole in the northern
hemisphere is located on a peninsula in northern 7. Which one of the following cities is nearest to the
Canada. equator? [UPSC CSE 2008]
3. Earth’s magnetic equator passes through (a) Colombo
Thumba in South India. (b) Jakarta
Which of the statements given above is/are correct? (c) Manila
(d) Singapore
(a) 1, 2 and 3
(b) 2 and 3 only 8. Which one of the following straits is nearest to the
(c) 2 only International Date Line? [UPSC CSE 2008]
(d) 3 only (a) Malacca Strait
4. Assertion (A): To orbit around the Sun, the planet (b) Bering Strait
Mars takes lesser time than the time taken by the (c) Strait of Florida
Earth (d) Strait of Gibraltar
Reason (R): The diameter of the Planet Mars is less 9. In order of their distance from the Sun, which
than that of the Earth. of the following planets lie between Mars and
[UPSC CSE Pre. 2006] Uranus? [UPSC CSE 2009]
(a) if both A and R are true and R is the correct (a) Earth and Jupiter
explanation of A (b) Jupiter and Saturn

1 Workbook
GEOGRAPHY

(c) Saturn and Earth (b) 1 4 2 3


(d) Saturn and Nept (c) 3 2 4 1
(d) 3 4 2 1
10. Who of the following scientists proved that the
stars with mass less than 1.44 times the mass of the 15. The 2004 Tsunami made people realize that
Sun end up as White Dwarfs when they die? mangroves can serve as a reliable safety hedge
[UPSC CSE 2009 ] against coastal calamities. How do mangroves
(a) Edwin Hubble function as a safety hedge? [UPSC CSE Pre. 2011]
(b) S Chandrashekhar (a) The mangrove swamps separate the human
(c) Stephen Hawking settlements from the sea by a wide zone in which
(d) Steven Weinberg people neither live nor venture out
(b) The mangroves provide both food and medicines
11. In the structure of the planet Earth, below the which people are in need of after any natural
mantle, the core is mainly made up of which one disaster.
of the following? [UPSC CSE Pre 2009] (c) The mangrove trees are tall with dense canopies
(a) Aluminium and serve as an excellent shelter during a cyclone
(b) Chromium or tsunami
(c) Iron (d) The mangrove trees do not get uprooted by
(d) Silicon storms and tides because of their extensive roots
12. Consider the following statements: 16. The jet aircrafts fly very easily and smoothly in the
[UPSC CSE Pre 2009] lower stratosphere. What could be the appropriate
1. The albedo of an object determines its visual explanation? [UPSC CSE Pre. 2011]
brightness when viewed with reflected light. 1. There are no clouds or water-vapour in the lower
2. The albedo of Mercury is much greater than the stratosphere.
albedo of the Earth. 2. There are no vertical winds in the lower
Which of the statements given above is/are correct? stratosphere.
(a) 1 only Which of the statements given above is/are correct
(b) 2 only in this context?
(c) Both 1 and 2 (a) 1 only
(d) Neither 1 nor 2 (b) 2 only
13. Consider the following statements: (c) Both 1 and 2
[UPSC CSE Pre 2009] (d) Neither 1 nor 2
1. In the world, the tropical deserts occur along the 17. What could be the main reason/reasons of the
western margins of continents within the trade formation of African and Eurasian desert belt?
wind belt. [UPSC CSE 2011]
2. In India the East Himalayan region gets high 1. It is located in the sub-tropical high-pressure
rainfall from north-east winds. cells.
Which of the statements given above is/are correct? 2. It is under the influence of warm ocean currents.
(a) a) 1 only Which of the statements given above is/are correct
(b) b) 2 only in this context?
(c) c) Both 1 and 2 (a) 1 only
(d) d) Neither 1 nor 2 (b) 2 only
14. Match List 1 and List II and select the correct (c) Both 1 and 2
answer using the code given below: (d) Neither 1 nor 2
[UPSC CSE Pre 2009] 18. Which is the difference between asteroids and
List I (Geographic fea- List II (Country) comets? [UPSC CSE 2011]
ture) 1. Asteroids are small rocky planetoids, while
comets are formed of frozen gases held together
A. Great Victoria Des- 1. Australia by rocky and metallic material.
ert 2. Asteroids are found mostly between the orbits of
B. Grand Canyon. 2. Canada Jupiter and Mars, while comets are found mostly
between Venus and Mercury.
C. Lake Winnipeg. 3. New Zealand
3. Comets show a perceptible glowing tail, while
D. Southern Alps. 4. USA asteroids do not.
ABCD Which of the statements given above is/are correct?
(a) 1 2 4 3 (a) 1 and 2 only

Workbook 2
GEOGRAPHY

(b) 1 and 3 only 25. Which of the following leaf modifications occurs/
(c) 3 only occur in desert areas to inhibit water loss?
(d) 1, 2 and 3 [UPSC CSE Pre 2013]
19. hrough which one of the following groups of 1. Hard and waxy leaves
countries does the Equator pass? 2. Tiny leaves or no leaves
[UPSC CSE Pre. 2011] 3. Thorns instead of leaves
(a) Brazil, Zambia and Malaysia Select the correct answer using the codes given
(b) Colombia, Kenya and Indonesia below.
(c) Brazil, Sudan and Malaysia (a) 1 and 2 only
(d) Venezuela, Ethiopia and Indonesia (b) 2 only
20. Consider the following factors: (c) 1 and 3 only
[UPSC CSE Pre. 2012] (d) 1, 2 and 3
1. Rotation of the Earth 26. Which one of the following pairs is correctly
2. Air pressure and wind matched? [UPSC CSE Pre. 2013]
3. Density of ocean water
Geographical Feature Region
4. Revolution of the Earth
(a) Abyssinian Plateau Arabia
Which of the above factors influence the ocean
currents? (b) Atlas Mountains North-Western Africa
(a) 1 and 2 only (c) Guiana Highlands South-Western Africa
(b) 1, 2 and 3 only
(c) 1 and 4 only (d) Okavango Basin Patagonia
(d) 2, 3 and 4 only
27. Variations in the length of daytime and night time
21. Which one of the following sets of elements was from season to season are due to
primarily responsible for the origin of life on the [UPSC CSE Pre. 2013]
Earth? [UPSC CSE 2012] (a) the earth’s rotation on its axis
(a) Hydrogen, Oxygen, Sodium (b) the earth’s revolution round the sun in an
(b) Carbon, Hydrogen, Nitrogen elliptical manner
(c) Oxygen, Calcium, Phosphorous (c) latitudinal position of the place
(d) Carbon, Hydrogen, Potassium (d) revolution of the earth on a tilted axis
22. A person stood alone in a desert on a dark night 28. On the planet earth, most of the freshwater exists
and wanted to reach his village which was situated as ice caps and glaciers. Out of the remaining
5km east of the point where he was standing. He freshwater, the largest proportion
had no instruments to find the direction but he [UPSC CSE Pre. 2013]
located the polestar, the most convenient way now (a) is found in atmosphere as moisture and clouds
to reach his village is to walk in the (b) is found in freshwater lakes and rivers
[UPSC CSE Pre. 2012] (c) exists as groundwater
(a) facing the polestar (d) exists as soil moisture
(b) opposite to the polestar 29. Consider the following: [UPSC CSE Pre. 2013]
(c) keeping the polestar to his left
(d) keeping the polestar to his right 1. Electromagnetic radiation
2. Geothermal energy
23. Which one of the following sets of elements was 3. Gravitational force
primarily responsible for the origin of life on the 4. Plate movements
Earth? [UPSC CSE Pre. 2012]
5. Rotation of the earth
(a) Hydrogen, Oxygen, Sodium
(b) Carbon, Hydrogen, Nitrogen 6. Revolution of the earth
(c) Oxygen, Calcium, Phosphorous Which of the above are responsible for bringing
(d) Carbon, Hydrogen, Potassium dynamic changes on the surface of the earth?
(a) 1, 2, 3 and 4 only
24. On the planet earth, most of the freshwater exists (b) 1, 3, 5 and 6 only
as ice caps and glaciers. Out of the remaining (c) 2, 4, 5 and 6 only
freshwater, the largest proportion (d) 1, 2, 3, 4, 5 and 6
[UPSC CSE Pre 2013]
(a) is found in atmosphere as moisture and clouds 30. Which of the following phenomena might have
(b) is found in freshwater lakes and rivers influenced the evolution of organisms?
(c) exists as groundwater [UPSC CSE Pre. 2014]
(d) exists as soil moisture 1. Continental drift

3 Workbook
GEOGRAPHY

2. Glacial cycles 35. The term “Levant” often heard in the news roughly
Select the correct answer using the code given below. corresponds to which of the following regions?
(a) 1 only [UPSC CSE Pre 2022]
(b) 2 only (a) Region along the eastern Mediterranean shores
(c) Both 1 and 2 (b) Region along North African shores stretching
(d) Neither 1 nor 2 from Egypt to Morocco
(c) Region along Persian Gulf and Horn of Africa
31. Consider the following statements: (d) The entire coastal areas of Mediterranean Sea
[UPSC CSE Pre. 2018]
1. The Earth’s magnetic field has reversed every few 36. Consider the following countries:
hundred thousand years. [UPSC CSE Pre 2022]
2. When the Earth was created more than 4000 1. Azerbaijan
million years ago, there was 54% oxygen and no 2. Kyrgyzstan
carbon dioxide. 3. Tajikistan
3. When living organisms originated, they 4. Turkmenistan
modified the early atmosphere of the Earth. 5. Uzbekstan
Which of the statements given above is/are correct? Which of the above have borders with Afghanistan?
(a) 1 only (a) 1, 2 and 5 only
(b) 2 and 3 only (b) 1, 2, 3 and 4 only
(c) 1 and 3 only (c) 3, 4 and 5 only
(d) 1, 2 and 3 (d) 1, 2, 3, 4 and 5
32. On 21st June, the Sun [UPSC CSE Pre. 2019] 37. In the northern hemisphere, the longest day of the
(a) does not set below the horizon at the Arctic year normally occurs in the : [UPSC CSE Pre 2022]
Circle (a) First half of the month of June
(b) does not set below the horizon at Antarctic (b) Second half of the month of June
Circle (c) First half of the month of July
(c) shines vertically overhead at noon on the (d) Second half of the month of July
Equator
(d) shines vertically overhead at the Tropic of 38. Consider the following pairs:
Capricorn [UPSC CSE Pre 2022]

33. TWith reference to the water on the planet Earth, Region often Country
consider the following statements: mentioned in the news
[UPSC CSE Pre. 2021] 1. Anatolia Turkey
1. The amount of water in the rivers and lakes is
more than the amount of groundwater. 2. Amhara Ethiopia
2. The amount of water in polar ice caps and glaciers 3. Cabo Delgado Spain
is more than the amount of groundwater.
4. Catalonia Italy
Which of the statements given above is/are correct?
How many pairs given above are correctly matched?
(a) 1 only
(a) Only one pair
(b) 2 only
(b) Only two pairs
(c) Both 1 and 2
(c) Only three pairs
(d) Neither 1 nor 2
(d) All four pairs
34. Consider the following statements:
[UPSC CSE Pre 2021] Practice Questions
1. In the tropical zone, the western sections of the
oceans are warmer than the eastern sections 1.2. The Earth’s crust & Formation of Earth
owing to the influence of trade winds.
2. In the temperate Zone, westerlies 39. Which of the following element is most abundantly
make the eastern sections of oceans found in the Earth’s crust?
warmer than the western sections (a) Iron
Which of the statements given above is/are (b) Oxygen
correct? (c) Silicon
(a) 1 only (d) Aluminium
(b) 2 only
40. With reference to Earth’s Geomagnetism consider
(c) Both 1 and 2
the following statements:
(d) Neither 1 nor 2

Workbook 4
GEOGRAPHY

1. It is caused by a giant bar magnet placed (a) 1 only


approximately along the axis of rotation of the (b) 2 only
earth. (c) Both 1 and 2
2. Earth’s magnetic field is generated in the earth’s (d) Neither 1 nor 2
outer core. 45. Consider the following statements:
3. It plays an important role in protecting the 1. All longitudes have equal length whereas length
planet from solar winds and cosmic radiation. of latitudes varies depending upon their location
4. The magnetic field of Mars is stronger than that 2. Two longitudes remain equidistant at all places
of Earth. whereas distance between latitudes varies
5. Currently, earth’s magnetic north pole is moving depending upon their location
from its current position in Canada towards 3. Latitudes determines the amount of insolation
Siberia. received whereas longitude determines the local
time at any place.
Select the correct answer using the code given below:
Which of the statements given above is/are correct?
(a) 1, 2, 3 and 5 only
(b) 1, 2 and 4 only (a) 1 and 2 only
(b) 2 and 3 only
(c) 2, 3 and 5 only
(c) 1 and 3 only
(d) 2, 3, 4 and 5 only
(d) 1, 2 and 3
41. The Earth’s radius at the equator is larger than 46. Consider the following statements:
at the poles. Which one of the following is the
1. Earth’s rotational axis makes an angle of 23.5°
primary reason for this? with the orbital plane of the earth.
(a) Rotation of Earth on its axis 2. Earth’s inclination causes variation in the length
(b) Revolution of Earth around the Sun of day and night.
(c) Pull of gravity is very high at poles 3. Axial parallelism of Earth’s Axis causes
(d) Tilted axis of Earth’s rotation Perihelion and Aphelion
42. What is ‘Mars Opposition’, sometimes seen in Which of the statements given above is/are Incorrect?
news? (a) 1 and 2 only
(a) An event when the Mars, Earth and the sun form (b) 2 and 3 only
a straight line during the course of their orbits (c) 1 and 3 only
(b) Gravity anomalies faced by an object on the (d) None of the above
surface of Mars 47. Consider the following statements regarding the
(c) An event when Mars and Earth are on opposite magnetic field of Earth:
sides of the Sun 1. Magnetic field is due to convective motion of
(d) The effect of precession in Mars causing the metallic fluids in the outer core of the earth.
north and south poles to move in a very large 2. The magnetic poles of the earth do not change
circle their position with time.
43. Which one of the following statements is correct Which of the statement(s) given above is/are correct?
regarding the ‘Heat Budget of the Earth’? (a) 1 only
(a) Earth receives even radiation across its surface (b) 2 only
during any given day of a year. (c) Both 1 and 2
(b) Less than one-fifth of the amount of insolation (d) Neither 1 nor 2
received by earth is absorbed by its surface. 48. Consider the following statements about
(c) The Earth as a whole does not accumulate or composition of Earth’s Crust:
loose heat. 1. The metals in the earth’s crust are usually found
(d) More than half of the total insolation is reflected in elemental form.
back to space even before reaching the earth’s 2. Oxygen has the highest concentration amongst
surface. the elements found in the crust.
44. Consider the following statements with reference Which of the statement(s) given above is/are correct?
to Earth’s geology: (a) 1 only
1. The Mid-Oceanic ridges are some of the most (b) 2 only
active volcanic regions. (c) Both 1 and 2
2. The oceanic crust is much older than the (d) Neither 1 nor 2
continental crust. 49. Which of the following statements are correct
Which of the statements given above is/are correct? regarding the rocks of Earth’s Crust?

5 Workbook
GEOGRAPHY

1. Igneous rocks are formed outof rock composition is thought to be the cause of this
recrystallizationand reorganization of existing discontinuity.
rocks. Which of the following discontinuity is being best
2. Sedimentary rocks are formed through the described in the above paragraph?
process of lithification. (a) Mohorovicic discontinuity
3. Metamorphic rocksform under the action of (b) Gutenberg discontinuity
pressure,volume and temperature changes. (c) Lehmann discontinuity
Select the correct answer using the code given below: (d) Conrad discontinuity
(a) 1 and 2 only 54. With reference to gases in atmosphere consider
(b) 2 and 3 only the following statements:
(c) 1 and 3 only
1. Argon is found naturally in atmosphere.
(d) 1, 2 and 3
2. Carbon Dioxide is the second most abundant
50. With respect to Igneous rocks, which of the gas after Nitrogen.
following statements is/are correct? Which of the statements given above is/are correct?
1. Felsic igneous rocks are denser and darker in (a) 1 only
color than Mafic rocks. (b) 2 only
2. They occur in strata and have a rich fossil (c) Both 1 and 2
presence. (d) Neither 1 nor 2
3. As compared to volcanic rocks, plutonic rocks
have a larger crystal size. 55. Consider the following statements:
1. The thickness of troposphere is greater at the
Select the correct answer using the code given below:
poles than at the equator.
(a) 1 and 3 only
2. The stratosphere contains majority of the ozone
(b) 2 only
layer.
(c) 3 only
(d) 2 and 3 only 3. In thermosphere temperature decreases rapidly
with increasing height.
51. With respect to rock cycle, consider the following Which of the statements given above is/are correct?
statements:
(a) 2 only
1. Sun plays an important role in the rock cycle on (b) 1 and 2 only
Earth. (c) 1 and 3 only
2. For a planet to have an active rock cycle, hot core (d) 1, 2 and 3
is needed to drive mantle convection.
3. Granite undergoes metamorphism and form 56. Which of the following statements is/are incorrect
Gneiss. with reference to Diastrophic forces of nature?
Which of the statements given above is/are correct? 1. They are very slow forces whose effects are
observable only after thousands of years.
(a) 1 and 2 only
(b) 1 and 3 only 2. They are a part of Exogenetic forces.
(c) 2 and 3 only 3. They are restricted to a small area under the
(d) 1, 2 and 3 influence of local weathering agents.
Select the correct answer using the code given below:
52. Consider the following statements with reference
to sedimentary rocks: (a) 1 only
(b) 1 and 2 only
1. Sedimentary rocks are formed at earth’s surface
(c) 3 only
by the hydrological system.
(d) 2 and 3 only
2. Formation of sedimentary rocks involves the
weathering of preexisting rocks. 57. Consider the following events with reference to
3. Sedimentary rocks contain fossils. the formation of planets:
4. Sedimentary rocks typically occur in layers. 1. Formation of localized clump of gases
Which of the statements given above are correct? 2. Accumulation of hydrogen gas to form nebula
(a) 1 and 2 only 3. Formation of Planetesimals
(b) 1 and 4 only 4. Formation of rotating disc of gases around core
(c) 2, 3 and 4 only What is the correct chronological order of the above
(d) 1, 2, 3 and 4 events, starting from the earliest time?
53. This discontinuity forms the boundary between (a) 1-2-3-4
the crust and the upper mantle. Its average depth (b) 2-1-4-3
is more beneath the continental surfaces and (c) 3-4-1-2
shallower beneath the ocean basins. Change in (d) 2-1-3-4

Workbook 6
GEOGRAPHY

58. With reference to the “solar system”, consider the (a) A force created by revolution of earth around the
following statements: sun.
1. The atmosphere of Jovian Planets is thin as (b) Centrifugal force due to earth’s rotation.
compared to Terrestrial planets. (c) Gravitational pull of the sun and the moon.
2. The terrestrial planets have lower gravity as (d) Earth’s internal magnetic force.
compared to Jovian planets. 64. Which one of the following statements about the
3. The terrestrial planets have relatively higher Great Red Spot is correct?
densities as compared to Jovian planets. (a) It is the largest volcano on Mercury.
Which of the statements given above is/are correct? (b) It is the largest sun spot on corona of the Sun.
(a) 1 and 3 only (c) It is a gigantic storm on Jupiter.
(b) 3 only (d) It is the deepest crater on Mars surface.
(c) 2 and 3 only 65. Consider the following statements regarding the
(d) 1, 2 and 3 Sea floor spreading:
59. Consider the following events during the 1. The age of rock increases as one moves away
“evolution of the earth”: from the centre of mid oceanic ridge.
1. Degassing 2. The deep trenches have deep-seated earthquakes
2. Differentiation while in the mid-oceanic ridge areas, the quake
3. Photosynthesis foci usually have shallow depths.
4. Loss of primordial atmosphere. Which of the statements given above is/are correct?
What is the correct chronological order of the above (a) 1 only
events, starting from the earliest time? (b) 2 only
(c) Both 1 and 2
(a) 4-2-1-3 (d) Neither 1 nor 2
(b) 4-1-2-3
(c) 4-1-3-2 66. Consider the following statements:
(d) 2-4-1-3 1. Tectonic Plate theory lacked a geological
60. Consider the following statements: mechanism to explain how the continents could
drift across the earth’s surface.
1. If a small asteroid or large meteoroid survives
its fiery passage through earth’s atmosphere and 2. Wegener’s hypothesis provided this missing
lands on earth’s surface, it is then called a meteor. geological mechanism to explain movement of
continents.
2. If a meteoroid enters the Earth’s atmosphere and
vaporizes, it becomes a meteorite. Which of the statements given above is/are correct?
(a) 1 only
Which of the statements given above is/are correct?
(b) 2 only
(a) 1 only (c) Both 1 and 2
(b) 2 only (d) Neither 1 nor 2
(c) Both 1 and 2
(d) Neither 1 nor 2 67. Consider the following statements with reference
to tectonic plates:
61. In the context Earth’s inner structure, the
1. The Lithosphere includes the crust and
Gutenberg discontinuity separates
uppermost part of the mantle.
(a) The lower crust from the upper mantle
2. Lithospheric plates include both continental and
(b) The upper mantle from the lower mantle
oceanic lithosphere.
(c) The lower mantle from the outer core
(d) The outer core from the inner core 3. The Lithospheric plates move over the
Asthenosphere.
62. With reference to Tillite rocks, consider the Which of the statements given above are correct?
following statements: (a) 1 and 2 only
1. These are igneous rocks formed out of magma. (b) 2 and 3 only
2. These rocks provide evidence for the theory of (c) 1 and 3 only
Continental Drift. (d) 1, 2 and 3
Which of the statements given above is/are correct? 68. Consider the following statements regarding
(a) 1 only albedo:
(b) 2 only 1. Albedo is the proportion of sunlight that is
(c) Both 1 and 2 reflected back into space.
(d) Neither 1 nor 2
2. Decreased albedo leads to greater melting of ice.
63. In the context of Alfred Wegener’s Continental 3. Albedo of water bodies is greater than albedo of
Drift theory, the ‘pole-fleeing force’ is: land.

7 Workbook
GEOGRAPHY

Which of the statements given above are correct? Which of the pairs given above is/are correctly
(a) 1 and 2 only matched?
(b) 2 and 3 only (a) 1 only
(c) 1 and 3 only (b) 1 and 2 only
(d) 1, 2 and 3 (c) 2 and 3 only
69. Consider the following statements: (d) 1, 2 and 3
1. Mid oceanic ridges are found near Convergent 74. Consider the following statements about the
plate boundary. interior of Earth:
2. Volcanic Island arcs are found near Divergent 1. Oceanic crust does not contain the rocks of
plate boundary. SIAL.
Which of the statements given above is/are 2. Outer core is liquid because of the very high
incorrect? pressure.
(a) 1 only 3. Secondary waves or S waves are absent in
(b) 2 only asthenosphere.
(c) Both 1 and 2
(d) Neither 1 nor 2 Which of the statements given above is/are correct?
(a) 1 only
70. The Nazca Plate is located between the (b) 1 and 2 only
(a) Pacific Plate and South American Plate (c) 2 and 3 only
(b) Pacific Plate and African Plate (d) 1, 2 and 3
(c) Pacific Plate and Eurasian Plate
(d) South American Plate and Scotia Plate 75. Which of the following factors contributed to the
formation of lithosphere?
71. Which of the following are direct source of
information about the Earth’s interior? 1. Increase in density of earth during primordial
1. Volcanic eruption stage
2. Magnetic field 2. The Big Splat
3. Gravitation 3. Degassing
4. Mining Select the correct answer using the code given below:
Select the correct answer using the code given below: (a) 1 only
(a) 1 and 4 only (b) 2 only
(b) 2 and 4 only (c) 1 and 2 only
(c) 1, 2 and 3 only (d) 1 and 3 only
(d) 1, 3 and 4 only 76. Which of the following evidence was not used
72. Consider the following statements about the by Wegener for supporting the continental drift
Earth’s crust: theory?
1. Silicon is the most abundant element found in (a) Occurrence of placer deposits and its source
earth’s crust. region in two different continents.
2. Oceanic crust is generally denser than (b) Slow movement of hot mantle below the
continental crust. continent.
Which of the statements given above is/are correct? (c) Rocks of same age found across the continents.
(a) 1 only (d) Identical fossils found on many continents.
(b) 2 only 77. Consider the following statements regarding
(c) Both 1 and 2 Polar lights (Auroras).
(d) Neither 1 nor 2
1. These are formed in the stratospheric layer of
73. Consider the following pairs with respect to atmosphere.
discontinuities found in earth’s crust: 2. Auroras are least visible during the height of
sunspot activities.
Discontinuity Boundary between
3. Colors of the aurora vary depending on the
1. Moho Crust and altitude.
Discontinuity Asthenosphere
Which of the statements given above is/are
2. Gutenberg Mantle and Outer Core incorrect?
Discontinuity (a) 1 only
3. Conrad Outer core and Inner (b) 1 and 2 only
Discontinuity Core (c) 3 only
(d) 1, 2 and 3

Workbook 8
GEOGRAPHY

78. Which of the following best describes the event Which of the statements given above are correct?
‘Laschamps Excursion’? (a) 1 and 4 only
(a) Sudden mass extinction that occurred at the end (b) 2 and 3 only
of the Cretaceous period. (c) 1, 2 and 4 only
(b) An event marking the breakup of Pangea. (d) 1, 2, 3 and 4
(c) An asteroid impact responsible for the extinction
of the dinosaurs. 83. Consider the following statements:
(d) A dramatic change in the Earth’s magnetic field. 1. In the absence of atmosphere, earth’s surface
temperature would increase considerably.
79. With reference to geomorphic processes, consider 2. Earth’s magnetic field is the result of convection
the following statements: currents in its deep inner core.
1. The movement of the lithospheric plates due to
the convectional currents in the mantle is the Which of the statements given above is/are correct?
cause behind endogenic processes. (a) 1 only
2. An example of endogenic processes include (b) 2 only
diastrophism which refers to deformation of the (c) Both 1 and 2
Earth’s crust. (d) Neither 1 nor 2
Which of the above statements is/are correct? 84. With reference to characteristics of the Sun,
(a) 1 only consider the following statements:
(b) 2 only 1. It was formed due to collapse of nebula under its
(c) Both 1 and 2 own gravity.
(d) Neither 1 nor 2 2. Its core generates energy by converting hydrogen
80. Consider the following statements with reference into helium through nuclear fusion.
to the Indian plate movement: 3. It emits steady stream of charged particles called
1. The Indian plate’s movement in the northern solar winds.
direction is a type of convergent boundary 4. Solar flares are large eruptions of electromagnetic
interaction. radiation from the Sun.
2. The boundary between Indian and the Antarctic Which of the statements given above is/are correct?
plate is a type of divergent boundary. (a) 1 and 2 only
Which of the statements given above is/are correct? (b) 1 only
(a) 1 only (c) 3 and 4 only
(b) 2 only (d) 1, 2, 3 and 4
(c) Both 1 and 2 85. Which of the following statements correctly
(d) Neither 1 nor 2 defines the term ‘’Orion arm’’?
81. Consider the following statements: (a) Ring of stars formed after the supernova
1. Direct sources provide the complete information explosion.
about the interior of the Earth. (b) Dense concentration of newly formed stars.
2. Meteors are direct sources of information about (c) Outer disc of the galaxy which emit relatively
the interior of the Earth. low energy.
(d) A minor arm of the Milky Way Galaxy containing
Which of the statement(s) given above is/are correct?
solar system.
(a) 1 only
(b) 2 only 86. Which of the following is the most accurate reason
(c) Both 1 and 2 for the occurrence of different seasons on Earth?
(d) Neither 1 nor 2 (a) Rotation of the earth and the impact of Coriolis
82. With reference to the evolution of earth, consider force
the following statements: (b) Revolution of the earth
(c) Revolution and inclination of earth’s axis in a
1. Earth’s early atmosphere primarily consisted of
fixed direction
hydrogen and helium.
(d) Rotation and gravitational force of the earth
2. Volcanoes released water vapour and ammonia
into the atmosphere. 87. Consider the following statements:
3. Photosynthesizing bacteria released oxygen into Statement 1: The troposphere is thickest at the
atmosphere around 2.7 billion years ago. equator and comparatively thinner at the Poles.
4. In the beginning, the earth almost entirely Statement 2: Heat difference on Earth’s surface cause
consisted of molten magma. the convectional currents to rise at greater heights at
warmer places.

9 Workbook
GEOGRAPHY

Which one of the following is correct in respect of (a) Both A and R are true and R is the correct
the above statements? explanation for A
(a) Both the Statement 1 and Statement 2 are correct (b) Both A and R are true and R is not the correct
and Statement 2 is the correct explanation for explanation for A
statement 1. (c) A is true and R is false
(b) Both statement 1 and Statement 2 are correct but (d) Both A and R are false
Statement 2 is not the correct explanation for
93. For which of the following reasons the Pluto was
Statement 1.
reclassified as a ‘dwarf planet’?
(c) Statement 1 is correct but Statement 2 is not
correct. (a) It is not in orbit around the Sun.
(d) Statement 1 is not correct but Statement 2 is (b) It has not sufficient mass to assume hydrostatic
correct. equilibrium.
(c) It does not fulfill the criteria of minimum mass
88. With reference to the troposphere, consider the standard.
following statements: (d) It has not cleared the neighbourhood around its
1. The majority of the mass of entire atmosphere is orbit.
contained in this layer.
94. Consider the following statements regarding the
2. It is always calm with little turbulence. effects of acid rains:
3. Most of the Earth’s clouds are located in this 1. Soils deficient in naturally occurring limestone
layer. are more affected by acid rain.
Which of the statements given above is/are correct? 2. Buildings made of granite and sandstone are
(a) 1 only less likely to be affected by acid deposition as
(b) 1 and 3 only compared to buildings made up of limestone.
(c) 2 only
Which of the statements given above is/are correct?
(d) 2 and 3 only
(a) 1 only
89. Which of the following is the reason for the (b) 2 only
bulging of the equator? (c) Both 1 and 2
(a) Revolution of the earth (d) Neither 1 nor 2
(b) Rotation of the Earth
95. Which of the following sources contribute to
(c) Tidal force
Space radiation?
(d) Magnetic pull of the earth
1. Solar flares
90. Which of the following is the primary feature of 2. Sun’s corona
the Earth’s atmosphere, that protects us from 3. Cell phones
meteoroids?
4. Sun’s photosphere
(a) Absorption of energy by stratosphere
5. Van Allen radiation belts.
(b) Presence of high-energy radiation in ionosphere
(c) Existence of ozone layer all over the planet 6. Exploding stars
(d) Presence of mesosphere 7. Microwave ovens
91. With reference to Black Holes, consider the Select the correct answer using the code given below:
following statements: (a) 1, 2, 3 and 4 only
1. It is formed when a massive star collapses. (b) 4, 5, 6 and 7 only
(c) 1, 2, 4, 5 and 7 only
2. They have such strong gravitational force that
(d) 1, 2, 4, 5 and 6 only
even light cannot escape it.
3. There is no black hole in our solar system. 96. “It consists of Hydrogen and Helium, as these are
Which of the statements given above is/are correct? the most common and stable compounds in the
Universe. These massive collections of hydrogen
(a) 1 and 2 only gas are pulled together by gravity to form star
(b) 2 and 3 only nurseries through the process of fusion.”
(c) 1 and 3 only
(d) 1, 2 and 3 The information given in above paragraph refers to
which of the following:
92. Consider the following statements: (a) Pulsar
Assertion (A) : Theoretically, it is possible to reach (b) Nebulae
the edge of Universe. (c) Planetary Nebulae
Reason (R) : Dark flow of matter limits the boundary (d) Quasar
of Universe. 97. Which of the following is the reason behind the
Select the correct answer using the code given below: gaseous nature of Outer planets?

Workbook 10
GEOGRAPHY

(a) These planets have lower gravity as compared to 2. Outgassing refers to release of previously
inner planets. dissolved, trapped and frozen gas along the mid-
(b) The presence of rings which are composed ocean ridge.
mainly of gases like hydrogen and helium. Which of the statements given above is/are incorrect?
(c) They have captured a large amount of hydrogen
and other gasses from the surrounding nebula. (a) 1 only
(d) The cores of all these planets are made of gaseous (b) 2 only
compounds. (c) Both 1 and 2
(d) Neither 1 Nor 2
98. Consider the following statements is/are correct
regarding the Moon of Earth: 103. Consider the following statements with reference
1. There is absence of atmosphere on the Moon. to Lithosphere:
2. Meteoroids that pass close to the Moon are easily 1. Lithosphere is composed of the crust and the
vaporized. portion of the upper mantle.
3. Impact crater on the lunar surface is mainly due 2. The main mineral constituents of lithosphere are
to plate tectonics. silica and alumina.
Which of the statements given above is/are incorrect? Which of the statements given above is/are correct?
(a) 1 only (a) 1 only
(b) 1 and 2 only (b) 2 only
(c) 2 and 3 only (c) Both 1 and 2
(d) 1, 2 and 3 (d) Neither 1 Nor 2
99. Which one of the following factors/processes is 104. Tectonic activity is responsible for which of the
related to the formation or modification of the following?
present atmosphere of earth? 1. Earthquakes
1. Loss of primordial atmosphere 2. Volcanoes
2. Degassing 3. Oceanic crust
3. Volcanic eruptions
Select the correct answer using the code given below:
4. Photosynthesis
(a) 1 and 3 only
5. Solar winds
(b) 1 and 2 only
Select the correct answer using the code given below: (c) 2 and 3 only
(a) 1, 2, 4 and 5 only (d) 1, 2 and 3
(b) 2, 3, 4 and 5 only
(c) 1, 2, 3 and 4 only 105. What will happen when the Earth’s Plate Tectonics
(d) 1, 2, 3, 4 and 5 movement stops?
1. The height of Mt Everest will get reduced
100. Which of the following is not the correct reason
behind the habitable nature of earth? gradually.
(a) Presence of Goldilocks zone. 2. The Formation of the new continental and
(b) Presence of an active plate tectonic system. oceanic crust will be stopped.
(c) It is orbiting a high-mass star. Select the correct answer using the code given below:
(d) Presence of molten core inside the earth. (a) 1 only
101. Which of the following is/are the similarities (b) 2 only
between earth and other terrestrial planets? (c) Both 1 and 2
1. They are made up of Rocks and metals. (d) Neither 1 Nor 2
2. They have relatively higher densities. 106. Consider the following statements with reference
3. They have thicker atmosphere than Jovian to evolution of earth:
planets. 1. Gravitational compression and radioactive
Select the correct answer using the code given below: decay contributed to the molten state of earth in
(a) 1 and 2 only its initial stage of evolution.
(b) 2 and 3 only 2. Asteroids that slammed into Earth and other
(c) 1 and 3 only inner planets contained a significant amount of
(d) 1, 2 and 3 water in their minerals.
102. Consider the following statements with reference Which of the statements given above is/are correct?
to earth’s crust formation: (a) 1 only
1. The oceanic crust originates from the Mid- (b) 2 only
oceanic Ridges and spreads towards the (c) Both 1 and 2
continents. (d) Neither 1 nor 2

11 Workbook
GEOGRAPHY

107. Which of the following elements are present in the 112. Consider the following statements:
inner core: 1. Plate Tectonics has empowered geologists to
1. Iron predict an earthquake a few days in advance.
2. Nickel 2. The point on the surface of the earth which first
3. Siderophiles experiences the earthquake waves is called the
4. Sulphur epicentre.
Select the correct answer using the code given below: Which of the statements given above is/are correct?
(a) 1 and 2 only (a) 1 only
(b) 1, 2 and 3 only (b) 2 only
(c) 3 and 4 only (c) Both 1 and 2
(d) 1, 2, 3, and 4 (d) Neither 1 nor 2

108. In the context Earth’s inner structure, the 1.3. Volcanism & Earthquakes
Gutenberg discontinuity separates
(a) The lower crust from the upper mantle 113. In the context of Hotspot vulcanism, consider the
(b) The upper mantle from the lower mantle following statements:
(c) The lower mantle from the outer core 1. It is fed by a region deep inside the earth which
(d) The outer core from the inner core acts as a source of Volcanic eruption.
2. They are always located near plate boundaries.
109. With reference to the interior structure of the
earth, consider the following statements: Which of the statements given above is/are correct?
1. It is from the outer core that molten rock (a) 1 only
materials reach the surface of Earth in the form (b) 2 only
of lava. (c) Both 1 and 2
(d) Neither 1 nor 2
2. The transfer of heat and material in the mantle
helps determine the landscape of Earth. 114. Consider the following statements regarding
Which of the statements given above is/are correct? earthquakes.
(a) 1 only 1. Earthquakes are not found along the transform
(b) 2 only plate boundary.
(c) Both 1 and 2 2. Deep-focus earthquakes are more common in
(d) Neither 1 nor 2 mid-oceanic ridges.
110. Consider the following events during the Which of the statements given above is/are correct?
“evolution of the earth”: (a) 1 only
(b) 2 only
1. Degassing
(c) Both 1 and 2
2. Differentiation of earth forming material. (d) Neither 1 nor 2
3. Photosynthesis
4. Loss of primordial atmosphere. 115. Consider the following statements regarding
Volcanic Landforms:
What is the correct chronological order of the above
1. The lava that cools within the crust are called as
events, starting from the earliest time?
plutonic rocks.
(a) 4-2-1-3
2. Sills are the vertical bodies of the intrusive
(b) 4-1-2-3
igneous rocks.
(c) 4-1-3-2
(d) 2-4-1-3 3. Laccoliths are found at the top of anticlines or at
the base of synclines.
111. In context of plate tectonics theory, which among Which of the statements given above is/are incorrect?
the following landform feature(s) is/are present (a) 1 only
along a subduction zone? (b) 2 only
1. Deep ocean trench (c) 2 and 3 only
2. Forearc basin (d) 1, 2 and 3
3. Chain of volcanic island arcs
116. These volcanoes are mostly made up of basalt
4. Accretionary wedges and are not steep. They are characterized by low
Select the correct answer using the code given below: explosivity. The upcoming lava moves in the form
(a) 1 and 2 only of a fountain. It throws out the cone at the top of
(b) 1, 3 and 4 only the vent and develops into a cinder cone. The type
(c) 3 and 4 only of volcano described is:
(d) 1, 2, 3, and 4 (a) Icelandic Volcano

Workbook 12
GEOGRAPHY

(b) Hawaiian Volcano (d) 1, 2 and 3


(c) Strombolian Volcano
(d) Plinian Volcano 121. With reference to earthquakes, consider the
following statements:
117. Consider the following statements: 1. An earthquake of magnitude 8 on Richter Scale
1. Fossils are abundant in both plutonic and is twice as great as an earthquake of magnitude
volcanic rocks. 4.
2. Volcanic rocks are acidic whereas Plutonic rocks 2. The Mercalli scale takes into account the visible
are basic in nature. damage caused by the event.
3. Volcanic rocks form crystals whereas Plutonic 3. The epicenter is the point on the earth’s surface
rocks are generally amorphous. vertically above the hypocenter where a seismic
Which of the statements given above is/are rupture begins.
incorrect? Which of the above statements are correct?
(a) 1 and 2 only (a) 1 and 2 only
(b) 2 only (b) 2 and 3 only
(c) 1 and 3 only (c) 1 and 3 only
(d) 1, 2 and 3 (d) 1, 2 and 3
118. Consider the following statements regarding the 122. With reference to volcanic landforms, consider the
Pacific ring of fire. following statements:
1. It is the region where massive Pacific Plate 1. Extrusive landforms are formed from the
interacts with less-dense plates surrounding it. material thrown out to the surface during
2. Transform faults do not occur in this region. volcanic activity.
3. Most of the active volcanoes on the Ring of Fire 2. Shield Type Volcanic Landforms are mostly
are found on its western edge. made up of basaltic lava and are not steep.
Which of the statements given above is/are correct? 3. A mid-oceanic ridge is an underwater
(a) 1 only mountain range, formed along convergent plate
(b) 1 and 2 only boundaries.
(c) 1 and 3 only Which of the above statements are correct?
(d) 1, 2 and 3 (a) 1 and 2 only
119. With reference to Volcanic arcs and Island arcs (b) 1 and 3 only
which of the following statements is/are correct? (c) 2 and 3 only
1. Convergence of two oceanic plates lead to the (d) 1, 2 and 3
formation of Island arcs. 123. Consider the following pairs:
2. Volcanic arc is formed when two continental
plates collide. Volcanic Landforms Definition
3. Volcanoes are absent on Island arc. 1. Batholiths Large mass of intrusive
Select the correct answer using the code given below: igneous rock that forms
(a) 1 and 2 only from solidification of
(b) 1 only hot magma deep in the
(c) 2 and 3 only Earth’s crust.
(d) 3 only 2. Laccoliths Large dome-shaped
120. With reference to earthquakes, consider the intrusive bodies
following statements: with a level base and
connected by a pipe like
1. P-waves creates density differences in the conduit from below.
material leading to stretching (rarefaction) and
squeezing (compression) of the material. 3. Sills Vertical intrusions
2. S-waves create troughs and crests in the material of molten magma as
through which they pass. narrow walls of igneous
rocks
3. Shadow zones of S-waves and P-waves have
helped scientists identify the composition of 4. Dykes Near horizontal bodies
earth’s interior. of the intrusive igneous
Which of the above pairs are correctly matched? rocks.
(a) 1 only Which of the above pairs are correctly matched?
(b) 1 and 2 only (a) 1 and 2 only
(c) 2 and 3 only (b) 1, 2 and 3 only

13 Workbook
GEOGRAPHY

(c) 3 and 4 only of propagation of the wave, while secondary


(d) 1, 2, 3 and 4 waves vibrate perpendicular to the direction of
propagation of the wave.
124. Consider the following statements regarding Mid
Oceanic Ridges: 2. Primary waves travel only through solid
materials while secondary waves travel through
1. These are the longest mountain chain on the solid, liquid and gaseous materials.
surface of the earth.
2. These ridges are tectonic in origin. Which of the statements given above is/are correct?
3. The quake foci in these ridges usually have a (a) 1 only
shallow depth. (b) 2 only
(c) Both 1 and 2
Which of the statements given above is/are correct? (d) Neither 1 nor 2
(a) 1 and 2 only
(b) 1 and 3 only 129. Which of the following causes may result in an
(c) 2 and 3 only earthquake?
(d) 1, 2 and 3 1. Sliding of rocks along the fault line
2. Eruption of a volcano
125. In the context of earthquakes and volcanoes,
the Ring of Fire is associated with which of the 3. Intense and unsustainable mining activity
following tectonic plates? 4. Explosion of nuclear devices
1. Indo-Australian Plate Select the correct answer using the code given below:
2. Nazca Plate (a) 1 only
3. Philippine Plate (b) 2 and 4 only
4. North American Plate (c) 1, 2 and 3 only
(d) 1, 2, 3 and 4
Select the correct answer using the code given below:
(a) 1 and 3 only 130. Which of the following are the effects of
(b) 1, 2 and 3 only earthquake?
(c) 2 and 4 only 1. Differential ground movement
(d) 1, 2, 3 and 4 2. Landslides
126. With reference to earthquakes, which of the 3. Soil liquefaction
following statements is/are correct? 4. Avalanches
1. They are caused by the movements along the Select the correct answer using the code given below:
faults which are limited to the Earth’s crust only. (a) 1 and 2 only
2. Intensity of an earthquake is the evaluation of (b) 1 and 4 only
severity of ground motion at a given location. (c) 2 and 3 only
3. The line joining the places which experience (d) 1, 2, 3 and 4
similar seismic intensity is called Isoseismic line. 131. Which of the following are extrusive volcanic
Select the correct answer using the code given below: structures?
(a) 1 and 2 only 1. Caldera
(b) 1 and 3 only 2. Batholith
(c) 2 and 3 only 3. Mid oceanic ridge
(d) 1, 2 and 3 4. Crater
127. With respect to Acidic lava, which of the following Select the correct answer using the code given below:
statements is/are correct? (a) 1, 3 and 4 only
1. It is rich in silica and have high viscosity. (b) 1, 2 and 3 only
2. It is fast moving and so forms a gentle sloping (c) 1, 2 and 4 only
flattened sheild. (d) 2, 3 and 4 only
3. Pyroclast is a feature of acidic lava volcanic 132. Why there are no volcanoes in the Himalayan
eruption. region?
Select the correct answer using the code given below: (a) Asthenosphere is absent beneath Himalayas.
(a) 1 and 2 only (b) Temperature beneath the Himalayas is not very
(b) 1 and 3 only hot because of the absence of radioactive decay.
(c) 2 and 3 only (c) Magma cannot penetrate the thick continental
(d) 1, 2 and 3 only crust of the Himalayas.
128. With reference to “Earthquake waves”, consider (d) Divergence of the two continental plates.
the following statements: 133. Consider the following statements regarding
1. Primary waves vibrate parallel to the direction Volcano Explosivity Index:

Workbook 14
GEOGRAPHY

1. The index measures how much volcanic material 138. In the context of volcanic eruptions, which of the
is ejected and how long the eruptions last. following statements is/are correct?
2. The index is based on a logarithmic scale. 1. Shield Volcanoes are characterized by highly
Which of the statements given above is/are correct? fluid basaltic lava flows.
(a) 1 only 2. Composite volcanoes are characterized by
(b) 2 only explosive eruptions.
(c) Both 1 and 2 Select the correct answer using the code given below:
(d) Neither 1 nor 2 (a) 1 only
134. Consider the following statements with reference (b) 2 only
to Geysers: (c) Both 1 and 2
1. Geysers are fountains of hot water and (d) Neither 1 nor 2
superheated steam. 139. Consider the following statements:
2. The water usually rises to surface without any 1. Evidence of volcanism has been found in every
explosion. terrestrial planet of our solar system.
3. Geysers are well distributed across all continents 2. Majority of volcanic activity on earth occurs
of the earth. along the mid-ocean ridges.
Which of the statements given above is/are Which of the statements given above is/are correct?
incorrect? (a) 1 only
(a) 1 only (b) 2 only
(b) 1 and 2 only (c) Both 1 and 2
(c) 2 and 3 only (d) Neither 1 nor 2
(d) 1, 2 and 3
140. Which of the following places fall within the
135. Consider the following statements: Pacific Ring of Fire?
1. The Mercalli scale measures the observed effects 1. Indonesia
of the earthquake and describes its intensity.
2. Mount Fuji
2. The Richter scale measures the seismic waves, or
3. Papua New Guinea
the energy released, causing the earthquake and
describes the quake’s magnitude. 4. Bolivia
Which of the statements given above is/are correct? Select the correct answer using the codes given
(a) 1 only below:
(b) 2 only (a) 1, 2 and 3 only
(c) Both 1 and 2 (b) 2, 3 and 4 only
(d) Neither 1 nor 2 (c) 1, 3 and 4 only
(d) 1, 2, 3 and 4
136. With reference to volcanism, consider the
following statements: 141. Which of the following are among the
1. Mountain slopes and valleys near eruption sites characteristics of ‘acidic’ lava?
are less fertile. 1. Low melting point
2. Often valuable minerals such as gold and 2. High in silicates
uranium are found near regions having volcanic 3. Covers longer distances
activity. 4. Explosive eruptions
Which of the statements given above is/are correct? Select the correct answer using the code given below.
(a) 1 only (a) 1, 2 and 3 only
(b) 2 only (b) 2, 3 and 4 only
(c) Both 1 and 2 (c) 2 and 4 only
(d) Neither 1 nor 2 (d) 1, 2, 3 and 4
137. In the context of various land movements, the
term ‘Lahar’ refers to which of the following? 1.4. Weathering
(a) It is a slow, downward progression of rock and
142. With reference to Sedimentary rocks, which of the
soil down a high-grade slope.
following statements is/are incorrect.
(b) These are the rocks or lava lumps ejected during
minor volcanic eruptions. 1. Sandstone is an example of organically formed
(c) It is a volcanic debris flow which consists of a sedimentary rocks.
significant fraction of clay sized particles 2. Foliated rocks are a kind of sedimentary rocks.
(d) It is a melt water flood resulting from a volcanic 3. Lithification is an important process in
eruption under a glacier. sedimentary rock formation.

15 Workbook
GEOGRAPHY

Select the correct answer using the code given below: 148. Consider the following statements about
(a) 1 and 2 only Biological Weathering:
(b) 2 only 1. Biological weathering is the process of removal
(c) 2 and 3 only of minerals from the weathering environment.
(d) 1, 2 and 3 2. Anthropological activity contributes to
143. Which of the following statements is/are correct biological weathering.
with reference to Exogenetic forces? Which of the statements given above is/are correct?
1. They are generated from the atmosphere due (a) 1 only
to varying combination of temperature and (b) 2 only
moisture. (c) Both 1 and 2
2. They are called destructional forces as they are (d) None of the above.
engaged in the destruction of relief features. 149. Which of the following are effects of weathering
Select the correct answer using the code given below: of rocks?
(a) 1 only 1. Soil formation
(b) 2 only 2. Soil erosion
(c) Both 1 and 2 3. Loss of minerals
(d) Neither 1 nor 2
4. Reduction of reliefs
144. Which of the following process is used to define Select the correct answer using the code given below:
the gradual disintegration of rocks in-situ by (a) 1 and 3 only
atmospheric or weather forces? (b) 2 and 3 only
(a) Erosion (c) 1, 2 and 4 only
(b) Deposition (d) 1, 2, 3 and 4
(c) Weathering
(d) Transportation 150. With reference to the erosion phenomena, which
of the following statements is/are correct?
145. Which of the following statements correctly 1. It involves acquisition and transportation of rock
describes the term “Hydration”? debris.
(a) A mechanical action that leads to weathering. 2. Erosion is a degradational process.
(b) Disintegration of rocks caused by chemical
addition of water. 3. Erosion is the consequence of deposition.
(c) The river load particles striking, colliding against Select the correct answer using the code given below:
each other and breaking down in the process. (a) 1 and 2 only
(d) Solid river load striking against rocks and (b) 2 and 3 only
wearing them down. (c) 3 only
(d) 1, 2 and 3
146. Which of the following processes are associated
with the chemical weathering of the rocks? 151. Consider the following statements regarding
1. Dissolution weathering of rocks:
2. Carbonation 1. Weathering influences enrichment of certain
3. Oxidation valuable ores.
4. Hydration 2. Weathering is a pre-requisite for mass
movements.
Select the correct answer using the code given below:
(a) 1, 2 and 3 only Which of the statements given above is/are correct?
(b) 1, 3 and 4 only (a) 1 only
(c) 2 and 3 only (b) 2 only
(d) 1, 2, 3 and 4 (c) Both 1 and 2
(d) Neither 1 nor 2
147. With reference to physical weathering of rocks,
consider the following statements:
1. This type of weathering involves mechanical
1.5. Mass Movements and Groundwater
disintegration of rocks. 152. This mass movement involves slow downslope
2. Temperature changes and water pressures does flowing soil mass or fine-grained rock debris
not have any effect on physical weathering. completely saturated with water. This process is
Which of the statements given above is/are correct? quite common in moist temperate areas. When the
(a) 1 only upper portions get saturated and when the lower
(b) 2 only parts are impervious to water percolation, flowing
(c) Both 1 and 2 occurs in the upper parts.
(d) Neither 1 nor 2 The mass movement described above is:

Workbook 16
GEOGRAPHY

(a) Soil creep (b) 2 only


(b) Earthflow (c) Both 1 and 2
(c) Mudflow (d) Neither 1 nor 2
(d) Solifluction
153. Consider the following statements regarding 1.6. Landforms
various mass movements.
157. These are the gently inclined rocky floors that
1. Solifluction is a type of mass movement that is
form close to the mountains at their foot. These
common in moist temperate areas.
can be with or without a thin cover of debris. These
2. Slump is included in the category of slow rocky floors formed through the erosion of the
movement.
mountain front through a combination of lateral
3. Mud flow is common in regions having thick erosion by streams and sheet flooding. These
vegetation cover. erosional landform features are known as:
Which of the statements given above is/are (a) Deflation Hollows
incorrect? (b) Pediments
(a) 1 only (c) Playas
(b) 2 and 3 only (d) Pedestal rocks
(c) 3 only
(d) 1, 2 and 3 158. Which of the following landforms are formed by
running water?
154. Consider the following pairs of types of mass
movements and their descriptions: 1. Gully
2. Cirques
Types Description 3. Ravine
1. Debris avalanche characteristic of humid 4. Moraines
regions with or without Select the correct answer using the code given below:
vegetation cover.
(a) 1 and 3 only
2. Slump slipping of rock debris (b) 1, 2 and 3 only
with a backward (c) 2, 3 and 4 only
rotation. (d) 1, 3 and 4 only
3. Rockslide sliding of individual 159. Which of the following are depositional
rock masses landforms?
Which of the pairs given above is/are correctly 1. Moraines
matched?
2. Barchans
(a) 1 only
(b) 1 and 2 only 3. Hanging valley
(c) 2 and 3 only 4. Sink holes
(d) 1, 2 and 3 Select the correct answer using the code given below:
155. With reference to the mass movements, consider (a) 1 and 2 only
the following statements: (b) 1 and 4 only
1. Mass movements are very active over weathered (c) 2 and 3 only
slopes rather than over un-weathered materials. (d) 1, 2 and 3 only
2. Rapid movements are mostly prevalent in humid 160. Consider the following statements:
climatic regions. 1. Structural Plains are formed by the deposition
Which of the statement given above is/are correct? of materials brought by various agents of
(a) 1 only transportation.
(b) 2 only 2. Erosional Plains are formed by horizontally
(c) Both 1 and 2 bedded rocks which are relatively undisturbed
(d) Neither 1 nor 2 by the crustal movements of the earth.
156. With reference to landslides, consider the 3. Erosional plains are hardly smooth and therefore
following statements: also called as peneplains.
1. Landslide is a sudden mass movement of soil. Which of the statements given above is/are correct?
2. Landslide occur due to loss of balance of land (a) 1 and 2 only
mass with respect to gravity. (b) 2 only
Which of the statements given above is/are incorrect? (c) 2 and 3 only
(a) 1 only (d) 3 only

17 Workbook
GEOGRAPHY

3. Sea adjoining the delta to be shallow.


1.7. Running Water 4. Presence of strong current running right angles
161. With reference to fluvial landforms, consider the to the river mouth.
following statements: Select the correct answer using the code given below:
1. Alluvial fans are formed when streams flowing (a) 1 and 3 only
from higher levels break into foot slope plains of (b) 1, 2 and 3 only
low gradient. (c) 2 and 4 only
2. Point bars are formed on the concave side of the (d) 1, 2, 3 and 4
meanders of large rivers.
3. In the old stage of a river, shallow V-shaped 1.8. Glaciers
valleys are formed due to high speed of water
flow. 166. Consider the following statements regarding
Which of the above statements are correct? Cirques:
(a) 1 only 1. They are deep, long and wide troughs or basins
(b) 1 and 3 only with very steep walls at its head as well as sides.
(c) 2 and 3 only 2. The headward erosion of the cirque walls leads
(d) 1, 2 and 3 to formation of horns.
162. Which of the following country is not part of the Which of the statements given above is/are correct?
drainage basin of the river Nile? (a) 1 only
(a) Libya (b) 2 only
(b) Egypt (c) Both 1 and 2
(c) Sudan (d) Neither 1 nor 2
(d) Ethiopia
167. Consider the following pairs:
163. With reference to different stages of River flow,
which of the following statements is/are correct? Lakes Formed by
1. Rapids and cataracts are more common in Kettle lakes Glaciation
youthful stage of river.
Tarns Volcanic activity
2. Meander becomes more pronounced in the
mature stage of the river. Karst lakes Erosion
3. River Piracy is a common feature observed in Which of the pairs given above are correctly
valley course. matched?
Select the correct answer using the code given below: (a) 1 and 3 only
(a) 1 only (b) 2 and 3 only
(b) 2 and 3 only (c) 1 and 2 only
(c) 2 only (d) 1, 2 and 3
(d) 1 and 3 only
168. With reference to landforms in glaciated lowlands,
164. Consider the following statements with reference which one of the following statements correctly
to River systems: describes the feature of ‘Eskers’?
1. In the mountain course the predominant action (a) They are deep, long and wide troughs with
of river is lateral erosion. vertical high walls at its head as well as sides.
2. Vertical erosion dominates in the valley course. (b) They are U-shaped valleys with broad floors and
3. Deposition is the major activity of river in the relatively smooth, and steep sides.
middle course. (c) They are unsorted glacial deposits comprising a
Which of the statements given above is/are range of eroded materials.
incorrect? (d) They are long and narrow ridges composed of
(a) 1 only sand and gravel.
(b) 1 and 2 only 169. Moraines are often very old material left behind
(c) 2 and 3 only by a moving glacier. In this context consider the
(d) 1, 2 and 3 following pairs:
165. Which of the following conditions are favorable
Type / Category Feature
for the formation of a ‘delta’?
1. Active vertical and lateral erosion in the upper 1. Lateral Moraine Often remain on the
course of the river. landscape long after
2. Unsheltered Coast with higher activities of tides. glacier retreat

Workbook 18
GEOGRAPHY

2. Supraglacial Made up of rocks and (a) 1 and 2 only


Moraine earth that have fallen (b) 2 and 3 only
on the glacier. (c) 1 and 3 only
(d) 1, 2 and 3
3. Ground Moraine Made up of sediment
brought by streams 174. Which one of the following is not an erosional
underneath the glacier. landform of desert regions?
(a) Mushroom rock
Which of the pairs given above is/are correctly
(b) Inselberg
matched? (c) Zeugen
(a) 1 and 2 only (d) Barchan
(b) 3 only
(c) 1 and 3 only 175. With reference to Barchans, which of the following
(d) 1, 2 and 3 statements is correct?
(a) It is a longitudinal dune which is formed when
170. Hanging Valley is a distinctive landform feature supply of sand is poor and wind direction is
that occurs in nature. It is formed: constant.
(a) when two different glacier flows interact with (b) It refers to the gently inclined rocky floors close
each other. to the mountains at their foot with or without a
(b) at the confluence of two rivers. thin cover of debris.
(c) due to the denudational effect of rivers carrying (c) It is a crescent shaped landform with the points
solid load. or wings directed away from wind direction.
(d) at the mouth of a river. (d) It refers to an off-shore bar which is exposed due
to further addition of sand.
171. Consider the following statements:
176. Consider the following statements:
1. Antarctic glaciers terminate in the sea as floating
ice shelves. 1. Most of the mid-latitude desert are found on the
plateau and are at a considerable distance from
2. Antarctic glaciers are not affected by ocean the sea.
currents.
2. Interior deserts, which are found in the heart
3. Antarctic region experiences some warming due of continents, exist because no moisture-laden
to tropospheric westerlies. winds reach them.
Which of the statements given above is/are correct? 3. Desert humidity is usually so low that not
(a) 1 only enough water vapor exists to form clouds.
(b) 1 and 3 only Which of the statements given above is/are correct?
(c) 2 only (a) 1 and 2 only
(d) 1, 2 and 3 (b) 2 and 3 only
(c) 1 and 3 only
172. Consider the following statements with reference
(d) All of the Above
to glacial topography:
1. Glacial till is deposited by melting ice as ground 177. Consider the following statements regarding the
moraine and is composed of sand and large ‘Desert’ ecosystem:
boulders. 1. Most of the animals in the hot desert ecosystem
2. An arete is depositional landform formed at the are diurnal.
conjunction of two overlapping glaciers. 2. Playas are temporary lakes formed in arid and
semi-arid areas.
Which of the statements given above is/are correct?
3. Many of the hot deserts of the world are located
(a) 1 only on the western coasts of continents.
(b) 2 only
(c) Both 1 and 2 Which of the statements given above is/are correct?
(d) Neither 1 nor 2 (a) 1 and 2 only
(b) 2 only
(c) 2 and 3 only
1.9. Deserts (d) 3 only
173. Which of the following are associated with Desert 178. Which of the following deserts are mainly an
landforms? outcome of its geographical location in the rain
1. Fjords shadow area?
2. Deflation hollows 1. Sahara Desert
3. Yardangs 2. Death Valley
Select the correct answer using the code given below: 3. Mojave Desert

19 Workbook
GEOGRAPHY

4. Thar Desert 3. In its purest form, limestone is made up of


Select the correct answer using the code given below. carbonates of calcium and magnesium.
(a) 1 and 3 only Which of the statements given above is/are correct?
(b) 2 and 4 only (a) 1 only
(c) 2 and 3 only (b) 1 and 2 only
(d) 1, 3 and 4 only (c) 2 and 3 only
(d) 1, 2 and 3
179. Deserts around the world receive very little
rainfall because: 183. Consider the following statements:
1. They are located in interior drainage basins far 1. A sinkhole is a surface depression in a region of
from oceans. limestone or chalk terrain.
2. They are located on the protected side of 2. Caverns are interconnected subterranean
mountain ranges. cavities in bedrock formed by the corrosion.
3. They are affected by cold ocean currents that 3. Stalactites are deposit that grow upward from
parallel the coast. the floor of the cavern
Select the correct answer using the code given below. Which of the statements given above is/are correct?
(a) 1 and 2 only (a) 1 only
(b) 2 and 3 only (b) 1 and 2 only
(c) 1 and 3 only (c) 1 and 3 only
(d) All of the above (d) 1, 2 and 3
180. Consider the following statements: 184. Which one of the following statements is correct
1. While warm deserts occur in tropical areas, cold regarding the human activities in Karst regions
deserts occur exclusively in areas beyond the across the world?
Arctic Circle. (a) These are one of the most productive equatorial
2. Sand dunes are not prominent features in cold regions for vegetative growth.
deserts. (b) These are remote areas of the Arctic occupied by
nomadic hunters and food gatherers.
Which of the above statements is/are correct? (c) The natural conditions of this region suit animal
(a) 1 only farming due to availability of pastoral lands.
(b) 2 only (d) It has scattered human settlements and the only
(c) Both 1 and 2 important mineral found here is lead.
(d) Neither 1 nor 2
181. Which of the following pairs are correctly 1.11. Lakes, Coastal landforms, Islands and
matched? Miscellaneous
List I List II 185. It is a long, deep, narrow body of water that
reaches far inland. They are often set in a U-shaped
1. Ventifacts These are pebbles faceted by valley with steep walls of rock on either side. The
sand-blasting and thoroughly coastline appears highly indented with extension
polished by wind abrasion. of water into the land where these landforms are
2. Yardangs These are vertical bands of present. They are found mainly in Norway, Chile,
rocks usually parallel to the New Zealand, Canada, Greenland, and the U.S.
prevailing wind direction. state of Alaska.
3. Bajadas These are narrow water Which landform is being described in the above
channels found in polar regions passage?
(a) Fjord
Select the correct answer using the code given below:
(b) Stalagmite
(a) 1 and 2 only (c) Bergschrund
(b) 3 only (d) Drumlin
(c) 1 only
(d) 1, 2 and 3 186. With reference to different types of coasts,
consider the following statements:
1. Ria coasts are formed due to upliftment of
1.10. Limestone continental self.
182. Consider the following statements: 2. Fjord coasts are the result of drowned glaciated
1. The most prominent stretch of limestone occurs valleys.
in Yugoslavia. 3. Dalmatian coasts are longitudinal coasts where
2. Limestone regions are identified by general mountains run parallel to the coast.
absence of surface drainage system. Which of the statements given above is/are correct?

Workbook 20
GEOGRAPHY

(a) 1 and 2 only Which of the statements given above is/are correct?
(b) 2 and 3 only (a) 1 and 2 only
(c) 1 and 3 only (b) 1 and 3 only
(d) 1, 2 and 3 (c) 2 and 3 only
187. With reference to coral reefs, consider the (d) 1, 2 and 3
following statements: 191. Consider the following pairs of different canals/
1. They thrive abundantly in tropical waters with straits and seas connected by them.
20-25ºC temperature.
2. They can grow only in deep waters. Straits/canals Oceans/seas
3. A river mouth is very ideal for their growth due 1. Seuz Canal Connects Red Sea to
to abundance of fresh water. Gulf of Aden.
Which of the statements given above is/are correct? 2. Bosporus strait Connects Sea of
(a) 1 only Marmara to Aegean
(b) 2 and 3 only Sea.
(c) 1 and 3 only
(d) 1, 2 and 3 3. Bab-el-Mandeb Connects Red Sea to
Mediterranean Sea.
188. Consider the following statements with reference
to effects of ocean currents: 4. Strait of Hormuz Connects Persian Gulf
1. Cold ocean currents result in desert formation with Indian ocean.
in west coast regions of the tropical continents. Which of the pairs given above is/are correctly
2. North Atlantic drift keeps the coasts of the North matched?
Sea warm. (a) 1, 2 and 4 only
3. The mixing of warm and cold currents favours (b) 2 and 3 only
the growth of planktons. (c) 2 and 4 only
Which of the statements given above is/are correct? (d) 4 only
(a) 1 and 2 only 192. Recently Lake Salda was in news for giving clue
(b) 2 and 3 only about the life on mars. It is located in which
(c) 3 only country?
(d) 1, 2 and 3 (a) Afghanistan
189. With reference to ocean currents, consider the (b) Turkey
following pairs: (c) China
(d) Iraq
Current Characteristics
193. With reference to tides, which of the following
1. Canaries current Cold current of Atlantic statements is/are correct?
Ocean 1. During perihelion, tidal ranges are much greater
2. Agulhas current Warm current of than average.
Southern Indian Ocean 2. During neap tides, both high and low tides are
3. Irminger currentCold current of lower than average.
Southern Pacific 3. In diurnal tide, successive high and low tides are
approximately of the same height.
Which of the pairs given above are correctly
matched? Select the correct answer using the code given below:
(a) 1 only (a) 1 only
(b) 1 and 2 only (b) 1 and 2 only
(c) 2 and 3 only (c) 2 and 3 only
(d) 1 and 3 only (d) 1 and 3 only
190. Consider the following statements: 194. Which of the following statement is incorrect
1. The Drake passage lies between South Africa and about Tidal currents?
the tip of the Antarctic Peninsula in Southern (a) It occurs in conjunction with the rise and fall of
Ocean. the tide.
2. The strong westerly winds blowing around the (b) It is the only type of current affected by the
Antarctica results in large-scale upwelling in the interactions of the Earth, sun, and moon.
Southern Ocean. (c) It is generated when tides are channeled between
3. A recent study showed that marine productivity islands.
in the Southern Ocean has increased due to (d) During full moon day the tidal current velocities
intense solar ultraviolet radiation. are weak.

21 Workbook
GEOGRAPHY

195. With reference to aquatic ecosystems, consider the Which of the pairs given above are correctly
following statements: matched?
1. Increased concentration of Suspended (a) 1 and 2 only
particulate matter limits the photosynthetic (b) 1 and 3 only
activity. (c) 1, 2 and 3 only
2. In Profundal zone there is a rapid increase in (d) 2, 3 and 4 only
photosynthetic activity. 200. In context to ‘Geomorphology’, which of the
3. Increase in the temperature of water body following are not an erosional landform?
decreases the amount of dissolved oxygen. (a) Capes and Bays
Which of the statements given above is/are correct? (b) Stack and Stumps
(a) 1 only (c) Gloup and Geos
(b) 2 and 3 only (d) Spits and Bars
(c) 1 and 3 only 201. Consider the following statements with reference
(d) 1, 2 and 3 to the ‘Continental Shelves’:
196. Consider the following statements: 1. Continental shelves can be formed either due to
1. Fringing reefs grow near the coastline and are emergence of continent or submergence of sea.
separated from the shore by narrow, shallow 2. Continental shelves can be almost absent along
lagoons. some of the coasts.
2. Atolls are rings of coral with shallow lagoon in 3. Continental shelves are major source of oil
middle. deposits around the world.
Which of the statements given above is/are correct? 4. Continental shelves houses rich fishing grounds.
(a) 1 only Which of the statements given above is/are correct?
(b) 2 only (a) 2 and 3 only
(c) Both 1 and 2 (b) 2, 3, and 4 only
(d) Neither 1 nor 2 (c) , and 4 only
(d) , 2, 3 and 4
197. Consider the following statements:
1. In a Lunar eclipse, Moon’s shadow falls on Earth, 202. With reference to the oceanic trenches, consider
while in a Solar eclipse, Earth casts a shadow the following statements:
onto Moon. 1. Ocean trenches occur at the bases of continental
2. Lunar eclipse can occur only during a new moon; slopes and along island arcs.
Solar eclipsecan occur only during a full moon. 2. They are the feature of convergent plate
Which of the statement(s) given above is/are boundaries.
Incorrect? Which of the statements given above is/are correct?
(a) 1 only (a) 1 only
(b) 2 only (b) 2 only
(c) Both 1 and 2 (c) Both 1 and 2
(d) Neither 1 nor 2 (d) Neither 1 nor 2
198. These type of clouds look like cotton wool and 203. With reference to the oceanic current system in
are generally formed at a height of 4,000 - 7,000 the Indian ocean region, which of the following is
m. They exist in patches and can be seen scattered incorrect?
here and there. The description is best suited for (a) Ocean current system in Indian ocean is strongly
which of the following type of cloud? impacted by landmass and monsoon system.
(a) Cumulus (b) North Indian Ocean has permanent and
(b) Cirrus consistent system of ocean currents.
(c) Stratus (c) Current in the Northern Indian Ocean changes
(d) Nimbus direction twice a year.
(d) Southern Equatorial current is less affected by
199. Consider the following pairs: seasonal reversal of monsoon wind.
Place Country 204. With reference to the Ocean heat budget, consider
the following statements:
1. Bhasan Char Bangladesh
1. Ocean is cooled by the oceanic convection.
2. Tigray Region Ethiopia 2. Ocean thermal energy conversion (OTEC) is a
3. Sana Yemen process which harnesses the thermal gradients
between ocean surface waters and deep ocean
4. Hagia Sophia Egypt waters.

Workbook 22
GEOGRAPHY

Which of the statements given above is/are correct? (d) All of the Above
(a) 1 only 208. Consider the following statements:
(b) 2 only
(c) Both 1 and 2 1. Cirque lakes or tarns are formed by the volcanic
(d) Neither 1 and 2 activity.
2. Lakes formed by landslides are short lived.
205. These are the western boundary ocean currents 3. Lakes are temporary features of the earth crust.
in the wind-driven, subtropical and subarctic
circulations of the North Pacific Ocean. One of Which of the statements given above is/are correct?
them is downwelling-dominant, low in biological (a) 1 and 2 only
productivity and is devoid of detritus and other (b) 2 and 3 only
organic material in the surface water, whereas (c) 1 and 3 only
the other is dominated by upwelling and fed by (d) All of the Above
nutrient-rich waters. The confluence of these 209. Consider the following statements:
currents is marked by fogbanks and has a great 1. The western coast of India underwent more
influence on fisheries and climate in the western erosion as compared to the eastern coast.
North Pacific. These currents are:
2. Coastlines of granite remains stable for centuries.
(a) Kuroshio Current and Oyashio Current
(b) California Current and Alaska Current Which of the statements given above is/are correct?
(c) Falkland Current and Humbolt Current (a) 1 only
(d) Labrador Current and Gulf Stream (b) 2 only
(c) Both 1 and 2
206. Consider the following pairs: (d) Neither 1 nor 2
Lake Characteristics 210. Consider the following pairs:
1. Vembanad Largest Freshwater lake Ocean Current Flowing Region
in India
1. Benguela Current Atlantic Ocean
2. Baikal Deepest lake in the
world 2. Agulhas Current Sea of Japan
3. Lake Superior Shared by Canada and 3. Gulf Stream Pacific Ocean
USA 4. Labrador Current North Sea
Which of the pairs given above is/are correctly Which of the pairs given above is/are correctly
matched? matched?
(a) 1 and 2 only (a) 1 only
(b) 2 and 3 only (b) 1 and 4 only
(c) 1 and 3 only (c) 2 and 3 only
(d) All of the Above (d) 1, 3 and 4 only
207. With reference to the features of the lake, consider 211. “This Canyon lies in the south-western portion of
the following statements: the Colorado Plateau of United States of America.
1. The most important chemicals in a lake are It is a steep-sided canyon carved by the Colorado
nitrogen and phosphorus. River. It is the largest canyon in the United States.
2. All freshwater lakes are open. Native peoples have inhabited the canyon area
3. A reservoir is a manmade lake that is created for thousands of years, and the Pueblo people
when a dam is built on a river. consider the area to be a holy pilgrimage site.”
4. The clearer water in the lake is an indication of The above passage describes which of the following
the fewer nutrients in lake. geographical locations?
Which of the statements given above is/are correct? (a) Royal Gorge Canyon, Colorado
(a) 1, 3 and 4 only (b) The Grand Canyon, Arizona
(b) 1, 2 and 3 only (c) Antelope Canyon, Arizona
(c) 2 and 4 only (d) King Canyon, California

23 Workbook
GEOGRAPHY

SOLUTIONS

can also be described as the northern subcontinent of


1.1. Previous Years Questions a single continent, America. North America covers an
1. Solution: (a) area of about 24,709,000 square kilometres (9,540,000
square miles), about 16.5% of the Earth’s land area and
Exp) Option a is correct.
about 4.8% of its total surface. North America is the third-
“Land area” refers to the total surface area of the land largest continent by area, following Asia and Africa, and
of a geographical region or country (which may include the fourth by population after Asia, Africa, and Europe.
discontinuous sections of land such as islands).
South America is a continent entirely in the Western
Statement 1 is correct. Earth’s total planimetric (flat) Hemisphere and mostly in the Southern Hemisphere, with
land area is approximately 147,539,063.133 km2 a relatively small portion in the Northern Hemisphere.
(57,505,693.767 sq. mi) which is about 29.2% of its total
Antarctica is Earth’s southernmost continent. It contains
surface, including that which is covered by ice. Water
the geographic South Pole and is situated in the Antarctic
covers approximately 70.8% of planimetric Earth’s surface,
region of the Southern Hemisphere, almost entirely south
mainly in the form of oceans and ice formations; but this
of the Antarctic Circle, and is surrounded by the Southern
proportion is decreased by the land’s increased terrain.
Ocean. At 14,200,000 square kilometres (5,500,000 square
Statement 2 is incorrect. The ratio of land and water on miles), it is the fifth-largest continent and nearly twice the
earth is approximately in the ratio of 1:2. size of Australia.
About 71 percent of the Earth’s surface is water-covered, Europe is a landmass variously recognised as part of
and the oceans hold about 96.5 percent of all Earth’s Eurasia or a continent in its own right, located entirely
water. Water also exists in the air as water vapor, in rivers in the Northern Hemisphere and mostly in the Eastern
and lakes, in icecaps and glaciers, in the ground as soil Hemisphere. Europe covers about 10.18 million km2 (3.93
moisture and in aquifers, and even in you and your dog. million sq mi), or 2% of the Earth's surface (6.8% of land
Statement 3 is correct. Pacific Ocean is the largest area), making it the second-smallest continent.
water mass on the planet. It covers more than 30 percent Australia is a sovereign country comprising the mainland
of the Earth’s surface. With a surface area of more than of the Australian continent, the island of Tasmania, and
155 million square kilometres (60 million square miles), numerous smaller islands.[13] It is the largest country
this ocean basin is larger than the landmass of all the by area in Oceania and the world’s sixth-largest country.
continents combined. Additionally, it contains almost Australia’s population of nearly 26 million, in an area of
twice as much water as the world’s second largest body of 7,617,930 square kilometres.
water, the Atlantic Ocean.
2. Solution: (b)
Exp) Option b is correct.
The correct sequence of the given Continents in the
decreasing order of their percentage of Earth’s land are
Africa – North America – South America – Europe.
Asia: It is a landmass variously described as part of Eurasia
or as Earth’s largest and most populous continent in its
own right, located primarily in the Eastern and Northern
Hemispheres. It shares the continental landmass of
Eurasia with the continent of Europe and the continental
landmass of Afro-Eurasia with both Europe and Africa.
Asia covers an area of 44,579,000 square kilometres
(17,212,000 sq mi), about 30% of Earth’s total land area
and 8.7% of the Earth’s total surface area. 3. Solution: (b)
Africa is the world’s second-largest and second-most Exp) Option b is correct.
populous continent, after Asia in both cases. At about Statement 1 is incorrect. The axis of earth is inclined
30.3 million km2 (11.7 million square miles) including to the plane of the ecliptic at an angle of 66 ½ degree,
adjacent islands, it covers 6% of Earth’s total surface area giving rise to different seasons and varying lengths of day
and 20% of its land area. and night. Earth rotates on its own axis, tilted at an angle
North America is a continent in the Northern Hemisphere of 23.4 degrees every 23 hour 56 minutes giving rise to
and almost entirely within the Western Hemisphere. It Day and Night.

Workbook 24
GEOGRAPHY

The axis of the Earth’s magnetic field is inclined at an Insolation, Planetary winds, Gravity, Salinity of ocean
angle of about 15 degrees with the geographical axis. water, Shape of coastline are some other important factors
which influence the movement of ocean currents.
5. Solution: (b)
Exp) Option b is correct.
Pair 1 is incorrectly matched. Continental drift is the
hypothesis that the Earth’s continents have moved over
geologic time relative to each other, thus appearing to
have “drifted” across the ocean bed. The speculation that
continents might have ‘drifted’ was first put forward by
Abraham Ortelius in 1596. A pioneer of the modern view
of mobilise was the Austrian geologist, Otto Ampferer.
The concept was independently and more fully
developed by Alfred Wegener in 1912, but his hypothesis
was rejected by many for lack of any motive mechanism.
Arthur Holmes later proposed mantle convection for
Statement 2 is correct. The North Magnetic Pole moves that mechanism. The idea of continental drift has since
over time due to magnetic changes in Earth\’s core. In been subsumed into the science of plate tectonics, which
2001, it was determined by the Geological Survey of studies the movement of the continents as they ride on
Canada to lie west of Ellesmere Island in northern plates of the Earth’s lithosphere.
Canada at 81.3°N 110.8°W. It was situated at 83.1°N Edwin Hubble contribution was in describing the
117.8°W in 2005. universe’s rate of expansion.
Statement 3 is correct. The Thumba Equatorial Rocket Pair 2 is incorrectly matched. The expansion of the
Launching Station (TERLS) is an Indian spaceport universe is the increase in distance between any two given
operated by the Indian Space Research Organisation. It is gravitationally unbound parts of the observable universe
located in Thumba, Thiruvananthapuram which is near with time. Edwin Hubble contributed in development
the southern tip of mainland India, very close to earth’s of expansion of universe theory. Hubble’s brilliant
magnetic equator. observation was that the red shift of galaxies was directly
4. Solution: (b) proportional to the distance of the galaxy from earth.
That meant that things farther away from Earth were
Exp) Option b is correct. moving away faster. In other words, the universe must be
Statement 1, 2 and 3 are correct. expanding. He announced his finding in 1929.
Rotation of Earth lead to Coriolis effect which influences Pair 3 is correctly matched. Photoelectric effect is
direction of ocean currents. phenomenon in which electrically charged particles
Coriolis causes freely moving objects to appear to move to are released from or within a material when it absorbs
the right in the Northern Hemisphere and to the left in the electromagnetic radiation. The effect is often defined as
Southern Hemisphere. The objects themselves are actually the ejection of electrons from a metal plate when light
moving straight, but the Earth is rotating beneath them, falls on it. In a broader definition, the radiant energy may
so they seem to bend or curve. be infrared, visible, or ultraviolet light, X-rays, or gamma
Water in the surface currents is pushed in the direction of rays; the material may be a solid, liquid, or gas. The
the major wind belts: released particles may be ions (electrically charged atoms
trade winds: east to west between the equator and 30 or molecules) as well as electrons. The phenomenon was
degrees N and 30 degrees S fundamentally significant in the development of modern
physics because of the puzzling questions it raised about
westerlies: west to east in the middle latitudes
the nature of light—particle versus wavelike behaviour—
polar easterlies: east to west between 50 degrees and 60 that were finally resolved by Albert Einstein in 1905.
degrees north and south of the equator and the north and
south pole 6. Solution: (d)
The density of ocean water influences the speed of the Exp) Option d is correct
ocean currents. Statement 1 is correct - Along high rocky coasts, waves
The density of seawater plays a vital role in causing ocean break with great force against the land shaping the hill
currents and circulating heat because of the fact that sides into cliffs. With constant pounding by waves, the
dense water sinks below less dense. Salinity, temperature cliffs recede leaving a wave-cut platform in front of the
and depth all affect the density of seawater. sea cliff. After a considerable period of cliff development,
Statement 4 is incorrect. Revolution of Earth does not a wave-built terrace would develop in front of wave-cut
influence the movement of Ocean Currents. terrace.

25 Workbook
GEOGRAPHY

Statement 2 is correct - The constant erosion of The international date line passes through the mid-
rocky headlands may produce a variety of particular Pacific Ocean and roughly follows a 180 degrees longitude
geomorphic structures, including sea arches and sea north-south line on the Earth. It is located halfway around
stacks. With prolonged erosion, sea arches may collapse the world from the prime meridian — the 0 degrees
to form sea stacks—steep pillars of rock a short distance longitude line in Greenwich, England.
from the mainland. Both sea stacks and sea arches are
Bering Strait: The Bering Strait is a strait between the
impermanent features that will eventually disappear with
continued erosion. Pacific and Arctic oceans, separating the Chukchi
Peninsula of the Russian Far East from the Seward
Statement 3 is correct - As the erosion along the coast Peninsula of Alaska.
takes place a good supply material becomes available to
longshore currents and waves to deposit them as beaches Bering Strait is nearest to the International Date line.
along the shore and as bars (long ridges of sand and/ or The International Date Line runs between the Diomede
shingle parallel to the coast) in the nearshore zone. Bars Islands and St. Lawrence Islands located in the middle of
are submerged features and when bars show up above the strait.
water, they are called barrier bars. The present Russia-United States maritime boundary is
Statement 4 is correct - Barrier bar which gets keyed up at 168° 58’ 37” W longitude, slightly south of the Arctic
to the headland of a bay is called a spit. When barrier Circle at about 65° 40’ N latitude.
bars and spits form at the mouth of a bay and block it,
a lagoon form. The lagoons would gradually get filled up
by sediments from the land giving rise to a coastal plain.
7. Solution: (d)
Exp) Option d is correct.
The equator is an imaginary line of latitude on the
spheroid (Earth), dividing it into southern and northern
hemispheres. On earth, this line is approximately 24,901
miles of which only 21.3% lies on land and the rest on
the water. The Equator runs through several countries,
mainly in Africa and South America. However, Indonesia
(Southeast Asia) is straddled the most by the equator.
Singapore is around one-and-a-half degrees north of
the equator, lying entirely between the 1st and 2nd Malacca Strait: The Strait of Malacca Straits of Malacca
parallels. It is nearest to the equator among the above is a narrow stretch of water, 580 mi (930 km) in length,
given cities. between the Malay Peninsula (Peninsular Malaysia) and
the Indonesian island of Sumatra.

Jakarta is the capital of Indonesia. Lying on the northwest


coast of Java (the world’s most populous island), its Strait of Florida: The Straits of Florida, Florida Straits,
Coordinates are 6°12S 106°49E. or Florida Strait is a strait located south-southeast of
Colombo is the capital of Sri Lanka by population. Its the North American mainland, generally accepted to be
Coordinates: 6°5604N 79°5034E. between the Gulf of Mexico and the Atlantic Ocean, and
Manila, is the capital of the Philippines, and its coordinates between the Florida Keys (U.S.) and Cuba. It is 93 mi
are 14°36N 120°59E. (150 km) wide at the narrowest point between Key West
and the Cuban shore, and has been sounded to a depth
8. Solution: (b) of 6,000 feet. The strait carries the Florida Current, the
Exp) Option b is correct. beginning of the Gulf Stream, from the Gulf of Mexico.

Workbook 26
GEOGRAPHY

10. Solution: (b)


Exp) Option b is correct.
Subrahmanyam Chandrasekhar, an Indian-born
scientist is most famous for coming up with the theory
that explains the death of the universe’s most massive
stars. Chandrasekhar showed that it is impossible
for a white dwarf star, which is supported solely by a
degenerate gas of electrons, to be stable if its mass is
greater than 1.44 times the mass of the Sun. If such a star
does not completely exhaust its thermonuclear fuel, then
this limiting mass may be slightly larger.
All direct mass determinations of actual white dwarf stars
have resulted in masses less than the Chandrasekhar limit.
A star that ends its nuclear-burning lifetime with a mass
greater than the Chandrasekhar limit must become either
a neutron star or a black hole.
Strait of Gibraltar: The Strait of Gibraltar also known as Before Chandrasekhar, scientists assumed that all
the Straits of Gibraltar is a narrow strait that connects the stars collapsed into white dwarfs when they died. He
Atlantic Ocean to the Mediterranean Sea and separates determined this isn’t so.
the Iberian Peninsula in Europe from Morocco in Africa. 11. Solution: (c)
Exp) Option c is correct.
The core is made almost entirely of metal—specifically,
iron and nickel.
The Core of Earth is the very hot, very dense centre of our
planet. The ball-shaped core lies beneath the cool, brittle
crust and the mostly-solid mantle. The core is found about
2,900 kilometres below Earth’s surface, and has a radius of
about 3,485 kilometres.
When Earth was formed about 4.5 billion years ago, it was
a uniform ball of hot rock. Radioactive decay and leftover
heat from planetary formation caused the ball to get even
9. Solution: (b) hotter. Eventually, after about 500 million years, Earth’s
temperature heated to the melting point of iron—about
Exp) Option b is correct.
1,538° Celsius. This pivotal moment in Earth’s history is
Jupiter and Saturn lie between Mars and Uranus. called the iron catastrophe.
The International Astronomical Union defines a true The shorthand used for the core’s iron-nickel alloys is
planet as a body that circles the sun without being some simply the elements’ chemical symbols—NiFe.
other object’s satellite; is large enough to be rounded
by its own gravity (but not so big that it begins to
undergo nuclear fusion, like a star); and has “cleared its
neighbourhood” of most other orbiting bodies.
The order of the planets in the solar system, starting
nearest the sun and working outward is the following:
Mercury, Venus, Earth, Mars, Jupiter, Saturn, Uranus
and Neptune.

Elements that dissolve in iron, called siderophiles, are


also found in the core. Because these elements are found
much more rarely on Earth’s crust, many siderophiles
are classified as “precious metals.” Siderophile elements
include gold, platinum, and cobalt. Another key element
in Earth’s core is sulphur—in fact 90% of the sulphur on
Earth is found in the core.

27 Workbook
GEOGRAPHY

12. Solution: (a) cause of high rainfall in the north-east Himalayas. The
Exp) Option a is correct. northeast monsoon is confined to south India and brings
rainfall from October to December over Tamil Nadu,
Albedo is the fraction of light that is reflected by a body
Puducherry, Karaikal, Yanam, Andhra Pradesh, Kerala,
or surface. It describes the reflective properties of planets,
Mahe and south interior Karnataka.
satellites, and asteroids.
Statement 1 is correct. Albedo is a non-dimensional, 14. Solution: (b)
unitless quantity that indicates how well a surface reflects Exp) Option b is correct.
solar energy. The albedo of an object will determine its
The Great Victoria Desert is a sparsely populated desert
visual brightness when viewed with reflected light. For
example, the planets are viewed by reflected sunlight ecoregion and interim Australian bioregion in Western
and their brightness depends upon the amount of light Australia and South Australia. It is the largest desert in
received from the sun and their albedo. When an object Australia and consists of many small sandhills, grassland
reflects most of the light that hits it and looks bright then plains, areas with a closely packed surface of pebbles
it has high albedo. (called desert pavement or gibber plains), and salt lakes.
Statement 2 is incorrect. Mercury receives the maximum The Grand Canyon is a steep-sided canyon carved by the
amount of sunlight, but its albedo is only 0.1 so it is not Colorado River in Arizona, United States. The Grand
as bright as it would be with a higher albedo. The albedo Canyon is 277 miles (446 km) long, up to 18 miles (29
of the Earth is 0.37. Therefore, Albedo of Earth is greater km) wide and attains a depth of over a mile (6,093 feet or
than Mercury. 1,857 meters).
Lake Winnipeg is a very large, relatively shallow
24,514-square-kilometre (9,465 sq mi) lake in North
America, in the province of Manitoba, Canada. Its
southern end is about 55 kilometres (34 mi) north of the
city of Winnipeg. Lake Winnipeg is Canada’s sixth-largest
freshwater lake and the third-largest freshwater lake
contained entirely within Canada. It is the eleventh-largest
13. Solution: (a) freshwater lake on Earth. The lake’s east side has pristine
Exp) Option a is correct. boreal forests and rivers that were in 2018 inscribed as
Pimachiowin Aki, a UNESCO World Heritage Site.
Statement 1 is correct. Tropical deserts are mostly
located between 100 to 300 north and south of Equator on The Southern Alps are a mountain range extending
the western margins of the continent. These regions lie along much of the length of New Zealand’s South Island,
in the belt of the trade winds. These winds blow from reaching its greatest elevations near the range’s western
east to west. These winds shed their moisture on the side. The name “Southern Alps” generally refers to the
eastern side of the continents. By the time they reach to entire range, although separate names are given to many
the western margin of the continents, they had shed their of the smaller ranges that form part of it.
moisture and hence do not cause any rainfall. Thus, most
of the tropical deserts like The Sahara Desert, the Kalahari 15. Solution: (d)
Desert, West Australian desert etc. are located on the Exp) Option d is correct.
western margin of the continent. Moreover, these deserts The sturdy root system of mangrove trees help form a
also lie on the leeward side of the mountains and hence natural barrier against disasters such as tsunamis and
the region is also hot and dry. floods. River and land sediment is trapped by the roots,
which stabilizes shoreline, coastline areas and slows
erosion.
16. Solution: (c)
Exp) Option c is correct.
Statement 1 is correct. The stratosphere is very dry,
air in lower stratosphere contains little water vapor and
there is almost complete absence of thunderstorms. Less
water vapour means less drag which helps maintain fuel
efficiency.
Statement 2 is correct. Unlike the troposphere,
there is almost complete absence of vertical winds or
thunderstorms in the lower stratosphere.
Statement 2 is incorrect. The average annual rainfall
in East Himalayan region is 10,000 mm (390 inches). 17. Solution: (a)
The Bay of Bengal Branch of Southwest Monsoon is the Exp) Option a is correct

Workbook 28
GEOGRAPHY

Statement 1 is correct. The sub-tropical high-pressure


belts are adiabatic in nature. Hence, they bring little
rainfall to the African and Eurasian desert region.
Majority of deserts are located in the subtropics due to the
warming and drying effect of the subsidence generated
by high pressure. The Sahara Desert and Arabian Deserts
are good examples of subtropical deserts that largely owe
their existence to this climatological feature.
The air converges and rise at the equator due to intense
heating, and circulate to the edge of the subtropics where
it descends again. This is called the Hadley Cell, and it
creates the subtropical high-pressure belt, where high
pressure and descending air are found for much of the The Equator passes through 13 countries: Ecuador,
year. Colombia, Brazil, Sao Tome & Principe, Gabon, Republic
of the Congo, Democratic Republic of the Congo, Uganda,
Statement 2 is incorrect. Warm ocean currents aren’t a Kenya, Somalia, Maldives, Indonesia and Kiribati. At
necessary condition for the desert belt. For instance, Gobi least half of these countries rank among the poorest in the
cold desert doesn’t come under the influence of any warm world.
currents. Moreover, African and Eurasian desert belts lie
20. Solution: (d)
under the influence of cold ocean currents.
Exp) Option d is correct.
18. Solution: (b) Assertion A is an incorrect statement.
Exp) Option b is correct. The orbital period (also revolution period) is the time
Asteroids, sometimes called minor planets, are rocky, a given astronomical object takes to complete one orbit
airless remnants left over from the early formation of our around another object, and applies in astronomy usually
solar system about 4.6 billion years ago. to planets or asteroids orbiting the Sun, moons orbiting
planets, exoplanets orbiting other stars, or binary
Comets are cosmic snowballs of frozen gases, rock, and stars. The time taken by Planet Mars to complete one
dust that orbit the Sun. revolution around Sun (687 days) is greater that the
Statement 1 and 3 are correct. time taken by Earth (365 days).
Asteroids are small, rocky objects that orbit the Sun. Planets that orbit closer to the Sun than Earth have shorter
years than Earth. Planets that orbit farther from the Sun
Comets are frozen leftovers from the formation of the than Earth have longer years than Earth. This happens
solar system composed of dust, rock and ices. They range for two main reasons.
from a few miles to tens of miles wide, but as they orbit
If a planet is close to the Sun, the distance it orbits around
closer to the sun, they heat up and spew gases and dust the Sun is fairly short. This distance is called an orbital
into a glowing head that can be larger than a planet. This path.
material forms a tail that stretches millions of miles.
The closer a planet travels to the Sun, the more the Sun’s
Statement 2 is incorrect. gravity can pull on the planet. The stronger the pull of the
The asteroid belt is located roughly between the orbits of Sun’s gravity, the faster the planet orbits.
the planets Jupiter and Mars. Statement R is correct.
Comets are not limited to any specific region; they are The diameter of the Planet Mars is less than that of the
spread all over the solar system. Earth. The diameter of Mars is 6792 Km and the diameter
of Earth is 12756.
19. Solution: (b)
21. Solution: (b)
Exp) Option b is correct. Exp) Option b is correct.
The equator is an imaginary line that circles the Earth, Carbon, Hydrogen, Nitrogen were primarily responsible
splitting it into the Northern and Southern hemispheres. for the origin of life on the Earth.
Because of this, the intersection point of any location by Hydrogen, Nitrogen, Oxygen, Phosphorus and Sulfur
the equator is equidistant from the North and South poles. combined with Carbon generated the first group of
Find out what life is like for countries along the equator. compounds that eventually formed the chemical basis of
Although the equator stretches 40,075 kilometres around life.
the world, it travels through just 13 countries, though The matter within every living Earth creature mainly
only the water controlled by two of these rather than the consists of just four chemical elements: hydrogen, oxygen,
landmasses themselves. carbon, and nitrogen.

29 Workbook
GEOGRAPHY

22. Solution: (c) 26. Solution: (b)


Exp) Option c is correct. Exp) Option b is correct.
The pole star also called the North Star indicates the
Constitutional Subject
North direction. Therefore, if the person walks keeping
Amendment
the polestar in his left, he will be travelling in the East
direction. Abyssinian Plateau Ethiopia
23. Solution: (b) Atlas Mountains North-Western Africa
Exp) Option b is correct. Guiana Highlands Omicoo basin (Brazil)
Carbon, Hydrogen, Nitrogen were primarily Okavango Basin South-West Africa
responsible for the origin of life on the Earth. Abyssinian Plateau is also known as Ethiopian Highlands.
Carbon is the main element in organic compounds.
Organic compounds make up the cells and other 27. Solution: (d)
structures of organisms and carry out life processes. Exp) Option d is correct.
Hydrogen, Nitrogen, Oxygen, Phosphorus and Sulfur
combined with Carbon generated the first group of Throughout the year, different parts of Earth receive
compounds that eventually formed the chemical basis of the Sun’s most direct rays. So, when the North Pole tilts
toward the Sun, it’s summer in the Northern Hemisphere
life.
and the days are longer in the Northern Hemisphere.
The matter within every living Earth creature mainly Similarly, when the South Pole tilts toward the Sun, it’s
consists of just four chemical elements: hydrogen, oxygen, winter in the Northern Hemisphere and nights are longer
carbon, and nitrogen. in the Northern Hemisphere. Thus, Revolution of Earth
on it’s tilted axis causes variations in the length of daytime
24. Solution: (c)
and night time from season to season.
Exp) Option c is correct. If the Earth was not tilted on its axis, it would be
The distribution of water on the Earth’s surface is permanently hot at the equator and cold at the poles.
extremely uneven. Also, the precipitation patterns would remain the same
Only 3% of water on the surface is fresh; the remaining throughout the year.
97% resides in the ocean. Of freshwater, 69% resides in
28. Solution: (c)
glaciers, 30% exists as groundwater, and less than 1% is
located in lakes, rivers, and swamps. Exp) Option c is correct.
The distribution of water on the Earth’s surface is extremely
25. Solution: (d) uneven. Only 3% of water is fresh and the remaining 97%
Exp) Option d is correct. resides in the ocean.
Following are leaf modifications of desert plant to Of freshwater, 69% resides in glaciers, 30% exists as
inhibit water loss for adaptations and survival in deserts: groundwater, and less than 1% is located in lakes,
Wax coatings on leaves prevent water loss through rivers, and swamps.
evaporation, which in the hot desert can cause loss of water 29. Solution: (d)
from both the surface and the inside of leaves. Leaves
are also smaller on desert plants, further reducing the Exp) Option d is correct answer.
possibility for water loss. All the above are responsible for bringing dynamic changes
Deciduous plants in desert ecosystems have adapted on the surface of the earth. Transfer from potential to
through the activity of their leaves. Leaves on these plants kinetic energy results in dynamic changes.
are typically smaller and coated with wax to prevent Option 1 is correct. Electromagnetic radiation from the
evaporation. sun is source of heat and energy on Earth. This energy
Plants such as aloes are equipped with fleshy leaves that is source of all life on earth. It leads to precipitation and
contain much of their water supply. Because of their moist weathering of different surfaces.
inner bodies, these plants are called succulents. They Option 2 is correct. Geothermal energy is heat derived
typically feel spongy and when cut open are filled with a within the sub-surface of the earth. It leads to melting of
pulpy flesh, protected by a waxy outer layer. ice caps, volcanism and other dynamic changes.
Many plants in the desert conserve water by not having Option 3 is correct. Due to the gravitational forces, the
any leaves at all. Cacti are the most prolific of this plant Sun, planets, and their moons interact with each other in
type. Many cacti have spines in place of leaves, which a way, so that the Solar system is stable. The tides in the
conduct photosynthesis and catch dew when the climate oceans are caused by gravitational force which leads to
is right. These small structures also reflect light, further erosion and deposition.
reducing water loss. During heavy rains, cacti will grow Option 4 is correct. Movements of plates leads
temporary root systems and absorb water. They will then to formation of islands, mountains, volcanoes,
shed the roots when the ground has dried. earthquakes.

Workbook 30
GEOGRAPHY

Option 5 is correct. Rotation of earth causes day and The Southern Hemisphere experiences the exact reverse
night, it also results in pressure differences due to conditions. Therefore, It is winter season in the Southern
differential heating of earth surface. This cause winds hemisphere, the days are shorter and the nights are longer
to flow and weathering of surfaces, as prominently seen
in deserts. Rotation also produces Coriolis affect, which 33. Solution: (b)
affects the movement of ocean currents. Exp) Option b is correct.
Option 6 is correct. Revolution of the earth results in Statement 1 is incorrect. Of the total freshwater, 69%
different seasons. When surfaces are exposed to different resides in glaciers, 30% underground, and less than
conditions it affects their structural integrity and aids in 1% is located in lakes, rivers, and swamps. Thus, the
erosion. amount of water in rivers and lakes is less than amount
of groundwater.
30. Solution: (c)
Exp) Option c is correct.
Continental drift and Glacial cycles had an impact on
the climate of different regions and thus influenced the
evolution of organisms on this planet.
31. Solution: (c)
Exp) option c is correct.
Statement 1 is correct. The study of paleomagnetism (the
magnetism residual in rocks) has provided evidence that
the earth’s magnetic field has reversed direction every
few hundred thousand years— i.e., the north and south
poles have switched polarity. Some scientists as well as
psychics predict this phenomenon may occur in the near
geologic future.
Statement 2 is incorrect. When Earth was first Statement 2 is correct.
created, there was no oxygen whatsoever. Our Earth’s About 2.1% of all of Earth’s water is frozen in glaciers.
atmosphere at that time consisted of mostly methane and About three-quarters of Earth’s freshwater is stored in
carbon dioxide. The early atmosphere largely contained glaciers. Therefore, glacier ice is the second largest
water vapour, nitrogen, carbon dioxide, methane, and reservoir of water on Earth and the largest reservoir of
ammonia. These gases and were released from the interior freshwater on Earth.
solid earth during its cooling off. The process through • 97.2% is in the oceans and inland seas
which the gases were outpoured from the interior is called
degassing. • 2.1% is in glaciers
Statement 3 is correct. The process of photosynthesis • 0.6% is in groundwater and soil moisture
modified the early atmosphere of the Earth. About 2.4 • less than 1% is in the atmosphere
billion years ago, a type of organism called cyanobacteria • less than 1% is in lakes and rivers
evolved on the early Earth and began carrying out
• less than 1% is in all living plants and animals.
photosynthesis. Photosynthesis uses carbon dioxide and
energy from the Sun to produce sugar and oxygen. The 34. Solution: (c)
cyanobacteria were very simple organisms but performed
Exp) Option c is correct.
an important role in changing Earth’s early atmosphere.
They carried out photosynthesis to produce the materials When trade winds move over warm tropical waters, they
they needed to grow. They gave off oxygen to the pick up moisture and bring heavy rainfall to the windward
atmosphere as they did this facing slopes of mountainous areas, contrasting with the
downward motion of dry air that creates desert areas on
32. Solution: (a) land.
Exp) Option a is correct. Statement 1 is correct. Warmer water is transported
On 21st June, Northern Hemisphere is tilted towards the westward in the ocean by the Northeast trade winds in
sun. As a result, Tropic of Cancer receives direct sunlight the Northern hemisphere of the tropical zone (as the
as sun is directly over Tropic of Cancer and areas beyond figure given below shows). So, in tropical zones, the
the Arctic circle till the pole experience continuous day western section of ocean is warmer than eastern sections
light for 6 months as the sun does not set below the due to trade winds.
horizon for 6 months. This signfies it is summer time in Statement 2 is correct. Westerlies play an important role
the Northern Hemisphere at this period of time. Places in in carrying the warm, equatorial waters and winds to the
Northern Hemisphere experience longer days and shorter western coasts of continents (that is eastern section of
nights. the Oceans in temperate zone). Thus, in temperate zones,

31 Workbook
GEOGRAPHY

westerlies make the eastern section of the ocean warmer


than the western sections.

35. Solution: (a) 37. Solution: (b)


Exp) Option a is the correct answer. Exp) Option b is the correct answer.
Levant, originally meant ‘the East’ or ‘Mediterranean In the northern hemisphere, the longest day of the year is
lands east of Italy’. It is borrowed from the French called the summer solstice.
levant ‘rising’, referring to the rising of the sun in the The maximum amount of sunlight received by the
east, or the point where the sun rises. Historically, the Northern Hemisphere during this time is usually on June
region along the eastern Mediterranean shores, roughly 20, 21 or 22.
corresponding to modern-day Israel, Jordan, Lebanon, The longest day of 2021 for those living north of the
Syria, and certain adjacent areas. Equator is June 21. This day is referred to as the summer
solstice, the longest day of the summer season. It occurs
when the sun is directly over the Tropic of Cancer, or
more specifically right over 23.5 degree north latitude.

38. Solution: (b)


Exp) Option b is the correct answer
36. Solution: (c)
Pair 1 is correctly matched: Anatolia, also called Asia
Exp) Option c is the correct answer.
Minor, the peninsula of land that today constitutes the
Afghanistan, officially the Islamic Republic of Afghanistan, Asian portion of Turkey. Because of its location at the
is a mountainous landlocked country in southern point where the continents of Asia and Europe meet,
Central Asia. It borders Tajikistan, Turkmenistan, and Anatolia was, from the beginnings of civilization, a
Uzbekistan to the north, Iran to the west, Pakistan to crossroads for numerous peoples migrating or conquering
the east and south. from either continent.

Workbook 32
GEOGRAPHY

Italy). Catalonia was formerly a principality of the crown


of Aragon, and it has played an important role in the
history of the Iberian Peninsula. From the 17th century it
was the centre of a separatist movement that sometimes-
dominated Spanish affairs.

Pair 2 is correctly matched: The Amhara Region is a


regional state in northern Ethiopia and the homeland of
the Amhara people. Its capital is Bahir Dar which is the
seat of the Regional Government of Amhara. Amhara is
bordered by Sudan to the west and northwest and by other
the regions of Ethiopia: Tigray to the north, Afar to the
east, Benishangul-Gumuz to the west and southwest, and
Oromia to the south.
1.2. The Earth’s crust & Formation of Earth
39. Solution: (b)
Exp) Option b is correct.
The earth is composed of various kinds of elements.
About 98% of the total crust is made up of eight elements
as oxygen, silicon, aluminium, iron, calcium, sodium,
potassium, and magnesium.
Earth’s crust consists of 46% Oxygen, 28% Silicon, 8%
Aluminium, 5% Iron and the rest others. Oxygen mostly
occurs in combined forms like silicates, metal and non-
metal oxides and hydroxides.
40. Solution: (c)
Exp) Option c is correct.
Statement 1 is incorrect. Earth’s Geomagnetism is not
Pair 3 is incorrectly matched: Cabo Delgado is the caused by a giant bar magnet placed along the axis
northernmost province of Mozambique (not Spain). The of rotation of the earth and deep in the interior. The
region is an ethnic stronghold of the Makonde tribe, with magnetic field is aroused due to electrical currents
the Makua and Mwani as leading ethnic minorities. produced by the convective motion of metallic fluids
(consisting mostly of molten iron and nickel) in the outer
core of the earth. This is known as the dynamo effect.
Statement 2 is correct. Earth’s magnetic field is
generated in the earth’s outer core. The Earth consists
of a solid iron core. Surrounding the iron core is an ocean
of hot, liquid metal. The liquid metal that flows in Earth’s
core creates electrical currents, which in turn creates our
magnetic field.
Statement 3 is correct. The Earth’s magnetic field plays
an important role in protecting the planet from solar
winds and cosmic radiation that are harmful. It is the
magnetosphere that protects us from harmful radiation.
Statement 4 is incorrect. Mars doesn’t have flowing
Pair 4 is incorrectly matched: Catalonia occupies a liquid metal in its core and thus it doesn’t produce the
triangular area in the north-eastern corner of Spain (not same dynamo effect. This leaves the planet with a very

33 Workbook
GEOGRAPHY

weak magnetic field, allowing for its atmosphere to be redistributed pole wards and as a result the tropics do not
stripped away by solar winds, leaving it uninhabitable. get progressively heated up due to the accumulation of
Statement 5 is correct. According to British Geological excess heat or the high latitudes get permanently frozen
Survey, the earth’s magnetic north pole is moving from its due to excess deficit.
current position in Canada towards Siberia. Option b is Incorrect. About half of the total insolation
received by earth is absorbed by its surface. If insolation
41. Solution: (a) received at the top of the atmosphere is 100 units, then 47
Exp) Option a is correct. unit is absorbed by earth’s surface.
Option a is correct. The radius at the equator is larger Option c is correct. The earth as a whole does not
than at the poles due to the long-term effects of the earth’s accumulate or loose heat. It maintains its temperature.
rotation. The difference in radius is due to the centrifugal This happen because the amount of heat received in the
force of the earth’s rotation which causes the bulging at the form of insolation equals the amount lost by the earth
equator and compression at the Poles. The Earth is about through terrestrial radiation.
43 km (27 mi) wider at the equator than pole-to-pole. Option d is Incorrect. Only about one-third of the total
Option b is incorrect. The Revolution of Earth around the insolation is reflected back to space even before reaching
sun does not causes bulging at the equator. The revolution the earth’s surface.Out of 100 units of total insolation
affects the occurrence of different seasons on Earth. received by earth, 35 units are reflected back to space
Option c is incorrect. This is one of the effects of the even before reaching the earth’s surface. Of these, 27
bulging of the Earth at equator. The poles are closer to units are reflected back from the top of the clouds and 2
the center due to the equatorial bulge, and thus have a units from the snow and ice-covered areas of the earth.
stronger gravitational field. This is due to the fact that the The reflected amount of radiation is called the albedo of
radius at the equator is larger than at the poles. the earth.
Option d is incorrect. Earth’s tilted axis causes the seasons 44. Solution: (a)
and not the bulging. Throughout the year, different parts Exp) Option a is correct
of Earth receive the Sun’s most direct rays. So, when
the North Pole tilts toward the Sun, it’s summer in the Statement 1 is correct. A mid-ocean ridge or mid-
Northern Hemisphere. And when the South Pole tilts oceanic ridge is an underwater mountain range, formed
by plate tectonics. This uplifting of the ocean floor occurs
toward the Sun, it’s winter in the Northern Hemisphere.
when convection currents rise in the mantle beneath the
It is also the reason behind variations in the length of
oceanic crust and create magma where two tectonic plates
day and night throughout the Earth. Earth’s axis is tilted
meet at a divergent boundary. The mid-oceanic ridges
23.5 degrees.
are some of the most active regions in terms of volcanic
42. Solution: (a) eruptions.
Exp) Option a is correct. Statement 2 is incorrect. The continental crust is much
older than the oceanic crust. As the continental crust is
Opposition is the event when the sun, Earth and a planet
rarely destroyed and recycled in the process of subduction,
(Mars in this case) are lined up, with the Earth in the
some sections of continental crust are nearly as old as the
middle.
Earth itself.
Opposition can happen only for planets that are farther
Oceanic crust is thinner and denser than continental
away from the sun than the Earth. In case of Mars, roughly
crust.
every two years, the Earth passes between sun and Mars,
this is when the three are arranged in a straight line. 45. Solution: (c)
Option c is incorrect. Solar conjunction is an event Exp) Option c is correct
which occurs every 26 months, when Mars and Earth are Statement 1 is correct – Longitudes are imaginary semi-
on opposite sides of the Sun, communication between the circles starting from north pole and ending at south
two planets is disrupted. pole. All longitudes are equal in length irrespective of
43. Solution: (c) their location. Latitudes on the other hand are imaginary
parallel lines which make a circle around theEarth. The
Exp) Option c is correct. equator represents the 0oLatitude. All parallel circles from
Option a is Incorrect. Earth do not receive even the equator to the poles are called parallels of latitudes.
radiation across its surface during any given day of a As we move away from the equator, the length of the
year. There are variations in the amount of radiation parallels of latitudes decreases
received at the earth’s surface. Some part of the earth has Statement 2 is Incorrect – The distance between two
surplus radiation balance while the other part has deficit. longitudes is maximum at the equator and decreases
Between 40 degrees north and south and the regions steadily pole wards until it becomes zero at the poles,
near the poles have a deficit. The There is a surplus of net where all meridians meet. Whereas latitudes are
radiation balance surplus heat energy from the tropics is equidistant from each other.

Workbook 34
GEOGRAPHY

Statement 3 is correct –Amount of insolation received Statement 2 is Incorrect:The magnetic poles inside the
at a particular place depends on its latitudinal position. earth change position with time. In a span of 240 years
Insolation in the tropics is greater than at the poles. from 1580 to 1820 AD, over which recordsare available,
Latitudes are used to classify Earth into various heat zones the magnetic declination at London has been found to
depending upon the insolation – Torrid zone, Temperate change by 3.5°. On the scaleof a million years, the earth’s
zone and Frigid zone. magnetic fields has been found to reverse its direction.
Longitudes are used to measure local time of the place. 48. Solution: (b)
At a particular time, sun is overhead all the places on a
longitude. Thus all the places will have same local time. By Exp) Option b) is correct.
convention, Greenwich Meridian which is also known as Statement 1 is Incorrect: The elements in the earth’s crust
Prime Meridian is taken as reference. All the longitudes are rarely found exclusively. They are usually combined
towards East of Greenwich is ahead of Greenwich time with other elements to make various substances. These
and those on the west are behind. substances are recognized as minerals.
Statement 2 is correct:Oxygen is found in highest
46. Solution: (c)
proportion (46.6%) in Earth’s crust. Next comes Silicon
Exp) Option c is correct (27.72%), Aluminium (8.13%), Iron (5%) and Calcium
Statement 1 is Incorrect - Earth’s rotational axis makes an (3.63%).
angle of 23.5° with the normal, i.e. it makes an angle of
66.5° with the orbital plane of the earth (ecliptic plane). 49. Solution: (b)
The plane in which Earth revolves around the sun is called Exp) Option b) is correct.
as orbital plane. Statement 1 is Incorrect: Igneous –in Latin means ‘Fire’.
Statement 2 is correct – Due to Earth’s axial tilt (or Igneous Rocks aresolidified form of magma and lava.
obliquity), day and night are not evenly divided. When magma in its upwardmovement cools and turns into
Hemisphere that is tilted towards the Sun is warmer solid form it iscalled igneous rock. The process of cooling
because sunlight travels moredirectly to the Earth’s surface andsolidification can happen in the earth’s crustor on the
so less gets scattered in the atmosphere. That means that surface of the earth.Granite, gabbro, pegmatite,basaltand
when itis summer in the Northern Hemisphere, it is tuff are some ofthe examples of igneous rocks.
winter in the Southern Hemisphere. The hemispheretilted Statement 2 is correct:Sedimentary rocks are formed
towards the Sun has longer days and shorter nights. That’s from sediments transported by exogenous agencies. These
why days are longer during thesummer than during the sediments are depositedin the form of layers and turn
winter. If the Earth’s axis were perpendicular to its orbital into rocksthrough compaction. This process is known as
plane aroundthe Sun, all places on Earth would experience lithification. Sedimentary rocks are also called stratified
equal amounts of day and night (i.e. 12 hours ofday and rocks. Sandstone, conglomerate, limestone, shale, loess
night, respectively) every day during the year and there are examples of Sedimentary rock.
would be no seasonal variability.So the primary cause of Statement 3 is correct:The word metamorphic means
changing of length of day and night is the inclination of ‘change of form’. Metamorphic rocksform under the
Earth’s Axis. action of pressure,volume and temperature changes.
Statement 3 is Incorrect – Throughout its revolution Metamorphism is a process by which alreadyconsolidated
around the sun, Earth’s axis maintains the same alignment rocks undergo recrystallization and reorganisation of
relative to the plane of the elliptic and to Polaris. In each materials within originalrocks.Marble and Gneiss are
position earth is revolving with the axis oriented parallel examples of these rocks.
to itself. This condition is known as axial parallelism. This
parallelism of axis of the Earth is the responsible for 50. Solution: (c)
apparent movement of the sun over the tropics. Exp) Option c is correct.
Perihelion and Aphelion positions occurs because Earth Statement 1 is incorrect. Felsic igneous rocks are made
is revolving around the sun in an elliptical orbit with sun up of acidic lava and have high portion of silica which
at one of its focus. At Perihelion position, sun is nearer to makes them less dense and lighter in color as compared
the earth, this occurs is month of January. At Aphelion to mafic rocks.
position, sun is farthest from the Earth, this occurs in the Mafic rocks are made of basic lava and have higher
month of July. portion of aluminum and iron.
47. Solution: (a) Statement 2 is incorrect. Igneous rocks normally have a
Exp) Option a) is correct. crystalline structure. They do not occur in strata. They
do not contain any fossils either. This is because they are
Statement 1 is correct:Themagnetic field is due to formed directly from lava or magma, not sediment.
electrical currents producedby convective motion of
metallic fluids (consisting mostly of molteniron and Sedimentary rocks have a rich fossil presence.
nickel) in the outer core of the earth. This is known as Statement 3 is correct. Volcanic rocks (extrusive
thedynamo effect. igneous rocks) are formed due to solidification of lava at

35 Workbook
GEOGRAPHY

the surface of Earth. They solidify rapidly and as a result 30 kilometers beneath continental surfaces. The cause
it has a small crystal size. Basalt is a common example of of this discontinuity is thought to be a change in rock
it. Whereas in plutonic rocks (intrusive igneous rocks), composition from rocks containing feldspar (above) to
solidification of magma is at great depths and rate of rocks that contain no feldspars (below).
cooling is much slow as a result of which size of crystal Option b is incorrect – Gutenberg discontinuity lies
is very large. Granite is an example of the plutonic rocks. between mantle and the outer core.
51. Solution: (d) Option c is incorrect – Lehmann discontinuity lies
Exp) Option d is correct. between outer and inner core.
Statement 1 is correct. The rock components of the Option d is incorrect – Conrad discontinuity is the
crust are slowly but constantly being changed from one transition zone between the upper and the lower part
of the lithosphere. It is observed in various continental
form to another and the processes involved is known as
regions at a depth of 15 to 20 km; however, it is not found
the rock cycle. It is driven by two forces: earth’s internal
in oceanic regions.
heat and hydrological cycle- movement of water which
is powered by sun. Thus, sun plays an important role in 54. Solution: (a)
the rock cycle on earth. Exp) Option a is correct
Statement 2 is correct. Rock cycle is still active on Earth Statement 1 is correct. Argon is found naturally in
because our core is hot enough to keep the mantle atmosphere it makes up 0.93% of the Earth’s atmosphere
moving, this leads to slow and significant changes on and is the third most abundant atmospheric gas.
earth’s crust. In case of moon, the rock cycle is virtually
dead because the core is no longer hot enough to drive Statement 2 is Incorrect. Carbon dioxide is the fourth
mantle convection. most abundant (not second) gas in atmosphere.
Statement 3 is correct. Granite is an igneous rock. Under 55. Solution: (a)
the influence of favorable conditions of temperature and Exp) Option a is correct
pressure it undergoes metamorphism and form Gneiss.
Statement 1 is incorrect. The thickness of troposphere
52. Solution: (d) is greater at the equator as the heated air rises to greater
heights.
Exp) Option d is correct.
Statement 2 is correct. Most ozone (about 90%) is found
Statement 1 is correct. Sedimentary rocks are formed in the stratosphere, which begins about 10–16 kilometers
at Earth’s surface by the hydrologic system. They (6–10 miles) above Earth’s surface and extends up to
form from fragments derived from other rocks and by about 50 kilometers (31 miles) altitude. The stratospheric
precipitation from water. region with the highest ozone concentration is
Statement 2 is correct. Their formation involves the commonly known as the “ozone layer”. The ozone layer
weathering of preexisting rock, transportation of the extends over the entire globe with some variation in
material away from the original site, deposition of the altitude and thickness.
eroded material in the sea or in some other sedimentary Statement 3 is incorrect. In thermosphere temperature
environment, followed by compaction and cementation. increases rapidly with increasing height. Temperature at
Statement 3 is correct. Fossils are the preserved remains its upper limit reaches up to 1700-degree C.
of ancient plants and animals. Sedimentary rocks
contain fossils because, unlike other rocks, they form at 56. Solution: (d)
temperatures and pressures that do not destroy animal Exp) Option d is correct
and plant remains. Statement 1 is correct: Diastrophic forces are caused due
Statement 4 is correct. They typically occur in layers, or to energy originating from within the Earth. They operate
strata, separated one from the other by bedding planes very slowly and their effects become recognizable only
and differences in composition. after thousands and millions of years.
Statement 2 is incorrect: Diastrophic force is a subpart
53. Solution: (a) of Endogenetic forces along with sudden forces. These
Exp) Option a is correct are created by forces deep inside the earth’s surface. It
Seismic discontinuities are the regions in the earth where is also sub-divided into epeiriogenetic and orogenetic
seismic waves behave a lot different compared to the movements.
surrounding regions due to a marked change in physical Statement 3 is incorrect: Diastrophic forces are also
or chemical properties. called as constructive forces which affect larger areas
Option a is correct- Mohorovicic discontinuity of globe and produce meso-level reliefs e.g. mountains,
forms the boundary between the crust and the upper plateaus, plains lakes, big faults, etc.
mantle (asthenosphere). It occurs at an average depth 1. Sudden Forces
of about 8 kilometers beneath the ocean basins and • Volcanic Eruption

Workbook 36
GEOGRAPHY

• Earthquakes 3. This was followed by the Process of Degassing, in


2. Diastrophic forces which the gases were outpoured from the interior of
• Eperiogenetic forces- emergence and submergence earth. The early atmosphere largely consisted of water
vapour, nitrogen, carbon dioxide, methane, ammonia
• Orogenetic forces- tensional forces (cracking and and very little of oxygen. Continuous volcanic
faulting) and compressional forces (wraping and eruptions contributed water vapour and gases.
folding)
4. As the earth cooled, the water vapour released
57. Solution: (b) started getting condensed. The carbon dioxide in
Exp) Option b is correct. the atmosphere got dissolved in rainwater and the
temperature further got decreased causing more
The sequence of events during the formation of stars condensation and more rains. The rainwater falling
and planets are onto the surface got collected into the depressions
1. Formation of Galaxy started by accumulating and gave rise to oceans. And then life began to evolve
hydrogen gas in the form of a very large cloud called along with the process of photosynthesis.
nebula.
2. Growing nebula develops localized clumps of gases. 60. Solution: (d)
These clumps continue to grow into even denser Exp) Option d is correct.
gaseous bodies, giving rise to formation of stars. Statement 1 is incorrect. If a small asteroid or large
3. The gravitational force within lumps leads to the meteoroid survives its fiery passage through earth’s
formation of a core to the gas cloud and a huge atmosphere and lands on earth’s surface, it is then called
rotating disc of gas and dust develops around the gas a meteorite.
core.
Statement 2 is incorrect. If a meteoroid enters the
4. Gas cloud start getting condensed and the matter
Earth’s atmosphere and vaporizes, it becomes a meteor;
around the core develops into small rounded
objects which form planetesimals by the process of a shooting star.
cohesion. 61. Solution: (c)
These larger number of small planetesimals accrete to Exp) Option c is correct.
form a few large bodies in the form of planets.
The Gutenberg discontinuity separates and divides the
58. Solution: (c) lower mantle from the outer core. The lower mantle
Exp) Option c is correct above the Gutenberg line is solid, but the outer core
below the line is liquid molten. Seismic waves change
Mercury, Venus, Earth and Mars are called as the inner their activity and secondary waves completely disappear
planets as they lie between sun and the belt of the asteroids. below this boundary. Beneath this boundary zone, the
They are also called as Terrestrial Planets (earth like). The molten outer core is much denser than the mantle above
rest four are called as Jovian or Gas Giant planets. Jovian
as a result of the heavy amounts of iron it contains, and
means Jupiter like.
below this layer is the inner core, which is composed of
Statement 1 is incorrect. Jovian planets have thick extremely hot solid nickel and iron.
atmosphere mainly consisting of Hydrogen and Helium
as the solar winds were not all that intense to cause 62. Solution: (b)
removal of gases from the Jovian planets. Exp) Option b is correct.
Statement 2 is correct. The terrestrial planets are Statement 1 is incorrect. Tillite is the sedimentary
smaller and thus have lower gravity. This is the reason rock formed out of deposits of glaciers. At the base the
that it could not hold the escaping gases and thus have Gondwana system there is thick tillite indicating extensive
rocky surfaces. and prolonged glaciation. The glacial tillite provides
Statement 3 is correct. The terrestrial planets are made unambiguous evidence of paleoclimates.
up of rocks and metals and so they have relatively higher Statement 2 is correct. The Gondwana system of
densities as compared to Jovian planets. sediments from India is known to have its counter
59. Solution: (d) parts in six different landmasses of the Southern
Hemisphere. The base of the system has thick tillite.
Exp) Option d is correct. Counter parts of this succession are found in Africa,
The sequence of events is Falkland Island, Madagascar, Antarctica and Australia
1. It is the process of Differentiation through which besides India. Overall resemblance of the Gondwana type
earth forming material got separated into different sediments clearly demonstrates that these landmasses had
layers, like Crust, Mantle, Outer Core and Inner Core. remarkably similar histories. Thus, it supports the theory
2. Then started the formation of atmosphere in which of Continental Drift.
the first stage is marked by the loss of Primordial
atmosphere i.e., loss of hydrogen and helium present 63. Solution: (b)
in early atmosphere by solar winds. Exp) Option b is correct

37 Workbook
GEOGRAPHY

According to Wegner, two types of forces are responsible polar-fleeing force relates to the rotation of the earth. The
for the drifting of continents: tidal force is due to the attraction of the moon and the sun
• equatorward due to the interaction of forces of gravity, that develops tides in oceanic waters. However, most of
pole-fleeing force (due to centrifugal force caused by scholars considered these forces to be totally inadequate.
earth’s rotation) and buoyancy (ship floats in water Therefore, it lacked a geological mechanism to explain
due to buoyant force offered by water), and how the continents could drift across the earth’s surface.
• westwards due to tidal currents because of the earth’s 67. Solution: (d)
motion (earth rotates from west to east, so tidal Exp) Option d is correct.
currents act from east to west, according to Wegener).
Statement 1 is correct. The internal structure of the earth
• Polar fleeing force: The polar-fleeing force relates to begins with the crust and the uppermost part of the
the rotation of the earth. Earth is not a perfect sphere;
mantle which is called Lithosphere. Its thickness range
it has a bulge at the equator. This bulge is due to the
varies between 5-100 km in oceanic parts and about 200
rotation of the earth (greater centrifugal force at the
km in the continental areas.
equator). Centrifugal force increases as we move from
poles towards the equator. This increase in centrifugal Statement 2 is correct. A tectonic plate, also called
force has led to pole fleeing, according to Wegener. Lithospheric plate, is a massive, irregularly-shaped slab
of solid rock generally composed of both continental and
64. Solution: (c) oceanic lithosphere. A plate may be referred to as the
Exp) Option c is correct. continental plate or oceanic plate depending on which of
The Great Red Spot is a persistent high-pressure region the two occupy a larger portion of the plate. Pacific plate
in the atmosphere of Jupiter, producing an anticyclonic is largely an oceanic plate whereas the Eurasian plate
storm that is the largest in the Solar System. may be called a continental plate.
It is a storm in Jupiter’s southern hemisphere with Statement 3 is correct. Lithospheric plates move
crimson-coloured clouds that spin counter clockwise at horizontally over the Asthenosphere as rigid units. The
wind speeds that exceed those in any storm on Earth. It is upper portion of the mantle is called asthenosphere.
currently about 1.3 times as wide as our planet. The word astheno means weak. It is considered to be
extending up to 400 km. It is the main source of magma.
65. Solution: (c) The lower mantle extends beyond the asthenosphere. It is
Exp) Option c is correct. in solid state.
Statement 1 is correct. The newest, thinnest crust on 68. Solution: (a)
Earth is located near the centre of mid-ocean ridge—the Exp) Option a is correct
actual site of seafloor spreading. The age, density, and
thickness of oceanic crust increases with distance from Statement 1 is correct – Albedo of a surface is the
the mid-ocean ridge. proportion of sunlight that the surface can reflect back
into space. The reflected amount of radiation by Earth as
Statement 2 is correct. The mid oceanic ridges are
a whole is called the albedo of the earth. Earth’s albedo is
subjected to frequent shallow earthquakes due to up thrust
about 0.3 or 30%.
of the lava. Whereas at the convergent plate boundaries,
deep focus earthquakes are found. Therefore at deep Statement 2 is correct – Greater is the albedo, more
trenches deep seated earthquakes are found. amount of heat is reflected back. However, as albedo
decrease, more and more heat is absorbed which causes
66. Solution: (d) greater local warming, which in turn leads to further ice
Exp) Option d is correct. melt. This in turn sets a loop and decreases albedo further.
Statement 1 is incorrect: The mobile rock beneath Statement 3 is incorrect – Albedo of land is much
the rigid plates of the Earth is believed to be moving greater than albedo of oceans and water bodies. Water
in a circular manner. The heated material rises to the bodies and oceans tends to absorb most of the heat that
surface, spreads and begins to cool, and then sinks falls on them, thus they have lower albedo (6%). Whereas
back into deeper depths. This cycle is repeated over and land, depending on texture and topography reflects far
over to generate what scientists call a convection cell or more amount of heat as compared to water bodies and
convective flow. Heat within the earth comes from two thus have higher albedo (10-40%).
main sources: radioactive decay and residual heat. The
slow movement of hot, softened mantle that lies below 69. Solution: (c)
the rigid plates is the driving force behind the plate Exp) Option c is correct.
movement. Thus, Plate tectonic provided the mechanism Statement 1 is incorrect: Mid-Ocean ridges occur along
to explain movement of continents divergent plate boundaries, where new ocean floor is
Statement 2 is incorrect: Wegener suggested that the created as the Earth’s tectonic plates spread apart. As
movement responsible for the drifting of the continents the plates separate, molten rock rises to the seafloor,
was caused by pole-fleeing force and tidal force. The producing enormous volcanic eruptions of basalt.

Workbook 38
GEOGRAPHY

Statement 2 is incorrect: When two oceanic plates 74. Solution: (a)


collide against each other, the older and therefore Exp) Option a is correct.
heavier of the two subducts beneath the other, initiating
Statement 1 is correct: “Crust” describes the outermost
volcanic activity in a manner similar to that which occurs
shell of a terrestrial planet. Oceanic crust, is mostly
at an oceanic-continental convergent plate boundary
composed of different types of basalts. Geologists often
and forming a volcanic island arc.
refer to the rocks of the oceanic crust as “sima”. Sima
70. Solution: (a) stands for silicate and magnesium, the most abundant
Exp) Option a is correct minerals in oceanic crust. Rocks of Sial is absent in
oceanic crust.
The Nazca plate include convergence towards the east
with the South-American plate and divergence with the Statement 2 is incorrect: The outer core of the Earth is
Antarctic, Pacific and Cocos Plates. in liquid state while the inner core is in solid state. The
metallic nickel–iron outer core is liquid because of the
71. Solution: (a) high temperature. However, the intense pressure, which
Exp) Option a is correct. increases towards the inner core, dramatically changes the
melting point of the nickel–iron, making it solid.
1 and 4 are correct. They are Direct sources of
information about the Earth’s interior -Volcanic Statement 3 is incorrect: The mantle contains a weaker
eruptions, Deep earth mining. zone called asthenosphere. It is from this that the molten
rock materials find their way to the surface. Asthenosphere
2 and 3 are incorrect. They are indirect sources of or low velocity zone because P and S waves travel more
information about the earth’s interior. Indirect slowly in this region and the S waves are attenuated or
sources – Seismic waves, Meteorites, Gravitation, weakened. Thus, S waves travels through it.
Radio activities etc.
75. Solution: (c)
72. Solution: (b)
Exp) Option c is correct.
Exp) Option b is correct
Only 1 and 2 are correct.
Statement 1 is incorrect. Oxygen is the most abundant
1. Increase in density of earth during primordial
element found in earth’s crust followed by silicon. About
stage - During the primordial stage of earth, the
98 per cent of the total crust of the earth is composed of
earth was mostly in volatile stage. So due to gradual
eight elements like oxygen, silicon, aluminium, iron,
increase in density, the temperature inside the earth
calcium, sodium, potassium and magnesium, and the
has increased. As a result, the material inside started
rest is constituted by titanium, hydrogen, phosphorous,
getting separated depending upon their densities.
manganese, sulphur, carbon, nickel and other elements.
This allowed heavier metals to sink towards the center
Statement 2 is correct. Continental crust is broadly of the earth and the lighter ones to move towards the
granitic in composition and, with a density of about 2.7 surface. This process is called differentiation. With
grams per cubic cm, is somewhat lighter than oceanic the passage of time, it cooled further and solidified
crust, which is basaltic (i.e., richer in iron and magnesium and condensed into a smaller size. This led to the
than granite) in composition and has a density of about development of the outer surface in the form of a
2.9 to 3 grams per cubic cm. crust.
73. Solution: (b) 2. The Big Splat - During the formation of moon, due
to giant impact called the “big splat” the earth was
Exp) Option b is correct. further heated up and then again, the process of
Pair 1 is correctly matched. Mohorovicic (Moho) differentiation started which resulted into layering of
discontinuity forms the boundary between the crust and materials depending upon different densities.
the asthenosphere (upper reaches of the mantle) where 3. Degassing - Degassing is not the reason behind the
there is a discontinuity in the seismic velocity. It occurs at formation of lithosphere. It is the reason behind the
an average depth of about 8 kilometres beneath the ocean evolution of atmosphere by which gases and water
basins and 30 kilometres beneath continental surfaces. vapor were released from the interior of the earth.
The cause of the Moho is thought to be a change in rock
composition from rocks containing feldspar (above) to 76. Solution: (b)
rocks that contain no feldspars (below). Exp) Option b is correct.
Pair 2 is correctly matched. Gutenberg Discontinuity Evidence which supports the continental drift theory
lies between the mantle and the outer core, about 2,880 are
kilometers (1,798 miles) beneath Earth’s surface. 1. Rich placer deposits of gold were found in the
Pair 3 is incorrectly matched. Conrad discontinuity lies Ghana coast but its source was not found. But the
between the hydrosphere and crust whereas Lehmann gold bearing veins in Brazil indicate that these were
discontinuity lies between outer and inner core. derived from the Brazilian plateau.

39 Workbook
GEOGRAPHY

2. Rocks of the same age were found across the continent The ultimate source of energy behind endogenic
like the earliest marine deposits along the coastline movements is earth’s internal heat which is a result of
of South America and Africa are of the Jurassic age. mainly radioactive decay. Differences in temperature and
3. Identical fossil remains were found on different pressure (temperature gradients or geothermal gradients
continents for example, the Glossopteris fossil was and pressure gradients) among various layers of the
found in Australia, Antarctica, India, South Africa, earth give rise to density differences and these density
and South America—all the southern continents. differences give rise to conventional currents. The Earth’s
However, the Slow movement of hot mantle below the rotation (Coriolis effect) can influence where convection
continent was not the findings of Wegener theory but currents travel. Thus, the movement of the lithospheric
of Arthur Holmes who said that this is the main driving plates due to the convectional currents is the cause behind
force behind the plate movement. endogenic movements.
Statement 2 is correct. Diastrophism and volcanism are
77. Solution: (b)
endogenic geomorphic processes since they help in the
Exp) Option b is correct. building of newer landforms on earth.
Statement 1 is incorrect. Aurora occurs in the All processes that move, elevate or build up portions of the
thermosphere layer. The thermosphere starts just above earth’s crust are collectively referred to as diastrophism.
the mesosphere. They include: (i) orogenic processes that help in mountain
Statement 2 is incorrect. Sunspots are the coldest part of building through process of folding of the crust; (ii)
the sun and appear as dark blobs on its white-hot surface. epeirogenic processes involving uplifting of the earth’s
Solar flares and coronal mass ejections are associated with crust; (iii) earthquakes (iv) plate tectonics involving
sunspots. Bright, consistent auroras are most visible horizontal movements of plates of the earth’s crust.Thus
during the height of sunspot activity. there is deformation of the Earth’s crust.
Statement 3 is correct. The colors of the aurora vary,
depending on altitude and the kind of atoms involved. 80. Solution: (c)
If ions strike oxygen atoms high in the atmosphere, the Exp) Option c is correct
interaction produces a red glow. A green-yellow hue, Statement 1 is correct. The Indian plate’s movement in
occurs as ions strike oxygen at lower altitudes. Reddish the northern direction is a type of convergent boundary
and bluish light that often appears in the lower fringes of interaction. The Indian plate is moving northward and
auroras is produced by ions striking atoms of nitrogen. collides with the Eurasian plate. The subduction zone
Ions striking hydrogen and helium atoms can produce along the Himalayas forms the northern plate boundary
blue and purple auroras. in the form of continent-continent convergence.
78. Solution: (d) Statement 2 is correct. The boundary between Indian
Exp) Option d is correct. and the Antarctic plate is a type of divergent boundary.
It runs in through West-East direction and merging into
The magnetic north pole (where a compass needle points
to) does not have a permanent location. Instead, it usually the spreading site, a little south of New Zealand.
wobbles around close to the geographic north pole (the The Indo-African boundary is also divergent.
point around which the Earth spins) over time due to
movements within the Earth’s core. For reasons still not 81. Solution: (d)
entirely clear, magnetic pole movements can sometimes Exp) Option d) is correct.
be more extreme than a wobble. One of the most dramatic Statement 1 is Incorrect:Direct sources only provide a
of these pole migrations took place some 42,000 years ago part of the information. The interior of the earth can be
and is known as the Laschamps Excursion, named after understood onlyby indirect evidences as neither any one
the village where it was discovered in the French Massif has noranyone can reach the interior of the earth. Direct
Central. and Indirect sources jointly provide us a large picture
A geomagnetic excursion, like a geomagnetic reversal, about the interior of the Earth.
is a significant change in the Earth’s magnetic field. Statement 2 is Incorrect:Meteors that reach the earth are
Unlike reversals, an excursion is not a “permanent” re- the indirect source of information about Earth’s interior.
orientation of the large-scale field, but rather represents
Although they do not come from the interior of Earth,
a dramatic, typically a (geologically) short-lived change
the material and thestructure observed in the meteors are
in field intensity. Excursion events typically only last a
similar to that of the earth.
few thousand to a few tens of thousands of years.
Increase in pressure and temperature with depth,
79. Solution: (c) gravitation, magnetic field, and seismic activity are the
Exp) Option c is correct other sources ofobtaining indirect information.
Statement 1 is correct. In general terms, the endogenic Gold mines, Deep Ocean Drilling Project, Volcanic
forces are mainly land building forces and the exogenic eruptions are Direct sources of information about
processes are mainly land wearing forces. interior of the Earth.

Workbook 40
GEOGRAPHY

82. Solution: (d) mild to very cold and at such low temperature life would
Exp) Option d is correct. not be able to exist.
When the solar system settled into its current layout about Statement 2 is incorrect. Earth’s magnetic field is
4.5 billion years ago, Earth formed when gravity pulled generated in the earth’s outer core. The temperature of
swirling gas and dust in to become the third planet from the outer core ranges from 4400 °C in the outer regions
the Sun. to 6000 °C near the inner core. There is lower pressure in
the outer core than the inner core which means the metal
Earth is one of the four inner, terrestrial planets in our in the outer core is fluid. This differences in temperature,
solar system. Just like the other inner planets—Mercury, pressure and composition within the outer core cause
Venus, and Mars—it is relatively small and rocky. Early convection currents in the molten iron of the outer core
in the history of the solar system, rocky material was the as cool, dense matter sinks while warm, less dense matter
only substance that could exist so close to the Sun and rises. This flow of liquid iron generates electric currents,
withstand its heat. which in turn produce magnetic fields.
Statement 1 is correct. Earth’s early atmosphere was
most likely composed of hydrogen and helium. The early 84. Solution: (d)
atmosphere is supposed to have been stripped off as a Exp) Option d is correct.
result of the solar winds. This happened not only in case At about 864,000 miles (1.4 million kilometres) wide, the
of the earth, but also in all the terrestrial planets, which sun is 109 times wider than Earth, and it accounts for
were supposed to have lost their primordial atmosphere more than 99.8 percent of the solar system’s total mass.
through the impact of solar winds.
Statement 1 is correct. The sun was formed more than
Statement 2 is correct. As the planet changed, and 4.5 billion years ago when a cloud of dust and gas called
the crust began to form, volcanic eruptions occurred a nebula collapsed under its own gravity. As it did, the
frequently. These continuous volcanic eruptions cloud spun and flattened into a disk, with sun forming at
pumped water vapour, ammonia, and carbon dioxide its centre. The disk’s outskirts later accreted into our solar
into the atmosphere around Earth. As the earth cooled, system, including Earth and the other planets.
the water vapour released started getting condensed.
The carbon dioxide in the atmosphere got dissolved in Statement 2 is correct. Deep in the sun’s core, nuclear
rainwater and the temperature further decreased causing fusion converts hydrogen to helium, which generates
more condensation and more rains. energy. Particles of light called photons carry this energy
to the top layer of the solar interior, the convection zone.
Statement 3 is correct. The rainwater falling onto the
There, hot plasmas rise and fall, which transfers energy to
surface got collected in the depressions to give rise to
the sun’s surface, called the photosphere.
oceans. Around 2.2 billion to 2.7 billion years before the
present, the process of photosynthesis got evolved and Statement 3 is correct. The steady stream of charged
primitive life began in those oceans. Tiny organisms particles that is radiated by the sun in addition to light
known as cyanobacteria, or blue-green algae later and heat are known as the solar wind. The wind blows
contributed oxygen into the atmosphere. about 280 miles (450 kilometres) a second throughout
the solar system, extending the sun’s magnetic field out
Statement 4 is correct. At its beginning, Earth was
more than 10 billion miles. Beyond that distance, the solar
extremely hot, to the point that the planet likely consisted
almost entirely of molten magma. Over the course of a wind gives way to the colder, dense material that drifts in
few hundred million years, the planet began to cool and between stars, forming a boundary called the heliopause.
oceans of liquid water formed. Heavy elements began Statement 4 is correct. Solar flares are large eruptions
sinking past the oceans and magma toward the centre of of electromagnetic radiation from the Sun lasting from
the planet. As this occurred, Earth became differentiated minutes to hours. It is a tremendous explosion on the Sun
into layers, with the outermost layer being a solid that happens when energy stored in ‘twisted’ magnetic
covering of relatively lighter material while the denser, fields (usually above sunspots) is suddenly released.
molten material sunk to the centre. They heat material to many millions of degrees and
produce a burst of radiation across the electromagnetic
83. Solution: (d)
spectrum, including from radio waves to x-rays and
Exp) Option d is correct. gamma rays.
The atmosphere is a layer of gases that surround the Earth.
Carbon dioxide (CO2), methane (CH4), and ozone (O3), 85. Solution: (d)
are kinds of greenhouse gases. Exp) Option d is correct
Statement 1 is incorrect. Greenhouse gases collect in the The Orion Arm is the minor spiral arm of the Milky
Earth’s atmosphere. Without the greenhouse effect, Earth’s Way Galaxy and is probably some 3,500 light-years wide.
average temperature would drop (and not increase). The Solar System (including the Earth and the Sun) is
Now, it is about 57 degrees Fahrenheit (14 degrees located in the Orion Arm - about one-third from the
Celsius). It could drop to as low as 0 degrees Fahrenheit centre (26,000 light years from the centre) of the Milky
(minus 18 degrees Celsius). The weather would go from Way galaxy.

41 Workbook
GEOGRAPHY

Our Milky Way galaxy is the island of stars. Our solar


system lies between two prominent spiral arms, in what
astronomers once thought was a mere bridge of stars, gas,
and dust clouds. In recent decades, research advances
have revealed that we live in our very own spiral arm of
the galaxy, albeit a relatively minor one. Our spiral arm
is the Orion-Cygnus Arm, or simply, the Orion Arm or
Local Arm.
The Orion Arm gets its name from the constellation
Orion the Hunter, which is one of the most prominent
constellations of Northern Hemisphere winter (Southern
Hemisphere summer).

86. Solution: (c)


Exp) Option c is correct
On earth, the four seasons—spring, summer, fall, and Statement 1 is correct. The majority of the mass of the
winter—follow one another regularly. Each has its own entire atmosphere is contained in the troposphere—
light, temperature, and weather patterns that repeat yearly. between approximately 75 and 80 percent.
The revolution of the earth and the inclination of the Statement 2 is incorrect. Stratosphere (and not
earth’s axis in a fixed direction cause seasons. Earth’s troposphere) is calm with little turbulence. Strong
axis is an invisible line that runs through its center, from horizontal winds blow in the stratosphere, but there is little
pole to pole. As the earth spins on its axis, producing turbulence. This is ideal for planes that can fly in this part
night and day, it also moves about the sun in an elliptical of the atmosphere. In contrast, the troposphere tends to
(elongated circle) orbit that requires about 365 1/4 days to change suddenly and violently. Air masses, areas of high-
pressure and low-pressure systems, are moved by winds
complete. The earth’s spin axis is tilted with respect to its
in the troposphere. These weather systems lead to daily
orbital plane. This is what causes the seasons. When the weather changes as well as seasonal weather patterns
earth’s axis points towards the sun, it is summer for that and climate systems, such as El Nino.
hemisphere. When the earth’s axis points away, winter can
Statement 3 is correct. Most of the water vapour in
be expected. the atmosphere, along with dust and ash particles, are
87. Solution: (a) found in the troposphere. Thus, most of Earth’s clouds
are located in this layer. Clouds, from low-lying fog
Exp) Option a is correct. to thunderheads to high-altitude cirrus, form in the
The troposphere is the lowermost layer of the atmosphere. troposphere. This is the most important layer for all
Its average height is 13 km and extends roughly to a height biological activities. All changes in climate and weather
of 8 km near the poles and about 18 km at the equator. take place in this layer. The temperature in this layer
decreases at the rate of 1°C for every 165m of height.
Option a is correct. The troposphere is thickest at the
equator, and much thinner at the North and South Poles 89. Solution: (b)
because equator is warmer and heat is transported to great Exp) Option b is correct
heights by strong convectional currents. Thus, convection The Earth has a rather slight equatorial bulge: it is about
currents transport heat to greater heights at the equator 43 km (27 mi) wider at the equator than pole-to-pole. The
than the poles due to heat difference on Earth. This also bulging of the equator and flattening at the poles is due to
implies that the warmer the weather, the thicker is the the rotation of the Earth which creates the centrifugal
troposphere. force. Centrifugal force increases as one moves from poles
towards the equator and is therefore greater at the equator.
88. Solution: (b) This increase in centrifugal force has led to pole fleeing.
Exp) Option b is correct. As a result, a person on the equator is over 13 miles farther
from Earth’s center than a person on the North Pole is.
The Earth’s atmosphere is divided into five layers based That difference is more than double the distance between
on characteristics like temperature. These layers are - sea level and the top of Mount Everest, but on a planetary
troposphere, stratosphere, mesosphere, thermosphere and scale, this bulge is virtually imperceptible. Centrifugal
exosphere. The troposphere is the lowest atmospheric force is the force which is perpendicular to the axis of
layer. the Earth on excluding the equator.

Workbook 42
GEOGRAPHY

90. Solution: (d) moving away faster than we could ever move towards it.
Exp) Option d is correct Reason is false.
Meteoroids are lumps of rock or iron that orbit the sun, Astronomers have found that whole clusters of galaxies
just as planets, asteroids, and comets do. Meteoroids, are moving in convoy towards a single point in the sky.
especially the tiny particles called micrometeoroids, are This unexpected motion appears to be separate from the
extremely common throughout the solar system. expansion of the universe and the researchers even suggest
Mesosphere protects the Earth from meteoroids. It that a force beyond the visible universe is the reason. The
is the third layer of the atmosphere which lies above discovery has been named “dark flow”.
the stratosphere. The mesosphere extends from the Cosmologists have observed two distinct effects caused by
stratopause (the upper boundary of the stratosphere) to invisible entities in the universe: dark matter is known to
about 85 kilometres (53 miles) above the surface of the affect the rotation of galaxies and dark energy seems to
Earth. Meteorites burn up in this layer on entering from be causing the expansion of the universe to accelerate.
the space. The mesosphere has the coldest temperatures In simple terms, the space between galaxies is also
in the atmosphere, dipping as low as -120 degrees Celsius expanding, which drives the edge of the universe further
(-184 degrees Fahrenheit, or 153 kelvin). and further.
91. Solution: (d) 93. Solution: (d)
Exp) Option d is correct Exp) Option d is correct.
A Black hole is a cosmic body of extremely intense gravity. There are three criteria of the International Astronomical
Statement 1 is correct. Black holes are created when Union (IAU) for a full-sized planet are:
a massive star reaches the end of its life and implodes, It is in orbit around the Sun.
collapsing in on itself. It is formed by the collapse of It has sufficient mass to assume hydrostatic equilibrium (a
individual stars that are relatively small, but incredibly nearly round shape).
dense. A black hole takes up zero space, but does have
mass — originally, most of the mass that used to be a star. It has “cleared the neighbourhood” around its orbit.
Statement 2 is correct. According to Einstein’s general Pluto meets only two of these criteria. It has not managed
theory of relativity, the gravity of a black hole is so to clear its neighbourhood. Clearing the neighbourhood
intense that nothing can escape it – not even light means that the planet has become gravitationally
cannot get out. The gravity is so strong because matter has dominant so that there are no other bodies of comparable
been squeezed into a tiny space and is extremely dense. size other than its own satellites or those otherwise under
They get “bigger” as they consume matter near them. The its gravitational influence, in its vicinity in space. But
bigger they are, the larger a zone of “no return” they have, Pluto shares its orbital neighbourhood with Kuiper belt
where anything entering their territory is irrevocably lost objects such as the plutinos.
to the black hole. This point of no return is called the There are no such criteria of fulfilling the minimum
event horizon. mass. Eris is also considered a dwarf planet, but it is
Statement 3 is correct. No black hole is present in the even more massive than Pluto. So, if Pluto is considered
or is close to the Solar system for the Earth and therefore as a planet, then Eris should also be granted planet status
Earth will not fall into a black hole. The newly discovered as well.
black hole is about 1,011 light-years from our solar system 94. Solution: (c)
in the star system HR 6819.
Exp) Option c is correct.
Besides, black holes do not go around the Universe
swallowing stars and planets. Suppose a black hole were Acid rain, or acid deposition, is a broad term that includes
to take the place of the Sun. The black hole would have any form of precipitation with acidic components, such
the same gravity as the sun. Earth would still not fall into as sulfuric or nitric acid that fall to the ground from the
it and would orbit the black hole as it would orbit the Sun. atmosphere in wet or dry forms. This can include rain,
snow, fog, hail or even dust that is acidic.
92. Solution: (d) Statement 1 is correct. Soils deficient in limestone are
Exp) Option d is correct. more affected by Acid rain. Due to naturally occurring
Assertion is false. As far as we can tell, there is no edge to limestone, soil is slightly basic (which has a pH of greater
the universe. Space spreads out infinitely in all directions. than 7). Bases counteract acids, these soils tend to balance
Furthermore, galaxies fill all of the space through-out out some of the acid rain’s acidity. Some of the Rocky
the entire infinite universe. Since universe is technically Mountains and parts of the north-western and south-
expanding faster than the speed of light as the space eastern United States, where limestone does not naturally
between galaxies is also expanding. It drives the edge occur in the soil, acid rain can harm the environment.
of the universe further and further from us every single Statement 2 is correct. Granite and sandstone are
second. Therefore, it is theoretically impossible to ever primarily composed of silicate minerals, which are
reach the “edge” of the universe, since it will always be resistant to acid attack. A few sandstones are less resistant

43 Workbook
GEOGRAPHY

because they contain a carbonate cement that dissolves Quasar- It stands for quasi-stellar object and that refers
readily in weak acid. Limestone and marble are primarily to any of the celestial objects that are similar to a star in
composed of the mineral calcite (calcium carbonate), appearance but has a comparatively high redshift.
which dissolves readily in weak acid. So, buildings made
of granite and sandstone are less likely to be affected by 97. Solution: (c)
acid deposition as compared to buildings made up of Exp) Option c is correct.
limestone. The outer planets are Jupiter, Saturn, Uranus, and
95. Solution: (d) Neptune. These gas giants are made up primarily of
hydrogen and helium, the same elements that make up
Exp) Option d is correct. most of the Sun. The inner planets, or terrestrial planets,
Space radiation is different from the kinds of radiation are the four planets closest to the Sun: Mercury, Venus,
we experience on Earth. Space radiation is comprised Earth, and Mars.
of atoms in which electrons have been stripped away Statement a is incorrect. Inner planets have lower
as the atom accelerated in interstellar space to speeds gravity as compared to Outer planets. So higher gravity
approaching the speed of light – eventually, only the can hold the escaping gases much stronger as compared
nucleus of the atom remains i.e. this radiation consists
to Inner planets.
primarily of ionizing radiation which exists in the form
of high-energy, charged particles. While radiation on Statement b is incorrect. The presence of rings does not
Earth consists of non- ionizing radiation decide the gaseous nature of planets. In fact, all four Outer
planets have rings, although only Saturn’s rings are easily
There are three naturally occurring sources of space
visible from Earth. Rings are composed of countless small
radiation- (1) particles trapped in the Earth’s magnetic
field (2) particles shot into space during solar flares pieces of rock and ice, each orbiting its planet like a tiny
(solar particle events) (3) galactic cosmic rays (from moon.
Supernova or exploding stars), which are high-energy Statement c is correct. As a result of their enormous
protons and heavy ions from outside our solar system. gravity, Outer planets captured a large amount of
All of these kinds of space radiation represent ionizing hydrogen and other gasses from the surrounding nebula
radiation. The photosphere is also the source of solar and thus these are gaseous in nature.
flares. CME (Coronal mass ejections) are huge bubbles of Statement d is incorrect. The cores of all four
plasma (ionized atomic matter with high kinetic energy) Outer planets or all inner planets are made of some
threaded with a magnetic field that is ejected from the combination of rock, metal and hydrogen compounds.
Sun’s corona (outer atmosphere). The inner radiation belt, The deeper you go, the hotter and denser it gets. An
or Van Allen Belt, consists of ionizing radiation in the increase in temperature and density means an increase in
form of very energetic protons—by-products of collisions pressure. Thus, these are not the deciding factors for the
between GCR and atoms of Earth’s atmosphere. gaseous nature of outer planets.
Cell phones and microwave ovens emit a type of
radiation called radiofrequency radiation. It’s a form of 98. Solution: (d)
electromagnetic radiation and Electromagnetic radiation Exp) Option d is correct.
is non-ionizing radiation. So, these are not the sources The Moon is Earth’s only natural satellite and the fifth
of space radiation. largest moon in the solar system. The Moon’s presence
96. Solution: (b) helps stabilize our planet’s wobble and moderate our
climate. The Moon’s distance from Earth is about 240,000
Exp) Option b is correct. miles (385,000km).
Nebulae are clouds of gas and dust which form in
Statement 1 is incorrect. Recent studies confirm that
interstellar space. They range in size from millions of
our moon does indeed have an atmosphere consisting of
miles across to hundreds of light years across. Nebulas
some unusual gases, including sodium and potassium,
are mostly composed of hydrogen along with smaller
which are not found in the atmospheres of Earth, Mars or
amounts of other gases and dust particles. Stars and
Venus. It’s an infinitesimal amount of air when compared
planetary systems are formed from the gas and dust
within nebulae under their own weight through fusion. to Earth’s atmosphere.
Like humans, stars are also born at a certain time and Statement 2 is incorrect. As Moon has very thin
place, and their birthplaces are what astronomers call atmosphere, meteoroids can easily strike its surface as
nebulae or star nurseries. nothing is obstructing its path. As a result, tiny craters are
Planetary Nebula form when a star runs out of fuel to quite common on lunar rocks.
burn. The star blows off its layers of gas in the shape of a However almost all the meteoritic material is vapourised
ring or bubble. It is a type of nebula. They are made of gas in Earth’s atmosphere, leaving a bright trail fondly called
and plasma. “shooting stars.”
Pulsar- It is a rotating neutron star, that is highly Statement 3 is incorrect. Impact crater on the lunar
magnetized and it emits a beam of electromagnetic surface is mainly due to the attacking of its surface by
radiation. meteors and not due to plate tectonics. Plate tectonics is

Workbook 44
GEOGRAPHY

absent on the Moon. The mass of the Moon is probably Statement d is correct. Earth has a molten core, which
too small for this to occur. The Moon’s Mantle, too cool is responsible for producing our Earth’s magnetosphere.
to move easily, has no convection and no active tectonic Our magnetosphere protects us from the sun’s harmful
plate motions, due the Moon’s smaller size and cooler rays that otherwise would spell disaster for any life on
interior than Earth. Earth. Because of this, Earth’s magnetic field played a vital
part in making Earth a habitable planet.
99. Solution: (d)
Exp) Option d is correct. 101. Solution: (a)
The present composition of earth’s atmosphere is chiefly Exp) Option a is correct.
contributed by nitrogen and oxygen. The process which is Our Solar system consists of eight planets. The nebula,
involved in the formation or modification of the present from which our Solar system is supposed to have been
atmosphere are as following: formed, started its collapse and core formation some time
Loss of Primordial atmosphere- It is the first stage. The 5-5.6 billion years ago and the planets were formed about
early atmosphere, with hydrogen and helium, is supposed 4.6 billion years ago.
to have been stripped off as a result of the solar winds.
Statements 1 and 2 are correct.
This happened not only in case of the earth, but also in all
the terrestrial planets, which were supposed to have lost Our solar system consists of the sun (the star), 8 planets,
their primordial atmosphere through the impact of solar more than 200 moons, millions of smaller bodies like
winds. asteroids and comets and huge quantity of dust-grains
Then the composition of atmosphere is modified by and gases.
the process of photosynthesis. Oceans began to have Out of the eight planets, mercury, Venus, earth and mars
the contribution of oxygen through the process of are called as the inner planets as they lie between the sun
photosynthesis. and the belt of asteroids the other four planets are called
Solar winds contributed in stripping the planets of the outer planets. Alternatively, the first four are called
hydrogen and helium. Terrestrial, meaning earth-like as they are made up of
Degassing- is the process through which gases like water rock and metals, and have relatively high densities.
vapour, nitrogen, methane, ammonia, carbon dioxide, Statement 3 is incorrect.
very little free oxygen are outpoured from the interior. The rest four planets are called Jovian or Gas Giant planets.
Volcanic eruptions –contributed to the evolution of Jovian means Jupiter-like. Most of them are much larger
water vapour and gases to the atmosphere. than the terrestrial planets and have thick atmosphere,
mostly of helium and hydrogen (thick atmosphere is
100. Solution: (c)
found in Jovian planets). All the planets were formed in
Exp) Option c is correct. the same period sometime about 4.6 billion years ago.
Statement a is correct. The goldilocks zone is the distance
that a planet must be from its star in order to support 102. Solution: (d)
liquid water. If a planet is too close to its sun, it will be Exp) Option d is correct
too hot – the water will evaporate. If it’s too far, it will be Statement 1 is correct. About 70% of the earth’s crust is
too cold, and the planet will freeze. Earth, as it just so covered by oceans. The oceanic crust originates from the
happens, lies in that goldilocks zone where water can exist
Mid-oceanic Ridges and spreads towards the continents
in its liquid state. A planet’s location in this habitable zone
and then it goes down to magma again and it melts.
plays a big role in what the planet will look like – whether
that is a dry, desert-like planet, or a humid ice planet. This could be the reason that we always find the younger
oceanic crust and it is not more than 200 million years
Statement b is correct. Unlike Venus or Mars, the Earth old. All of the oceanic crusts are made up of basalt rock
has an active plate tectonic system, which continuously which is a volcanic rock. Some deposits of continental
produces and destroys the crust. The same system also
sediments near the continents and organic depositions
gives us active water, oxygen and carbon dioxide cycles, as
in the deep-sea zone are also found there. Oceanic
material is subducted down into the Earth’s interior, and
eventually re-erupted from volcanoes. However, while crusts are heavier and its density is 3.0 g/cm3. Primary
tectonics give the Earth active water and gas cycles, the wave (P) travels through this layer at a speed of 7 km/s.
tectonic system itself moves too slowly to be the process Mineral compositions of the rocks are mainly silica and
that actively maintains the climate. magnesium. This layer is known as Sima.
Statement c is incorrect. The high-mass stars emit too Statement 2 is correct. Outgassing is the release of gas,
much ultraviolet radiation. The lifetime of a high-mass previously dissolved, trapped, frozen or absorbed in
star is too short. High-mass stars live only a few million some material (e.g., rock). It occurs mainly along mid-
years, and life probably cannot arise until a planet is at ocean ridges, subduction zones and at magma hotspots.
least a couple hundred million years old. Smaller stars like Pockets of carbon dioxide exist in the Earth’s crust.
the Sun survive far longer than do high-mass stars. Thus, Volcanic eruptions and earthquakes can release these gas
it is not a reason for habitable nature of earth. pockets.

45 Workbook
GEOGRAPHY

103. Solution: (c) Pacific Plate), while continental crust buoyantly projects
Exp) Option c is correct. above sea level.
The lithosphere is the rigid, outermost shell on Earth.
Statement 1 is correct. It is composed of the crust and
the portion of the upper mantle that behaves elastically
on time scales of thousands of years or greater.
The outermost shell of a rocky planet, the crust, is defined
on the basis of its chemistry and mineralogy. This includes
the crust and the uppermost mantle, which constitute the
hard and rigid outer layer of the Earth. The uppermost
part of the lithosphere that chemically reacts to the
atmosphere, hydrosphere, and biosphere through the soil-
forming process is called the pedosphere.
Statement 2 is correct. The lithosphere comprises two
distinct parts: The upper part consists of granite rocks and 105. Solution: (c)
forms the continents and its main mineral constituents
are silica and alumina and referred to as the Sial with Exp) Option c is correct.
an average density of 2.7 The lower part is a continuous According to plate tectonic theory fold mountains
zone of denser basaltic rocks forming the ocean floor, are formed due to collision of two convergent plates.
comprising mainly silica, iron and magnesium and is Fold mountains are formed along the destructive plate
therefore called sima with an average density of 3.0.The boundaries. It is obvious that the process of mountain
sial and sima together form the earth’s crust which varies building is associated with destructive plate boundaries of
in thickness from only 3-4 miles beneath the oceans to two convergent plates. The plate tectonic theory envisages
as much as 30 miles under some parts of the continents. the formation of mountains due to compression of
sediments caused by the collision of two convergent plate
104. Solution: (d) boundaries.
Exp) Option d is correct Statement 1 is correct. The Himalayan Mountain range
Statement 1 is correct. An earthquake is shaking of the was formed as the Indian tectonic plate collided into the
earth. It is a natural event. It is caused due to release of Eurasian plate about 50 million years ago. The process
Energy due to the tectonic activity beneath the earth. continues even today, which causes the height of the
This generates waves that travel in all directions. The mountain range to rise every year. The peak of Mt Everest
release of energy occurs along a fault. A fault is a sharp rises in height by approximately 2 cm each year due to
break in the crustal rocks. Rocks along a fault tend to move plate tectonics.
in opposite directions. As the overlying rock strata press
However, in the absence of such plate movements, there
them, the friction locks them together. However, their
is a possibility that the height of Mt Everest may gradually
tendency to move apart at some point of time overcomes
reduce as a result of erosion. Erosion would wear the
the friction. As a result, the blocks get deformed and
mountains down and with no tectonic activity to refresh
eventually, they slide past one another abruptly. This
them, over a few million years they would erode down to
causes a release of energy, and the energy wave’s travel in
low rolling hills. So, as a result the whole planet would be
all directions.
much flatter in such a case.
Statement 2 is correct. Volcanoes form when one
tectonic plate moves under another. Usually a thin, Statement 2 is correct. According to the Plate Tectonic
heavy (denser) oceanic plate sub ducts, or moves under, Theory, the oceanic crust has its origin at the mid oceanic
a thicker continental plate. When this happens, the ridges. The oceanic crust originates from the Mid-oceanic
ocean plate sinks into the mantle. A volcano is a place Ridges and spreads towards the continents and then it
where gases, ashes and/or molten rock material – lava – goes down to magma again and it melts.
escape to the ground. The mantle contains a weaker zone As with oceanic crust, continental crust is created by plate
called asthenosphere. It is from this that the molten rock tectonics. At convergent plate boundaries, where tectonic
materials find their way to the surface. The material in the plates crash into each other, continental crust is thrust up
upper mantle portion is called magma. in the process of orogeny, or mountain-building. Thus, in
Statement 3 is correct. Oceanic crust is formed at the absence of plate tectonics movement there will be no
sea-floor spreading centres, and continental crust is new formation of oceanic and continental crust.
formed through arc volcanism and accretion of terrines 106. Solution: (c)
through tectonic processes. Oceanic crust is denser than
continental crust owing to their different compositions. Exp) Option c is correct.
As a result of this density stratification, oceanic crust The Sun and its family of planets formed when a cloud
generally lies below sea level (for example most of the of dust and gas condensed 4.6 billion years ago. Several

Workbook 46
GEOGRAPHY

hundred million years after the Earth took form, an outer • It has a radius of 1220 km.
crust developed.
108. Solution: (c)
From the outset, heat and gravity shaped the evolution of
the planet. Exp) Option c is correct.
Statement 1 is correct. The materials that accreted The Gutenberg discontinuity separates and divides the
into the early Earth were probably added piecemeal, lower mantle from the outer core. The lower mantle
without any particular order (though some models above the Gutenberg line is solid, but the outer core
call on sequential accretion of metallic and then silicate below the line is liquid molten. Seismic waves change
materials). The early Earth was very hot from gravitational their activity and secondary waves completely disappear
compression and Radioactive decay (much more than below this boundary. Beneath this boundary zone, the
today). molten outer core is much denser than the mantle above
The early Earth was probably partially or largely as a result of the heavy amounts of iron it contains, and
molten. The denser metallic liquids sank to the centre of below this layer is the inner core, which is composed of
the Earth and less dense silicate liquids rose to the top. extremely hot solid nickel and iron.
In this way the Earth very quickly differentiated into a
metallic, mostly iron core and a rocky silicate mantle.
Statement 2 is correct. It is believed that during the
early formation of Earth, asteroids were continuously
bombarding the planet, and could have been carrying
with them an important source of water. Scientists believe
the asteroids that slammed into Earth contained a
significant amount of water in their minerals.
Some suggest that nearly 30 per cent of the water
contained initially in the asteroids would have remained
in the fragmented sections of rock on Earth, even after 109. Solution: (b)
impact. Exp) Option b is correct.
Earth consists of three main layers: the crust, the mantle,
107. Solution: (d)
and the core. The core accounts for almost half of Earth’s
Exp) Option d is correct. radius, but it amounts to only 16.1% of Earth’s volume.
The solid spheroid that comprises the inner core of the Most of Earth’s volume (82.5%) is its mantle, and only a
Earth is made of different metals, specifically, mostly small fraction (1.4%) is its crust.
iron and nickel. Siderophiles, or elements that dissolve Statement 1 is incorrect. The layer below the solid crust
in iron, are also found in the inner core. These elements is mantle. It has higher density than that of the crust. The
include gold, cobalt, and platinum. Most of the Earth’s mantle contains a weaker zone called asthenosphere. It
sulphur is in the core as well. is from this layer that the molten rock materials find
Composition of the Inner Core: their way to the surface. The material in the upper mantle
portion is called magma.
Element type Weight %
Once it starts moving towards the crust or it reaches
1. Iron 80% the surface, it is referred to as lava. The material that
2. Nickel 5 to 15 % reaches the ground includes lava flows, pyroclastic debris,
volcanic bombs, ash and dust and gases such as nitrogen
3. Siderophiles 2 to 3 % compounds, sulphur compounds and minor amounts of
4. Sulphur and Oxygen 5 to 10 % chlorene, hydrogen and argon.
Formation of the Inner Core: Statement 2 is correct. The transfer of heat and material
The inner core and other layers of the Earth were in the mantle helps determine the landscape of Earth.
formed during the accretion of the planets in the solar Activity in the mantle drives plate tectonics, contributing
system by the separation of lighter silicate minerals to volcanoes, seafloor spreading, earthquakes, and
and more dense metallic minerals. The light elements orogeny(mountain-building).
naturally rose to the top of the planet, while the denser
ones coalesced in the center and created the inner core. 110. Solution: (d)
This process was largely exothermic, or heat releasing, Exp) Option d is correct.
causing the entire Earth to heat by some 2000°C.The
The correct chronological sequence of the above events
inner core has the following characteristics:
is -
• It is a solid spheroid
It is the process of Differentiation through which earth
• It is approximately 5200°C (9800°F). forming material got separated into different layers, like
• It generates its own magnetic field. Crust, Mantle, Outer Core and Inner Core.

47 Workbook
GEOGRAPHY

Then started the formation of atmosphere in which the plate, it heats up and becomes magma. The magma will
first stage is marked by the loss of Primordial atmosphere rise through the crust until it reaches the surface. This
i.e., loss of hydrogen and helium present in early magma creates a chain of volcanoes or a volcanic arc near
atmosphere by solar winds. the boundary of the top plate. There are two types of arcs:
This was followed by the Process of Degassing, in which the island arcs and continental arcs.
gases were outpoured from the interior of earth. The early Statement 4 is correct. Accretionary wedges are pieces
atmosphere largely consisted of water vapour, nitrogen, of the subducting plate that have broken off at the trench.
carbon dioxide, methane, ammonia and very little of
oxygen. Continuous volcanic eruptions contributed water 112. Solution: (b)
vapour and gases. Exp) Option b is correct
As the earth cooled, the water vapour released started An earthquake is an intense shaking of Earth’s surface.
getting condensed. The carbon dioxide in the atmosphere The shaking is caused by movements in Earth’s outermost
got dissolved in rainwater and the temperature further got layer.
decreased causing more condensation and more rains. The Statement 1 is incorrect. As for now, we cannot predict
rainwater falling onto the surface got collected into the when an earthquake will happen. The only thing scientists
depressions and gave rise to oceans. And then life began can do is to forecast an earthquake: they know that on
to evolve along with the process of photosynthesis. certain segments of certain faults an earthquake is due,
because of the past earthquake records (in corals and
111. Solution: (d)
sediments), and because of the strain built-up by the
Exp) Option d is correct plates’ motions. Knowing whether that earthquake will
The location on which sinking of a tectonic plate occurs happen tomorrow, in ten years or in a hundred years is
is called a subduction zone. There are three ways in which currently impossible.
convergence can occur. These are: Statement 2 is correct. When the blocks of earth get
between an oceanic and continental plate; deformed and eventually, slide past one another abruptly,
between two oceanic plates; and it causes a release of energy, and the energy waves travel
in all directions. The point where the energy is released is
between two continental plates. called the focus of an earthquake, alternatively, it is called
Deep ocean trenches, Fold mountains with active the hypocentre. The energy waves travelling in different
volcanoes, island arcs, and seismic fault lines are directions reach the surface. The point on the surface,
examples of features that can form along converging plate nearest to the focus, is called epicentre. It is the first one
tectonic boundaries. Other features include accretionary to experience the waves. It is a point directly above the
wedges, forearc basins, backarc basins and remnant arcs. focus.

Statement 1 is correct. Oceanic trenches are formed at 1.3. Volcanism & Earthquakes
subduction zones. Oceanic plates meet continental plates
in the water, so trenches are formed as the oceanic plate 113. Solution: (a)
goes under the continental plate. These trenches can be Exp) Option a is correct
very deep if the plate that is subducting (going down) is Statement 1 is correct. A hot spot is fed by a region deep
an older and colder plate. Younger oceanic plates are less within the Earth’s mantle from which heat rises through
dense and the angle will be shallower. the process of convection. This heat facilitates the melting
Statement 2 is correct. Forearc basins are between the of rock at the base of the lithosphere. The melted rock,
island arc and the trench, whereas the backarc is behind known as magma, often pushes through cracks in the
the island arc. These basins catch sediment (dirt and small crust to form volcanoes.
rocks that wash away in rain) runoff from the island arcs. Statement 2 is incorrect. Hot spot volcanism is unique
Statement 3 is correct. Volcanic arcs form parallel to because it does not occur at the boundaries of Earth’s
subduction zones. As one plate descends under another tectonic plates, where all other volcanism occurs.

Workbook 48
GEOGRAPHY

Instead, it occurs at abnormally hot centers known as Option d is incorrect – In Plinian eruptions, dissolved
mantle plumes. volatile gases stored in the magma are channeled to the
top through a narrow conduit (pipe-like structure). The
114. Solution: (d) gases erupt into a massive column of the gas plume that
Exp) Option d is correct. reaches up to 2 to 45 km into the atmosphere.
Statement 1 is incorrect. Earthquakes are very abundant
117. Solution: (d)
in subduction zone of convergent plate boundary.
Earthquakes are also common along transform faults, Exp) Option d is correct.
such as the San Andreas Fault. Also, earthquakes do Statement 1 is incorrect. In order to become fossilized,
occur along the divergent boundaries like the mid- an organism must be buried quickly before it is destroyed
Atlantic ridge. by erosion or eaten by other organisms. This is why fossils
Statement 2 is incorrect. Shallow focus earthquakes are found almost exclusively in sediment and sedimentary
mostly occur on divergent plate boundaries. Mid- rocks. Plutonic and Volcanic rocks are kind of igneous
oceanic ridges occur on divergent plate boundaries so rocks.
the earthquakes here are shallow focus. Deep-focus Igneous rocks, which form from cooling magma or lava,
earthquakes are most common on convergent plate and metamorphic rocks, which have been altered by heat
boundaries. Therefore, it is found in areas of circum- and pressure, are unlikely to contain fossils (but may
pacific belt as well as Alpine-Himalayan belt. contain under special circumstances). Thus, fossils are
rare (and not abundant) in plutonic and volcanic rocks.
115. Solution: (c)
As igneous rocks have their origin under conditions of
Exp) Option c is correct.
high temperature thus, they are unfossiliferous.
Statement 1 is correct – The cooling may take place
Statement 2 is incorrect. It is not necessary that volcanic
either on reaching the surface or also while the lava is
rocks are acidic. It may be acidic or basic in nature
still in the crustal portion. Depending on the location
depending upon the quantity of silica present. When
of the cooling of the lava, igneous rocks are classified as
rocks contain high proportion of silica, they are said to be
volcanic rocks (cooling at the surface) and plutonic rocks
acidic. When rocks contain a greater proportion of basic
(cooling in the crust).
oxides of iron, aluminium, or magnesium, they are said
Statement 2 is incorrect – The near horizontal bodies to be basic.
of the intrusive igneous rocks are called sill or sheet,
Statement 3 is incorrect. When volcanic rocks make
depending on the thickness of the material. The thinner
contact with the atmosphere they cool quickly, so the
ones are called sheets while the thick horizontal deposits minerals do not have time to form large crystals resulting
are called sills. in a natural amorphous rock with few or no crystals.
Statement 3 is incorrect – Laccoliths are large dome Whereas Plutonic rocks cools slowly without ever
shaped intrusive bodies connected to a pipe-like conduit reaching the surface. They have large crystals that are
from below. Phacoliths are found at the base of syncline usually visible without a microscope.
or at the top of anticlines.
118. Solution: (c)
116. Solution: (b) Exp) Option c is correct.
Exp) Option b is correct. Statement 1 is correct. It is also known as the Circum-
Option a is incorrect – The Icelandic type eruption is Pacific Belt. It traces the meeting points of many less
characterized by effusions of molten basaltic lava that flow dense tectonic plates, including the Eurasian, North
from long, parallel fissures. Such outpourings often build American, Juan de Fuca, Cocos, Caribbean, Nazca,
lava plateaus. E.g. Deccan Traps, Siberian Traps. Antarctic, Indian, Australian, Philippine, and other
Option b is correct. Hawaiian Volcanoes are a type of smaller plates, which all encircle the large Pacific Plate.
Shield volcanoes. These volcanoes are mostly made up of Statement 2 is incorrect. Plates overlap at convergent
basalt, a type of lava that is very fluid when erupted. For boundaries called subduction zones along much of this
this reason, these volcanoes are not steep. They become ring of fire. A significant exception is the border between
explosive if somehow water gets into the vent; otherwise, the Pacific and North American Plates. This stretch of
they are characterized by low explosivity. The upcoming the Ring of Fire is a transform boundary, where plates
lava moves in the form of a fountain and throws out the move sideways past one another. So, transform faults
cone at the top of the vent and develops into a cinder also form near this ring of fire.
cone. Statement 3 is correct. Most of the active volcanoes on
Option c is incorrect – Strombolian eruptions are a The Ring of Fire are found on its western edge, from the
type of volcanic eruption, named after Stromboli (Lipari Kamchatka Peninsula in Russia, through the islands of
Islands, Italy). Strombolian eruptions are driven by the Japan and Southeast Asia, to New Zealand.
continuous formation of large gas bubbles within the
magma. Upon reaching the surface, the bubbles burst 119. Solution: (b)
with a loud pop, throwing magma in the air. Exp) Option b is correct.

49 Workbook
GEOGRAPHY

Statement 1 is correct. When two oceanic plates collide other words, an earthquake of magnitude 8 is not twice
as in the western Pacific Ocean, the volcanoes form a as great as an earthquake of magnitude 4 but 10,000
curved line of islands, known as an island arc, that is times as great.
parallel to the trench, as in the case of the Mariana Islands Statement 2 is correct. The intensity scale called the
and the adjacent Mariana Trench. Mercalli scale, takes into account the visible damage
Statement 2 is incorrect. Volcanic arc is formed when caused by the event. The range of intensity scale is from
oceanic plate collides with continental plate e.g., 1-12.
volcanoes in Andes of western south America. On the Statement 3 is correct. The point where the energy
contrary when two continental plates collide volcanic during the earthquake is released is called the focus or
arcs not are formed. hypocenter of an earthquake. The energy waves travelling
Statement 3 is incorrect. Island arcs are the marine in different directions reach the surface. The point on
subset of currently active or formerly active volcanoes the surface, nearest to the focus, is called epicentre. It is
located near the boundary between two converging the first one to experience the waves. It is a point directly
tectonic plates. When the crust of the overlying plate is above the focus.
less than ~25 km thick, most of the volcanoes lie below sea
level. Only the largest volcanoes form islands, which, 122. Solution: (a)
therefore, constitute an island arc. Exp) Option a is correct
Statement 1 is correct. Extrusive landforms are formed
120. Solution: (d) from material thrown out to the surface during volcanic
Exp) Option d is correct. activity. The materials thrown out include lava flows,
Statement 1 is correct. There are two types of body pyroclastic debris, volcanic bombs, ash, dust and gases
waves. They are called P and S-waves. P-waves also called such as nitrogen compounds, Sulphur compounds and
as compressional waves as they produce rarefaction and minor amounts of chlorine, hydrogen and argon.
compression when travelling through the medium. Statement 2 is correct.Shield type volcanic landform
P-waves move faster and are the first to arrive at the is formed by the eruption of highly fluid (low viscosity)
surface and hence are used to detect earthquakes. The lava, which travels farther and forms thinner flows than
P-waves are similar to sound waves. They can travel the more viscous lava erupted from a stratovolcano and
through gaseous, liquid and solid materials and travel in therefore not steep and mostly made of basaltic lava.
longitudinal direction. Examples of shield volcanoes are Kilauea and Mauna Loa
in Hawaii.
Statement 2 is correct. S-waves are also called as
transverse waves or shear waves or distortional Statement 3 is Incorrect. A mid-oceanic ridge occurs
waves. They are analogous to water ripples or light along divergent plate boundaries (not convergent plate
waves. Transverse waves or shear waves mean that the boundaries), where new ocean floor is created as the
direction of vibrations of the particles in the medium Earth’s tectonic plates spread apart. As the plates separate,
is perpendicular to the direction of propagation of the molten rock rises to the seafloor, producing enormous
wave. Hence, they create troughs and crests in the material volcanic eruptions of basalt. The speed of spreading
through which they pass (they distort the medium). affects the shape of a ridge – slower spreading rates result
in steep, irregular topography while faster spreading rates
Statement 3 is correct.Shadow zone is defined as that
produce much wider profiles and more gentle slopes. The
area where these waves are not detected due to their
mid-ocean ridge is the most extensive chain of mountains
inability to reach there depending on their characteristic
on Earth with more than 90 percent of the mountain
features. The zone between 105° and 145° from epicentre
range lying in the deep ocean.
was identified as the shadow zone for both the types of
waves. S-waves travel in a transverse direction. The entire 123. Solution: (a)
zone beyond 105° does not receive S-waves and is shadow
Exp) Option a is correct
zone for S-waves. Differences in arrival times, waves
taking different paths than expected (due to refraction) Pair 1 is correctly matched. A batholith is a huge mass
and absence of the seismic waves in certain regions of igneous rocks usually granite, which after removal of
called as shadow zones, allow mapping of the Earth’s the overlying rocks form a massive and restaurant upland
interior. region. Example - Wicklow mountains of Ireland
Pair 2 is correctly matched. Laccolith is an igneous
121. Solution: (b) mound with a dome shaped upper surface and a level base
Exp) Option b is correct. fed by a pipe-like conduit from below. Example - Henry
Statement 1 is Incorrect. The earthquake events are scaled Mountains in Utah, USA
either according to the magnitude or intensity of the Pair 3 and pair 4 are Incorrectly matched. Sills are near
shock. The magnitude scale is known as the Richter scale. horizontal bodies of the intrusive igneous rocks along the
The magnitude relates to the energy released during the bedding planes of sedimentary rocks. If a sill is very thin,
quake. The magnitude is expressed in absolute numbers, it is called a SHEET. Example - Great Whin Sill of N.E.
0-10. The Richter Scale is a base-ten logarithmic scale. In England.

Workbook 50
GEOGRAPHY

Dykes are vertical intrusions of molten magma as 127. Solution: (b)


narrow walls of igneous rocks. Example - Cleveland Exp) Option b is correct
Dyke of Yorkshire England.
Statement 1 is correct: Acidic lavas are highly viscous
All the above are examples of intrusive volcanic landforms. with a high melting point. They are rich in silica, have a
Extrusive landforms include the various types of volcanoes light color and low density.
that exist on the earth. Statement 2 is incorrect: Due to their high viscosity,
124. Solution: (d) acidic lavas are slow moving. As a result, the cones
formed by them are steep-sided.
Exp) Option d is correct.
It is basic volcanic eruption in which the lava is highly
Statement 1 is correct – Mid Oceanic Ridges forms an fluid and forms flattened shields or domes.
interconnected chain of mountain systems within the
ocean. These are the longest mountain chain on the Statement 3 is correct: Another feature of acidic lavas
surface of the earth though submerged under the oceanic is that they are explosive in nature. Highly viscous
waters. lava sometimes solidifies inside the vents and creates
an obstruction to out pouring lava resulting in loud
Statement 2 is correct – The rift system at the crest is explosions, throwing out many volcanic bombs or
the zone of intense volcanic activity. The mid-oceanic pyroclasts. Spine or plug is another feature which is
ridges were found to be most active in terms of volcanic formed due to highly viscous lava.
eruptions. The dating of the rocks from the oceanic crust
revealed the fact that they are much younger than the 128. Solution: (a)
continental areas. Exp) Option a is correct.
Statement 3 is correct – In general, the foci of the Statement 1 is correct. Primary waves also called P-waves
earthquake in the areas of mid-oceanic ridges are at vibrate parallel to the direction of propagation of the
shallow depths. While those along the Alpine-Himalayan wave, while secondary waves vibrate perpendicular to
belt as well as the rim of the Pacific, the earthquakes are the direction of propagation of the wave. Primary waves
deep-seated ones. are the first to arrive at the surface. They are similar to
125. Solution: (d) sound waves.
Exp) Option d is correct. Statement 2 is incorrect. Secondary waves also called
S-waves travel only through solid materials while
The Ring of Fire, also referred to as the Circum-Pacific Primary waves travel through solid, liquid and gaseous
Belt, is a path along the Pacific Ocean characterized by materials.
active volcanoes and frequent earthquakes. Its length is
approximately 40,000 kilometers. It traces boundaries 129. Solution: (d)
between several tectonic plates. This includes the Pacific,
Exp) Option d is correct.
Juan de Fuca, Cocos, Indo-Australian, Nazca, North
American, and Philippine Plates. Statement 1 is correct. Sliding of rocks along the
fault line - The most common ones are the tectonic
Seventy-five percent of Earth’s volcanoes are located
earthquakes. These are generated due to sliding of rocks
along the Ring of Fire. Ninety percent of Earth’s
along a fault plane.
earthquakes occur along its path.
Statement 2 is correct. Eruption of a volcano - A special
126. Solution: (c) class of tectonic earthquake is sometimes recognized as
Exp) Option c is correct volcanic earthquake. However, these are confined to
areas of active volcanoes.
Statement 1 is incorrect: Earthquakes are generally
considered to be a phenomenon which is limited only Statement 3 is correct. Intense and unsustainable
till the movement along faults at earth’s crust. But in mining activity - In the areas of intense mining activity,
some circumstances the focus of earthquake is as deep sometimes the roofs of underground mines collapse
as 670 km. This occurs when oceanic crust slips under the causing minor tremors. These are called collapse
continental crust at Wadati-Benioff Zone. earthquakes.
Statement 2 is correct: Intensity of an earthquake is Statement 4 is correct. Explosion of nuclear devices -
the evaluation of severity of ground motion at a given Ground shaking may also occur due to the explosion
location. It is measured by a Modified Mercali Scale. of chemical or nuclear devices. Such tremors are called
Whereas magnitude of an earthquake is related to the explosion earthquakes.
energy released at the hypocenter. It is measured in
Richter scale. 130. Solution: (d)
Statement 3 is correct: Isoseismic line joins the areas Exp) Option d is correct.
which have similar seismic intensity. On the other hand, The effects of earthquake are as follow –
homoseismal line joins the places where earthquake is 1. Differential ground movement - The differential
experienced at the same time. movement, also called as surface faulting, can be of

51 Workbook
GEOGRAPHY

the two sides of a fracture at the Earth’s surface. They index. This index measures volume of material ejected
are of three types - strike-slip, normal, and reverse (or with height of an eruption column and duration of
thrust). eruptions.
2. Landslides – Many times landslides take place in Statement 2 is correct. The scale is open-ended and
conjunction with earthquakes. The most abundant logarithmic, or based on 10; therefore, an increase of “1”
types of earthquakes induced landslides are rock falls on the scale indicates an eruption 10 times more powerful
and slides of rock fragments that form on steep slopes. than the number before it on the scale.
3. Soil liquefaction – In liquefaction, clay-free soil
deposits, primarily sands and silts, temporarily lose 134. Solution: (c)
strength and behave as viscous fluids rather than as Exp) Option c is correct.
solids. Liquefaction takes place when seismic shear Statement 1 is correct. A geyser is a rare kind of hot
waves pass through a saturated granular soil layer, spring that is under pressure and erupts, sending jets
distort its granular structure, and cause some of the of water and steam into the air. Geysers are made from
void spaces to collapse. a tube-like hole in the Earth›s surface that runs deep into
4. Avalanches – Snow avalanches are moving sources of the crust. The tube is filled with water. Near the bottom
infrasonic and seismic energy. They can be triggered of the tube is molten rock called magma, which heats
by many different mechanisms that include the the water in the tube. Water in the lower part of the tube,
shaking produced by earthquakes. close to the magma, becomes superhot. Gradually, it
begins to boil. Some of the water is forced upward. The
131. Solution: (a) boiling water begins to steam, or turn to gas. The steam
Exp) Option a is correct jets toward the surface. Its powerful jet of steam ejects the
Statement 1 is correct. Caldera: In some volcanoes, column of water above it.
the magma chamber below the surface may be emptied Statement 2 is incorrect: The jet of water is usually
after volcanic eruptions. The volcanic material above the emitted with an explosion and is often triggered by gases
chamber collapses into the empty magma chamber, and seeping out of the heated rocks.
the collapsed surface appears like a large cauldron-like
Statement 3 is incorrect: Almost all the world geysers are
hollow (tub shaped) called the caldera. It is an extrusive
confined to 3 areas Iceland, Rotorua district of North
volcanic structure.
Island of New Zealand and Yellow stone park of USA.
Statement 2 is incorrect. Batholith is a large mass of
intrusive igneous rock that forms from cooled magma 135. Solution: (c)
deep in the Earth’s crust. Exp) Option c is correct.
Statement 3 is correct. Mid oceanic ridge: Found Statement 1 is correct. Mercalli scale is a twelve-point
along divergent plate boundaries where basaltic magma scale for expressing the local intensity of an earthquake,
continuously oozes out. It is an extrusive volcanic ranging from I (virtually imperceptible) to XII (total
structure. destruction). The Mercalli scale bases its measurement on
Statement 4 is correct. Crater is an inverted cone-shaped the observed effects of the earthquake and describes its
vent through which the magma flows out. It is an extrusive intensity. It is a linear measurement.
volcanic structure. Statement 2 is correct. The Richter scale measures
132. Solution: (c) the seismic waves, or the energy released, causing the
earthquake and describes the quake’s magnitude.
Exp) Option c is correct. The magnitude of an earthquake is determined from
In case of formation of present-day South Asia, the the logarithm of the amplitude of waves recorded by
collision involved two continental plates and not an seismographs.
oceanic plate. In such cases, due to folding and faulting,
even those seismic activity is retained which leads to 136. Solution: (b)
frequent earthquakes, but the zone of subduction is Exp) Option b is correct
as deep as 50 to 70 kilometres. The magma from the
Volcanism is the eruption of molten rock from inside the
asthenosphere cannot penetrate such a thick continental
crust and hence it stays in the crust, as a result of which Earth to the surface. Volcanism occurs because of Earth’s
there are no volcanoes in India and the entire of South internal heat, and is associated with tectonic processes
Asia. and a part of the rock cycle.
Statement 1 is incorrect. Volcanic eruptions result
133. Solution: (c) in ash being dispersed over wide areas around the
Exp) Option c is correct. eruption site. And depending on the chemistry of the
Statement 1 is correct. The Volcanic Explosivity Index magma from which it erupted, this ash will be contain
(VEI) is a relative measure of the explosiveness of varying amounts of soil nutrients.
volcanic eruptions. Volcano event is measured through In addition, eruptions release bits of rock such as
simple descriptive index called as volcano explosivity potolivine, pyroxene, amphibole, and feldspar, which

Workbook 52
GEOGRAPHY

are in turn rich in iron, magnesium, and potassium. As Statement 1 is correct. In shield volcanoes, very high
a result, regions that have large deposits of volcanic soil fluid basaltic lava flows. The lava spreads in a thin layer
(i.e. mountain slopes and valleys near eruption sites) are before cooling . These are the largest of all volcanoes of
quite fertile.
the world. A large, broad cone with sides sloping gently
Statement 2 is correct. In addition to fertile soil, volcanic away from the centre are the features of these volcanoes.
action also leads to the formation of gems, precious
They become explosive if somehow water gets into the
stones and valuable minerals such as gold and uranium.
These have a wide array of uses from making jewellery, vent; otherwise, they are characterised by low-explosivity.
electronics and weapon manufacture. Volcanism has Statement 2 is correct. Composite volcanoes often result
also led to geothermal energy as an alternative source of in explosive eruptions. Along with lava, large quantities of
energy which contributes a significant quota of electricity
pyroclastic material and ashes find their way to the surface.
production in some countries like Iceland, Kenya, and the
Philippines. This material accumulates in the vicinity of the vent
openings leading to formation of layers, and this makes
137. Solution: (c) the mounts appear as composite volcanoes. Andesitic lava
Exp) Option c is correct. along with pyroclastic material accumulates in the vicinity
Option a is incorrect. Soil creep is the slow, downward of the vent openings leading to the formation of layers,
progression of rock and soil down a low-grade slope. and this makes the mounts appear as a composite volcano
Solifluction is the gradual movement of wet soil or other or a stratovolcano (divided into layers).
material down a slope, especially where frozen subsoil
acts as a barrier to the percolation of water. 139. Solution: (c)
Option b is incorrect. Ballistics refers to rocks or Exp) Option c is correct.
lava lumps ejected during major and minor volcanic
eruptions. Ballistic projectiles are one potentially lethal Volcanism is the eruption of molten rock (magma) onto
and damaging hazard produced in volcanic eruptions. the surface of a planet.
Ballistics are fragments of lava (bombs) or rock (blocks) Statement 1 is correct. Planetary scientists have found
ejected in explosive eruptions. evidence of volcanism on every terrestrial planet and on
Option c is correct. Debris avalanches are the fast moving, many of the moons and even some asteroids. While the
gravity driven currents of partially or fully water land Moon and Mercury have relatively small volcanoes, those
saturated volcanic debris. If the debris flow consists of
a significant fraction of clay sized particles it is called a on Venus, Earth and Mars are large and numerous.
lahar or Mudflow. Lahars are extremely destructive. They Statement 2 is correct. The mid-ocean ridge is a
can flow tens of metres of debris per second and have been continuous range of undersea volcanic mountains more
known to be up to 140 metres (460 ft.) deep. than 70,000 km long that stretches through all the ocean
An avalanche is typically triggered when material on a basins. It includes a collection of volcanic ridges, rifts,
slope breaks loose from its surroundings. This material fault zones, and other geologic features.
then quickly collects and carries additional material down
the slope. There are various kinds of avalanches, including It is formed and evolved as a result of spreading in Earth’s
rock avalanches (which consist of large segments of lithosphere at the divergent plate boundaries. The vast
shattered rock), ice avalanches (which typically occur in majority of volcanic activity on the planet occurs along
the vicinity of a glacier), and debris avalanches (which the mid-ocean ridge, and it is the place where the crust
contain a variety of unconsolidated materials, such as of the Earth is born.
loose stones and soil).
Option d is incorrect. Ground deformation is the melt 140. Solution: (d)
water flood resulting from a volcanic eruption under Exp) Option d is correct.
a glacier. Ground deformation measurements provide
an important indicator about what is happening beneath Ring of Fire is a long horseshoe-shaped seismically active
a volcano. As magma accumulates in an underground belt of earthquake epicentres, volcanoes, and tectonic
reservoir before an eruption, the ground surface typically plate boundaries that fringes the Pacific basin.
swells.
Seventy-five percent of Earth’s volcanoes are located along
138. Solution: (c) the Ring of Fire. Ninety percent of Earth’s earthquakes
Exp) Option c is correct. occur along its path, including the planet’s most violent
A volcano is an opening in the earth’s crust through which and dramatic seismic events. The abundance of volcanoes
gases, molten rocks materials (lava), ash, steam etc. are and earthquakes along the Ring of Fire is caused by the
emitted outward in the course of an eruption. amount of movement of tectonic plates in the area.

53 Workbook
GEOGRAPHY

Statement 3 is correct. Lithification is the process found


in sedimentary rocks. The process by which sediments
are slowly compacted into rocks from the weight of
overlying sediments is called lithification.
143. Solution: (c)
Exp) Option c is correct.
Statement 1 is correct: Exogenetic forces are called as
leveler forces which are generated from the atmosphere
due to varying combination of temperature and
moisture. Erosion and weathering are its two principal
processes.
Statement 2 is correct: Endogenetic forces are credited
for creation of relief features, that’s why they are called
Statements 1, 3 and 4 are correct. Bolivia, Chile, variability developers. These variables are continuously
Ecuador, Peru, Costa Rica, Guatemala, Mexico, United destructed by exogenetic forces, reason why they are also
States, Canada, Russia, Japan, Philippines, Australia, called destructional forces.
Papua New Guinea, Indonesia, New Zealand, and
Antarctica are some of the important places located in 144. Solution: (c)
the ring of the fire. Exp) Option a is incorrect. Erosion is the process in
Statement 2 is correct. Mount Fuji, Japan’s tallest and which there is actual wearing away of the earth’s surface
most famous mountain is an active volcano in the Ring by moving agents like running water, wind, ice and
of Fire. Mount Fuji sits at a “triple junction”, where three waves. It is dynamic process of disintegration of earth’s
tectonic plates (the Amur Plate, Okhotsk Plate, and surface which involves both, the removal of material and
Philippine Plate) interact. their transportation.
Option b is incorrect. Denudation is broader process
141. Solution: (c) of general lowering and leveling out of the surface.
Exp) Option c is correct This is done by process of wearing away of the earth.
Volcanoes are openings, or vents where lava, tephra (small It is carried out in four phases: weathering, erosion,
rocks), and steam erupt on to the Earth’s surface. transportation and deposition. Deposition is a process in
which sediments, soils and rocks are added to a lanform
Acidic magmas have high melting points, high viscosity or landmass.
(stickiness), high in silicates, high gas pressure, but low
Option c is correct. Weathering is the process of gradual
gas solubility at low containing pressure.
disintegration of rocks by atmospheric or weather
Therefore, acidic volcanism is characterized by explosive forces. Opposed to erosion, it is a static process which
eruptions in which large amounts of pyroclastic material involves disintegration and decomposition of rocks in-
are generated. situ.
It creates a steep sided dome. Acidic lava flows are usually Option d is incorrect. Transportation is the process of
associated with Composite volcanoes. shifting of the eroded debris to new positions. This is
Acidic lava is highly viscous and flows only for a short done by climatic and biotic factors. The material shifted
distance. new place then gets dumped in certain parts of earth,
Basic lava has low viscosity so, it can flow very easily where it may accumulate to form new rocks, this is called
as deposition.
over long distances.
145. Solution: (b)
1.4. Weathering Exp) Option b is correct
142. Solution: (a) Hydration is a form of chemical weathering in which
the chemical bonds of the mineral are changed as it
Exp) Option a is correct. interacts with water. One instance of hydration occurs
Statement 1 is incorrect. Sandstone is not an example of as the mineral anhydrite reacts with groundwater. The
organically formed sedimentary rocks. It is an example of water transforms anhydrite into gypsum, one of the most
mechanically formed sedimentary rocks. Sandstone is common minerals on Earth.
formed from layers of sandy sediment that is compacted
and lithified. 146. Solution: (d)
Statement 2 is incorrect. Foliated rocks are a kind of Exp) Option d is correct
metamorphic rocks. Metamorphic rocks are classified Chemical weathering involves chemical decomposition
into two major groups — foliated rocks and non-foliated of rocks and soil. This process includes dissolution,
rocks. solution, carbonation, hydration, oxidation and

Workbook 54
GEOGRAPHY

reduction that act on the rocks to decompose, dissolve or of rocks and deposits helps in the enrichment and
reduce them to a fine state concentrations of certain valuable ores of iron,
• Dissolution is a process where a solute in gaseous, manganese, aluminium, copper etc. It also results in
liquid, or solid phase dissolves in a solvent to form a transportation of minerals from one region to another
solution resulting in the loss of minerals in a particular region.
• Carbonation is a process in which atmospheric Statement 4 is correct. Reduction of relief and changes
carbon dioxide leads to solution weathering. in landforms are a consequence of weathering and
Carbonation refers to reactions of carbon dioxide to erosion.
give carbonates, bicarbonates, and carbonic acid.
150. Solution: (a)
• Oxidation means a combination of a mineral with
Exp) Option a is correct.
oxygen to form oxides (rusting in case of iron) or
hydroxides. Red soils appear red due to the presence Statement 1 is correct. Erosion involves acquisition
of iron oxides. and transportation of rock debris. When massive rocks
break into smaller fragments through weathering and any
• Hydration is the chemical addition of water to
other process, erosional geomorphic agents like running
the atoms and molecules of a mineral. When rock
water, groundwater, glaciers, wind and waves remove
minerals take up water, the increased volume creates
and transport it to other places.
physical stresses within the rock.
Statement 2 is correct. By erosion, relief degrades, i.e., the
147. Solution: (a) landscape is worn down. That means, though weathering
Exp) Option a is correct. aids erosion it is not a pre-condition for erosion to
Statement 1 is correct: Physical Weathering involves take place. Weathering, mass-wasting and erosion are
Mechanical Disintegration of rocks due to temperature degradational processes.
changes, freeze-thaw cycles, wet-dry cycles, crystallization Statement 3 is incorrect. Deposition is a consequence of
of salts, animal and plant activity, etc. erosion. The erosional agents lose their velocity and hence
Statement 2 is incorrect: The applied forces could be: energy on gentler slopes and the materials carried by
them start to settle themselves. The coarser materials get
• Gravitational forces such as overburden pressure, load deposited first and finer ones later. The same erosional
and shearing stress agents viz., running water, glaciers, wind, waves and
• Expansion forces due to temperature changes, groundwater act as depositional agents also.
crystal growth or animal activity
151. Solution: (a)
• Water pressures controlled by wetting and drying
cycles. Exp) Option a is correct.
Many of these forces are applied both at the surface and Statement 1 is correct. Weathering of rocks and deposits
within different earth materials leading to rock fracture. helps in enrichment and concentrations of certain
valuable ores of iron, manganese, aluminium, copper etc.
148. Solution: (c) Statement 2 is incorrect. Weathering is not a pre
Exp) Option c is correct. requisite for mass movement though it aids mass
Statement 1 is correct. Biological weathering is the movement. Mass movements are aided by gravity. No
process contribution to or removal of minerals and ions geomorphic agent like running water, glaciers, wind,
from the weathering environment and physical changes waves and currents participate in the process of mass
due to growth or movement of organisms. movements. This shows that mass movement do not come
Statement 2 is correct. Human beings by disturbing under erosion though there is shift of materials from one
vegetation, ploughing and cultivating soils, also help in place to another.
mixing and creating new contacts between air, water and
minerals in the earth materials. 1.5. Mass Movements and Groundwater
149. Solution: (d) 152. Solution: (d)
Exp) Option d is correct. Exp) Option d is correct.
Statement 1 is correct. Soil formation - Weathering Option a is incorrect – Soil creep is the slow, downward
is the first step in the formation of soil from rocks. progression of rock and soil down a low-grade slope. It
Weathering processes are responsible for breaking down can also refer to the slow deformation of such materials
the rocks into smaller fragments and preparing the way as a result of prolonged pressure and stress.
for formation of regolith and soils. Option b is incorrect – Earth flow refers to the movement
Statement 2 is correct. Soil erosion - Erosion cannot be of water-saturated clayey or silty earth materials down
significant if the rocks are not weathered. That means, low-angle terraces or hillsides. It is a rapid movement.
weathering aids mass wasting and erosion. Option c is incorrect – Mudflow - In the absence of
Statement 3 is correct. Loss of minerals - Weathering vegetation cover and with heavy rainfall, thick layers of

55 Workbook
GEOGRAPHY

weathered materials get saturated with water and either influence of gravity. It may range from slow to rapid,
slowly or rapidly flow down along definite channels. It affecting shallow to deep columns of materials and include
looks like a stream of mud within a valley. creep, flow, slide and fall. Mass movements are very active
Option d is correct - Solifluction is the gradual over weathered slopes rather than over un-weathered
movement of wet soil or other material down a slope, materials.
especially where frozen subsoil acts as a barrier to Statement 2 is correct. Rapid mass movements are
the percolation of water. It involves slow downslope mostly prevalent in humid climatic regions and occur
flowing soil mass or fine-grained rock debris completely over gentle to steep slopes. Earthflow, mudflow, debris
saturated or lubricated with water. This process is avalanche are some of the rapid movements.
quite common in moist temperate areas where surface
melting of deeply frozen ground and long-continued rain 156. Solution: (d)
respectively, occur frequently. When the upper portions Exp) Option d is correct.
get saturated and when the lower parts are impervious
to water percolation, flowing occurs in the upper parts. Statement 1 is correct. The sudden mass movement
of soil is called a landslide. These are relatively rapid
153. Solution: (b) and perceptible movements. The materials involved are
Exp) Option b is correct. relatively dry. The size and shape of the detached mass
depends on the nature of discontinuities in the rock,
Statement 1 is correct. Solifluction involves slow the degree of weathering and the steepness of the slope.
downslope flowing soil mass or fine-grained rock debris Statement 2 is correct. Landslides occur due to
saturated or lubricated with water. It is quite common in
instability or loss of balance of land mass with respect
moist temperate areas where surface melting of deeply
to gravity. The loss in balance occurred mainly due to
frozen ground and long continued rain occur frequently.
excessive water or moisture in the earth mass. Gravity
When the upper portions get saturated and when the
lower parts are impervious to water percolation, flowing acts on such an unstable landmass and causes the large
occurs in the upper parts. chunks of surface materials such as soil and rocks to slide
down rapidly.
Statement 2 is incorrect. There are two classes of mass
movements- Slow movements and Rapid movements.
Slump is included in the category of Rapid movements. 1.6. Landforms
Slump is slipping of one or several units of rock debris 157. Solution: (b)
with a backward rotation with respect to the slope over
which the movement takes place. Exp) Option b is correct.
Statement 3 is incorrect. Mud Flow is common in regions Option a is incorrect.
having no vegetation cover and having heavy rainfall. Due to persistent movement of wind current in one
Thick layers of weathered materials get saturated with direction weathered mantle from rocks or soil gets blown
water and either slowly or rapidly flow down along definite out. This process may create a shallow depression called as
channels. It looks like a stream of mud within a valley. deflation hollows.
154. Solution: (d) Option b is correct. Gently inclined rocky floors close
to the mountains at their foot with or without a thin
Exp) Option d is correct.
cover of debris are called pediments. Such rocky floors
Pair 1 is correctly matched. Debris avalanche, which is form through the erosion of the mountain front through
more characteristic of humid regions with or without a combination of lateral erosion by streams and sheet
vegetation cover and occurs in narrow tracks on steep flooding.
slopes. This debris avalanche can be much faster than the
mudflow. Debris avalanche is similar to snow avalanche. Option c is incorrect.
Pair 2 is correctly matched. Slump is slipping of one or Playas are by far the most prominent landforms in the
several units of rock debris with a backward rotation deserts. There is a gradual deposition of sediment from
with respect to the slope over which the movement takes basin margins, a nearly level plain forms at the centre of
place. Rapid rolling or sliding of earth debris without the basin. In times of sufficient water, this plain is covered
backward rotation of mass is known as debris slide. up by a shallow water body. Such types of shallow lakes are
called as playas. Water is retained only for a short duration
Pair 3 is correctly matched. Sliding of individual
due to evaporation.
rock masses down bedding, joint or fault surfaces is
rockslide. Over steep slopes, rock sliding is very fast and Option d is incorrect.
destructive. Wind deflation and abrasion wore out the rocks of the
deserts leaving behind some remnants of resistant rocks.
155. Solution: (c)
It is polished beautifully in the shape of a mushroom with
Exp) Option c is correct. a slender stalk and a broad and rounded pear-shaped cap
Statement 1 is correct. Mass movements transfer the above. Sometimes, the top surface is broad like a tabletop
mass of rock debris down the slopes under the direct and quite often, the remnants stand out like pedestals.

Workbook 56
GEOGRAPHY

158. Solution: (a) Depositional Plains are formed by the deposition of


Exp) Option a is correct. materials brought by various agents of transportation.
The landforms created as a result of degradational action Statement 3 is correct. The surface of erosional plains is
(erosion and transportation) or aggradational work hardly smooth. These are therefore also called peneplains
(deposition) of running water are called fluvial landforms. which mean almost a plain.
Statement 1 is correct A gully is a landform created by
running water. Gullies resemble large ditches or small 1.7. Running Water
valleys but are metres to tens of metres in depth and width.
161. Solution: (a)
Statement 2 is incorrect A cirque is basically a bowl-
Exp) Option a is correct.
shaped depression formed by the erosional activity of
a glacier. Alternative names for this landform are corrie Statement 1 is correct. An alluvial fan is a triangle-
and cwm. shaped deposit of gravel, sand, and even smaller pieces of
sediment, such as silt. This sediment is called alluvium.
Statement 3 is correct A ravine is a landform narrower
Alluvial fans are usually created as flowing water interacts
than a canyon and is often the product of stream cutting
erosion. Ravines are typically classified as larger in scale with mountains, hills, or the steep walls of canyons. They
than gullies, although smaller than valleys. are formed when streams flowing from higher levels
break into foot slope plains of low gradient.
Statement 4 is incorrect Moraines are accumulations of
dirt and rocks that have fallen onto the glacier surface Statement 2 is Incorrect. Point Bar is formed by alluvium
or have been pushed along by the glacier as it moves. The that accumulates in a linear fashion on the inside bends
dirt and rock composing moraines can range in size from of streams and rivers below the slip-off slope.Point bars
powdery silt to large rocks and boulders. are also known as meander bars. They are found on
the convex side of meanders of large rivers and are
159. Solution: (a) sediments deposited in a linear fashion by flowing waters
Exp) Option a is correct. along the bank.
Statement 1 is correct. Moraines - Depositional Statement 3 is Incorrect. Shallow V-shaped valleys are
landform by glaciers: moraines, drumlins, eskers, formed in the youthful stage of the river because the
outwash plain, etc. Moraines are long ridges of deposits river flows at high velocity and has the requisite power to
of glacial till. cut valleys in rocks which is not possible in the old stage
of river where the flow of water is very slow.
Statement 2 is correct. Barchans - Depositional
landform by Wind: Sand dunes, Ripple marks, Barchans, 162. Solution: (a)
etc. Barchans are Crescent shaped dunes with the points or
Exp) Option a is correct
wings directed away from wind direction i.e., downwind.
Statement 3 is incorrect. Hanging valley is an erosional The drainage basin of the Nile river covers 11 countries:
feature. A characteristic feature of glaciated mountain Egypt, Sudan, South Sudan, Eritrea, Ethiopia, Kenya,
topography, a hanging valley is a tributary to a main valley Congo, Burundi, Rwanda, Uganda and Tanzania.
which has been deeply scoured by glacial ice, leaving the 163. Solution: (a)
tributary valley “hanging” above the main valley.
Exp) Option a is correct.
Statement 4 is incorrect. Sinkholes are cavities in the
ground that form when water erodes an underlying rock Statement 1 is correct. When a river runs over alternating
layer. It is an erosional feature. layers of hard and soft rock, rapids may form. Due to the
unequal resistance of hard and soft Rocks the outcrop of
160. Solution: (d) hard rock may cause a river to jump or fall down a stream
Exp) Option d is correct. forming rapids. Similar falls of greater dimensions are also
Statement 1 is incorrect. Structural plains are relatively referred to as cataracts. Rapids and cataracts may occur
undisturbed horizontal surfaces of the Earth. They are at any part of river course, but they are most common
structurally depressed areas of the world that make up in the youthful stage of the river.
some of the most extensive natural lowlands on the Earth’s Statement 2 is incorrect. A meander is a winding curve or
surface. Example- Great Plains of USA bend in a river. Meanders are the result of both erosional
The difference between structural plains and depositional and depositional processes. They are typical of the middle
plains is that the structural plains are relatively and lower course of a river. This is because vertical
undisturbed plains and depositional plains are erosion is replaced by lateral erosion, plus deposition
continuously happening as forces of deposition like within the floodplain. In the lower course (old stage) of
wind, water etc. are constantly at work the river the meanders become more pronounced.
Statement 2 is incorrect. Erosional Plains are formed by Statement 3 is incorrect. River Piracy is a common
the continuous and a long-time erosion of all sorts of feature observed in the mountain course of the river or
upland. the youthful stage of river.

57 Workbook
GEOGRAPHY

If a greater gradient stream (Stream A) enlarges its own 167. Solution: (a)
basin at the expense of the weaker stream (Stream B) due Exp) Option a is correct.
to its greater erosive power. Then it is called river piracy
Pair 1 is correctly matched. Kettle lakes are depressions
i.e., stream A captures or pirates stream B.
in the outwash plain left by melting of masses of stagnant
164. Solution: (d) water. They are formed by glaciation.
Exp) Option d is correct. Pair 2 is incorrectly matched. Cirque lakes or tarns
are formed by glaciation. A glacier on its way down the
Statement 1 is incorrect: In the upper or mountain valley leaves behind circular hollows in the heads of the
course the predominant action of river is vertical valleys up in the mountains. Such hollows are cirques or
erosion. The valley developed is deep, narrow and corries. Their over deepened floor may be filled with water
V-shaped. Down cutting takes place so rapidly that lateral to form cirque lake or tarns.
erosion cannot keep pace.
Pair 3 is correctly matched. The solvent action of rain
Statement 2 is incorrect: In the middle or valley course water on lime stone carves out solution hollows. When
lateral erosion tends to replace vertical erosion. Active these become clogged with debris, karst lakes may form in
erosion of banks widens the V-shaped valley. The work them. They are formed by erosion.
of the river is predominantly transportation with some
deposition. 168. Solution: (d)
Statement 3 is incorrect: The work of the river is Exp) Option d is correct.
predominantly transportation with some deposition in Eskers: These are long, narrow, sinuous ridges composed
the middle or valley course. In the lower or plain course, of sand and gravel which mark the former sites of sub-
the work of river is mainly deposition. glacial melt-water streams.
165. Solution: (a) Process of formation: When glaciers melt in summer,
the water flows on the surface of the ice or seeps down
Exp) Option a is correct. along the margins or even moves through holes in the ice.
When a river reaches the sea, the fine materials it has not These waters accumulate beneath the glacier and flow like
yet dropped are deposited at its mouth, forming a fan- streams in a channel beneath the ice. Such streams flow
shaped alluvial area called a delta. over the ground (not in a valley cut in the ground) with ice
The following are conditions necessary for formation of forming its banks. Very coarse materials like boulders
river delta: and blocks along with some minor fractions of rock
1. Active vertical and lateral erosion in the upper debris carried into this stream settle in the valley of ice
course of the river. The river needs to have enough beneath the glacier and after the ice melts can be found
material to deposit at the river mouth otherwise all as a sinuous ridge called esker.
materials will be eroded by ocean current or being
deposited before they reach the river mouth.
2. Sheltered Coast with low to no activities of tides
(tideless)
3. Sea adjoining the delta to be shallow or the load will
disappear in the deep waters
4. There should be no strong current running right
angles to the river mouth, washing away the diluents.

1.8. Glaciers
166. Solution: (c)
Exp) option c is correct.
Landforms of glaciated lowlands
Statement 1 is correct. Cirques are deep, long and wide
troughs or basins with very steep concave to vertically Option a is incorrect.
dropping high walls at its head as well as sides. There Cirque: They are deep, long and wide troughs or basins
can be two or more cirques one leading into another down with very steep concave to vertically dropping high walls
below in a stepped sequence. at its head as well as sides.
Statement 2 is correct. Horns form through head ward Option b is incorrect.
erosion of the cirque walls. If three or more radiating
glaciers cut headward until their cirques meet, high, Glacial Valleys/Troughs: Glaciated valleys are trough-like
sharp pointed and steep sided peaks called horns form. and U-shaped with broad floors and relatively smooth,
The highest peak in the Alps, Matterhorn and the highest and steep sides. The valleys may contain littered debris or
peak in the Himalayas, Everest are in fact horns formed debris shaped as moraines with swampy appearance.
through headward erosion of radiating cirques. Option c is incorrect.

Workbook 58
GEOGRAPHY

Boulder clay or glacial till: This is an unsorted glacial side valley is at an elevation (appears hanging) from the
deposit comprising a range of eroded materials — main valley below.
boulders, angular stones, sticky clay and fine rock flour. Hanging valleys are smaller as compared to the main
It is spread out in sheets, not mounds, and forms gently valley. It is a shallow canyon formed over a large canyon.
undulating till or drift plains.
Hanging Valleys are found high up on the sides of larger
169. Solution: (d) U-shaped valleys.
Exp) Option d is correct 171. Solution: (b)
A moraine is material left behind by a moving glacier. This Exp) Option b is correct
material is usually soil and rock. Just as rivers carry along
all sorts of debris and silt that eventually builds up to form Antarctica is the world’s largest ice sheet. Much of the
deltas, glaciers transport all sorts of dirt and boulders that Antarctic ice sheet surface lies above 3000 m above sea
build up to form moraines. level. This massive thickness of ice drowns whole mountain
ranges, and numerous volcanoes exist underneath the icey
Moraines are divided into four main categories: lateral exterior.
moraines, medial moraines, supraglacial moraines, and
terminal moraines. Statement 1 is correct. The Antarctic Ice Sheet is drained
by fast-flowing ice streams, which respond quickly to
Pair 1 is correct. Lateral moraine forms along the sides climate change; they can thin, accelerate, recede, or even
of a glacier. As the glacier scrapes along, it tears off stop flowing entirely. Beneath the thick skin of ice, there is
rock and soil from both sides of its path. This material flowing water and subglacial lakes.
is deposited as lateral moraine at the top of the glacier’s
edges. Lateral moraines are usually found in matching Antarctic glaciers terminate on land or in the sea, as
ridges on either side of the glacier. The glacier pushes floating ice shelves or grounded or floating outlet
material up the sides of the valley at about the same glaciers.
time, so lateral moraines usually have similar heights. Statement 2 is incorrect. Antarctica is affected by ocean
If a glacier melts, the lateral moraine will often remain currents. Parts of West Antarctica are grounded well
as the high rims of a valley. below sea level, which can act to destablise the ice sheet.
Pair 2 is correct. Supraglacial moraine is material on Warm ocean currents are able to melt ice shelves from
the surface of a glacier. Lateral and medial moraines below, and cause a retreat of the glaciers. The Antarctic
can be supraglacial moraines. Supraglacial moraines Circumpolar Current (ACC) is an ocean current that
are made up of rocks and earth that have fallen on flows clockwise from west to east around Antarctica.
the glacier from the surrounding landscape. Dust and Statement 3 is correct. Warming over the Antarctic
dirt left by wind and rain become part of supraglacial Peninsula is exacerbated by a strengthening of the
moraines. Sometimes the supraglacial moraine is so Antarctic Oscillation, which is a periodic strengthening
heavy, it blocks the view of the ice river underneath. and weakening of the tropospheric westerlies that
If a glacier melts, supraglacial moraine is evenly surround Antarctica. Changing pressure patterns result
distributed across a valley. in flow anomalies, with cooling over East Antarctica and
Pair 3 is correct. Ground moraines often show up as warming over the Antarctic Peninsula.
rolling, strangely shaped land covered in grass or other
vegetation. They don’t have the sharp ridges of other 172. Solution: (a)
moraines. A ground moraine is made of sediment Exp) Option a is correct
that slowly builds up directly underneath a glacier by Statement 1 is correct. Glacial till is the sediment
tiny streams, or as the result of a glacier meeting hills
deposited by a glacier. Glacial till, deposited by melting
and valleys in the natural landscape. When a glacier
ice as ground moraine, forms a thin mantle, and is often
melts, the ground moraine underneath is exposed.
composed of loamy sand, rock fragments and large
Ground moraines are the most common type of moraine
and can be found on every continent. boulders.
Statement 2 is incorrect. An arete is a thin, crest of rock
170. Solution: (a) left after two adjacent glaciers (erosional feature, not
Exp) Option a is correct depositional) have worn a steep ridge into the rock. A
Glaciers carve a set of distinctive landform features as horn result when glaciers erode three or more arêtes,
they erode namely U-shaped valleys, fjords, and hanging usually forming a sharp-edged peak. Cirques are concave,
valleys. circular basins carved by the base of a glacier as it erodes
Hanging valleys are formed by two different glacier the landscape.
flows that interact with each other. A glacier with
the relatively small amount of material flows into the 1.9. Deserts
main glacier with the more glacial material. With time,
the bottom of the main glacier becomes lower than the 173. Solution: (b)
elevation of the small glacier. As the ice melts leaving the Exp) Option b is correct.

59 Workbook
GEOGRAPHY

Only 2 and 3 are correct. 176. Solution: (d)


1. Fjords: A glacially over deepened valley, usually Exp) Option d is correct.
narrow and steep sided, extended below sea level Deserts are characterized by their lack of rainfall. Most
and naturally filled with seawater. U-shaped valleys, deserts get less than ten inches of precipitation each year
fjords, and hanging valleys are examples of the kinds and evaporation usually exceed rainfall. Deserts – regions
of valleys glaciers can erode. where more water evaporates from the ground than is
2. Deflation hollows: When the sand is carried away over replaced by precipitation are generally extremely hot, but
long distances by the wind, depressions are formed in some, like the Gobi in Asia, experience very cold winters.
the area from which is lands in. These depressions are
The mid-latitude desert is an excessively dry region with
called deflation hollows and are commonly found in an arid climate located between 30 degrees and 60 degrees
non-mountainous arid regions. north and south of the equator.
3. Yardangs: They are the result of sand grains pushed
Statement 1 is correct. Mid-latitude deserts are mostly
by persistent winds to form low ridges of sand carved
situated on plateaus and are a part of continental
in soft rock. This is commonly found in flat deserts
interiors. Mid-latitude deserts lie at far away locations
where steady winds blow away dust and silt.
from the coast or are blocked by high mountains
174. Solution: (d) surrounding them, they are cut off from the moisture-
laden winds blowing from the seas. E.g., Gobi Desert is
Exp) Option d is correct.
form due to continentally and Patagonia desert due to
Option d is correct. Mushroom rock, inselbergs and rain-shadow effect.
zeugen are all landforms caused by wind erosion in
Statement 2 is correct. Interior deserts, which are found
deserts. Barchans are formed due to the depositional
in the heart of continents, exist because no moisture-
force of wind.
laden winds reach them. By the time air masses from
• A mushroom rock, also called rock pedestal or a coastal areas reach the interior, they have lost all their
pedestal rock, is a naturally occurring rock formed moisture. Interior deserts are sometimes called inland
due to sand blasting effects of wind. deserts.
• An inselberg is an isolated hill, knob, ridge, outcrop, Statement 3 is correct. In desert, humidity is usually so
or small mountain that rises abruptly from a gently low that not enough water vapor exists to form clouds.
sloping or virtually level surrounding plain. Light rains often evaporate in the dry air, never reaching
• Zeugen are tabular masses of rock found in arid and the ground. The sun’s rays beat down through cloudless
semi-arid areas formed when the more resistant rock skies and bake the land. The ground heats the air so
is reduced at a slower rate than softer rocks around it. much that air rises in waves you can actually see. These
• Barchans are crescent-shaped dunes, whose shimmering waves confuse the eye, causing travelers to
windward side is convex whereas the leeward side is see distorted images called mirages.
concave and steep.
177. Solution: (c)
175. Solution: (c) Exp) Option c is correct.
Exp) Option c is correct Deserts are areas that receive very little precipitation.
Option c is correct. A Barchan is a crescent shaped Although some deserts are very hot, with daytime
landform with the points or wings directed away from temperatures as high as 54°C (130°F), other deserts have
wind direction. They are wind depositional landforms cold winters or are cold year-round.
found in the deserts. Statement 1 is incorrect. Desert habitats are difficult to
Option a is Incorrect. It refers to Longitudinal dunes live in. Most of the animals living in deserts are nocturnal
which are formed when supply of sand is poor and (active during night). They hide underground during
wind direction is constant. They appear as long ridges of the day when it is very hot and emerge at night. Most of
considerable length but low in height. the animals who live in the desert are insects, scorpions,
Option b is Incorrect. It refers to a wind landform called reptiles and spiders. Most of the mammals who live in
Pediment. It is an erosional landform. Gently inclined the desert are very small, but large mammals like camels,
rocky floors close to the mountains at their foot with or gazelles and donkeys have adapted to deal with the very
without a thin cover of debris, are called pediments. Such dry conditions and can survive for long periods of time
rocky floors form through the erosion of mountain front without water.
through a combination of lateral erosion by streams and Animals that are diurnal are usually awake and active
sheet flooding. during the day.
Option d is Incorrect. It refers to what is called a Barrier Statement 2 is correct. Playas are temporary lakes
Bar which is a type of a depositional landform. Barrier formed when water collects in intermontane basins
bars or beaches are exposed sandbars that may have in arid and semi-arid areas. The playas range in areal
formed during the period of high-water level of a storm extent from a few square metres to tens of square
or during the high-tide season. kilometres. These are called ‘khabari’ and ‘mamlaha’

Workbook 60
GEOGRAPHY

in Arabian deserts while they are known as ‘shatts’ pressure zones. These deserts are in interior drainage
in Sahara. Playa lakes may last for days, weeks or basins far from oceans and have a wide range of annual
even longer before they are completely dried up by temperatures. The Sonoran Desert of southwestern
evaporation. Evaporation of playa lake water results in the North America is a typical midlatitude desert.
formation of encrustation of alkaline materials (sodium Statement 2 is correct. Rain shadow deserts are formed
bicarbonate or sodium carbonate), salt deposits such as because tall mountain ranges prevent moisture-rich
borax. Such salt-covered playa beds are called salinas. clouds from reaching areas on the lee, or protected
Statement 3 is correct. The major hot deserts of the world side, of the range. As air rises over the mountain, water
are located on the western coasts of continents between is precipitated and the air loses its moisture content. A
latitudes 15° and 30°N. and S. They include the biggest desert is formed in the leeside “shadow” of the range. The
Sahara Desert, Great Australian Desert, Arabian Desert, Turpan Depression is the rain shadow desert of the Tian
Iranian Desert, Thar Desert, Kalahari and Namib Deserts. Shan of China.
On the western coasts, the presence of cold currents gives Statement 3 is correct. Coastal deserts generally are
rise to mists and fogs by chilling the on-coming air. This found on the western edges of continents near the Tropics
air is later warmed by contact with the hot land, and little of Cancer and Capricorn. They are affected by cold ocean
rain falls. These hot deserts lie along the Horse Latitudes currents that parallel the coast. Because local wind
or the Sub-Tropical High-Pressure Belts where the air is systems dominate the trade winds, these deserts are less
descending, a condition least favorable for precipitation stable than other deserts. A coastal desert, the Atacama
of any kind to take place. of South America, is the Earth’s driest desert. They cool
178. Solution: (c) the air and make it harder for the air to hold moisture.
Most moisture falls as rain before it reaches the land, eg
Exp) Option c is correct the Namib Desert in Africa.
Rain shadow deserts are created when mountain ranges
The trade winds in two belts on the equatorial sides of the
lie parallel to moist, coastal areas. Prevailing winds
Horse Latitudes heat up as they move toward the Equator.
moving inland cool as air is forced to rise over the
These dry winds dissipate cloud cover, allowing more
mountains. Carried moisture falls on slopes facing the
sunlight to heat the land. Most of the major deserts of the
winds. When the winds move over the crest and down
the far side, they are very dry. Descending air also makes world lie in areas crossed by the trade winds. The world’s
it hard for additional clouds and precipitation to form. largest desert, the Sahara of North Africa.
Without another source of moisture, rain shadow deserts 180. Solution: (b)
are formed on the far side of these mountain ranges.
Exp) Option b is correct
The Mojave Desert is a rain shadow desert. The Mojave
Desert lies in the rain shadow of the Sierra Nevada In hot and dry deserts, also known as arid deserts, the
Mountains. The hot, moist air from the Pacific Ocean temperatures are warm and dry year-round. Some famous
goes up the Sierra Nevadas and is turned back by the arid deserts include the Sahara Desert.
cold air in the mountains. Although some of the rain Cold deserts are still dry but have extremely low
goes over the mountains, most of it is evaporated by the temperatures in comparison to the other types of deserts.
hot air of the desert before it can reach the ground. The The Antarctic is an example of a cold desert.
Mojave Desert is considered a dry desert because of the Statement 1 is incorrect. Cold deserts are also found in
rain shadow effect. mid latitudes. Gobi Desert is a mid-latitude desert. The
Death Valley, a desert in the U.S. states of California mid-latitude desert is an excessively dry region with an
and Nevada, is so hot and dry because it is in the rain arid climate located between 30 degrees and 60 degrees
shadow of the Sierra Nevada Mountain range. The north and south of the equator. Cold deserts are also
Tibetan Plateau, a rain shadow in Tibet, China, and India found in Tropical area. For example, Ladakh region has
has the enormous Himalaya Mountain range to thank for a cold desert in India. The Tibetan Plateau, a rain shadow
its dry climate. in Tibet, China, and India has the Himalayan mountain
Sahara Desert and Thar desert does not lie in the rain range to thank for its dry climate.
shadow area. Statement 2 is correct. Sand dunes are not prominent
Thar is a desert area because it receives scanty rainfall features in cold deserts, but snow dunes occur commonly
from the South West monsoon winds in India. in areas where precipitation is locally more abundant.
In the western margin of the Sahara desert the cold
181. Solution: (a)
Canary Current reduces air temperatures, thereby
reducing convectional rainfall. Exp) Option a is correct
Pair 1 is correct. Ventifacts or dreikanter: These are
179. Solution: (d)
pebbles faceted by sand-blasting and thoroughly
Exp) Option d is correct polished by wind abrasion. Rock fragments are moved
Statement 1 is correct. Mid-latitude deserts occur between by wind and smoothed on the windward side and another
30° and 50° N. and S., poleward of the subtropical high- facet is developed if wind direction changes. Such rocks

61 Workbook
GEOGRAPHY

have characteristic flat facets with sharp edges. Amongst Statement 2 is correct: Caverns are interconnected
the ventifacts, those with three wind-faceted surfaces are subterranean cavities in bedrock formed by the corrosion
called dreikanter form the desert pavement. action of underground water on limestone.
Pair 2 is correct. Yardangs are steep-sided deeply They are found near Dehradun in Uttarakhand and in
undercut over hanging rock ridges separated from one Almora in Kumaon Himalayas.
another by long grooves or corridors or passageways cut Statement 3 is incorrect: Stalactites form as calcium
in desert floors of relatively softer rocks. It’s size ranges carbonate drips from the ceiling of a cave, forming
from a metre to kilometre They are usually formed in the beautiful icicle-like formations. The word stalactite forms
areas which are characterized by directional steady winds. from the ceiling. Stalagmites form a calcium carbonate
They are formed where alternate bands of hard and soft drips from the ceiling to the floor of a cave and then
rocks are vertical or inclined to the horizontal plane. grow upwards. The g in stalagmite means it forms on the
The intervening beds of softer rocks are abraded and ground.
eroded materials are blown away by deflation process. The
184. Solution: (d)
most characteristic feature of yardangs is their parallelism.
They are formed by the action of wind, usually parallel to Exp) Option d is correct.
the wind direction. Option a is incorrect. The equatorial regions are generally
Pair 3 is incorrect. Bajada is the gently sloping sparsely populated. In the forests most primitive people
depositional plain between pediments and playa. Bajadas live as hunters and collectors and the more advanced one’s
are common in arid climates (and not in polar regions). practise shifting cultivation. Food is so abundant in such
Bajada is formed due to coalescence of several alluvial a habitat that many people worry very little about the life
of the next day.
fans. Thus, bajada is a wholly depositional feature. The
slope in its upper part ranges between 8º and 100 but it Option b is incorrect. Nomadic hunters and food
becomes 10 to zero in its lowermost portion touching the gatherers are the desert inhabitants. Of the primitive
playa. tribes, the Bushmen and the Bindibu are the best known.
Both the tribes are nomadic hunters and food gatherers,
growing no crops and domesticating no animals.
1.10. Limestone Option c is incorrect. The natural conditions suit animal
182. Solution: (b) farming in temperate grasslands. With the development
of refrigerated ships in the late nineteenth century, the
Exp) Option b is correct. temperate grasslands became major pastoral regions,
Statement 1 is correct – The north west region of exporting large quantities of beef, mutton, wool. hides.
Yugoslavia is one of the prominent stretches of Option d is correct. Karst regions are often barren and at
limestone. The karst landform derives its name from the best carry a thin layer of soil. The porosity of the rocks and
Karst district of Yugoslavia. the absence of surface drainage make vegetative growth
Statement 2 is correct –There is a general absence of difficult, so that limestone can usually support only poor
surface drainage as most of the surface water has gone grass and short turf; some sheep grazing is possible.
underground. Streams here only flow over limestone for Limestone vegetation in tropical regions, however, is
a short distance and then, disappears underground. For luxuriant because of the heavy rainfall all the year round.
Settlements are scattered and the population is often
the greater part of their course, they cut their way along
sparse. The only mineral of importance is lead which
the joints and fissures of the rock wearing out a system of
occurs in veins in association with limestone.
underground channels. The surface valleys are therefore
dry.
Statement 3 is incorrect – In its pure state, limestone
1.11. Lakes, Coastal landforms, Islands and
is made up of calcite or calcium carbonate, but where
Miscellaneous
magnesium is also present it is termed dolomite. Chalk is 185. Solution: (a)
a very pure form of limestone, white, and rather soft. Exp) Option a is correct.
183. Solution: (b) Option A is correct. A fjord is a deep, narrow and
elongated sea or lake drain, with steep land on three sides.
Exp) Option b is correct
This shallow threshold in the mouth of the fjord, is the
Statement 1 is correct: Sinkholes are well defined reason that fjords often are quieter than the open sea.
depressions in the karst landscape. Sinkholes are pits Thus, fjords are often natural harbors. The longest fjord in
in the ground that form in areas where water gathers the world is Scoresby Sund in Greenland (350 km).
without external drainage. Sinkholes mainly occur as Option B is Incorrect. A stalagmite is a type of rock
water drains below ground. It can dissolve subterranean formation that rises from the floor of a cave due to the
caverns, particularly in areas where the bedrock is made accumulation of material deposited on the floor from
of water-soluble evaporate rocks such as salt or gypsum or ceiling drippings. Stalactites hang from the ceiling of a
of carbonate rocks such as limestone or dolomite. cave while stalagmites grow from the cave floor.

Workbook 62
GEOGRAPHY

Option C is Incorrect. A Bergschrund is a deep crack at


the head of the glacier.
Option D is Incorrect. Drumlins are glacial landforms
whichare smooth oval shaped ridge-like features
composed mainly of glacial till with some masses of
gravel and sand. The drumlins form due to dumping of
rock debris beneath heavily loaded ice through fissures in
the glacier.

186. Solution: (b)


Exp) Option b is correct.
187. Solution: (a)
Statement 1 is incorrect. Ria Coasts are drowned river
Exp) Option a is correct.
valleys. Ria coast is a coastal inlet formed by the partial
Statement 1 is correct. Corals thrive in tropical waters
submergence of an unglaciated river valley that remains between 30°N and 30°S latitudes. The temperature
open to the sea & is a coastline having several parallel of water is around 20°C where diurnal and annual
rias separated by prominent ridges, extending a distance temperature ranges are very narrow. However temperate
and extra tropical varieties of corals are also found.
inland.
Statement 2 is incorrect. Coral requires fairly good
amount of sunlight to survive. The ideal depths for coral
growth are 45 m to 55 m below sea surface, where there is
abundant sunlight available.
Statement 3 is incorrect. Coral reefs require clean
sediment free water as both fresh water and highly saline
water are harmful. Sediments from river water can clog
their mouth and may become fatal.
188. Solution: (d)
Exp) Option d is correct.
Ocean currents have a number of direct and indirect
influences on human activities -
• West coasts of the continents in tropical and
subtropical latitudes are bordered by cool waters.
Cold ocean currents have a direct effect on desert
formation in west coast regions of the tropical and
Statement 2 is correct. Fjord coasts are drowned glaciated sub-tropical continents.
valleys, which were created by glaciers that moved very • Warm ocean currents bring rain to coastal areas and
slowly over time, and greatly altered the landscape when even interiors. Example: The North Atlantic Drift
they moved through an area carving deep valley. Fjord keeps the coasts of the North Sea warm which is
unusual for such high latitudes.
coast is formed when a glacier retreats, after carving its
• Warm currents flow parallel to the east coasts of the
typical U-shaped valley with the sea filling the resulting continents in tropical and subtropical latitudes. This
valley floor. results in warm and rainy climates. These areas lie in
the western margins of the subtropical anti-cyclones.
• They are responsible for moderate temperatures at
coasts. Ex: Canary cold current brings cooling effect
to Spain, Portugal etc.
• The mixing of warm and cold currents helps to
Statement 3 is correct. Dalmatian coasts are a longitudinal replenish the oxygen and favour the growth of
coast where the mountains run parallel to the coast, planktons, the primary food for fish population.
where the submergence of the coastline produces long,
189. Solution: (b)
narrow inlets with a chain of islands parallel to the coast.
Exp) Option b is correct.
Here the elongated islands are the crests of former
Pair 1 is correct. Canary Current is a cold current and
ranges & the narrow inlets were the former longitudinal a part of a clockwise-setting ocean-current system in the
valleys. North Atlantic Ocean.

63 Workbook
GEOGRAPHY

Pair 2 is correct. Agulhas current is a warm current of Statement 2 is incorrect. During neap tides, the bulge of
southern hemisphere. It flows south along the east coast the ocean caused by the sun partially cancels out the bulge
of Africa in Indian Ocean. of the ocean caused by the moon. This produces moderate
Pair 3 is Incorrect.Irminger current is a cold current and a tides meaning that high tides are a little lower and low
part of northern Atlantic Ocean (not southern). tides are a little higher than average.
Statement 3 is correct. In diurnal tide there is only one
190. Solution: (a) high tide and one low tide during each day. The successive
Exp) Option a is correct. high and low tides are approximately of the same height.
Statement 1 is correct. The Southern Ocean is made up
of the portions of the world ocean south of the Pacific, 194. Solution: (d)
Atlantic, and Indian oceans and their tributary seas Exp) Option d is correct.
surrounding Antarctica below 60° S. It is unbroken by any Statement a is correct. Tidal currents occur in
other continental landmass. Southern Ocean’s narrowest conjunction with the rise and fall of the tide. The vertical
constriction is the Drake Passage, 600 miles (about 1,000 motion of the tides near the shore causes the water to
km) wide, between South America (not South Africa) move horizontally, creating currents.
and the tip of the Antarctic Peninsula.
Statement b is correct. Tidal currents are the only type
Statement 2 is correct. Large-scale upwelling is found of current affected by the interactions of the Earth, sun,
in the Southern Ocean. Strong westerly (eastward) winds and moon. Tidal currents, just like tides, are affected by
blow around Antarctica, driving a significant flow of water the different phases of the moon.
northwards (Ekman transport). This is actually a type of
coastal upwelling. Since there are no continents in a band Statement c is correct. Tidal currents are the tides which
of open latitudes between South America and the tip of are channeled between islands or into bays and estuaries.
the Antarctic Peninsula, some of this water is drawn up Statement d is incorrect. When the moon is at full or
from great depths. new phases, tidal current velocities are strong and are
Statement 3 is incorrect. Increased solar ultraviolet called “spring currents.” When the moon is at first or
radiation resulting from the Antarctic ozone hole has third quarter phases, tidal current velocities are weak
reduced marine primary productivity (phytoplankton) and are called “neap currents.”
by as much as 15% and has started damaging the DNA 195. Solution: (c)
of some fish.
Exp) Option c is correct
191. Solution: (d) Statement 1 is correct. Suspended particulate matters
Exp) Option d is correct. such as clay, silt, phytoplankton, etc make the water
Pair 1 is incorrect. Seuz canal connects Port Said on the turbid which consequently limits the light penetration
Mediterranean Sea with the Gulf of Suez, an arm of the and the photosynthetic activity in water thus causing
Red Sea. decrease in productivity of aquatic ecosystem.
Pair 2 is incorrect. Bosphorus strait connects Black Sea Statement 2 is Incorrect. Aphotic zone also known as the
to Sea of Marmara. profundal zone has low light penetration and is poor for
Pair 3 is incorrect. Bab-el-Mandeb connects red sea to photosynthesis, thus has low productivity.
Gulf of Aden and Arabian sea. Statement 3 is correct. The amount of dissolved oxygen
Pair 4 is correct. Strait of Hormuz connects the Persian in water is also influenced by temperature. Oxygen is
Gulf to the Arabian Sea, part of the northern Indian less soluble in warm water. Warm water also enhances
Ocean. the decomposition activity. Therefore, increasing
the temperature of water body increase the rate at
192. Solution: (b) which oxygen is depleted from water. Hence, as the
Exp) Option b is correct. temperature of the water increases, dissolved oxygen
levels decrease.
NASA says the minerals and rock deposits at lake Salda
(Turkey) are the nearest match on earth to those around 196. Solution: (c)
the Jezero Crater of Mars. According to NASA, the Mars
Exp) Option c is correct
Perseverance mission will search for signs of ancient life
on the Martian surface similar to rocks found along the Statement 1 is correct.Fringing reefs grow near the
shoreline of Lake Salda in Turkey. coastline are separated from the shore by narrow-shallow
lagoons.Fringing reefs are the most common type of reef
193. Solution: (d) that we see.
Exp) Option d is correct. Statement 2 is correct.Atolls are rings of coral with
Statement 1 is correct. During perihelion that means shallow lagoon in middle.Atolls usually form when islands
when earth is closest to sun, tidal ranges are much surrounded by fringing reefs sink into the sea or the sea
greater than average. level rises around them (these islands are often the tops

Workbook 64
GEOGRAPHY

of underwater volcanoes). The fringing reefs continue to in Istanbul, Turkey. Recently, Turkey allowed for the
grow and eventually form circles with lagoons inside. conversion of the nearly 1,500 year-old Hagia Sophia
from a museum into a mosque.
197. Solution: (c)
Exp) Option c) is correct. 200. Solution: (d)
Statement 1 is Incorrect: When earth orbits the sun, it Exp) Option d is correct.
moves between the sun and the moon. If this happens, Erosion is the wearing away of the landscape by different
earth blocks the sunlight that normally is reflected by the agents like water, wind and ice. Different landforms
moon and Earth’s shadow falls on it. created on the surface of the earth because of erosion are
called erosional landforms.
When the moon orbits Earth, it moves between the sun
and Earth. When this happens, the moon blocks the light Option a is incorrect. Capes and Bays are erosional
of the sun from reaching Earth. This causes an eclipse of features of coastlands. On exposed coasts, the continual
the sun. action of waves on the rocks of varying resistance causes
the coastline to be eroded irregularly. This is particularly
Statement 2 is Incorrect: A Lunar eclipse can occur only pronounced where hard rocks occur in alternative band
when the moon is full, while a solar eclipse can only with softer rocks. The softer rocks are worn back into
happen at a new moon inlets or bays & the harder ones persist as headlands or
198. Solution: (a) capes. Even where the coastline consists of one rock type,
irregularities will be caused by variation within the rock.
Exp) Option a is correct
Option b is incorrect. Stack & Stump are erosional
Cumulus clouds look like cotton wool and are generally features of coastlands. Prolonged waves attack on
formed at a height of 4,000 - 7,000 m. They exist in patches the base of the cliff & excavate holes in regions of local
and can be seen scattered here and there. They also have weakness called cave. When 2 caves approach each other
a flat base from either side of headland & unite, they form an arch.
Option b is Incorrect – Cirrus clouds are formed at high Further erosion by waves will lead to total collapse of the
altitudes(8,000 - 12,000m). They are thin and detached arch. The seaward portion of the headland will remain as
clouds having a feathery appearance. They arealways a pillar of rock known as stack. With the course of time,
white in colour. these stubborn stacks will gradually be eroded, leaving
Option c is Incorrect – Stratus clouds are layered clouds behind the stumps, which are only just visible above the
covering large portions of the sky. These clouds are sea level.
generally formed either due to loss of heator the mixing of Option c is incorrect. Geos & Gloups are erosional
air masses with different temperatures features of coastlands. The occasional splashing of the
Option d is Incorrect – Nimbus clouds are black or dark waves against the roof of a cave may enlarge the joints
grey. They form at middle levels or very near to the surface when air is compressed & released repeatedly inside them.
of the earth. These are extremely dense and opaque to the A natural shaft is thus formed which may eventually pierce
rays of the sun. Sometimes, the clouds are so low that they through the surface. Waves breaking into the cave may
seem to touch theground. Nimbus clouds are shapeless force water or air out of this hole. Such a shaft is termed
masses of thick vapour as Gloup or blow hole. The enlargement of blow-holes &
continual action of waves weakens the cave roof. When
199. Solution: (c) the cave roof collapses, a long, narrow creek may develop
Exp) Option c is correct. known as Geos.
Pair 1 is correct. Bhasan Char is a specifically developed Option d is correct. Spits and Bars are the depositional
island in Bangladesh to accommodate 1,00,000 of the features of wave action.
1 million Rohingya who have fled from neighbouring A spit is an extended stretch of sand or shingle jutting
Myanmar. out into the sea from the land. Spits occur when there is
Pair 2 is correct. Tigray Region is a northern region a change in the shape of the landscape or there is a river
of Ethiopia in news due to ongoing conflict between mouth.
Ethipoian Military and Rebels. Sometimes a spit can grow across a bay, and joins two
Pair 3 is correct. Sana is the largest city in Yemen. headlands together. This landform is known as a bar. They
Under the Yemeni constitution, Sanaa is the capital of can trap shallow lakes behind the bar, these are known as
the country, although the seat of the Yemeni government lagoons. Lagoons do not last forever and may be filled up
moved to Aden, the former capital of South Yemen in the with sediment.
aftermath of the Houthi occupation.
A Saudi-led military coalition mounted air strikes 201. Solution: (b)
on Yemen’s Houthi-controlled capital Sana’a after it Exp) Option b is correct.
intercepted 10 drones launched by the Iran-backed rebels. Continental shelf is a seaward extension of the continent
Pair 4 is incorrect. Hagia Sophia is an important mosque from the shoreline to the continental edge. These extended

65 Workbook
GEOGRAPHY

margins of each continent are occupied by relatively subduction between a plate carrying continental crust
shallow seas and gulfs. Gradient of continental is of 1° or and a plate carrying oceanic crust. Continental crust is
even less. always much more buoyant than oceanic crust, and
Statement 1 is incorrect. Continental shelves can be oceanic crust will always subduct.
formed either due to emergence of sea or submergence
of continent. Part of a continent may drop in elevation,
becoming a portion of continental shelf that borders the
dry mainland. At the same time, sea level can also rise,
submerging a portion of the continent.
Statement 2 is correct. The width of the Continental
Shelves varies greatly. In some places where the coasts
are extremely mountainous the continental shelf may
be entirely absent. The Continental shelves are almost
absent or very narrow along some of the margins like
the coasts of Chile, the west coast of Sumatra, etc. On
the contrary, the Siberian shelf in the Arctic Ocean, the
largest in the world, stretches to 1,500 km in width.
Statement 3 is correct. Continental shelves are major
source of oil deposits around the world. More than a 203. Solution: (b)
third of the oil and gas extracted worldwide comes from Exp) Option b is correct
offshore sources. Petroleum is not distributed evenly
Statement a is correct: The current systems of the Indian
around the world. Slightly less than half of the world’s
ocean are largely controlled and modified by landmasses
proven reserves are located in the Middle East.
and monsoon winds. Here, Landmass includes African,
Statement 4 is correct. Continental shelves are of great Australian and Indian Subcontinent. The most striking
geographical significance. Their shallowness enables
feature of the Ocean current system in Indian ocean is the
sunlight to penetrate through the water, which encourages
the growth of minute plants and other microscopic seasonal reversal of the ocean currents in the northern
organisms. The continental shelves are therefore the hemisphere. The absence of a temperate and polar region
richest fishing grounds in the world, e.g., the Grand north of the equator due to presence of landmass is
Banks off Newfoundland, the North Sea and the Sunda another peculiarity in Indian ocean.
Shelf. Their limited depth and gentle slope keep out cold Statement b is incorrect: Indian ocean being surrounded
under-currents and increase the height of tides. Most of by the Indian Subcontinent, Africa and Australia does not
the world’s greatest seaports including Southampton, present most favorable conditions for the development
London, Hamburg, Rotterdam, Hong Kong and Singapore
of permanent and consistent system of ocean currents.
are located on continental shelves.
It is because of presence of monsoon wind system which
202. Solution: (c) results in seasonal reversal of ocean current twice a year.
Exp) Option c is correct In summer, South-West monsoon current is found and in
winter, N-E monsoon current is found.
Ocean trenches are long, narrow depressions on the
seafloor. These are the deepest parts of the ocean and
some of the deepest natural spots on Earth.
Statement 1 is correct: Ocean deeps which are generally
called as Submarine trenches are the deepest part of the
ocean. They are long, narrow, steep sided and flat-floored
depressions on the ocean floor. They are some 3-5 km
deeper than the surrounding ocean floor. They occur at
the bases of continental slopes and along island arcs
and are associated with active volcanoes and strong
earthquakes.
The Pacific Ocean has the largest number of trenches for
example, The Mariana Trench in ‘the Pacific Ocean is the
deepest known part of the oceans.
Statement 2 is correct: They are typically the feature of
convergent plate boundaries, where two or more tectonic
plates meet. It may be oceanic-oceanic convergence and
oceanic-continental convergence. In case of oceanic-
continental convergence, trenches are formed by

Workbook 66
GEOGRAPHY

In the Pacific Ocean, near Hokkaido, Japan the colder


Oyashio Current converges from the north with the
warmer Kuroshio Current, which flows from the south.
The Kuroshio and Oyashio Currents are the western
boundary currents in the wind-driven, subtropical and
subarctic circulations of the North Pacific Ocean.
The subarctic North Pacific Ocean, on the other hand,
is dominated by upwelling. The upwelled, nutrient-rich
water feeds the Oyashio from the north.
The confluence of these currents is marked by fogbanks
and have great influence on fisheries and climate in the
western North Pacific.

Statement c is correct: Unlike the Atlantic and Pacific,


both of which have strong currents circulating clockwise
north of the Equator, Northern Indian Ocean has surface
currents which changes their directions with the seasonal
monsoon. The currents in northern Indian ocean change
their flow of directions twice a year due to northeast and
southwest monsoon winds.
Statement d is correct: The current of the south Indian
ocean is least affected by seasonal change in the
direction of the monsoon wind. It includes The Indian 206. Solution: (b)
equatorial current that flows from east to west between
10°Sand 15°S latitudes from Australian coast towards Exp) Option b is correct.
African coast. It is divided into many branches after Pair 1 is incorrect. Wular Lake from Jammu & Kashmir
being obstructed by Madagascar, such as Agulhas current is the largest Freshwater lake in India. In fact, it is the
(warm) while the other branch is directed towards the largest Freshwater Lake in Asia.
north. It is because role of landmass and monsoon system • Formed by tectonic activity (during Pleistocene).
is more pronounced in Northern Indian Ocean, than
Southern Indian Ocean. • Jhelum River feeds the lake.
• Tulbul project is a navigation lock cum-control
204. Solution: (b) structure at the mouth of Wular Lake.
Exp) Option b is correct On the other hand, Vembanad is a brackish lake and is
The ocean water is heated by three processes. First includes the longest lake in India. It is also the largest lake in Kerala.
Absorption of radiation from the sun. It is maximum over Pair 2 is correct. Lake Baikal located in Siberia, Russia is
low latitude regions due to vertical insolation and longer the deepest lake in the world. It is also the world’s largest
duration of daylight. However, it reduces steadily towards
freshwater lake by volume.
poles. Secondly, Kinetic energy is a form of energy which
is produced due to friction caused by the surface wind and Other Important Lakes in the World
the tidal currents. It increases stress on the water body. Lake Tanganyika – longest Lake in the world. It is also the
Thus, the ocean water is heated. second largest by volume. It is the second deepest lake in
Statement 1 is incorrect: Oceanic water is heated by the world, after Lake Baikal.
convectional currents in the water body. So, a convectional The highest large lake in the world is the Pumoyong Tso
oceanic circulation at the bottom layers of ocean water (Pumuoyong Tso), in the Tibet.
takes place causing circulation of heat in water.
The world’s highest commercially navigable lake is Lake
Statement 2 is correct: Ocean thermal energy Titicaca in Peru and Bolivia border at 3,812 m. It is also
conversion (OTEC) is a process or technology which the largest lake in South America.
harnesses the temperature differences (thermal gradients)
between ocean surface waters and deep ocean waters for The world’s lowest lake is the Dead Sea, bordering Israel
producing energy. and Jordan at 418 metres below sea level. It is also one of
the lakes with highest salt concentration.
205. Solution: (a) Pair 3 is correct. Lake Superior is the largest of the five
Exp) Option a is correct Great Lakes of North America.

67 Workbook
GEOGRAPHY

It is shared by Ontario, Canada, to the north, and states the Bay of Bengal, compared to the western coast, which
in the United States in the south. remained largely stable. The eastern coast also sees a lot
Great Lakes of North America are a series of interconnected of rain which keeps the seas rough throughout most
freshwater lakes which connect to the Atlantic Ocean of the year. Besides the Southwest Monsoon (June to
through the Saint Lawrence Seaway. Consisting of Lakes September), the eastern coast also witnesses the Northeast
Superior, Michigan, Huron, Erie, and Ontario. Monsoon from October to December and brings rain to
coastal Andhra Pradesh and Tamil Nadu.
207. Solution: (d) Statement 2 is correct. Coastal changes can take hundreds
Exp) Option d is correct. of years. The way coasts are formed depends a lot on what
A lake is a body of water that is surrounded by land. kind of material is in the land and water. The harder the
There are millions of lakes in the world. They are found material in the land, the harder it is to erode. Coastlines of
on every continent and in every kind of environment—in granite, a hard rock, stay stable for centuries. Sugarloaf
mountains and deserts, on plains, and near seashores. Mountain, on the coast of Rio de Janeiro, Brazil, is made
mostly of granite and quartz. It has been a landmark for
Statement 1 is correct. The most important chemicals in a centuries.
lake are nitrogen and phosphorus. These chemicals allow
nutrient-rich plants and algae to grow. Other organisms 210. Solution: (a)
feed off these plants and algae, creating a complex, healthy Exp) Option a is correct
ecosystem.
Pair 1 is correct. Benguela Current flows northward
Statement 2 is correct. All lakes are either open or closed.
in the South Atlantic Ocean along the west coast of
If water leaves a lake by a river or other outlet, it is said
southern Africa nearly to the Equator before merging with
to be open. All freshwater lakes are open. If water only
the westward-flowing Atlantic South Equatorial Current.
leaves a lake by evaporation, the lake is closed. Closed
lakes usually become saline, or salty. This is because as the Pair 2 is incorrect. Agulhas Current, warm oceanic
water evaporates, it leaves behind solids—mostly salts. current that forms the western boundary current of the
Statement 3 is correct. A reservoir is a manmade lake southern Indian Ocean. It flows southward along the
that is created when a dam is built on a river. River water southeast coast of Mozambique and the coast of South
backs up behind the dam creating a reservoir. Africa before turning eastward to join the flow from
Africa to Australia.
Statement 4 is correct. Generally, the clearer the water
in the lake, the fewer nutrients it has. Lakes that are very Pair 3 is incorrect. The Gulf Stream is an ocean current
nutrient-rich are cloudy and hard to see through; this that brings warm water from the Gulf of Mexico into the
includes lakes that are unhealthy because they have too Atlantic Ocean.
many nutrients. Lakes need to have a balance of nutrients. Pair 4 is incorrect. The Labrador Current is a cold
oceanic current in the North Atlantic Ocean that
208. Solution: (b) originates at the Davis Strait.
Exp) Option b is correct.
Statement 1 is incorrect. Cirque lakes or tarns are formed
by glaciation (not of volcanic activity). A glacier on its
way down the valley leaves behind circular hollows in the
heads of the valleys up in the mountains. Such hollows are
cirques or corries. Their over deepened floor may be filled
with water to form cirque lake or tarns.
Statement 2 is correct. Lakes formed due to landslides
screes & avalanches are known as barrier lakes. Such
Lakes formed by landslides are short lived because of
loose fragments that pile up across the valleys will soon
rupture under pressure & will give way to water. When
they suddenly give way, the dammed water rushes down
causing floods.
Statement 3 is correct. Lakes are only temporary
features of earth crust; they will eventually be eliminated
by the process of draining and slitting up. Example in hot 211. Solution: (b)
deserts lakes disappear altogether by combined process of Exp) Option b is correct.
evaporation, percolation and outflow.
A canyon is a deep, narrow valley with steep sides. The
209. Solution: (b) movement of rivers, the processes of weathering and
erosion, and tectonic activity create canyons.
Exp) Option b is correct.
Option a is incorrect. Royal Gorge is one of the deepest
Statement 1 is incorrect. The eastern coast underwent
and narrowest canyons in Colorado. This regal ravine is
more erosion due to frequent Cyclonic Activities from
divided by the Arkansas River and is a hotspot for rock

Workbook 68
GEOGRAPHY

climbing, base-jumping, and bungee-jumping. The Royal Option c is incorrect. Antelope Canyon is a slot canyon
Gorge Bridge is actually the highest official jump platform in the America’s Arizona. It is divided into two main
in the world. upper and lower photogenic sections: The Crack and The
Option b is correct. The Grand Canyon is located in the Corkscrew. This stunning rock formation was formed by
U.S. state of Arizona. It is a product of tectonic uplift. The erosion of Navajo Sandstone due to flash flooding and
Grand Canyon is the largest canyon in the United States. erosion caused by airborne sand. The canyon is on Navajo
It has been carved as the Colorado River cuts through the Tribe land and has only been accessible to the public since
Colorado Plateau. It lies in the south-western portion of
1997.
the Colorado Plateau.
The Plateau’s arid climate produced many striking Option d is incorrect. Situated in Kings Canyon National
erosional forms, culminating in the Grand Canyon. The Park, this canyon actually features the deepest gorge in
Canyon’s mile-high walls display a largely undisturbed the country. The park is rich in unspoiled backcountry
cross section of the Earth’s crust extending back some two hiking, climbing, and scenic lookout points. Spring is
billion years. one of the most beautiful times to visit this stunning area,
The Pueblo people considered the Grand Canyon a holy as the warmer weather encourages the mountainside to
site and made pilgrimages to it. sprout blankets of wildflowers and tender green grass.

69 Workbook
GEOGRAPHY

GEOGRAPHY
OCEANOGRAPHY
Which of the above factors influence the ocean
2.1. Previous Years Questions currents?
1. Consider the following statements: (a) 1 and 2 only
[UPSC CSE Pre. 2007] (b) 1, 2 and 3 only
1. The annual range of temperature is greater in the (c) 1 and 4 only
Pacific Ocean than that in the Atlantic Ocean (d) 2, 3 and 4 only
2. The annual range of temperature is greater in the 5. The most important fishing grounds of the world
Northern Hemisphere than that in the Southern are found in the regions where
Hemisphere. [UPSC CSE Pre. 2013]
Which of the statements given above is/are correct? (a) warm and cold atmospheric currents meet
(a) 1 only (b) rivers drain out large amounts of fresh water into
(b) 2 only the sea
(c) Both 1 and 2 (c) warm and cold oceanic currents meet
(d) Neither 1 nor 2 (d) continental shelf is undulating
2. Between India and East Asia, the navigation-time 6. The annual range of temperature in the interior
and distance can be greatly reduced by which of of the continents is high as compared to coastal
the following? [UPSC CSE Pre. 2011] areas. What is / are the reason / reasons?
1. Deepening the Malacca straits between Malaysia [UPSC CSE Pre. 2013]
and Indonesia. 1. Thermal difference between land and water
2. Opening a new canal across the Kra isthmus 2. Variation in altitude between continents and
between the Gulf of Siam and Andaman Sea. oceans
Which of the statements given above is/are correct? 3. Presence of strong winds in the interior
(a) 1 only 4. Heavy rains in the interior as compared to coasts
(b) 2 only Select the correct answer using the codes given
(c) Both 1 and 2 below.
(d) Neither 1 nor 2 (a) 1 only
(b) 1 and 2 only
3. La Nina is suspected to have caused recent floods
(c) 2 and 3 only
in Australia. How is La Nina different from El (d) 1, 2, 3 and 4
Nino? [UPSC CSE Pre. 2011]
1. La Nina is characterised by unusually cold ocean 7. Which of the following have coral reefs?
temperature in equatorial Indian Ocean whereas [UPSC CSE Pre. 2014]
El Nino is characterised by unusually warm 1. Andaman and Nicobar Islands
ocean temperature in the equatorial Pacific 2. Gulf of Kachchh
Ocean. 3. Gulf of Mannar
2. El Nino has adverse effect on south-west 4. Sunderbans
monsoon of India, but La Nina has no effect on
Select the correct answer using the code given below.
monsoon climate.
(a) 1, 2 and 3 only
Which of the statements given above is/are correct? (b) 2 and 4 only
(a) 1 only (c) 1 and 3 only
(b) 2 only (d) 1, 2, 3 and 4
(c) Both 1 and 2
(d) Neither 1 nor 2 8. Turkey is located between [UPSC CSE Pre. 2014]
(a) Black Sea and Caspian Sea
4. Consider the following factors: (b) Black Sea and Mediterranean Sea
[UPSC CSE Pre. 2012] (c) Gulf of Suez and Mediterranean Sea
1. Rotation of the Earth (d) Gulf of Aqaba and Dead Sea
2. Air pressure and wind 9. What is the correct sequence of occurrence of the
3. Density of ocean water following cities in South-East Asia as one proceeds
4. Revolution of the Earth from south to north? [UPSC CSE Pre. 2014]

Workbook 70
GEOGRAPHY

1. Bangkok Which of the pairs given above are correctly


2. Hanoi matched?
3. Jakarta (a) 1 and 2
4. Singapore (b) 1 and 4
(c) 2 and 3
Select the correct answer using the code given below:
(d) 3 and 4
(a) 4-2-1-3
(b) 3-2-4-1 15. Consider the following pairs:
(c) 3-4-1-2 [UPSC CSE Pre. 2018]
(d) 4-3-2-1
Regions sometimes Country
10. Tides occur in the oceans and seas due to which mentioned in news
among the following? [UPSC CSE Pre. 2015]
1. Gravitational force of the Sun 1. Catalonia Spain
2. Gravitational force of the Moon 2. Crimea Hungary
3. Centrifugal force of the Earth 3. Mindanao Philippines
Select the correct answer using the code given below.
4. Oromia Nigeria
(a) 1 only
(b) 2 and 3 only Which of the pair given above are correctly matched?
(c) 1 and 3 only (a) 1, 2 and 3
(d) 1, 2 and 3 (b) 3 and 4 only
(c) 1 and 3 only
11. Which one of the following countries of South- (d) 2 and 4 only
West Asia does not open out to the Mediterranean
Sea? [UPSC CSE Pre. 2015] 16. Consider the following statements:
(a) Syria [UPSC CSE Pre. 2018]
(b) Jordan 1. Most of the world’s coral reefs are in tropical
(c) Lebanon waters.
(d) Israel 2. More than one-third of the world’s coral reefs are
12. Mediterranean Sea is a border of which of the located in the territories of Australia, Indonesia
following countries? [UPSC CSE Pre. 2017] and Philippines.
1. Jordan 3. Coral reefs host far a greater number of animal
2. Iraq phyla than those hosted by tropical rainforests.
3. Lebanon Which of the statements given above is/are correct?
4. Syria (a) 1 and 2 only
(b) 3 only
Select the correct answer using the code given below:
(c) 1 and 3 only
(a) 1, 2 and 3 (d) 1, 2 and 3
(b) 2 and 3 only
(c) 3 and 4 only 17. Consider the following pairs:
(d) 1, 3 and 4 only [UPSC CSE Pre. 2019]
13. Which of the following is geographically closest to Sea Bordering country
Great Nicobar? [UPSC CSE Pre. 2017]
(a) Sumatra 1. Adriatic Sea Albania
(b) Borneo 2. Black Sea Croatia
(c) Java
3. Caspian Sea Kazakhstan
(d) Sri Lanka
4. Mediterranean Sea Morocco
14. Consider the following pairs:
[UPSC CSE Pre. 2018] 5. Red Sea Syria
Which of the pairs given above are correctly
Towns sometimes Country mentioned in matched?
news (a) 1, 2 and 4 only
1. Aleppo Arabia (b) 1, 3 and 4 only
(c) 2 and 5 only
2. Kirkuk Yemen
(d) 1, 2, 3, 4 and 5
3. Mosul Palestine
18. Consider the following pairs:
4. Mazar-i-sharif Afghanistan [UPSC CSE Pre. 2020]

71 Workbook
GEOGRAPHY

River Flows into (b) 1 and 3 only


(c) 1, 2 and 4 only
1. Mekong Andaman Sea (d) 1, 2, 3 and 4
2. Thames Irish Sea
22. Which of the following factor(s) influence the flow
3. Volga Caspian Sea of Ocean Currents?
4. Zambezi Indian Ocean 1. Planetary winds
Which of the pairs given above is/are correctly 2. Temperature difference between the ocean
matched? waters at equator and poles
(a) 1 and 2 only 3. Variation in salinity of ocean water from place
(b) 3 only to place
(c) 3 and 4 only 4. Rotation of Earth
(d) 1, 2 and 4 only 5. Presence of Landmass
19. With reference to Ocean Mean Temperature Select the correct answer using the code given below:
(OMT), which of the following statements is/are (a) 1, 2 and 4 only
correct [UPSC CSE Pre. 2020] (b) 2, 3 and 5 only
1. OMT is measured up to a depth of 26°C isotherm (c) 1, 3, 4 and 5 only
which is 129 meters in the south -western Indian (d) 1, 2, 3, 4 and 5
Ocean during January–March.
2. OMT collected during January –March can be 23. With reference to cold-water corals, consider the
used in assessing whether the amount of rainfall following statements:
in monsoon will be less or more than a certain 1. There are more cold-water coral reefs than
long -term mean. tropical reefs in the oceans.
Select the correct using the code given below: 2. Deep water corals requires zooxanthellae to
(a) 1 only survive.
(b) 2 only 3. Lophelia reefs are deep water corals which can
(c) Both 1 and 2 host species of fish and invertebrates.
(d) Neither 1 nor 2 Which of the statements given above is/are correct?
Practice Questions (a) 1 and 2 only
(b) 2 only
(c) 1 and 3 only
2.2. Movement, relief, temperature and (d) 2 and 3 only
salinity of Ocean Water
24. This ocean relief feature is a seaward extension of
20. It is a warm Ocean current, carrying tropical water
the continent from the shoreline to the continental
towards the pole. It flows in the southwest Indian
edge. Its width varies greatly and may be entirely
ocean along the east coast of Africa. The ocean
absent in some areas. They form the richest fishing
current described is
grounds in the world. The ocean relief feature
(a) Benguela current described in the above paragraph is?
(b) Falkland current (a) Continental shelf
(c) Canaries current (b) Continental slope
(d) Agulhas current (c) Continental rise
21. Consider the following pairs: (d) Abyssal plain
25. With reference to Ocean Floor Configuration,
Desert Bordering Ocean
which of the following statements is incorrect?
Current
(a) Abyssal plains are the largest habitat on Earth.
1. Kalahari Desert Canary current (b) The continental slope connects the continental
2. Sahara Desert Benguela current shelf and the ocean basins.
(c) Abyssal plains are more productive ecosystems
3. Atacama Desert Peru Current than those along the continental shelf.
4. Australian Desert West Australian (d) Mid-ocean ridge forms the longest mountain
Current range on Earth.
Which of the pairs given above is/are correctly 26. Which of the following are located in the Indian
matched? Ocean?
(a) 3 and 4 only 1. Socotra-Chagos Ridge

Workbook 72
GEOGRAPHY

2. Reunion Island Which of the statements given below is/are correct?


3. Diamantina Trench (a) 1 and 2 only
4. Mariana Trench (b) 2 only
Select the correct answer using the code given below: (c) 1 and 3 only
(d) 2 and 3 only
(a) 1 and 4 only
(b) 1, 2 and 3 only 31. Variation is salinity in various seas and oceans
(c) 2, 3 and 4 only is/are directly affected by which of the following
(d) 1, 2, 3 and 4 factors?
27. Consider the following statements about Sargasso 1. Amount of addition of fresh water.
Sea: 2. Rate of evaporation.
1. It has no land boundary and is defined by ocean 3. Rotation of earth.
currents. Select the correct answer using the code given below:
2. It is located in Atlantic Ocean. (a) 1 only
3. It is named after critically endangered tortoise (b) 1 and 2 only
species called Sargassum. (c) 2 and 3 only
Which of the statements given above is/are correct? (d) 1, 2 and 3
(a) 1 only 32. Consider the following statements regarding
(b) 1 and 2 only Ocean salinity.
(c) 2 and 3 only 1. Sea ice contains more salt than Sea water.
(d) 1, 2 and 3
2. Halocline is a distinct zone where salinity
28. Consider the following statements with regard to decreases sharply.
the Continental slope: 3. Salinity is lower than the average in equatorial
1. Its boundary indicates the end of the continents. waters.
2. Canyons and trenches are observed in this Which of the statement given above is/are incorrect?
region. (a) 1 only
Which of the statements given above is/are correct? (b) 2 and 3 only
(a) 1 only (c) 1 and 2 only
(b) 2 only (d) 1, 2 and 3
(c) Both 1 and 2
33. Consider the following statements regarding the
(d) Neither 1 nor 2
temperature of ocean water.
29. Consider the following pairs, with reference to 1. The surface temperature of ocean water is
relief of the Ocean Floor. maximum at the Equator.
2. In higher latitudes the temperature in open sea is
Feature Characteristic
lower than in the enclosed sea.
1. Seamounts chains of mountains separated 3. Oceans in northern hemisphere receives more
by a large depression heat than oceans of southern hemisphere.
2. Guyots a flat-topped seamount Which of the statements given above is/are
3. Atoll low islands found in the incorrect?
tropical oceans (a) 1 only
(b) 1 and 2 only
Which of the pairs given above is/are correctly (c) 2 and 3 only
matched? (d) 1, 2 and 3
(a) 1 only
(b) 2 only 34. Which of the following factors contribute to the
(c) 3 only movement of ocean currents?
(d) 2 and 3 only 1. Temperature difference of oceanic water
30. Consider the following statements: 2. Planetary winds
1. The rate of decrease of temperature of oceans 3. Earth’s rotation
with depths is lower at the equator than at the 4. Gravity
poles. Select the correct answer using the code given below:
2. The winds blowing from the land towards the (a) 1, 2 and 3 only
ocean helps in upwelling of cold water at coast. (b) 2, 3 and 4 only
3. The enclosed seas in the high latitudes record (c) 1 and 4 only
relatively higher temperature than the open seas. (d) 1, 2, 3 and 4

73 Workbook
GEOGRAPHY

35. With reference to the minor relief features of the (a) 1, 2 and 3 Only
ocean floor, consider the following statements: (b) 1, 2 and 4 Only
1. Submarine Canyons are deep valleys that cut (c) 2 and 3 Only
across the continental shelves and slopes. (d) 1, 3 and 4 Only
2. Guyots are flat-topped sea mountains.
Which of the statements given above is/are correct? 2.3. Others
(a) 1 only 40. With reference to ocean currents, consider the
(b) 2 only following pairs:
(c) Both 1 and 2
(d) Neither 1 nor 2 Current Characteristics
36. With reference to Ocean Currents, consider the 1. Canaries current Cold current of Atlantic
following statements: Ocean
1. Ocean Currents are stronger at the surface than 2. Agulhas current Warm current of
at the depths. Southern Indian Ocean
2. Rotation of Earth have an influence on the 3. Irminger current Cold current of
movement of Ocean Currents. Southern Pacific
3. Difference in water density affect the mobility of
Ocean Currents. Which of the pairs given above are correctly
matched?
Which of the statements given above is/are correct? (a) 1 only
(a) 1 and 2 only (b) 1 and 2 only
(b) 3 only (c) 2 and 3 only
(c) 1 and 3 only (d) 1 and 3 only
(d) 1, 2 and 3
41. Which of the following pairs is/are correctly
37. Why thermohaline circulation is called as the matched?
“global conveyer Belt”?
(a) It drives the tidal movement across the globe. Sea Basin Countries
(b) It results in movement of aquatic biota at global 1. North Sea Russia
level.
(c) It results in the development of trade wind. 2. Bering Sea Norway
(d) It drives the ocean current in the deep ocean and 3. Red Sea Eritrea
on the surface.
4. Caspian Sea Iraq
38. With reference to the life forms in the deep ocean,
Select the correct answer using the code below:
consider the following:
(a) 3 Only
Assertion: Aquatic life form is found even in the (b) 1, 2 and 3 Only
submarine trenches. (c) 1 and 4 Only
Reason: Aquatic life forms rely upon the (d) 2 and 4 Only
chemosynthesis for energy.
42. Consider the following pairs:
(a) Both A and R are true and R is the correct
explanation of A Strait Joining Seas/Water
(b) Both A and R are true but R is not the correct bodies
explanation of A
(c) A is true but R is false 1. Bab-el-Mandeb Persian Gulf and Gulf of
(d) A is false but R is true Oman
2. Cook Tasman Sea and South
39. With reference to the bottom ocean topography of
Pacific Ocean
Pacific, Atlantic and Indian Ocean, consider the
following statements: 3. Sunda Java Sea and Indian Ocean
1. Ninety East Ridge is found in Pacific Ocean. 4. Formosa South China Sea and East
2. New Caledonia reef system is located in South China Sea
Pacific Ocean. Which of the pair/pairs given above is/are correctly
3. Siberian shelf located in Arctic Ocean is the matched?
largest continental shelf on the earth. (a) 1 and 2 only
4. The Pacific Ocean does not have oceanic ridges (b) 4 only
like Atlantic and Indian Ocean. (c) 2 and 3 only
Which of the statements given above is/are correct? (d) 2, 3 and 4 only

Workbook 74
GEOGRAPHY

43. Consider the following statements about oceanic following statements is/ are correct?
routes: 1. The International Maritime Organisation was
1. Suez Canal connects the Persian Gulf to the created under this convention.
Mediterranean Sea. 2. The convention regulates trading activities
2. The proposed Vladivostok to Chennai maritime through maritime regions.
route passes through the strait of Gibraltar. 3. The decisions of the conventions are binding on
3. The maritime routes passing through the coast all the members.
of Somalia are vulnerable to piracy. Select the correct answer using the code given below:
Which of the statements given above is/are correct? (a) 1 and 3 only
(a) 1 and 2 only (b) 3 only
(b) 2 and 3 only (c) 2 and 3 only
(c) 3 only (d) 1 only
(d) 1, 2 and 3
48. Consider the following statements about World
44. What is the correct sequence of following Seas as Meteorological Organization (WMO)
one proceeds from north to south? 1. It is not a specialized agency of the United
1. Celebes Sea Nation.
2. Baltic Sea 2. World Meteorological congress meets every four
3. Red Sea years to elect president and vice-president of the
organization.
4. Yellow Sea
3. It has linked recent locust swarms with increased
Select the correct answer using the code given below. frequency of extreme weather events.
(a) 1-2-4-3
Which of the statements given above is/are correct?
(b) 1-2-3-4
(c) 1-4-2-3 (a) 1 only
(d) 2-4-3-1 (b) 1 and 2 only
(c) 2 and 3 only
45. Consider the following statements in respect of (d) 1 and 3 only
the United Nations Convention on the Law of the
Sea (UNCLOS): 49. In which of the following areas tides holds
significance?
1. The contiguous zone extends seaward up to 12
nautical miles (nm) from the baselines. 1. Navigation of ships
2. Each coastal state has full sovereignty over its 2. Fishing
Exclusive Economic Zone (EEZ). 3. Removal of polluted water from river estuaries
3. It established International Seabed Authority to 4. Generation of energy
regulate exploitation of deep seabed minerals. Select the correct answer using the code given below:
Which of the statements is/are correct? (a) 1 and 2 only
(a) 1 and 2 only (b) 1, 3 and 4 only
(b) 2 and 3 only (c) 2 and 4 only
(c) 3 only (d) 1, 2, 3 and 4
(d) 2 only 50. Which of the following statements is incorrect
46. Consider the following statements in respect to with regard to the phenomena of Waves?
the Arctic Council: (a) Water moves from one place to another through
1. It is a public-private partnership which brings waves.
together researchers across the globe for (b) Steep waves are fairly young while slow and
promoting research in arctic zone. steady waves originate from faraway places.
2. Both India and China are members of the (c) The maximum wave height is determined by the
council. strength of the wind.
3. The council implements Arctic Monitoring and (d) Wave breaks when the depth of water is less than
Assessment Programme (AMAP). half the wavelength of the wave.
Which of the following statements is/are correct? 51. Consider the following pairs of types of tides and
(a) 1 and 3 only their explanation:
(b) 3 only
(c) 1 and 2 only Type of tide About
(d) 1, 2 and 3 only 1. Semi-diurnal tide Most common tidal
47. With reference to the United Nation Convention pattern with two high
on the Law of the Sea (UNCLOS), Which of the tides and two low tides

75 Workbook
GEOGRAPHY

2. Mixed tide These tides have (c) Indian Ocean is a land-locked ocean
variations in height (d) Indian Ocean has greater variation in salinity

3. Spring tide They occur twice a 54. Which of the following statements is/are incorrect
month, on full moon about the variability in insolation with latitude?
and on new moon 1. Due to transparency and motion of water it gets
heated up much faster than land.
4. Neap tide They occur when the
2. Meridional circulation occurs due to variation in
sun, the moon and the
insolation across the earth.
earth are in a straight
line Select the correct answer using the code given below:
Which of the pairs given above is/are correctly (a) 1 only
matched? (b) 2 only
(c) Both 1 and 2
(a) 1 and 3 only
(d) Neither1 nor 2
(b) 1, 2 and 3 only
(c) 1, 2 and 4 only 55. Consider the following pairs of different canals/
(d) 2, 3 and 4 only straits and seas connected by them.
52. Consider the following statements: Straits/canals Oceans/seas
1. Land breeze occurs when there is low pressure
area over the land. 1. Seuz Canal Connects Red Sea to
Gulf of Aden.
2. Sea breeze occurs when the pressure gradient is
from the land to the sea 2. Bosporus strait Connects Sea of
3. The land and sea breeze occur due to phenomena Marmara to Aegean
of convection. Sea.
Which of the statements given above is/are correct? 3. Bab-el-Mandeb Connects Red Sea to
(a) 1 and 2 only Mediterranean Sea.
(b) 2 only 4. Strait of Hormuz Connects Persian Gulf
(c) 3 only with Indian ocean.
(d) 2 and 3 only
Which of the pairs given above is/are correctly
53. Which one of the following factors is responsible matched?
for the change in the regular direction of the ocean (a) 1, 2 and 4 only
currents in the Indian Ocean? (b) 2 and 3 only
(a) Indian Ocean is half an ocean (c) 2 and 4 only
(b) Indian Ocean has monsoon drift (d) 4 only

Workbook 76
GEOGRAPHY

SOLUTIONS

Pacific compared to El Nino Whereas El Nino is


2.1. Previous Years Questions characterized by unusually warm ocean temperatures in
1. Solution: (b) the Equatorial Pacific.
Exp) Option b is correct. Statement 2 is incorrect. La Nina is favourable to Indian
Statement 1 is incorrect. The size of the oceans and the monsoon whereas, El Nino has adverse effect on south-
seas affects annual range of temperature e.g., bigger the west monsoon.
size, lower the annual range and vice versa. The Atlantic 4. Solution: (b)
Ocean records relatively higher annual range of
temperature than the Pacific Ocean due to this reason. Exp) Option b is correct.
Statement 2 is correct. The temperature of ocean water Option 1 is correct. Rotation of Earth leads to Coriolis
varies in the northern and the southern hemispheres effect which influences direction of ocean currents. The
because of dominance of land in the former and water in Coriolis effect causes air and water to move towards
the latter. The oceans in the northern hemisphere receive the right in the northern hemisphere and the left in the
more heat due to their contact with larger extent of land southern hemisphere.
than their counterparts in the southern hemisphere and Option 2 is correct. Wind is the primary force driving
thus the temperature of surface water is comparatively surface currents in the ocean. Warm air masses form
higher in the former than the latter. where the sun’s radiation is most intense, which is at the
Other factors affecting the distribution of temperature equator, this becomes an area of low pressure. Cold air
of ocean water include: masses form at the poles, where the sun’s radiation is less
Latitudes: The temperature of surface water decreases intense, this becomes an area of high pressure. Rising
from equator towards the poles because the sun’s rays warm air travels from low pressure to high pressure,
become more and more slanting and thus the amount of resulting in wind. The dominant wind patterns drive
insolation decreases poleward accordingly. oceanic currents.
Prevailing wind: Wind direction largely affects the Option 3 is correct. The density of ocean water influences
distribution of temperature of ocean water. The winds the speed of the ocean currents. Also, denser water tends
blowing from the land towards the oceans and seas (e.g., to sink, while relatively lighter water tends to rise.
offshore winds) drive warm surface water away from the Option 4 is incorrect. Revolution of Earth does not
coast resulting into upwelling of cold bottom water from influence the movement of Ocean Currents.
below.
Insolation, Planetary winds, Gravity, Salinity of ocean
Ocean currents: Surface temperatures of the oceans are water, Shape of coastline are some other important factors
controlled by warm and cold currents. Warm currents which influence the movement of ocean currents.
raise the temperature of the affected areas whereas cool
currents lower down the temperature. For example, the 5. Solution: (c)
Gulf Stream raises the temperature near the eastern coasts Exp) Option c is correct.
of N. America and the western coasts of Europe.
The meeting of the cold and warm water, on the other
2. Solution: (b) hand, produces favourable conditions for the growth of
Exp) Option b is correct. plankton, on which fish depend directly or indirectly for
Statement 1 is incorrect. The current depth of Strait of their food supply. Thus, the areas where warm and cold
Malacca is 25 metres, which prohibits entry of some of currents meet are the best fishing grounds in the world.
the biggest ships in the world. However, deepening it will For instance, Grand Banks in North America is an
not reduce navigation-time and distance to be travelled. international fishing ground where the cold Labrador
Statement 2 is correct. The Kra Isthmus is the narrowest Current and the relatively warm Gulf Stream.
part of the Malay Peninsula. Kra canal is a proposed canal
which would reduce the travel time by eliminating need 6. Solution: (a)
of going around the Malay peninsula, and reducing a Exp) Option a is correct.
distance of nearly 1,200 kilometres. Statement 1 is correct. The continents get heated faster
3. Solution: (d) and get cooled faster in comparison to the Oceans. The
annual range of temperature is high in the interior of the
Exp) Option d is correct.
continent because places located in the interior of the
Statement 1 is incorrect. La Nina is characterised by continent are far away from the moderating influence of
unusually cold ocean temperatures in the Equatorial the sea.

77 Workbook
GEOGRAPHY

Statements 2 is incorrect. Altitude affects the daily 7. Solution: (a)


range of temperature and annual mean temperature but
Exp) Option a is correct.
has negligible impact on annual range of temperature.
Latitude affects the annual range of temperature. The Option 1, 2 and 3 are correct. Coral reefs ideally require
annual range of temperature increases with increasing sunlight, clean water to sunlight through, Salt water, and
latitude. abundant Planktons. In India, the major reef formations
Statement 3 is incorrect. Wind are generally stronger are found in the Gulf of Mannar, Palk bay, Gulf of Kutch,
near the coasts compared to interior areas of continents. Andaman and Nicobar Islands and the Lakshadweep
Statement 4 is incorrect. Rainfall in the interiors of the islands.
Continents is generally low as compared to Coasts. It is
because the rain bearing winds from seas and oceans tend Option 4 is incorrect. Due to the presence of high volume
to loss most of the moisture till they reach the interiors. of freshwater, coral reefs are not formed in Sunderbans.

Fig: Distribution of coral reefs along Indian Coast


8. Solution: (b) The sequence of the given cities from South to North is
Exp) Option b is correct. Jakarta, Singapore, Bangkok and Hanoi.
Turkey is located between Mediterranean Sea to the
South-West and Black Sea to the North.

10. Solution: (d)


Exp) Option d is correct.
9. Solution: (c) Statement 1 and 2 are correct. Tides are the rise and
Exp) Option c is correct. fall of sea levels caused by the combined effects of the

Workbook 78
GEOGRAPHY

gravitational forces exerted by the moon and the sun, and


the rotation of the earth.
Statement 3 is correct. Tides occur due to an imbalance
between the various forces acting on the ocean water at
a point in time. In general, the tide-generating force
is the difference between these two forces; i.e. the
gravitational attraction due to the mass of the moon
and the centrifugal force due to rotation of the earth.
Together, the gravitational pull and the centrifugal force
are responsible for creating the two major tidal bulges Fig. Countries surrounding Mediterranean Sea
on the earth. On the side of the earth facing the moon, a
tidal bulge occurs while on the opposite side though the 13. Solution: (a)
gravitational attraction of the moon is less as it is farther Exp) Option b is correct.
away, the centrifugal force causes tidal bulge on the other Sumatra, and Java are islands located in Indonesia.
side. Option a is correct. With distance of 1192 kms Sumatra
11. Solution: (b) is geographically closest to the southernmost and
largest of the Nicobar Islands i.e. Great Nicobar.
Exp) Option b is correct.
Option b is incorrect. Distance between Great Nicobar
21 countries including Albania, Algeria, Bosnia and and Borneo islands is 2398 kms. Borneo islands lies in
Herzegovina, Croatia, Cyprus, Egypt, France, Greece, Malay peninsula and it is shared by Malaysia, Indonesia
Israel, Italy, Lebanon, Libya, Malta, Monaco, Montenegro, and Brunie.
Morocco, Slovenia, Spain, Syria, Tunisia, and Turkey Option c is incorrect. Distance between Great Nicobar
share their border with Mediterranean Sea. and Java islands is 1817 kms.
Option d is incorrect. Distance between Great Nicobar
and Sri Lanka is 1437 kms.

Fig. Countries surrounding Mediterranean Sea

12. Solution: (c)


Exp) Option c is correct.
Option 1 is incorrect. Jordan is bounded to the north by 14. Solution: (b)
Syria, to the east by Iraq, to the southeast and south by Exp) Option b is correct.
Saudi Arabia, and to the west by Israel and the West Bank. Pair 1 is correctly matched. Aleppo is a city in northern
It opens into the Red Sea at its southern most point. Syria. It used to be largest city of Syria before civil war
Option 2 is correct. Iraq is bordered to the north by broke out in 2011.
Turkey, to the east by Iran, to the west by Syria and Jordan, Pair 2 and 3 are incorrectly matched. Kirkuk and Mosul
and to the south by Saudi Arabia and Kuwait. It has access are cities located in Iraq. Kikuk is a region in Iraq which is
to Persian Gulf in the South claimed by the Kurds, who controlled it for a brief period
in 2017. Mosul, located in Iraq witnessed major battles to
Option 3 and 4 are correct. Following countries have a
defeat ISIS. Mosul was retaken from ISIS in 2017.
shoreline at the Mediterranean Sea:
Pair 4 is correctly matched. Mazar-i-sharif is a city in
European countries: Spain, France, Italy, Malta, Monaco, Afghanistan, which witnessed a massive Taliban attack on
Slovenia, Croatia, Bosnia and Herzegovina, Montenegro, Afghanistan Armed Forces in 2017. Also, Indian consulate
Albania, and Greece. in Mazar-i-Sharif was attacked in 2016.
Western Asia (Middle East):Turkey, Cyprus, Syria, Thus, all the places mentioned in the options are
Lebanon, Israel, and the Palestine Gaza Strip. conflict ridden areas.

79 Workbook
GEOGRAPHY

Statement 3 is correct: It is estimated that more than


25,000 described species from thirty-two of the world’s
thirty-three animal phyla live in coral reef habitats. It is
around four times the number of animal phyla found in
tropical rain forests.
17. Solution: (b)
Exp) Option b is correct.
Option 1 is correct. Adriatic Sea borders Albania.
Option 2 is incorrect. Black Sea borders Russia, Turkey,
Georgia, Romania, Ukraine and Bulgaria. Adriatic Sea
borders Croatia.
Option 3 is correct. Russia, Iran, Azerbaijan, Kazakhstan
and Turkmenistan are countries bordering Caspian Sea.
Option 4 is correct. Morocco which lies in North
15. Solution: (c) western Africa borders the Mediterranean Sea near
Exp) Option c is correct. Strait of Gibraltar.
All the regions mentioned have been in news for seeking Option 5 is incorrect. Red Sea borders Egypt,
autonomy from the countries they are located in. Sudan, Yemen, Saudi Arabia, Eritrea and Djibouti
Option a is correct. Catalonia is a autonomous region in The Mediterranean Sea borders Syria.
Spain. They are seeking complete independence.
18. Solution: (c)
Option b is incorrect. Till 2014, Crimea was part of
Exp) Option c is correct.
Ukraine. Following pro-Russian demonstrations, it was
annexed by Russia. Option 1 is incorrectly matched. Mekong river originates
Option c is correct. Mindanao is an Island in in Tibet, flows through China, Myanmar, Laos, Thailand,
Philippines. Centuries of conquest first by Spanish and Cambodia and Vietnam before draining into the
American colonial forces that had ruled the Philippine South China Sea. It does not reach the Andaman Sea.
archipelago followed by Filipino Christian settlers have Option 2 is incorrectly matched. Thames is the longest
gradually turned Muslims into a minority group in river of England. It flows through the Thames Estuary
Mindanao, triggering conflict over land, resources and before draining into the North Sea.
sharing of political power. Option 3 is correctly matched. Volga is the longest
Option d is incorrect. Oromia is a state in Ethiopia, river of Europe. It Rises in the Valdai Hills northwest of
which has seen struggle for autonomy in the past by the Moscow, and discharges into the Caspian Sea
Oromo people. Option 4 is correctly matched. Zambezi is a major river
16. Solution: (d) of Africa. It originates in Zambia, flows through Angola,
Namibia, Botswana, Zimbabwe and Mozambique before
Exp) Option d is correct
draining into the Indian Ocean.
Coral reefs are large underwater structures composed of
the skeletons of colonial marine invertebrates called coral. 19. Solution: (b)
Coral polyps, the animals primarily responsible for Exp) Option b is correct
building reefs, can take many forms: large reef building Ocean Mean Temperature (OMT) is an important
colonies, graceful flowing fans, and even small, solitary climatic parameter required for atmospheric and oceanic
organisms.
studies like cyclone and monsoon predictions and ocean
Statement 1 is correct: Coral reefs are predominantly heat transport estimations.
present in tropical shallow waters less than 50 meters deep.
Coral reefs can be found in tropical destinations around Statement 1 is incorrect. OMT is measured up to a depth
the world, mostly in areas around the equator where the of 26°C isotherm, is seen at depths varying from 50-100
water is warmer. More than 100 countries have a coral metres. During January-March, the mean 26°C isotherm
reef within their borders, and over half of the world’s coral depth in the South-western Indian ocean is 59 metres.
reefs are found within six countries: Australia, Indonesia, Statement 2 correct. OMT collected during January —
Philippines, Papua New Guinea, Fiji, and the Maldives. March can be used in assessing whether the amount of
Statement 2 is correct: Global distribution of corals- rainfall in monsoon will be less or more than a certain
Australia-17% Indonesia-16% Philippines-9% So, long-term mean. Using OMT data collected during
Australia, Indonesia and Philippines together host more January-March 2018, it was able to predict with greater
than one-third of world’s corals. probability of the pattern of monsoon.

Workbook 80
GEOGRAPHY

Statement 3 is correct. The Salinity of ocean waters varies


2.2. Movement, relief, temperature and from place to place. Waters of high salinity are denser
salinity of Ocean Water than waters of low salinity. Hence, waters of low salinity
20. Solution: (d) flow on the surface of waters of high salinity while waters
of high salinity flow at the bottom towards waters of low
Exp) Option d is correct. salinity.
Agulhas current is the warm current that flows through Statement 4 is correct. The earth’s rotation deflects freely
the east coast of Africa in the southwest Indian ocean. It moving objects, including ocean currents, to the right. In
finally merges with the west wind drift. the northern hemisphere (ex- the circulation of the Gulf
Option a is incorrect – Benguela current is a cold current Stream Drift and the Canaries Current), this is a clockwise
that flows near the western coast of Africa in the southeast direction and in the Southern hemisphere, it is an anti-
Atlantic Ocean. It is associated with the Namib desert. clockwise direction (ex- the Brazilian Current and the
Option b is incorrect – Falkland current is a cold current West Wind Drift).
that flows near the Argentinian coast in the southwest Statement 5 is correct. A landmass always obstructs
Atlantic Ocean. and diverts a current. For instance, the tip of southern
Option c is incorrect – Canaries current is a cold current Chile diverts part of the West Wind Drift northwards as
that flows near the Spain-Portugal coast and North- the Peruvian Current. Similarly, the ‘shoulder’ of Brazil
at Cape Sao Roque, divides the west-flowing equatorial
Western Africa in the Atlantic Ocean.
currents into the Cayenne Current which flows north-
21. Solution: (a) westwards and the Brazilian Current which flows south-
westwards.
Exp) Option a is correct
The hot Deserts are generally associated with cold 23. Solution: (c)
currents in its vicinity which creates the desiccating effect Exp) Option c is correct.
and hence low levels of rainfall. Statement 1 is correct: According to the United Nations
Pair 1 is incorrectly matched. Kalahari Desert – Benguela Environment Programme, there are more cold-water
current. It is a cold current. Benguela current moves coral reefs worldwide than tropical reefs. The largest cold-
from the Southern Ocean (around Antartica) and flows water coral reef is the Rost Reef off Norway.
northwards along the West Coast of Africa. Statement 2 is incorrect: Like tropical corals, they provide
Pair 2 is incorrectly matched. Sahara Desert - Canary habitat to other species, but deep-water corals do not
current. It is a cold current. The Canary Current flows require zooxanthellae to survive.
along the African coast from north to south. Statement 3 is correct: Deep-sea corals together with
Pair 3 is correctly matched. Atacama Desert – Peru other habitat-forming organisms host a rich fauna of
current. Also known as Humboldt current. It is a cold associated organisms. Lophelia reefs can host up to 1,300
ocean current of low salinity that flows in the north-west species of fish and invertebrates. Various fish aggregate on
direction along the coast of South America. deep sea reefs. Deep sea corals, sponges and other habitat-
Pair 4 is correctly matched. Australian desert –West forming animals provide protection from currents and
Australian current. It is relatively cold surface current of predators, nurseries for young fish, and feeding, breeding
the southeast Indian Ocean, part of the general counter and spawning areas for numerous fish and shellfish
clockwise movement in the southern section of that ocean. species.

22. Solution: (d) 24. Solution: (a)


Exp) Option d is correct. Exp) Option a is correct
Option a is correct - Continental shelf is a seaward
Statement 1 is correct. Planetary winds play dominant
extension of the continent from the shoreline to the
influence on the flow of ocean currents. Trade winds
continental edge. Its width varies greatly, from a few
move equatorial water poleward and westwards and warm
miles in North Pacific off the continent of North America,
the eastern coasts of continents.
to over 100 miles off North-west Europe. In some places
In the temperate latitudes, though westerlies are less where the coasts are extremely mountainous, such as rocky
consistent than the trade winds, they result in a north- mountain and Andean coasts, the continental shelf may
easterly flow of water in the northern hemisphere. be entirely absent. Their shallowness enables sunlight to
Statement 2 is correct. There is difference in the penetrate through the water, which encourages the growth
temperature of ocean waters at the equator and at the of minute plants and other microscopic organisms. They
poles. As warm water is lighter and rises, and cold water are thus rich in planktons and form the richest fishing
is denser and sinks, warm equatorial waters move slowly grounds in the world.
along the surface polewards, while the heavier cold waters Option b is incorrect – Continental slope connects the
of the polar regions creep slowly along the bottom of the continental shelf and the ocean basins. It begins where the
sea equatorwards. bottom of the continental shelf sharply drops off into a

81 Workbook
GEOGRAPHY

steep slope. The gradient of the slope is about 1 in 20. The Option 4 is incorrect: The Mariana Trench is a crescent-
depth of the slope region varies between 200 and 3,000 m. shaped trench in the Western Pacific, just east of the
Option c is incorrect – The seaward edge of the Mariana Islands near Guam.
continental slope loses gradient at this depth and gives 27. Solution: (b)
rise to continental rise. It has very little slope.
Exp) Option b is correct.
Option d is incorrect – With increasing depth the rise
becomes virtually flat and merges with the abyssal plain. Statement 1 is correct. While all other seas in the world
These are the flattest and smoothest regions of the world are defined at least in part by land boundaries, the Sargasso
because of terrigenous [denoting marine sediment eroded Sea is defined only by ocean currents.
from the land] and shallow water sediments that buries Statement 2 is correct. It lies within the Northern
the irregular topography. It covers nearly 40% of the ocean Atlantic Subtropical Gyre. The Gulf Stream establishes the
floor. Sargasso Sea’s western boundary, while the Sea is further
defined to the north by the North Atlantic Current, to the
25. Solution: (c) east by the Canary Current, and to the south by the North
Exp) Option c is correct. Atlantic Equatorial Current.
Statement a is correct. An abyssal plain is an underwater Statement 3 is incorrect. The Sargasso Sea is a vast patch
plain on the deep ocean floor, usually found at depths of ocean named for a genus of free-floating seaweed
between 3,000 metres and 6,000 metres. They are called Sargassum.
generally found between the foot of a continental rise and
a mid-ocean ridge. Abyssal plains are the largest habitat 28. Solution: (c)
on earth. Exp) Option c is correct.
Statement b is correct. The continental slope connects Statement 1 is correct. The Continental slope connects
the continental shelf and the ocean basins. It begins the continental shelf and the ocean basins. It begins where
where the bottom of the continental shelf sharply drops the bottom of the continental shelf sharply drops off into
off into a steep slope. The gradient of the slope region a steep slope. The slope boundary indicates the end of the
varies between 2-5°. The depth of the slope region varies continents.
between 200 and 4000 m. The slope boundary indicates Statement 2 is correct. Continental slopes are
the end of the continents. Canyons and trenches are characterized by numerous submarine canyons and
observed in this region. trenches.
Statement c is incorrect. The waters along the The gradient of the continental slope region varies
continental shelf are usually very productive ecosystem. between 2-5 degrees. The depth of the slope region varies
This is due to sunlight and nutrients from upwelling and between 200 and 3000 m.
land water runoff.
Abyssal plains are less productive ecosystems than 29. Solution: (d)
those along the continental shelf. This is mainly because Exp) Option d is correct.
sunlight does not penetrate to the sea floor; thus, Pair 1 is incorrectly matched. Seamounts is a mountain
it supports very less plants, making these deep, dark with pointed summits, rising from the seafloor that does
ecosystems less productive. not reach the surface of the ocean, unlike mid oceanic
Statement d is correct. The mid-ocean ridge system ridge composed of two chains of mountains separated by
forms the longest and largest mountain range on Earth. a large depression.
The mid-ocean ridge consists of thousands of individual Pair 2 is correctly matched. Guyots are flat-topped
volcanoes or volcanic ridge segments which periodically seamount. They show evidences of gradual subsidence
erupt. through stages to become flat topped submerged
26. Solution: (b) mountains.
Exp) Option b is correct. Pair 3 is correctly matched. Atolls are low islands
found in the tropical oceans consisting of coral reefs
Option 1 is correct: Submarine ridges in this ocean surrounding a central depression. It may be a part of
include the Lakshadweep-Chagos Ridge [Reunion the sea (lagoon), or sometimes form enclosing a body of
Hotspot], the Socotra-Chagos Ridge, the Seychelles fresh, brackish, or highly saline water.
Ridge, the South Madagascar Ridge, Carlsberg Ridge etc.
Option 2 is correct: The islands of Indian ocean include 30. Solution: (b)
the Andaman and Nicobar, Sri Lanka, Madagascar and Exp) Option b is correct.
Zanzibar. The Lakshadweep and Maldives are coral Statement 1 is incorrect. The rate of decrease of
islands and Mauritius and the Reunion Islands are of temperature with depths is greater at the equator than
volcanic origin. at the poles. Though the surface temperature of the
Option 3 is correct: Trenches of Indian Ocean includes seas decreases from equator towards the poles but the
Sunda and Diamantina. temperature at the ocean bottoms is uniform from the

Workbook 82
GEOGRAPHY

equator towards the pole, which means that the rate of have lower temperature than the open seas. It is because
decrease of temperature with increasing depth is more of the influence of ocean currents, prevailing winds, land-
rapid near the equator than towards the poles. water expanse and latitudinal differences.
Statement 2 is correct. The winds blowing from the land Statement 3 is correct. Oceans in the northern
towards the oceans drive warm surface water away from hemisphere receives more heat due to their contact
the coast resulting in the upwelling of cold water from with larger extent of land than the oceans in the southern
below. hemisphere.
Statement 3 is incorrect. The enclosed seas in the low 34. Solution: (d)
latitudes record relatively higher temperature than the
open seas. Whereas the enclosed seas in the high latitudes Exp) Option d is correct
have lower temperature than the open seas. Statement 1 is correct.Temperature difference of
oceanic water results in movements of oceanic water.
31. Solution: (b) Heating by solar energy causes the water to expand. That
Exp) Option b is correct. causes piling up of water near the equator than in the
Factors that affect the salinity of the oceans can be broadly middle latitudes. This results in a slight gradient and water
grouped into two categories, (a) factors that increase tends to flow down the slope.
salinity and (b) factors that decrease salinity. Statement 2 is correct.Winds are responsible for both
The factors responsible for increase of salinity are: magnitude and direction. Wind blowing on the surface
1. evaporation Option 2 is correct of the ocean pushes the water to move. Friction between
the wind and the water surface affects the movement of
2. Ice formation,
the water in ocean.
3. advection of more saline water
Statement 3 is correct.Earth’s rotation deflects freely
4. mixing with more saline deep water and
moving objects including ocean currents to the right in the
5. solution of salt deposits. northern hemisphere and left in the southern hemisphere.
The factors that are responsible decrease of salinity are: Statement 4 is correct. Gravity causes movement of
1. Precipitation, water in ocean.Gravity causes the denser water to fall,
2. melting of ice pushing away the less dense water.
3. advection of less saline water, 35. Solution: (c)
4. mixing with less saline deep water and Exp) Option c is correct.
5. inflow of fresh water from land Option 1 is correct Statement 1 is correct. Submarine Canyons are deep
Option 3 is incorrect. Rotation of earth has no direct valleys that cut across the continental shelves and
impact on variation of salinity of seas or oceans. slopes. They are very similar to the gorges found on the
continents. The underwater landslide is one of the reasons
32. Solution: (c) for the formation of the submarine canyons. The Hudson
Exp) Option c is correct. Canyon is the best known submarine canyon in the world.
Statement 1 is incorrect. Sea ice contains very little salt, Statement 2 is correct. Guyot is a flat topped seamount
about a tenth of the amount of salt that sea water has. that forms with the subsidence of the volcanic mountains.
This is because ice will not incorporate sea salt into its They are usually submerged mountains unlike seamounts
crystal structure. Therefore, sea ice is actually drinkable. that may reach the surface of the ocean.
Statement 2 is incorrect. Salinity, generally, increases with 36. Solution: (d)
depth and there is a distinct zone called the halocline,
where salinity increases sharply. Exp) Option d is correct.
Statement 3 is correct. Salinity is lower than average in Ocean currents are the continuous, predictable, directional
equatorial waters because of the heavy daily rainfall and movement of seawater. It is driven by factors like gravity,
high relative humidity. wind (Coriolis Effect), and water density.
Statement 1 is correct. Ocean currents are generally
33. Solution: (b) referred to by their “drift”. Ocean currents are stronger at
Exp) Option b is correct. the surface and may attain speeds over 5 knots. At depths,
Statement 1 is incorrect. The temperature of Ocean water currents are generally slow with speed less than 0.5 knots.
decreases from equator to poles but highest temperature Statement 2 is correct. Rotation of Earth leads to Coriolis
is not found at the equator but at the tropics. Due to effect which influences direction of ocean currents. The
high rainfall in the equatorial region, temperature is not Coriolis effect causes air and water to move towards
so high. the right in the northern hemisphere and the left in the
Statement 2 is incorrect. The enclosed seas in the low southern hemisphere.
latitudes record relatively higher temperature than the Statement 3 is correct. Differences in water density affect
open seas; whereas the enclosed seas in the high latitudes the mobility and speed of ocean currents. Denser water

83 Workbook
GEOGRAPHY

tends to sink whereas lighter water tends to rise. Cold- direction parallel to 90-degree North longitude up to
water ocean currents occur when the cold water at the 40-degree South where it merges with Amsterdam-St Paul
poles sinks and slowly moves towards the equator. Warm- Plateau.
water currents travel out from the equator along the Central Ridges or Mid Oceanic Ridge in Indian Ocean
surface, flowing towards the poles to replace the sinking which is known as Mid Indian Oceanic Ridges extends
cold water. from Southern tip of Indian Peninsula in the North to
37. Solution: (d) Arctica in the South almost in North-South direction and
forms a continuous chain of highlands.
Exp) Option d is correct
Statement 2 correct. The New Caledonia Barrier Reef
Thermohaline circulation is an oceanographic
which is located in the South Pacific is one of the longest
phenomenon which drives a global-scale system
reefs in the world, covering a length of 930 miles. It is a
of currents. Thus, it is can be said to be the “global
home to more than 1000 different species-- some of which
conveyor belt”.
are still unclassified which live within the reef.
There is constant motion in the ocean in the form of a
global ocean conveyor belt. This motion is caused by a Statement 3 is correct. Pacific continental shelf of South
combination of thermohaline currents in the deep ocean America is represents the example of narrow shelf (16km).
and wind-driven currents on the surface. Cold, salty The Atlantic continental shelf off the east coast of North
water is dense and sinks to the bottom of the ocean while America represents the example of medium size shelves
warm water is less dense and remains on the surface. (96-120km) and extensive shelves having width of a few
hundred kilometres are found off the coast of in Arctic
In this, the upper part of the Atlantic thermohaline Sea, Arafura Sea, China Sea Adriatic Sea etc. The world’s
circulation carries warm, upper ocean water through the largest continental shelf extends 1,500 km (about 930
tropics and subtropics toward the north pole while the miles) from the coast of Siberia into the Arctic Ocean.
deep part carries cold, dense polar water move southward
through the Atlantic, around the tip of Africa, and into Statement 4 is incorrect. The Pacific Ocean does have
the ocean beyond. oceanic ridges. Pacific-Antarctic Ridge (PAR) is a
divergent tectonic plate boundary located on the
38. Solution: (a) seafloor of the South Pacific Ocean. Also, the east pacific
Exp) Option a is correct ridge known as Albatross Plateau is 1600 km wide and it
Assertion is correct: Oceanic exploration have revealed extends from the North of New Zealand to the California,
the life forms do exist even in trenches. For instance, as
per study of Mariana Trench which is the deepest location 2.3. Others
in earth’s crust about which although little is known about
the life forms living in the Trench but despite the lack 40. Solution: (b)
of light, acidic and freezing conditions, more than 200 Exp) Option b is correct.
known micro-organisms and small creatures, including Pair 1 is correct. Canary Current is a cold current and
crustaceans and amphipods, have been known to be a part of a clockwise-setting ocean-current system in the
living there. The three most common organisms at the North Atlantic Ocean.
bottom of the Mariana Trench are xenophyophores,
amphipods and small sea cucumbers (holothurians). Pair 2 is correct. Agulhas current is a warm current of
southern hemisphere. It flows south along the east coast
Reason is correct: The deep-sea fauna/life form in of Africa in Indian Ocean.
trenches has found source of energy alternative to sunlight.
To compensate for the absence of photosynthesis, they Pair 3 is Incorrect.Irminger current is a cold current and a
rely on chemosynthesis carried out by various bacteria, part of northern Atlantic Ocean (not southern).
in which they turn inorganic substances into organic.
41. Solution: (a)
Chemosynthesis is the process by which certain microbes
Exp) option a is correct.
create energy by mediating chemical reactions. So the
animals that live around hydrothermal vents make their Sea Basin Countries
living from the chemicals coming out of the seafloor in
the vent fluids. Red Sea Djibouti, Egypt, Eritrea, Saudi
Arabia, Sudan, Yemen
Some have developed much sharper vision while some
have abandoned the need of vision, as they rely on touch Caspian Sea Armenia, Azerbaizan, Georgia,
and vibration. Iran, Kazakhstan, Russia, Turkey,
Turkmenistan, Uzbekistan
39. Solution: (c)
Bering Sea Russia, United States of America
Exp) Option c is correct
Statement 1 is incorrect. Indian ocean has the Ninety North Sea UK, Norway, Germany, Denmark,
East Ridge which extends from continental shelf of Netherland, Begium, luxemborg,
Irrawaddy River mouth. It runs almost North-South France, Switzerland, Italy, Austria

Workbook 84
GEOGRAPHY

42. Solution: (d)


Exp) Option d is correct.
Pair 1 is incorrectly matched. The Bab al-Mandab
strait is the narrow waterway that separates the Arabian
Peninsula from the Horn of Africa. It links the Red Sea to
the Gulf of Aden and the Indian Ocean. At its narrowest
point, the strait is only 29 km wide between Yemen on one
side and Djibouti and Eritrea on the other.
Strait of Hormuz links Persian Gulf and Gulf of Oman.

43. Solution: (c)


Exp) Option c is correct.
Statement 1 is incorrect – Suez Canal lying in Egypt
between Port Said in the north and Port Suez in the south
links the Mediterranean Sea and the Red Sea and not
the Persian Gulf.

Pair 2 is correctly matched. Cook Strait separates the


North and South Islands of New Zealand. The strait
connects the Tasman Sea on the northwest with the
South Pacific Ocean on the southeast. It is considered Fig: Suez Canal
one of the most dangerous and unpredictable waters in Statement 2 is incorrect – A memorandum of Intent was
the world. signed to open a full-fledged maritime route between
Pair 3 is correctly matched. The Sunda Strait is the strait Russia’s eastern port city Vladivostok and Chennai on
between the Indonesian islands of Java and Sumatra. India’s Eastern seaboard. Vladivostok to Chennai route
It connects the Java Sea to the Indian Ocean. Anak passes through the Strait of Malacca, to emerge into the
Bay of Bengal and then cuts across through the Andaman
Krakatau, a volcano that erupted recently is located on
and Nicobar archipelago to Chennai. The Strait of
this strait. Gibraltar connects the Mediterranean Sea to the Atlantic
Ocean.

Fig: Chennai to Vladivostok route


Statement 3 is correct – As per International Maritime
Pair 4 is correctly matched. Formosa Strait (Taiwan Organisation, maritime routes passing through the
Strait) joins South China Sea & East China Sea. It is Gulf of Aden and Offshore coast of Somalia are highly
Located between China-Taiwan. prone to piracy.

85 Workbook
GEOGRAPHY

44. Solution: (d) Statement 3 is correct: The Convention has created three
Exp) Option d is correct. new institutions:
The correct order from north to south is Baltic sea, Yellow 1. The International Tribunal for the Law of the Sea, the
sea, Red sea and Celebes sea. Commission on the Limits of the Continental Shelf
and the International Seabed Authority.
2. The International Seabed Authority regulates the
exploration and exploitation of deep seabed minerals.
46. Solution: (b)
Exp) Option b is correct.
Statement 1 is incorrect: It is an inter-governmental
forum consisting of Eight countries whose territory lies
in the arctic zone.
Members: Canada, Denmark (Greenland), Norway,
Russia, The United States of America, Iceland, Finland,
and Sweden.
Why these seas are important? Statement 2 is incorrect: Non-arctic state can obtain
1. Celebes sea: Earthquake hit in April 2021, Observer status in the council. Observers have no voting
2. Yellow Sea: Oil spill in Last week of April, 2021 right. Observers: Germany, Netherlands, Poland, United
Kingdom, France, Spain, China, India, Italy, Japan, South
3. Red sea: Sudan suspended an agreement which
Korea, Singapore and Switzerland.
allowed Russia to build naval base in Red Sea.
4. Baltic Sea: Construction of Baltic pipe project to Statement 3 is correct: The work of the Council is
transport natural gas. primarily carried out in six Working Groups.
1. Arctic Contaminants Action Program (ACAP): to
45. Solution: (c) encourage national actions to reduce emissions and
Exp) Option c is correct. other releases of pollutants.
Statement 1 is incorrect: The territorial sea extends 2. Conservation of Arctic Flora and Fauna Working
seaward up to 12 nautical miles (nm) from the baselines Group (CAFF): it addresses the conservation of Arctic
of the coastal state. The coastal states have sovereignty and biodiversity, working to ensure the sustainability of
jurisdiction over the territorial sea. the Arctic’s living resources.
Beyond the 12-nautical-mile (22 km) limit, there is a 3. Emergency Prevention, Preparedness and Response
further 12 nautical miles (22 km) from the territorial sea Working Group (EPPR): it works to protect the
baseline limit which is called the contiguous zone. Arctic environment from the threat or impact of an
accidental release of pollutants or radionuclides.
4. Protection of the Arctic Marine Environment
(PAME) Working Group: it is the focal point of the
Arctic Council’s activities related to the protection
and sustainable use of the Arctic marine environment.
5. Sustainable Development Working Group (SDWG):
it works to advance sustainable development in
the Arctic and to improve the conditions of Arctic
communities as a whole.
6. Arctic Monitoring and Assessment Programme
(AMAP): it monitors the Arctic environment,
ecosystems and human populations, and provides
scientific advice to support governments as they
tackle pollution and adverse effects of climate change.
47. Solution: (b)
Exp) Option (b) is the correct answer.
Statement 1 is incorrect. UNCLOS has created three new
institutions:
International Tribunal for the Law of Sea
Statement 2 is incorrect: The exclusive economic zone International Seabed Authority
extends 200 nautical mile seawards from the baseline.
Within this area, the coastal nation has sole exploitation Commissions on the limit of Continental Shelf
rights over all natural resources. However, the coastal Statement 2 is incorrect. The objectives of the convention
state does not have full sovereign right over the area. are:

Workbook 86
GEOGRAPHY

To facilitate International Communications; 50. Solution: (a)


To enable equitable and efficient utilisation of ocean Exp) Option a is correct.
resources; Statement a is incorrect. Water in the waves does not
To protect and preserve the marine environment; move, but the wave trains move ahead. Waves are
To promote Maritime safety. actually the energy, not the water as such, which moves
across the ocean surface. Water particles only travel in a
It does not regulate trading activities. small circle as a wave passes. Energy to surface water is
Statement 3 is correct. The decision of the convention is provided by the waves.
binding on its members. It is through the ocean currents that the water moves
48. Solution: (c) ahead from one place to another.
Exp) Option c is correct. Statement b is correct. A wave size and shape reveal its
origin. Steep waves are fairly young ones and probably
Statement 1 is incorrect. WMO is an intergovernmental formed by local wind. Slow and steady waves originate
organization with a membership of 193 Member States from faraway places, possibly in another hemisphere.
and Territories. Established by the ratification of the
Statement c is correct. Most of the waves are caused
WMO Convention on 23 March 1950, WMO became
by the wind driving against water. The maximum wave
the specialized agency of the United Nations for
height is determined by the strength of the wind, i.e.
meteorology. how long it blows and the area over which it blows in a
Statement 2 is correct. World Meteorological Congress single direction.
assembles delegates of Members once every four years: Waves travel because wind pushes the water body in
1. to determine general policies for the fulfilment of the its course while gravity pulls the crests of the waves
purposes of the Organization downward. The falling water pushes the former troughs
2. to consider membership of the Organization upward, and the wave moves to a new position.
3. to determine the General, Technical, Financial and Statement d is correct. As a wave approaches the beach, it
Staff Regulations slows down. This is due to the friction occurring between
4. to establish and coordinate the activities of constituent the dynamic water and the sea floor. And when the depth
bodies of the Organization. of water is less than half the wavelength of the wave, the
5. to approve long-term plans and budget for the wave breaks.
following financial period
51. Solution: (b)
6. to elect the President and Vice-Presidents of the
Organization and members of the Executive Council Exp) Option b is correct.
7. to appoint the Secretary-General Pair 1 is correctly matched. Semi-diurnal tide is the
most common tidal pattern, featuring two high tides
Statement 3 is correct. WMO said while desert locusts and two low tides each day. The successive high or low
have been here since biblical times, recent intense tides are approximately of the same height.
outbreaks can be linked to anthropogenic climate
change and the increased frequency of extreme weather Pair 2 is correctly matched. Mixed tides are those tides
events. that have variations in height. These tides generally
occur along the west coast of North America and on many
49. Solution: (d) islands of the Pacific Ocean.
Exp) Option d is correct. Pair 3 is correctly matched. Spring tides occur when
Statement 1 is correct. Tides help in navigation; they the sun, the moon and the earth are in a straight line, the
raise the water level close to the shores which helps the height of the tide will be higher. These are called spring
tides and they occur twice a month, one on full moon
ships to enter harbors especially near rivers and within
period and another during new moon period.
estuaries having shallow bars at the entrance.
Pair 4 is incorrectly matched. Sun, moon and earth are
Statement 2 is correct. Commercial and recreational
in a straight line in the case of Spring tide, not in the
fishermen use their knowledge of the tides and tidal
neap tide. In the Neap tide, the sun and moon are at right
currents to help them improve their catches. Depending
angles to each other and the forces of the sun and moon
on the species and water depth in a particular area, fish tend to counteract one another.
may concentrate during ebb or flood tidal currents. In
some areas, strong tidal currents concentrate bait and 52. Solution: (c)
smaller fish, attracting larger fish. Exp) Option c is correct.
Statement 3 is correct. Tides help in desilting the sediment Statement 1 is incorrect. During the day the land heats
and removal of polluted water from river estuaries. up faster and becomes warmer than the sea. Therefore,
Statement 4 is correct. Tides are used to generate over the land, the air rises giving rise to a low-pressure
electrical power in many countries. area, whereas the sea is relatively cool and the pressure

87 Workbook
GEOGRAPHY

over sea is relatively high. Thus, pressure gradient from the northeast monsoon ocean currents disappear and
sea to land is created, and the wind blows from the sea south-west monsoon ocean currents are developed in the
to the land as the sea breeze. northern Indian Ocean.
Statement 2 is incorrect. In the night the reversal of
condition takes place. The land loses heat faster and is 54. Solution: (a)
cooler than the sea. The pressure gradient is from the Exp) Option a is correct.
land to the sea which results in land breeze. Statement 1 is incorrect. The rate of heating is different
Statement 3 is correct. Land and Sea breeze occur due over land and water. Land is opaque and allows greater
to the process of convection. The vertical heating of the absorption; also, all the radiant heat is concentrated at
atmosphere is known as convection. the surface which leads to rapid heating of land. On the
other hand, water is transparent so heat is absorbed
53. Solution: (b) slowly and due to the mobile nature, the absorbed heat
Exp) Option b is correct is distributed over a greater depth and area. This is the
Monsoon Current, also called Monsoon Drift is the reason behind water getting heated up much slower than
surface current of the northern Indian Ocean. Unlike land.
the Atlantic and Pacific, both of which have strong Statement 2 is correct: Insolation is responsible for
currents circulating clockwise north of the Equator, movement of air, ocean currents, exogenetic forces and
the northern Indian Ocean has surface currents that many more. In case of earth as an entire system, there is
change with the seasonal monsoon. The Monsoon winds heat surplus at the equator and deficit at the poles. This
led to the change in the direction of ocean currents. imbalance gives rise to meridional circulations which
These currents are categorized into North-East Monsoon
works to transfer excessive heat from tropics to the
Current and South-West Monsoon Current.
colder Polar Regions.
North-East Monsoon Current:
North-east monsoon winds blow from land to the ocean 55. Solution: (d)
during winter season in the northern Indian Ocean and Exp) Option d is correct.
thus westward blowing north-east monsoon currents are Pair 1 is incorrect. Seuz canal connects Port Said on the
produced between Andaman and Somali. This current Mediterranean Sea with the Gulf of Suez, an arm of the
flows to the south of 5°N latitude. Red Sea.
South-West Monsoon Current:
Pair 2 is incorrect. Bosphorus strait connects Black Sea
There is complete reversal in the direction of monsoon to Sea of Marmara.
winds during summer season. In other words, north-
easterly direction of winter monsoon winds becomes Pair 3 is incorrect. Bab-el-Mandeb connects red sea to
south-westerly during summer season in the northern Gulf of Aden and Arabian sea.
Indian Ocean. This reversal of direction of monsoon Pair 4 is correct. Strait of Hormuz connects the Persian
winds also reverses the direction of ocean currents of Gulf to the Arabian Sea, part of the northern Indian
northern Indian Ocean during summer season. Thus, Ocean.

Workbook 88
GEOGRAPHY

GEOGRAPHY
CLIMATOLOGY
1. There are no clouds or water-vapour in the lower
3.1. Previous Years Questions stratosphere.
1. Westerlies in southern hemisphere are stronger 2. There are no vertical winds in the lower
and persistent than in northern hemisphere. Why? stratosphere.
[UPSC CSE Pre. 2011] Which of the statements given above is/are correct
1. Southern hemisphere has less landmass as in this context?
compared to northern hemisphere. (a) 1 only
2. Coriolis force is higher in southern hemisphere (b) 2 only
as compared to northern hemisphere (c) Both 1 and 2
(d) Neither 1 nor 2
Which of the statements given above is/are correct?
(a) 1 only 5. The seasonal reversal of winds is the typical
(b) 2 only characteristic of [UPSC CSE Prelims 2012]
(c) Both 1 and 2 (a) Equatorial climate
(d) Neither 1 nor 2 (b) Mediterranean climate
(c) Monsoon climate
2. La Nina is suspected to have caused recent floods (d) All of the above climates
in Australia. How is La Nina different from El
Nino? [UPSC CSE Pre. 2011] 6. Normally, the temperature decreases with the
1. La Nina is characterised by unusually cold ocean increase in height from the Earth’s surface,
because [UPSC CSE Pre. 2012]
temperature in equatorial Indian Ocean whereas
El Nino is characterised by unusually warm 1. the atmosphere can be heated upwards only
ocean temperature in the equatorial Pacific from the Earth’s surface
Ocean. 2. there is more moisture in the upper atmosphere
2. El Nino has adverse effect on south-west 3. the air is less dense in the upper atmosphere
monsoon of India, but La Nina has no effect on Select the correct answer using the codes given
monsoon climate. below:
Which of the statements given above is/are correct? (a) 1 only
(a) 1 only (b) 2 and 3 only
(b) 2 only (c) 1 and 3 only
(d) 1, 2 and 3
(c) Both 1 and 2
(d) Neither 1 nor 2 7. Which one of the following is the characteristic
climate of the Tropical Savannah Region?
3. The 2004 Tsunami made people realize that
mangroves can serve as a reliable safety hedge [UPSC CSE Pre. 2012]
against coastal calamities. How do mangroves (a) Rainfall throughout the year
function as a safety hedge? [UPSC CSE Pre. 2011] (b) Rainfall in winter only
(c) An extremely short dry season
(a) The mangrove swamps separate the human
(d) A definite dry and wet season
settlements from the sea by a wide zone in which
people neither live not venture out. 8. Variations in the length of daytime and night time
(b) The mangroves provide both food and medicines from season to season are due to [UPSC CSE 2013]
which people are in need of after any natural (a) the earth’s rotation on its axis
disaster. (b) the earth’s revolution round the sun in an
(c) The mangrove trees are tall with dense canopies elliptical manner
and serve as an excellent shelter during a cyclone (c) latitudinal position of the place
or tsunami. (d) revolution of the earth on a tilted axis
(d) The mangrove trees do not get uprooted by
9. The annual range of temperature in the interior
storms and tides because of their extensive roots.
of the continents is high as compared to coastal
4. The jet aircrafts fly very easily and smoothly in the areas. What is/are the reason/reasons?
lower stratosphere. What could be the appropriate [UPSC CSE Pre. 2013]
explanation? [UPSC CSE Pre. 2011] 1. Thermal difference between land and water

89 Workbook
GEOGRAPHY

2. Variation in altitude between continents and (a) The Earth’s rotation on its axis
oceans (b) Convergence of the two equatorial currents
3. Presence of strong winds in the interior (c) Difference in salinity of water
4. Heavy rains in the interior as compared to coasts (d) Occurrence of the belt of calm near the equator
Select the correct answer using the codes given 15. Consider the following statements:
below. [UPSC CSE Pre. 2015]
(a) 1 only 1. The winds which blow between 30 N and 60
(b) 1 and 2 only S latitudes throughout the year are known as
(c) 2 and 3 only westerlies.
(d) 1, 2, 3 and 4 2. The moist air masses that cause winter rains
10. Which of the following is/are unique characteristic/ in North-Western region of India are part of
characteristics of equatorial forests? westerlies.
[UPSC CSE Pre. 2013] Which of the statements given above is/are correct?
1. Presence of tall, closely set trees with crowns (a) 1 only
forming a continuous canopy (b) 2 only
2. Coexistence of a large number of species (c) Both 1 and 2
3. Presence of numerous varieties of epiphytes (d) Neither 1 nor 2
Select the correct answer using the code given below: 16. “Each day is more or less the same, the morning is
(a) 1 only clear and bright with a sea breeze; as the Sun climbs
(b) 2 and 3 only high in the sky, heat mounts up, dark clouds form,
(c) 1 and 3 only then rain comes with thunder and lightning. But
(d) 1, 2 and 3 rain is soon over.” Which of the following regions
is described in the above passage?
11. “Climate is extreme, rainfall is scanty and the
[UPSC CSE Pre. 2015]
people used to be nomadic herders.”
(a) Savannah
[UPSC CSE Pre. 2013]
(b) Equatorial
The above statement best describes which of the (c) Monsoon
following regions? (d) Mediterranean
(a) African Savannah
(b) Central Asian Steppe 17. Why are dewdrops not formed on a cloudy night?
(c) North American Prairie [UPSC CSE Prelims 2019]
(d) Siberian Tundra (a) Clouds absorb the radiation released from the
Earth’s surface.
12. During a thunderstorm, the thunder in the skies is (b) Clouds reflect back the Earth’s radiation.
produced by the [UPSC CSE Pre. 2013] (c) The Earth’s surface would have low temperature
1. meeting of cumulonimbus clouds in the sky on cloudy nights.
2. lightning that separates the nimbus clouds (d) Clouds deflect the blowing wind to ground level.
3. violent upward movement of air and water 18. Consider the following statements:
particles
[UPSC CSE Pre. 2020]
Select the correct answer using the codes given 1. Jet streams occur in the Northern Hemisphere
below.
only.
(a) 1 only
2. Only some cyclones develop an eye.
(b) 2 and 3
(c) 1 and 3 3. The temperature inside the eye of a cyclone is
(d) None of the above produces the thunder nearly 10°C lesser than that of the surroundings.
Which of the statements given above is/are correct?
13. In the South Atlantic and South-Eastern Pacific
(a) 1 only
regions in tropical latitudes, cyclone does not
(b) 2 and 3 only
originate. What is the reason?
(c) 2 only
[UPSC CSE Pre. 2015] (d) 1 and 3 only
1. Sea surface temperatures are low
2. Inter-tropical Convergence Zone seldom occurs 19. Consider the following statements :
3. Coriolis force is too weak 1. High clouds primarily reflect solar radiation and
cool the surface of the Earth.
4. Absence of land in those regions
2. Low clouds have a high absorption of infrared
14. What explains the eastward flow of the equatorial radiation emanating from the Earth’s surface and
counter-current? [UPSC CSE Pre. 2015] thus cause warming effect.

Workbook 90
GEOGRAPHY

Which of the statements given above is/are correct ? (a) 1 and 2 only
(a) 1 only (b) 2 and 3 only
(b) 2 only (c) 1 and 3 only
(c) Both 1 and 2 (d) 1, 2 and 3
(d) Neither 1 nor 2
24. Consider the following statements:
Practice Questions 1. Thunderstorms and Tornadoes are atmosphere’s
adjustments where unstable atmosphere returns
3.2. Weather to stable state.
2. Thunderstorms are common in middle latitudes
20. With reference to Polar Vortex, consider the whereas Tornadoes are common in high
following statements: latitudes.
1. It is a whirling cone of high pressure over the Which of the statements given above is/are correct?
poles.
(a) 1 only
2. When it becomes strong, a wave of cold air is (b) 2 only
pushed southwards always. (c) Both 1 and 2
3. The term vortex refers to counterclockwise flow (d) Neither 1 nor 2
of air.
25. In the context of role of Tropical easterly jet stream
Which of the statements given above is/are correct?
on Indian Monsoons, consider the following
(a) 1 and 2 only statements:
(b) 1 and 3 only
1. These streams play an important role in the
(c) 3 only
reversal of upper air circulation patterns.
(d) 2 and 3 only
2. The streams steer the tropical depressions into
21. Consider the following statements regarding India.
Sudden Stratospheric Warming (SSW): 3. The tracks of these streams in India are the areas
1. It occurs due to disruption and weakening of the of highest rainfall in India.
Polar vortex.
Which of the statements given above is/are correct?
2. It can affect the shape of the jet stream in the
(a) 1 only
upper Troposphere.
(b) 1 and 3 only
3. It is connected to extreme warm events on the (c) 2 and 3 only
Earth’s surface. (d) 1, 2 and 3
Which of the statements given above is/are correct?
26. In the context of naming of tropical cyclones,
(a) 1 only
consider the following statements:
(b) 2 only
(c) 1 and 2 only 1. Names are selected from the list of names
(d) 1, 2 and 3 prepared by the South Asian Forum for
Environment.
22. Consider the following statements: 2. Names of cyclones in Bay of Bengal are suggested
1. The actual amount of water vapor present in the by Bay of Bengal littoral countries only.
atmosphere is known as relative humidity. 3. Cyclone Tauktae is the name suggested by
2. When the temperature of the air is reduced Myanmar.
below its dew point, condensation occurs. Which of the statements given above is/are correct?
3. On cooling the air, its relative humidity increases.
(a) 1 only
Which of the statements given above is/are correct? (b) 2 and 3 only
(a) 1 and 2 only (c) 1 and 3 only
(b) 2 only (d) 3 only
(c) 2 and 3 only
(d) 3 only 27. Consider the following statements about animals
of the colder climate/polar region:
23. With reference to cyclone consider the following 1. To survive the harsh winters of Antarctica, male
statements: Emperor Penguins have adapted to make milk.
1. It helps in balancing global heat distribution. 2. During hibernation there is a drop-in breathing
2. It facilitates Ecological Succession in ecosystem. rate and temperature of an animal.
3. It is responsible for seed dispersal of plant 3. Large Tundra animals have hollow hairs which
species. trap air close to the body and keep them warm.
Which of the statements given above is/are correct? 4. Penguins breed during the Antarctic winter.

91 Workbook
GEOGRAPHY

Which of the statements given above is/are correct? 1. Fog contains more moisture than the mist.
(a) 1 and 2 only 2. Mists are frequent over mountains and fogs are
(b) 1, 2 and 3 only prevalent where warm currents of air meet cold
(c) 2, 3 and 4 only currents.
(d) 1, 2, 3 and 4 3. Fog has visibility of less than 1km.
28. Which of the following is incorrect about the Which of the statements given above is/are correct?
Koeppen’s scheme regarding climatic regions of (a) 1 only
India? (b) 1 and 2 only
(a) It is based upon annual and monthly values of (c) 2 only
precipitation only. (d) 2 and 3 only
(b) It accepts the native vegetation as the best
expression of totality of a climate. 33. Which of the following statement is correct?
(c) It classifies tropical climates as those where (a) In the mesosphere, temperature increases with
mean monthly temperature throughout the year the increase in altitude.
is over 18 degrees Celsius. (b) Radio waves transmitted from the earth are
(d) It classifies the Gangetic plains and most of the reflected back to the earth by Troposphere.
North East India in a same category. (c) The stratosphere contains the ozone layer.
29. While moving from equator towards poles, which (d) The zone separating the troposphere from the
of the following factors experience an increase? stratosphere is known as the stratopause.
1. Coriolis Force 34. Which of the following conditions are necessary
2. Dawn and Twilight for the formation of dew:
3. Seasonal variation in day length 1. High relative humidity.
4. Insolation 2. Steady air flow.
Select the correct answer using the codes given 3. Cold and long nights.
below:
4. Dew point below freezing point.
(a) 1 and 2 only
(b) 1, 2 and 3 only Select the correct answer using the code given below:
(c) 2, 3 and 4 only (a) 1 and 2 only
(d) 1, 2, 3 and 4 (b) 1 and 3 only
(c) 2, 3 and 4 only
30. Weather front formation is characterised by which
of the following factors? (d) 1, 2 and 3 only
1. Two contrasting air masses. 35. Which of the following statements is/are correct
2. Prominent in middle latitudes. regarding Temperature inversion?
3. Steep temperature gradient. 1. Visibility may be greatly reduced below the
4. Negligible change in pressure. inversion layer.
5. Shift in wind direction. 2. It causes very heavy rainfall.
Select the correct answer from the codes given 3. It improves the air quality below the inversion
below: layer.
(a) 1, 2, 4 and 5 only Select the correct answer using the code given below:
(b) 1, 3 and 5 only (a) 1 only
(c) 1, 2, 3 and 5 only (b) 1 and 2 only
(d) 1, 2, 3, 4 and 5 (c) 2 and 3 only
31. Consider the following statements, with reference (d) 1 and 3 only
to the relief rain:
36. Which of the following could be considered as the
1. The leeward slope of the mountain receive
most appropriate reason behind the naming of
greater rainfall.
Westerlies as Roaring Forties, Furious Fifties, and
2. The winds capacity to take in moisture increases Shrieking sixties in the southern hemisphere?
on the leeward slope of the mountain.
(a) They are more variable in the Southern
Which of the statements given above is/are correct? hemisphere than the northern hemisphere
(a) 1 only (b) They flow with a much greater force and
(b) 2 only regularity in the Southern hemisphere.
(c) Both 1 and 2 (c) They bring hot winds to the western coast of
(d) Neither 1 nor 2
continents in the southern hemisphere.
32. Consider the following statements regarding the (d) They bring much precipitation to the western
difference between Fog and Mist: coast of continents in the southern hemisphere.

Workbook 92
GEOGRAPHY

37. Consider the following statements regarding 2. This convergence zone moves north or south
Anti-cyclones. with the apparent movement of the sun.
1. They can form only in winter, not in summer. Which of the statements given above is/are correct?
2. Clockwise movement of wind occurs in northern (a) 1 only
hemisphere. (b) 2 only
3. Anticyclones normally results in fine weather. (c) Both 1 and 2
Which of the statements given above is/are correct? (d) Neither 1 nor 2
(a) 1 and 2 only
(b) 2 only 42. Which of the following factors control temperature
(c) 2 and 3 only distribution on the surface of the earth?
(d) 1 and 3 only 1. Latitude of the place
2. Air-circulation and ocean currents
38. Consider the following statements regarding
permafrost: 3. Altitude of the place
1. It is defined solely based on temperature and 4. Distance from sea
duration. 5. Local Aspects
2. It contains large quantities of organic leftover Select the correct answer using the code given below:
from thousands of years. (a) 1, 2 and 3 only
3. In northern hemisphere, it is found in higher (b) 1, 2 and 4 only
latitudes only. (c) 2, 3, 4 and 5 only
4. Thawing of permafrost is a major contributor to (d) 1, 2, 3, 4 and 5
climate change.
Which of the statements given above is/are correct? 43. Which of the following statements is/are correct
(a) 1 and 3 only regarding the composition of atmosphere?
(b) 1, 2 and 4 only 1. The concentration of water vapour is more in
(c) 2, 3 and 4 only temperate zones as compared to equatorial
(d) 1, 2, 3 and 4 zones.
39. With reference to the different types of Fronts, 2. The concentration of dust particles is more
consider the following statements: in temperate zones as compared to equatorial
zones.
1. Warm front is the contact zone that develops
when the cold air moves towards the warm air 3. Nitrogen constitutes the maximum amount
mass. of gas present in the atmosphere followed by
2. Occluded front develops when the front remains carbon dioxide.
stationery. Select the correct answer using the code given below:
Which of the following statements is /are correct? (a) 1 only
(a) 1 only (b) 2 only
(b) 2 only (c) 1 and 2 only
(c) Both 1 and 2 (d) 2 and 3 only
(d) Neither 1 nor 2
44. Which of the following statements is/are correct
40. If you need to condense a given moist volume of regarding Troposphere?
air, which of the following actions one will take? 1. Commercial jet aircraft doesn’t fly in this region
1. Increase the temperature of the given volume of due to the presence of turbulence in this region.
air.
2. Most of the mass of atmosphere is present in this
2. Add more moisture through evaporation. layer.
3. Reduce both volume and temperature.
3. It is the wettest layer of the atmosphere.
4. Bring the given volume of air in contact with
colder object. 4. The height of this layer is highest at the equator
and lowest over the poles.
Select the correct answer using the code given below:
(a) 1, 2 and 3 only Select the correct answer using the code given
(b) 1, 2 and 4 only below:s
(c) 2 and 3 only (a) 1 and 4 only
(d) 2, 3 and 4 only (b) 2 and 3 only
(c) 1, 2 and 4 only
41. Consider the following statements about the Inter (d) 1, 2, 3 and 4
Tropical Convergence Zone (ITCZ):
1. It is the zone of convergence of trade winds from 45. Which of the following statements is/are correct
two hemispheres. about the stratosphere?

93 Workbook
GEOGRAPHY

1. Materials like CFCs (chlorofluoro carbons), (b) 1 and 3 only


aerosols etc. stay in the stratosphere for long (c) 2 and 3 only
times due to the lack of vertical convection. (d) 1, 2 and 3
2. Temperature rises as one move upward through 50. Which of the following statements is/are correct?
the stratosphere due to the presence of Ozone
gas. 1. Farther from the equator, water vapor
concentrations are high in the hemisphere
Select the correct answer using the code given below: experiencing summer and low in the one
(a) 1 only experiencing winter.
(b) 2 only 2. In winter months water vapor amounts decrease
(c) Both 1 and 2 less over land area than adjacent ocean areas.
(d) Neither 1 nor 2
Select the correct options from the code given below:
46. Consider the following statements regarding La (a) 1 only
Nina. (b) 2 only
1. It brings heavy rains and flooding in Canada. (c) Both 1 and 2
2. La Nina Modoki is associated with warming in (d) Neither 1 nor 2
the central tropical Pacific.
51. Which of the following pairs is/are incorrectly
Which of the statements given above is/are correct? matched?
(a) 1 only 1. Conduction- two bodies of unequal temperature
(b) 2 only are in contact and there is flow of energy from
(c) Both 1 and 2 higher temperature to lower temperature.
(d) Neither 1 nor 2 2. Advection- air which rises vertically upon
47. Consider the following statements with reference heating in the form of currents and transmit heat
to Madden Julian oscillation (MJO). vertically in the atmosphere.
1. It is as an eastward moving disturbance of clouds, 3. Convection-it is the process of transfer of heat
rainfall, winds and pressure near the equator. through horizontal movement of air.
2. When MJO stays over Pacific Ocean, it brings Select the correct answer using the code given below:
good rainfall in India. (a) 1 only
3. There can be multiple MJO events within a (b) 1 and 2 only
season. (c) 2 and 3 only
Which of the statements given above is/are correct? (d) 1, 2 and 3
(a) 1 only 52. With respect to heat budget of the earth which of
(b) 2 and 3 only the following statements is/are correct?
(c) 1 and 3 only
1. Net radiation is the difference between all
(d) 1, 2 and 3
incoming solar energy and all outgoing
48. Which of the following statements is/are correct terrestrial energy.
about heat budget of the earth? 2. Sensible heat transfer from energy surplus
1. Heat budget is constant over the entire earth. to energy deficit areas is accomplished by
2. Atmosphere is heated only by the incoming atmospheric circulations and ocean currents.
short wave and long wave terrestrial radiations. Select the correct answer using the code given below:
Select the correct answer using the code given below: (a) 1 only
(a) 1 only (b) 2 only
(b) 2 only (c) Both 1 and 2
(c) Both 1 and 2 (d) Neither 1 nor 2
(d) Neither 1 nor 2
53. Consider the following statements regarding
49. Which of the following processes are due to temperature variability in the atmosphere:
inversion of temperature? 1. The temperature decreases with the increase in
1. Accumulation of smoke and dust particles in altitude throughout the atmosphere.
lower atmosphere. 2. The air temperature at the tropopause is greater
2. Occurrence of dense fog which disappears with over the equator than over the poles.
the heat of the sun. Which of the statements given above is/are incorrect?
3. Creates stability in the lower layers of the (a) 1 only
atmosphere. (b) 2 only
Select the correct answer using the code given below: (c) Both 1 and 2
(a) 1 and 2 only (d) Neither 1 nor 2

Workbook 94
GEOGRAPHY

54. Consider the following statements regarding 3. They are stronger in the Northern hemisphere as
influence of various factors on temperature of any opposed to the Southern hemisphere.
place: Which of the statements given above is/are incorrect?
1. A steep slope experiences a more rapid change in (a) 1 only
temperature than a gentle one. (b) 1 and 2 only
2. North facing slope of the mountain ranges in (c) 2 and 3 only
Northern hemisphere show higher temperature (d) 1, 2 and 3
than south facing slope.
Which of the statements given above is/are correct? 59. Consider the following statements regarding Land
and Sea breezes:
(a) 1 only
(b) 2 only 1. These are seasonal rhythms caused by differential
(c) Both 1 and 2 heating of land and sea.
(d) Neither 1 nor 2 2. Fishermen in the tropics often take advantage of
the sea breeze to sail out.
55. With reference to effect of soil on local temperature,
Which of the statements given above is/are correct?
consider the following statements:
(a) 1 only
1. Light soils reflect more heat than darker ones.
(b) 2 only
2. Dry soils like sands are more sensitive to (c) Both 1 and 2
temperature changes than wet soils. (d) Neither 1 nor 2
Which of the statements given above is/are correct?
(a) 1 only 60. Consider the following statements regarding
(b) 2 only Katabatic wind:
(c) Both 1 and 2 1. It is the dense air descending into the valley.
(d) Neither 1 nor 2 2. It is a day-time phenomenon occurring on the
leeward side of mountain slopes.
56. With reference to Ferrel’s Law of Deflection, 3. Fohn is a type of Katabatic wind.
consider the following statements:
1. It tells about the effect of revolution of the earth Which of the statements given above is/are correct?
on direction of wind movement. (a) 1 and 2 only
2. In Northern Hemisphere, winds deflect to the (b) 1 and 3 only
right and in the southern hemisphere to their (c) 2 and 3 only
left. (d) 1, 2 and 3
3. Coriolis force is maximum along the equator 61. Which of the following is/are the characteristics of
and decreases progressively towards the poles. an Air Mass?
Which of the statements given above is/are correct? 1. It has almost uniform physical properties.
(a) 1 and 2 only 2. Their properties are determined by the
(b) 2 only underlying region from where they originate.
(c) 1 and 3 only Select the correct answer using the code given below.
(d) 2 and 3 only (a) 1 only
57. The pattern of planetary winds largely depends on (b) 2 only
which of the following factors? (c) Both 1 and 2
1. Latitudinal variation of atmospheric heating (d) Neither 1 nor 2
2. Migration of pressure belts 62. With reference to different forms of condensation,
3. Revolution of earth consider the following pairs:
4. Ocean Currents
Condensation Forms Conditions of
Select the correct answer using the code given below. condensation
(a) 1 and 2 only
1. Dew Dew point above the
(b) 1, 2 and 4 only
freezing point
(c) 1, 2 and 3 only
(d) 1, 2, 3 and 4 2. Frost Dew point below the
freezing point
58. With reference to Westerlies, consider the
following statements: 3. Fog Sudden rise in air mass
1. They are winds which form part of Hadley cell temperature
circulation. Which of the pairs given above are correctly
2. They bring desiccation (dryness) effect to the matched?
western coast of the continents. (a) 1 and 2 only

95 Workbook
GEOGRAPHY

(b) 1 and 3 only 68. “This is a hot. dry dusty windwith a temperature of
(c) 2 and 3 only over 105°F which originates in the Sahara Desert.
(d) 1, 2 and 3 It blows outwards in a southerly direction from the
63. This is a very low cloud, uniformly grey and thick, desert interiors into the cooler MediterraneanSea.
which appears like a low ceiling or highland fog. It Its scorching breath damages vegetation and
brings dull weather with light drizzle. It reduces crops.”
the visibility of aircraft and is thus a danger. It is Identify the local wind from the information given
(a) Cirrus cloud above and select the correct option:
(b) Stratus cloud (a) Fohn wind
(c) Cumulus cloud (b) Sirocco wind
(d) Cumulonimbus cloud (c) Mistral wind
64. It is formed wherever the saturated air mass is (d) Chinook wind
forced to ascend a barrier. The air expands, and 69. Consider the following statements:
the temperature falls. Consequently, condensation 1. During the day time, sea becomes warmer
takes place forming clouds and eventually rain. leading to blowing of wind from the land to the
The description describes which of the following
sea.
types of rainfall:
2. During the night time, the slopes get cooled
(a) Convectional rainfall
(b) Relief rainfall and the dense air descends into the valley as the
(c) Cyclonic rainfall mountain breeze.
(d) None of the Above Which of the following statements given above is/are
correct?
65. Consider the following conditions:
(a) 1 only
1. Presence of Coriolis force
(b) 2 only
2. A high humidity level in the middle troposphere. (c) Both 1 and 2
3. Small variations in the vertical wind speed (d) Neither 1 nor 2
4. Upper convergence above the sea level system
70. Consider the following statements:
Which of the above conditions are favourable for
formation and intensification of tropical cyclones? 1. Air mass is a large body of air having little
horizontal variation in temperature and
(a) 1, 2 and 3 only
(b) 1 and 3 only moisture.
(c) 1, 3 and 4 only 2. The contact zone between the cold and warm
(d) 1, 2, 3 and 4 mass, as the cold air moves towards the warm air
is called Cold front.
66. With reference to Cyclones, consider the following 3. Warm Fronts brings cloudless skies and results
statements:
into drought like situation.
1. Bomb cyclone is named so because of the severe
lightning and thunderstorms associated with it. Which of the following statements given above is/are
2. Temperate Cyclones move under the influence correct?
of the Westerlies. (a) 1 and 2 only
3. Cyclonic winds circulate in anticlockwise (b) 2 and 3 only
direction in southern hemisphere. (c) 1 and 3 only
(d) 1, 2 and 3
Which of the statements given above is/are incorrect?
(a) 1 and 2 only 71. With reference to ‘Tropical Cyclones’, consider the
(b) 2 and 3 only following statements:
(c) 1 and 3 only 1. A pre-existing weak low-pressure area is a
(d) 3 only required condition for development of tropical
67. Consider the following statements: cyclones.
1. A sudden rise in the pressure of an area signals 2. Landfall of the cyclone intensifies the storm with
the approach of a cyclone the continuous supply of the moisture.
2. Anticyclone is characterised by clear sky and 3. Strong spirally circulating wind around the
calm air. centre is the characteristic of a mature tropical
Which of the statements given above is/ are correct? cyclone.
(a) 1 only Which of the following statements given above is/are
(b) 2 only correct?
(c) Both 1 and 2 (a) 1 and 2 only
(d) Neither 1 nor 2 (b) 1 and 3 only

Workbook 96
GEOGRAPHY

(c) 2 and 3 only 3. Temperature decreases with increasing


(d) 1, 2 and 3 heights due to increase in the concentration of
greenhouse gases.
72. Consider the following statements:
1. Thunderstorms are caused by intense convection Select the correct answer using the code given below:
on moist hot days. (a) 1 and 2 only
2. Cirrus clouds are responsible for producing (b) 2 only
thunder and lightning during a thunderstorm (c) 1 and 3 only
event. (d) 1, 2 and 3
3. Tornadoes are spiralling winds with a low 76. Match the following pairs:
pressure at its centre.
4. Tornadoes occurs at only upper latitudes with Term Description
high Coriolis force.
1. Absolute humidity It is the weight of water
Which of the following statements given above is/are vapour per unit volume
correct? of air.
(a) 1, 2 and 3 only
(b) 1 and 3 only 2. Relative humidity It is the amount of
(c) 2 and 4 only water vapour in volume
(d) 1, 2, 3 and 4 of air.
3. Dew point It is the temperature at
73. Consider the following pairs: which the air becomes
Rainfall type Reason of occurrence fully saturated
1. Conventional Intense heating of Earth’s Which of the above pairs is/are correctly matched?
surface (a) 2 only
(b) 1 and 3 only
2. Orographic Adiabatic heating caused (c) 1, 2 and 3
when cold, humid air raises (d) 3 only
due to the presence of a
barrier 77. Consider the following statements with respect to
3. Cyclonic Rising winds form the ice age:
cumulonimbus clouds, 1. The temperate zones are restricted to the lower
bringing torrential rains equatorial latitudes during ice ages.
Which of the pairs given above is/are correctly 2. There can be colder and warmer periods during
matched? the overall ice-age period.
(a) 1 and 2 only Which of the statements given above is/are correct?
(b) 2 and 3 only (a) 1 only
(c) 1 and 3 only (b) 2 only
(d) None of these (c) Both 1 and 2
74. Which of the following is/ are the reason(s) behind (d) Neither 1 nor 2
the phenomenon of Twilight? 78. Consider the following statements:
1. Tilt of Earth’s axis
Statement 1: Earth’s temperature remains stable over
2. Revolution of Earth around the sun long periods of time.
3. Rotation of Earth about its axis
Statement 2: The oceans absorb most of the excess
4. Spherical shape of Earth
heat from greenhouse gas emissions.
Select the correct answer using the code given below:
Which one of the following is correct in respect of
(a) 1, 2 and 3 only
the above statements?
(b) 2, 3 and 4 only
(c) 1, 3 and 4 only (a) Both the Statement 1 and Statement 2 are correct
(d) 1, 2, 3 and 4 and Statement 2 is the correct explanation for
statement 1
75. With reference to the factors affecting the normal (b) Both statement 1 and Statement 2 are correct but
lapse rate, which of the following statements is/are Statement 2 is not the correct explanation for
correct? Statement 1
1. Terrestrial radiation is absorbed by the (c) Statement 1 is correct but Statement 2 is not
greenhouse gases which heats up the atmosphere. correct
2. Fall in atmospheric pressure leads to decline in (d) Statement 1 is not correct but Statement 2 is
temperature at higher altitudes. correct

97 Workbook
GEOGRAPHY

79. What will be the impact on insolation distribution Which of the statements given above is/are correct?
on Earth if it is not tilted at an angle on its axis? (a) 1 and 2 only
(a) One half of the Earth would be too hot with (b) 1 and 3 only
continuous insolation, while other half would be (c) 3 only
frozen due to absence of insolation. (d) 2 and 3 only
(b) All the places on different latitudes will receive
the same amount of insolation. 84. With reference to frontal zones, consider the
(c) The hot places would become cold due to following statements:
reduced insolation and vice versa. 1. Cold fronts are usually associated with
(d) There would be no seasonal variations in thunderstorms and extreme weather.
temperatures of a place. 2. The stationary fronts can be transformed into
80. With reference to the mechanisms of heat transfer both warm and cold fronts.
in Earth’s atmosphere, consider the following 3. An occluded front is formed when a cold front
statements: overtakes a warm front.
1. Advection is the main mechanism of heat Which of the statements above is/are correct?
redistribution in the middle latitudes. (a) 1 and 2 only
2. Convective transfer of heat energy is confined (b) 1 and 3 only
only to the troposphere. (c) 2 and 3 only
Which of the statements given above is/ are correct? (d) 1, 2 and 3
(a) 1 only 85. In context to Air Masses, which of the following
(b) 2 only statements is correct?
(c) Both 1 and 2
(a) It is large body of air with highly variable
(d) Neither 1 nor 2
temperature and moisture.
81. With reference to the planets of the Solar System, (b) It can extend from the ground level to the
consider the following statements: stratosphere.
1. Mars does not experience any seasons. (c) Air masses are not formed in the polar region.
2. Venus has a very hot and high-pressure (d) Air masses move from one region to another
atmosphere. solely under the influence of Coriolis force.
Which of the statements given above is/ are correct? 86. The process where water vapor changes directly
(a) 1 only into snowflakes and frost is known as
(b) 2 only (a) Sublimation
(c) Both 1 and 2 (b) Deposition
(d) Neither 1 nor 2 (c) Evaporation
82. In which of the following ways do the oceans affect (d) Evapotranspiration
climate and weather on earth? 87. These are the highest clouds in the Earth’s
1. It helps in absorbing solar radiation and atmosphere, located in the mesosphere. They are
releasing heat needed to drive the atmospheric normally too faint to be seen, and are visible only
circulation. when illuminated by sunlight from below the
2. It helps in distributing the absorbed heat around horizon while the lower layers of the atmosphere
the globe. are in the Earth’s shadow.
3. Carbon dioxide is dissolved in cold ocean water Above paragraph best describes which of the
and the warm ocean water releases carbon following types of cloud?
dioxide back to the atmosphere.
(a) Cumulonimbus clouds
Select the correct answer using the code given below. (b) Noctilucent clouds
(a) 1 and 2 only (c) Stratocumulus clouds
(b) 3 only (d) Altocumulus clouds
(c) 2 and 3 only
(d) 1, 2 and 3 88. Which among the following cities has the longest
day in the month of December?
83. With reference to Polar Vortex, consider the (a) Paramaribo
following statements: (b) San Salvador
1. It is a whirling cone of high pressure over the (c) Lisbon
poles. (d) Malta
2. When the vortex becomes strong, a wave of cold
air is always pushed southwards. 89. Consider the following statements:
3. The term vortex refers to counterclockwise flow 1. The factors that affect wind direction is pressure-
of air. gradient force and the Coriolis force only.

Workbook 98
GEOGRAPHY

2. The friction force on the wind is more at the land 94. Arrange the following surfaces in increasing order
surface than on the sea surface. of their albedo:
Which of the above statements is/are correct? 1. Fresh snow
(a) 1 only 2. Sand
(b) 2 only 3. Grass
(c) Both 1 and 2 4. Earth and Atmosphere
(d) Neither 1 nor 2 5. Thin Cloud
90. Which of the following statements is/are correct Select the correct answer using the code given below:
regarding the ‘anti-cyclones’? (a) 3-5-4-2-1
1. Anti-cyclones are the regions of high pressure (b) 3-2-4-1-5
around which air circulates. (c) 2-3-4-1-5
2. Anti-cyclones always brought heavy rain, thus (d) 2-3-4-5-1
leading to flood like conditions.
95. With reference to ‘Geostrophic winds’, consider
3. Anti-cyclones are more frequent during the
the following statements:
summer season.
1. In geostrophic wind, the Coriolis force is
Select the correct answer using the code given below: balanced by horizontal pressure gradient force.
(a) 1 and 3 only 2. Geostrophic winds flow at great speeds due to
(b) 2 and 3 only low friction.
(c) 1 only
(d) 1, 2 and 3 3. Jet streams are also a form of geostrophic winds.
Which of the statements is/are correct in the above
91. With reference to the various forms of context?
precipitation, consider the following statements:
(a) 1 and 2 only
1. Temperature inversion in the atmosphere helps (b) 2 and 3 only
in the formation of sleet. (c) 1, 2 and 3
2. Sleet generally forms in winter, while hail is a (d) 1 and 3 only
precipitation of warm season.
3. Size of pellets of hailstones is normally larger 96. “Coriolis force is an apparent force caused by
than the pellets of sleet. the earth’s rotation.” Which of the following
statements is/are incorrect regarding Coriolis
Which of the statements given above is/are correct?
force?
(a) 1 and 2 only
(a) Coriolis force can cause the deflection of winds.
(b) 2 only
(c) 2 and 3 only (b) The Coriolis force is zero at the equator and
(d) 1, 2 and 3 maximum at the poles.
(c) The rotation of the cyclones in different
92. “A spacecraft witnessed a phenomenon where directions is due to Coriolis force.
instead of water, rainfall of methane was observed (d) The Coriolis force has no effect on deep ocean
on a celestial body. This discovery has led to a currents.
renewed interest of scientist about the possibility
of life outside Earth in our solar system.” 97. Which of the following statements regarding Fog
and Mist is/ are correct?
Which one of the following celestial bodies is being
talked about in the above given paragraph? 1. Fogs are drier compared to mists.
(a) Venus 2. Mists are prevalent where warm currents of air
(b) Juno come in contact with cold currents.
(c) Titan Select the correct answer using the code given below:
(d) Mars (a) 1 only
(b) 2 only
93. Consider the following statements regarding
(c) Both 1 and 2
grasslands:
(d) Neither 1 nor 2
1. A high annual range of temperature is found all
over the temperate grasslands. 98. Consider the following statements regarding
2. Savannas Grasslands are characterized by high Tropical Evergreen Forests:
diurnal range of temperature. 1. The tropical evergreen forests usually occur in
Which of the statements given above is/are correct? areas receiving more than 200 cm of rainfall.
(a) 1 only 2. These forests grow only in regions having
(b) 2 only temperature above 40 degrees Celsius.
(c) Both 1 and 2 3. The unique feature of these type of forest is that
(d) Neither 1 nor 2 the trees are hardwood along with being tall.

99 Workbook
GEOGRAPHY

Which of the statements given above is/are correct? Which of the statements given above is/are correct?
(a) 1 only (a) 2 and 3 only
(b) 1 and 3 only (b) 1, 2 and 4 only
(c) 2 and 3 only (c) 2, 3 and 4 only
(d) 1, 2 and 3 (d) 1, 2, 3 and 4
99. “This type of climate is entirely confined to the 103. Consider the following statements with reference
western portion of continental masses, between to El Nino-Southern Oscillation (ENSO):
30° and 40° north and south of equator. The basic 1. ENSO is a periodic fluctuation in sea surface
cause of this type of climate is shifting of wind temperatures and the air pressure of the
belts. It is characterised by dry summer and wet overlying atmosphere across the Atlantic Ocean.
winter.” 2. El Niño events represent periods of below-
Which one of the following climatic regions is average sea surface temperatures across the east-
described in the above given paragraph? central Equatorial Pacific.
(a) China Type Climate Which of the statements given above is/are correct?
(b) Hawaiian Type Climate (a) 1 only
(c) Mediterranean Type Climate (b) 2 only
(d) Savannah Type Climate (c) Both 1 and 2
(d) Neither 1 nor 2
100. With reference to ‘British Type of Climate’,
consider the following statements:
1. The amount of rainfall decreases eastwards with
3.3. Climate Phenomena
increasing distance from the sea. 104. Consider the statements with reference to Glacial
2. There is only one uniform season throughout the Lake Outburst Flood (GLOF):
year, which is usually not seen in the tropics. 1. A glacial lake is formed by the accumulation of
3. The open nature of the forests in this type of water in a depression created by moraines.
climate promotes Lumbering industry. 2. Remote sensing can be used to assess the possible
Which of the statements given above is/are correct? risk of future outbursts.
(a) 1 only 3. India’s first GLOF Early Warning System has
(b) 2 and 3 only been established in Sikkim.
(c) 1 and 3 only Which of the above statements is/are correct?
(d) 1, 2 and 3 (a) 1 and 2 only
(b) 1 and 3 only
101. Which of the following will result from the absence
(c) 2 and 3 only
of the ozone in the atmosphere?
(d) 1, 2 and 3
1. Photosynthesis would become impossible
2. Extra-terrestrial objects like meteors would 105. Fujiwhara effect is a phenomenon related to which
reach earth unfiltered of the following?
3. Cataracts in eyes (a) Tsunami
(b) Volcanoes
4. Disruption in marine food chain
(c) Polar vortex
Select the correct answer using the code given below: (d) Cyclones
(a) 1, 3 and 4 only
(b) 1 and 4 only 106. Recently “Death Valley” a city in California
registered a temperature of 54.4°C. Which among
(c) 2 and 3 only
the following is the most probable cause for this?
(d) 1, 2, 3 and 4
(a) Weakening of stratospheric Polar vortex event.
102. With reference to the carbon dioxide, consider the (b) Recent occurrences of forest fires in California.
following statements: (c) Heat trapping by high-pressure circulation in
1. It is found in all the layers of atmosphere. the atmosphere.
2. Carbon dioxide remains in the atmosphere for (d) Cumulative effect of high-pressure atmospheric
hundreds of years. condition and influences of EL-Nino.
3. Its concentration in the atmosphere has 107. Which of the following statement is correct with
increased since the beginning of the industrial respect to Heat waves?
age. (a) Heat waves occur when actual maximum
4. European Space Agency’s (ESA) Copernicus temperature is ≥47°C
Anthropogenic Monitoring mission is aimed at (b) Heat waves is a period of abnormally high
measuring atmospheric carbon dioxide. temperatures that occurs during the whole year

Workbook 100
GEOGRAPHY

(c) Severe heat waves occur when temperature 3. La Nina leads to decreased upwelling across the
departure from normal is greater than 6.4°C. west coast of South America.
(d) To declare a heat wave, the set criteria should be Which of the statements given above is/are correct?
met in all Meteorological stations of the district. (a) 1 only
108. “Monsoon rainfall in Indian subcontinent is (b) 2 and 3 only
adversely impacted during ‘El Nino Modoki’ (c) 1 and 3 only
phase.” In this context, El Nino Modoki is (d) 1, 2 and 3
associated with strong anomalous warming in 113. With reference to the distribution of rainfall
which one of the following regions? around the world, consider the following
(a) Eastern Equatorial Pacific Ocean statements:
(b) Western Indian Ocean 1. The rainfall between the latitudes 35-40 degrees
(c) Central Tropical Pacific Ocean North and South of the equator, is heavier on the
(d) Eastern Indian Ocean western coasts and goes on decreasing towards
109. With reference to different forms of precipitation, the east.
consider the following statements: 2. The rainfall between 45-65 degrees North and
South of equator, is heavier on the eastern
1. Temperature inversion in the atmosphere helps
margins of the continents and it goes on
in the formation of sleet.
decreasing towards the west.
2. Hailstones are clumps of layered ice that form in
updraughts (rising air) within thunderstorms. Which of the statements given above is/are correct?
3. Hailstones are larger pellets than the pellets that (a) 1 only
make up sleet. (b) 2 only
(c) Both 1 and 2
Which of the statements given above is/are correct? (d) Neither 1 nor 2
(a) 1 and 2 only
(b) 2 only 114. Regarding the pressure gradient force, which of
(c) 2 and 3 only the statements is/are incorrect?
(d) 1, 2 and 3 1. The vertical pressure gradient force is smaller
than that of the horizontal pressure gradient.
110. Which of the following is/are the characteristics of 2. The difference between vertical and horizontal
the convectional rainfall? pressure gradient force is balanced by the
1. It is very common in the interior parts of opposite gravitational force.
the continents, particularly in the northern Select the correct answer using the code given below:
hemisphere.
(a) 1 only
2. It is mostly associated with winter and colder air (b) 2 only
in the atmosphere. (c) Both 1 and 2
Select the correct answer using the code given below: (d) Neither 1 nor 2
(a) 1 only 115. Which of the following factors affect the wind
(b) 2 only movement?
(c) Both 1 and 2
1. Buoyant Force
(d) Neither 1 nor 2
2. Friction Force
111. Consider the following statements regarding Fog. 3. Coriolis Force
1. It is caused due to unequal refraction of light. 4. Centripetal Acceleration
2. Dense fogs are more common in high and Select the correct answer using the code given below:
middle latitudes. (a) 1 and 3 only
3. It only occurs in the lower strata of atmosphere. (b) 1, 2 and 3 only
Which of the statements given above is/are correct? (c) 2, 3 and 4 only
(a) 1 and 2 only (d) 1, 2, 3 and 4
(b) 2 only 116. With reference to the geostrophic winds, consider
(c) 2 and 3 only the following statements:
(d) 1 and 3 only 1. Geostrophic wind result from the balance
112. Consider the following statements: between Coriolis force and pressure gradient
1. Positive Indian Ocean dipole usually brings force.
good precipitation in India. 2. Jet streams are geostrophic winds.
2. Strong El Nino events contribute to higher than Which of the statements given above is/are correct?
average monsoon rainfall in India. (a) 1 only

101 Workbook
GEOGRAPHY

(b) 2 only 2. These forests provide hard and durable timber.


(c) Both 1 and 2 3. These forests are confined only to the Sundarbans
(d) Neither 1 nor 2 delta.
117. Consider the following statements in the context Which of the statements given above is/are correct?
of relative humidity: (a) 1 and 2 only
1. It is the weight of water vapour per unit volume (b) 2 only
of air. (c) 2 and 3 only
2. It increases with the decrease in temperature of (d) 1, 2 and 3
the water vapour content.
122. Which of the following statements is/are correct
3. It is greater over the oceans and least over the regarding megadiverse countries?
continents.
1. A megadiverse country must contain at least
Which of the statements given above is/are correct? 1500 species of vascular plants as endemics.
(a) 1 only 2. It is identified by Conservation International.
(b) 1 and 2 only
3. A Group of Like-Minded Megadiverse Countries
(c) 2 only
(d) 2 and 3 only are formed through the Cancun initiative.
4. All 17 megadiverse countries are members of the
118. Which of the following are the factors responsible like-minded megadiverse countries.
for variability in insolation?
Select the correct answer using the code given below:
1. Rotation of earth on its axis
(a) 1, 3 and 4 only
2. Angle of inclination of the sun’s rays
(b) 2 and 3 only
3. Length of the day (c) 2, 3 and 4 only
4. Transparency of the atmosphere (d) 3 and 4 only
Select the correct answer using the code given below
123. Which of the following sets of pair(s) is/are
(a) 1 and 2 only
correctly matched?
(b) 1, 3 and 4 only
(c) 2, 3 and 4 only Index Plant Most favourable Climate region
(d) 1, 2, 3 and 4
1. Teak Tropical Monsoon Climate
119. Which of the following statement is incorrect
about terrestrial radiations? 2. Olive Warm Temperate Western
Margin Climate
(a) It is responsible for the heating of lower portion
of the atmosphere. 3. Cactus Hot Desert Climate
(b) 23% of energy in the terrestrial heat budget is 4. Spruce Warm Temperate Eastern
lost through terrestrial radiations. Margin Climate
(c) Greenhouse gases traps the outgoing terrestrial
radiation. Select the correct answer using the code given below:
(d) It is reflected back from the earth’s surface as (a) 3 only
short wave radiations. (b) 1 and 3 only
(c) 2 and 4 only
3.4. Different forms of Vegetation (World) (d) 1, 2 and 3 only

120. With reference to Amazon rainforests consider 124. With reference to precipitation pattern around the
the following statements: world, consider the following pairs:
1. They are also known as lungs of the planet. Climate Climate type
2. It is the home of tribes like the Crow, Cree and Characteristics
Pawnee.
3. They are characterized by hot and wet climate 1. Precipitation with Savanna climate
throughout the year. Winter maxima
Which of the statements given above is/are correct? 2. Absence of distinct Equatorial climate
(a) 1 and 2 only dry season
(b) 2 and 3 only 3. A definite dry and Monsoon climate
(c) 1 and 3 only wet season
(d) 1, 2 and 3
Which of the pairs given above are correctly
121. Consider the following statements regarding matched?
Littoral and Swamp forests: (a) 1 and 2 only
1. These forests can survive only in brackish water. (b) 2 only

Workbook 102
GEOGRAPHY

(c) 2 and 3 only 129. With reference to tribes around the world,
(d) 1, 2 and 3 consider the following pairs:
125. Consider the following statements regarding Tribe Associated region
equatorial climatic region:
1. It has two periods of maximum rainfall which 1. Indian tribe Congo
occurs shortly after Solstices. 2. Orang Asli Malaysia
2. Equatorial forests occurs in multiple layers where 3. Bindibu Australia
each layer has pure stands of single species only.
3. The equatorial region is densely populated. 4. Bushmen Siberia
Which of the statements given above is/are incorrect? Which of the pairs given above are correctly
(a) 1 and 2 only matched?
(b) 2 and 3 only (a) 1 only
(c) 1 and 3 only (b) 2 and 3 only
(d) 1, 2 and 3 (c) 3 and 4 only
(d) 1, 3 and 4 only
126. With reference to Savanna type climate, consider
the following statements: 130. Consider the following statements regarding
1. Extreme Diurnal range of temperature is the grasslands:
characteristic feature of Savanna type climate. 1. Tropical grasslands are considered as granaries
2. Savanna Landscape is characterised by trees with of the world.
small wide leaves and absence of shade. 2. Temperate grasslands are famous for nutritious
3. Savanna type climate is also found in Brazil and grasses.
Australia. Which of the above statements is/are correct?
Which of the statements given above is/are correct? (a) 1 only
(a) 1 and 2 only (b) 2 only
(c) Both 1 and 2
(b) 2 only
(d) Neither 1 nor 2
(c) 1 and 3 only
(d) 1, 2 and 3 131. With reference to temperate grasslands of the
world, consider the following pairs:
127. Consider the following statements about the Hot
Desert type climatic regions: Grassland Associated region
1. Aridity of hot deserts is mainly due to effects of
off-shore trade winds. 1. Pustaz Hungary
2. Desert vegetation have highly specialized means 2. Velds New Zealand
of adaptation to arid environment.
3. Pampas Argentina
3. Mineral wealth of Deserts which make them
economically attractive. 4. Prairies Brazil
Which of the statements given above is/are correct? Which of the pairs given above are correctly
(a) 1 and 2 only matched?
(b) 2 only (a) 1 and 3 only
(c) 1 and 3 only (b) 1 and 4 only
(d) 1, 2 and 3 (c) 2 and 3 only
(d) 2 and 4 only
128. With reference to deserts around the world,
consider the following pairs: 132. Consider the following statements about Warm
Temperate Eastern Margin climate:
Desert Associated Cold ocean current 1. It is typified by warm, dry summer and cool,
1. Namib Desert Benguela current moist winter.
2. Regions experiences low rainfall and desert type
2. Mohave Desert Californian current vegetation.
3. Gobi Desert Oya Shio current 3. It is found in the gulf states of middle east.
4. Sahara Desert Canaries current Which of the statements given above is/are incorrect?
Which of the pairs given above are correctly (a) 1 only
matched? (b) 2 and 3 only
(c) 1 and 3 only
(a) 1 only (d) 1, 2 and 3
(b) 2 and 3 only
(c) 1, 2 and 4 only 133. Which of the following is not the characteristics of
(d) 2, 3 and 4 only the Laurentian climate?

103 Workbook
GEOGRAPHY

(a) It has features of both maritime and the 2. They support a diverse heterotrophic food chain.
continental climates. 3. Mangroves for Future is an intergovernmental
(b) Predominant vegetation is cool temperate organisation of SAARC and ASEAN member
forests. countries.
(c) Agriculture is the predominant economic
activity. Which of the following statements given above is/are
(d) In Asiatic region it shows similarity with correct?
monsoon type of climate. (a) 1 and 2 only
(b) 2 and 3 only
134. Consider the following statements about the Cool (c) 1 and 3 only
temperate western margin climate: (d) 1, 2 and 3
1. It is also referred to as Natal type of climate.
138. Consider the following statements:
2. It is found in the Northern hemisphere only.
1. Coral reefs are mainly found between latitudes 30
3. The natural vegetation of this climatic type is degree North and South on mud free coastlines.
deciduous forest
2. Increase of ammonia and nitrate in the water
Which of the statements given above is/are correct? leads to coral bleaching.
(a) 1 and 2 only Which of the following statements given above is/are
(b) 3 only correct?
(c) 1 and 3 only
(a) 1 only
(d) 1, 2 and 3
(b) 2 only
135. With reference to world distribution of rainfall, (c) Both 1 and 2
consider the following statements: (d) Neither 1 nor 2
1. Generally, rate of rainfall decreases steadily from 139. Consider the following factors:
equator to poles.
1. Acidic Podzolized soils
2. The coastal areas receive more rainfall than the
2. Evergreen vegetation
interior of the continents.
3. Low temperature
3. Western coast receives more rainfall between 35
degree and 40-degree N and S latitudes. 4. Absence of direct sunlight
Which of the statements given above is/are correct? Which of the above factors are responsible for little
undergrowth in the coniferous Siberian forests?
(a) 1 and 2 only
(b) 2 and 3 only (a) 1 and 2 only
(c) 1 and 3 only (b) 1, 3 and 4 only
(d) 1, 2 and 3 (c) 3 and 4 only
(d) 1, 2, 3 and 4
136. With reference to local winds of the world,
consider the following pairs: 140. Consider the following statements about the Cool
Temperate Eastern Margin Climatic region:
Local winds Region 1. This climatic region hosts one of the world’s best
fishing grounds.
1. Sirocco Hot and Dry wind in
North America 2. The predominant vegetation of this climate is
mahogany, ebony, rosewood.
2. Buran Cold winds in Eurasia
Which of the statements given above is/are correct?
3. Mistral Cold winds from (a) 1 only
Mediterranean (b) 2 only
4. Prairies Brazil (c) Both 1 and 2
(d) Neither 1 nor 2
Which of the pairs given above is/are correctly
matched? 141. Consider the following statements about Polar
(a) 1 only climatic vegetation:
(b) 2 only 1. Deficiency of heat is the greatest inhibiting
(c) 1 and 3 only factor for vegetation in this region.
(d) 2 and 3 only 2. Convectional rainfall is generally absent in this
137. Consider the following statements regarding region.
‘Mangrove forests’: 3. There are no trees found in the tundra region.
1. Mangroves forests requires sedimentation Which of the statements given above is/are correct?
covered with brackish water for a part or all of (a) 1 and 2 only
the day. (b) 2 and 3 only

Workbook 104
GEOGRAPHY

(c) 1 and 3 only (b) They are temporary depressional wetlands.


(d) 1, 2 and 3 (c) Spring produced by geothermally heated
142. Which of the following statement is incorrect with groundwater.
reference to Mangrove vegetation? (d) Lake found under a glacier, typically beneath an
(a) This vegetation mainly grows at tropical and ice cap or ice sheet.
subtropical latitudes. 145. In this forest type, tree growth is hindered by
(b) Oviparous mode of reproduction is very low temperatures and short growing seasons.Itis
common in mangrove species. composed of dwarf shrubs, sedges and grasses,
(c) Leaves of many mangrove species have salt
mosses, and lichens. The animals found are
secreting glands.
(d) In India, they are found along Andaman and reindeer, lemming and ptarmigan. Environmental
Nicobar Island. conditions here are very severe.
The above statement best describes which of the
143. Consider the following statements with reference
to coral reefs: following forest ecosystem?
1. They are the counterpart to the tropical rain (a) African Savannah
forest in terms of species diversity. (b) Central Asian Steppe
2. They exhibit different colors because of (c) Boreal forest
Zooxanthellae algae. (d) Siberian Tundra
3. Malvan coral reef is located along Karnataka 146. Which of the following are the features of plants in
coast.
desert ecosystem?
Which of the above statements is/are correct? 1. Poorly developed root system
(a) 1 and 2 only
2. Succulent leaves and stem
(b) 2 and 3 only
(c) 1 only 3. Presence of ephemeral lifestyle
(d) 1, 2 and 3 Select the correct answer using the code given below:
144. Which of the following best describes Vernal (a) 1 and 2 only
pools? (b) 2 and 3 only
(a) Large pools of liquid rock beneath the surface of (c) 1 and 3 only
the Earth (d) 1, 2 and 3

105 Workbook
GEOGRAPHY

SOLUTIONS

Even under such extreme circumstances, mangroves’


3.1. Previous Years Questions dense root and branch networks could help dissipate
1. Solution: (a) tsunamis, reducing their devastation.
Exp) Option a is correct. 4. Solution: (c)
Statement 1 is correct. Land comprises only 19.1% Exp) Option c is correct.
of Southern Hemisphere, In Northern Hemisphere, The stratosphere is found above the tropopause and
the majority of area is composed of land masses. Less extends up to a height of 50 km. One important feature of
landmass in Southern Hemisphere results in lesser the stratosphere is that it contains the ozone layer.
frictional drag in motion of westerlies, thus resulting in Statement 1 is correct. The stratosphere is very dry and
stronger and persistent Westerlies. air there contains little water vapor and there is almost
Statement 2 is incorrect. Coriolis force is equal in complete absence of clouds and thunderstorms. Less water
both the hemispheres at their respective latitudes. The vapour and absence of clouds means less drag which helps
magnitude of the Coriolis force depends on the speed of maintain fuel efficiency.
the object and its latitude. The Coriolis force is zero at the Statement 2 is correct. Unlike the troposphere,
equator and increases toward the poles. there is almost complete absence of vertical winds or
thunderstorms in the lower stratosphere.
2. Solution: (d)
Exp) Option d is correct. 5. Solution: (c)
La Nina events represent periods of below-average sea Exp) Option c is correct.
surface temperatures across the east-central Equatorial Monsoon refers to the seasonal change in the direction of
Pacific. It is indicated by sea-surface temperature decrease the prevailing winds of a region. Monsoons cause wet and
by more than 0.90 for at least five successive three-month dry seasons throughout much of the tropics. Monsoons
seasons. are most often associated with the Indian Ocean.
Monsoons always blow from cold to warm regions. The
Statement 1 is incorrect. La Nina is characterised by
summer monsoon and the winter monsoon determine
unusually cold ocean temperatures in the Equatorial
the climate for most of India and Southeast Asia.
Pacific compared to El Nino, which is characterized by
unusually warm ocean temperatures in the Equatorial Option c is correct.
Pacific. A monsoon is a seasonal reversal in wind patterns over a
Statement 2 is incorrect. La Nina is favourable to Indian region. The seasonal wind shift is usually accompanied by
a dramatic change in precipitation. The phenomenon is
monsoon whereas, El Nino has adverse effect on south-
best developed over the Indian subcontinent.
west monsoon. La Nina leads to heavy floods in Australia.
Heavy floods often lead to waterlogging resulting in poor During winter, there is a high-pressure area north of
agricultural output. There are increased temperatures in the Himalayas. Cold dry winds blow from this region
Western Pacific, Indian Ocean and off the Somalian coast. to the low-pressure areas over the oceans to the south.
In the western Pacific, La Nina increases the potential for In summer, a low-pressure area develops over interior
landfall in those areas most vulnerable to their effects, and Asia, as well as, over north-western India. Thus, wind in
India blows from the northeast during cooler months and
especially into continental Asia and China.
reverses direction to blow from the southwest during the
3. Solution: (d) warmest months of the year.
Exp) Option d is correct 6. Solution: (c)
Mangroves are a group of trees and shrubs that live in the Exp) Option c is correct.
coastal intertidal zone. Statement 1 is correct. The atmosphere is generally heated
Mangrove forests can be recognized by their dense tangle from the terrestrial radiation emitted by the Earth’s surface
of prop roots that make the trees appear to be standing on as gases in the atmosphere do not absorb the incoming
stilts above the water. The roots also slow the movement solar radiation. Therefore, temperature is maximum near
of tidal waters, causing sediments to settle out of the water the surface of the Earth.
and build up the muddy bottom. Statement 2 is incorrect. As the air rises upwards it cools
The sturdy root system of mangrove trees helps form down, as result of this its water holding capacity decreases.
a natural barrier against disasters such as tsunamis and Statement 3 is correct. Due to decreased atmospheric
floods. pressure the density of air decreases as we go upward due

Workbook 106
GEOGRAPHY

to low pressure and hence, less heat can be absorbed. Also, in comparison to the Oceans. The annual range of
as the warm air rises it cools down adiabatically. \ temperature is high in the interior of the continent
because places located in the interior of the continent are
7. Solution: (d)
far away from the moderating influence of the sea.
Exp) Option d is correct.
The tropical savanna climate has alternating dry and wet 10. Solution: (d)
seasons. The wet summer season lasts 6 to 8 months and Exp) Option d is correct.
during these days, there is plenty of rainfall. Winter lasts Statement 1 is correct. Equatorial forests are found within
for 4 to 6 months and there might be no rains in winter 5 degrees north and south of the equator. Primary tropical
this results in frequent forest fires. rainforest is vertically divided into at least five layers: the
Option a is incorrect. Rainfall throughout the year is a overstory, the canopy, the understory, the shrub layer, and
characteristic of Equatorial region. the forest floor. The canopy is the dense ceiling of closely
Option b is incorrect. The Mediterranean region falls spaced trees and their branches. It is the primary layer of
under the influence of wet westerlies during winter season the forest and resembles a roof of the forest.
and receive rainfall in winters only. Statement 2 is correct. Tropical rainforests support the
Option c is incorrect. Tropical Savannah region greatest diversity of living organisms on Earth. Although
experience dry conditions for a longer duration than wet they cover less than 2 percent of Earth’s surface,
conditions. Extremely short dry season is a characteristic rainforests house more than 50 percent of all plants and
of regions experiencing Tropical Monsoon climate. animals found on land. The primary reason behind this
abundance of diversity is the high amount of sunlight this
8. Solution: (d) region receives.
Exp) Option d is correct. Statement 3 is correct. An important characteristic of
Variations in the length of daytime and night time from the canopy system is the presence of plants known as
season to season are due to revolution on Earth on tilted epiphytes, that grow on canopy trees. Epiphytes are not
axis. Day and night are the consequence of the earth’s parasitic because they draw no nutrients away from the
rotation on its axis. But the axis is tilted 23.5 degrees. Due host, but climb the host tree to access direct sunlight.
to this reason, during the earth’s revolution around the 11. Solution: (b)
sun, polar regions spend long periods pointed towards the
Exp) Option b is correct.
sun in the summers and long periods pointed away from
the sun during winters. The same is true for other regions, Central Asian Steppe are dry, grassy plain. Steppes are
thus causing variations in length of day and night. located in the heart of the continents, meant they
have little maritime influence. Their climate is thus
9. Solution: (a) continental with extremes of temperature. With
Exp) Option a is correct. recorded temperatures as high as 45 °C in summer and in
winter falls to −55 °C. Trees are very scarce in the steppes,
The annual range of temperature is defined as the
because of the scanty rainfall (25-50 cm annually).
difference between the hottest and coldest months at a
place, taking monthly mean temperatures in each case. It is The lands do not allow people to settle permanently and
given approximately by the difference between the average the thinly scattered population live in small nomadic
of the January maximum and minimum temperatures, groups, herding cattle, goats, horses, camels, and sheep.
and the corresponding average for July. These grasslands were dominate by nomadic and semi-
nomadic peoples like the kirghiz.
Statement 1 is correct. The continents get heated faster and
get cooled faster in comparison to the Oceans. The annual 12. Solution: (d)
range of temperature is high in the interior of the continent
Exp) Option d is correct.
because places located in the interior of the continent
are far away from the moderating influence of the sea. All the statements are incorrect.
Statements 2 is incorrect. Altitude affects the daily range Thunder is caused by the rapid expansion of the air
of temperature and annual mean temperature but has surrounding the path of a lightning bolt. It is caused by
negligible impact on annual range of temperature. Latitude intense convection on moist hot days. A thunderstorm is a
affects the annual range of temperature. The annual well-grown cumulonimbus cloud producing thunder and
range of temperature increases with increasing latitude. lightning.
Statement 3 is incorrect. Wind is generally stronger near Many small bits of ice or water droplets(frozen raindrops)
the coasts compared to interior areas of continents. bump into each other as they move around in the air. All
Statement 4 is incorrect. Rainfall in the interiors of the of those collisions create an electric charge. After a while,
Continents is generally low as compared to Coasts. It is the whole cloud fills up with electrical charges.
because the rain bearing winds from seas and oceans tend The positive charges or protons form at the top of the
to loss most of the moisture till they reach the interiors. cloud and the negative charges or electrons form at the
The continents get heated faster and get cooled faster bottom of the cloud. Since opposites attract, that causes

107 Workbook
GEOGRAPHY

a positive charge to build up on the ground beneath the North western India by the Westerly Jet Stream causing
cloud. winter rains.
The grounds electrical charge concentrates around 16. Solution: (b)
anything that sticks up, such as mountains, people, or
single trees. Once the negative charge at the bottom of the Exp) Option b is correct.
cloud gets large enough, a flow of negative charge called A tropical rainforest climate or equatorial climate is a
a stepped leader rushes toward the Earth. The positive tropical climate usually found within 10 to 15 degrees
charges at the ground are attracted to the stepped leader, latitude of the equator. They experience high mean annual
so positive charge flows upward from the ground. When temperatures and small temperature ranges. Annual
the stepped leader and the positive charge meet, a strong rainfall is high, amounting to more than 2000 mm that
electric current carries positive charge up into the cloud. falls throughout the year, there is almost absence of any
This electric current is known as the return stroke. We see seasonal variation. A tropical rainforest climate is typically
it as the bright flash of a lightning bolt. hot, very humid, and wet. Due to high temperature and
high humidity, and convergence of trade winds which
13. Solution: (b) uplift the air, there is rainfall throughout the year.
Exp) Option b is correct.
17. Solution: (b)
The South Atlantic and South-Eastern Pacific are as void
of cyclonic activity largely due to the ITCZ having a Exp) Option b is correct.
tendency to stay near or north of the equator. Dew is small drops of water that form on the ground and
For a tropical cyclone to occur in South Atlantic and other surfaces outdoors during the night.
South-Eastern Pacific regions, a broad convergence zone is Option b is correct. Dewdrops are formed when the
needed to reach about 5 degrees of latitude away from the moisture is deposited in the form of water droplets on
equator in order for the Coriolis Force to have sufficient cooler surfaces of solid objects such as stones, grass blades
intensity to organize a full-fledged tropical cyclone, and and plant leaves. The ideal conditions for its formation
the Atlantic ITCZ almost never shifts that far south. are clear sky, calm air, high relative humidity, and
cold and long nights. When the sky is clear and the
14. Solution: (b) trees and plants are cooler at nights, there is more
Exp) Option b is correct. evaporation of water and hence more dew formation.
There are two equatorial current flowing from east to For the formation of dew, it is necessary that the dew
west - the North EC and South EC. In between the two, point is above the freezing point. The air containing
there flows a equatorial counter current in the opposite moisture to its full capacity at a given temperature is
direction, i.e from west to east. Most of the water from said to be saturated. The temperature at which saturation
North EC and South EC travels polewards, however occurs in a given sample of air is known as dew point.
some of the water gets deflected towards equators leading Dew forms when the temperature becomes equal to the
to piling up of waters in the area near Brazil, due to dewpoint. This often happens first at ground level for
convergence of the two equatorial currents give rise to the two reasons. First, longwave emission causes the earth’s
surface to cool at night. Condensation requires the
equatorial counter current.
temperature to decrease to the dewpoint. Second, the soil
is often the moisture source for the dew. Warm and moist
soils will help with the formation of dew as the soil cools
overnight. Cloudy skies reflect back the Earth’s radiation
and that prevents earth’s surface to cool at night.
18. Solution: (c)
Exp) Option c is correct.
Statement 1 is incorrect. Jet Streams occur in both the
Fig. Flow of Equatorial Counter Current hemisphere. Jet streams are high altitude westerly wind
system blows at a height of 6 to 14 km, with very high
15. Solution: (b) speed up to 450 km/hr in wavy form in both hemispheres.
Exp) Option b is correct. At most times in the Northern and Southern Hemispheres,
Statement 1 is incorrect. The Westerlies are prevailing there are two jet streams: a subtropical jet stream centered
winds from the west toward the east in the middle at about 30 degrees latitude and a polar-front jet stream
latitudes between 30°N and 60°N, and 30°S and 60° S. whose position varies with the boundary between polar
They originate from the high-pressure areas in the horse and temperate air.
latitudes and tend towards the poles and steer extra Statement 2 is correct. Only tropical cyclones have
tropical cyclones in this general manner. eye. The eye is a region of mostly calm weather at the
Statement 2 is correct. Western disturbances which centre of tropical cyclones. A mature tropical cyclone
originate over the Mediterranean Sea are steered towards is characterised by the strong spirally circulating wind

Workbook 108
GEOGRAPHY

around the centre, called the eye. The diameter of the the stratosphere disperses, it can affect the shape of the
circulating system can vary between 150 and 250 km. jet stream as the cold air sinks from the stratosphere
Statement 3 is incorrect. The eye experiences the higher into the troposphere. It is this change in the jet stream
temperature and lower pressure as compared to the that causes our weather to change.
surroundings. The eye is a region of calm with subsiding Statement 3 is incorrect – Extreme cold, winter snow
air. Around the eye is the eye wall, where there is a strong events have all been connected to the surface effects of
spiralling ascent of air to greater height reaching the sudden stratospheric warming. Eg. Extreme cold events
tropopause. The wind reaches maximum velocity in this in the year 2009-10, 2013 as well as ‘beast from the east’
region, reaching as high as 250 km per hour. Torrential in 2018.
rain occurs here.
22. Solution: (c)
19. Solution: (d)
Exp) Option c is correct.
Exp) Option d is the correct answer.
Statement 1 is incorrect – The actual amount of water
Whether a given cloud will heat or cool the surface
depends on several factors, including the cloud’s altitude, vapor present in the atmosphere is known as absolute
its size, and the make-up of the particles that form the humidity. The percentage of moisture present in the
cloud. atmosphere as compared to its full capacity at a given
temperature is known as the relative humidity. With the
Statement 1 is incorrect: Low, thick clouds primarily
change of air temperature, the capacity to retain moisture
reflect solar radiation and cool the surface of the Earth.
increases or decreases and the relative humidity is also
Statement 2 is incorrect: High, thin clouds primarily affected.
transmit incoming solar radiation; at the same time,
they trap some of the outgoing infrared radiation emitted Statement 2 is correct – Condensation takes place: (i)
by the Earth and radiate it back downward, thereby when the temperature of the air is reduced to dew point
warming the surface of the Earth. with its volume remaining constant; (ii) when both
the volume and the temperature are reduced; (iii) when
moisture is added to the air through evaporation.
3.2. Weather
Statement 3 is correct – When a particular air mass
20. Solution: (c) is cooled, its water holding capacity decreases. Thus,
Exp) Option c is correct. with decreasing temperature the relative humidity of
Statement 1 is incorrect. Polar vortex is described as a air increases. If its temperature is further decreased to
whirling cone of low pressure over the poles. the dew point temperature, the air becomes saturated, i.e
Relative humidity becomes 100%.
Statement 2 is incorrect. Normally, when the vortex is
strong and healthy, it helps to keep a current of air (the jet 23. Solution: (d)
stream) travelling around the globe in almost a circular
path. This current keeps the cold air up north and warm Exp) Option d is correct.
air down south. When the vortex becomes weak there is Statement 1 is correct. Cyclone helps in balancing global
a lack of a strong low-pressure system, resulting in jet heat distribution. Cyclone maintains the temperature
stream losing the hold to keep it in line, and becoming balance between the poles and the equator.
wavy and all of a sudden, a river of cold air is pushed Statement 2 is correct.
down south.
1. Cyclone facilitates Ecological Succession in
Statement 3 is correct. The term vortex refers to ecosystem.
counterclockwise flow of air that keeps the colder air
2. For example, A cyclone flattens a deciduous forest
near the poles.
which may seem as an act of destruction but such
21. Solution: (c) disturbances are a natural and necessary part of
Exp) Option c is correct. ecosystem function.
3. The defoliation of mature canopy trees allows sunlight
Statement 1 is correct – Every year in winter, strong
westerly winds circle the pole high up in the stratosphere. to reach the previously dark understory, permitting
This is called the stratospheric polar vortex and it shade-intolerant species to proliferate.
circulates cold air high over the Arctic. 4. Thus, it helps the shade-tolerant trees to create a
In some years, the winds in the polar vortex temporarily canopy again.
weaken. The cold air then descends very rapidly in the 5. Such cycling of vegetation communities or succession
polar vortex and this causes the temperature in the promotes biodiversity by giving more species the
stratosphere to rise very rapidly, as much as 50°C over chance to occupy a given ecosystem and maintaining
only a few days; hence the term sudden stratospheric landscape mosaics of greater complexity.
warming. Statement 3 is correct. Cyclones help in the dispersal of
Statement 2 is correct – As the cold air from high up in seeds of plant species.

109 Workbook
GEOGRAPHY

24. Solution: (a) make “milk” in the esophagus which can be used to feed
Exp) Option a is correct chicks in the winter before the female arrives back from
fishing.
Statement 1 is correct. Thunderstorms and Tornadoes
are atmosphere’s adjustments where restless Statement 2 is correct. Certain animal species have
atmosphere returns to stable state. They are of short evolved an adaptation that allows them to weather long
duration, occurring over a small area but are violent. In stretches of time when food is scarce — they enter a state
Thunderstorms and Tornadoes, the potential and heat known as hibernation. The animal’s heart and breathing
energies are converted into kinetic energy. rates slow down, and its body temperature drops.
Depending on the species, days or even weeks may pass
Statement 2 is incorrect. Thunderstorms and Tornadoes
without the animal waking to drink, eat or relieve itself.
are both common in mid latitudes. In fact, Tornadoes
are severe thunderstorms having spiralling wind Statement 3 is correct. Large Tundra animals like Musk
descending like a trunk of an elephant with great force, Ox and Polar Bear, hairs are unique in that they’re hollow.
with very low pressure at the centre, causing massive These hollow hairs help to trap air close to the body
destruction on its way. where it keeps the animals warm.
Statement 4 is correct. Penguins breed during the
25. Solution: (d) depths of the Antarctic winter, so the chicks are large
Exp) Option d is correct. enough to become independent during the summer
Statement 1 is correct – The Easterly jet streams are held abundance of food.
responsible for the burst of monsoon in India as formation
28. Solution: (a)
of these streams results in the reversal of upper air
circulation patterns from high pressure to low pressure. Exp) Option a is correct.
Statement 2 is correct – The easterly jet stream steers Option a is incorrect. The Koeppen’s scheme of Climatic
the tropical depressions into India. These depressions classification is based upon annual and monthly values
play a significant role in the distribution of monsoon of temperature and precipitation, not only precipitation.
rainfall over the Indian subcontinent. Option b is correct. The scheme accepts the native
Statement 3 is correct – The tracks of these depressions vegetation as the best expression of totality of a climate.
steered by easterly jet streams are the areas of Therefore, many of the climatic boundaries are based
highest rainfall in India. The frequency at which these upon vegetation. Koeppen has expressed the view that
depressions visit India, their direction and intensity, all go the effectiveness of precipitation in vegetation growth
a long way in determining the rainfall pattern during the depends not only upon precipitation, but also upon the
southwest monsoon period. intensity of evaporation and transpiration.
Option c and d are correct. The scheme identifies five
26. Solution: (d) major climatic types. Eg. Tropical Climates, where mean
Exp) Option d is correct. monthly temperature throughout the year is above
Statement 1 is incorrect –Cyclones are officially named by 18-degree Celsius.
one of the World Meteorological Organization’s (WMO) Each type is further sub-divided into sub-types on the
warning centres based across the globe. The WMO/ basis of seasonal variations in the distributional pattern
United Nations Economic and Social Commission for of rainfall and temperature. The small letters demote:
Asia and Pacific Panel on tropical cyclones names the f (sufficient precipitation), m (rain forest despite a dry
cyclone for Bay of Bengal and Arabian sea region since monsoon season), w (dry season in winter), h (dry and
2000. hot), c (less than four months with mean temperature
Statement 2 is incorrect – Names of cyclones in the over 10°C), and g (Gangetic plain).
regions are suggested by India along with 12 other Both, the Gangetic plains and most of the North
countries. Other nations that are part of this network East India, are classified under ‘Cwg’ climatic region:
include Bangladesh, Maldives, Myanmar, Oman, Iran, Monsoon with dry winter.22)
Pakistan, Sri Lanka, Qatar, Thailand, Saudi Arabia, United
29. Solution: (b)
Arab Emirates and Yemen. Thus, it is not just the Bay of
Bengal littoral countries. Exp) Option b is correct.
Statement 3 is correct – The name ‘Tauktae’ has been Point 1 is correct. At the equator, the Coriolis force is
suggested by Myanmar. The name comes from the zero. It increases as we move away from equator.
Burmese language and it means a ‘gecko’ or a very hitch- The Coriolis force acts perpendicular to the pressure
pitched lizard. gradient force. The pressure gradient force is perpendicular
to an isobar. At the equator the wind blows perpendicular
27. Solution: (d) to the isobars. Thus, the Coriolis force is zero at equator
Exp) Option d is correct. and it increases moving towards poles.
Statement 1 is correct. As part of the adaptations to Point 2 is correct. At the equator, sun rises and sets in
survive the harsh climate male Emperor Penguins can a vertical path which makes the period of refracted light

Workbook 110
GEOGRAPHY

short. But in higher latitudes, the sun rises and sets in an 32. Solution: (d)
oblique path and the period of refracted light is longer. Exp) Option d is correct.
Period of dawn and twilight increases while moving
from equator to poles. Statement 1 is incorrect. Mist contains more moisture
than the fog. Because, in mist each nucleus contains a
Point 3 is correct. Length of the day almost remains thicker layer of moisture than the fog.
constant on the equator throughout the year, while
there is a drastic change in the length of the day on Statement 2 is correct. Mists are frequent over
poles. In the northern hemisphere in summer, there is mountains as the rising warm air up the slopes meet
light for the whole day in midsummer on 21st June and a cold surface. Fogs are drier than mist and they are
in winter, the sun never rises for the whole day. There is prevalent where warm currents of air come in contact
a larger variation in day’s length while moving from with cold currents. Fogs are mini clouds in which
equator towards poles. condensation takes place around nuclei provided by the
dust, smoke, and the salt particles.
Point 4 is incorrect. Insolation decreases with increase
in latitude. The amount of insolation received depends Statement 3 is correct. Fog has visibility less than 1 Km
upon the angle of inclination of the rays. This depends and mist has visibility of 1km and above. Because of the
on the latitude of a place. The higher the latitude the fog and mist, the visibility becomes poor to zero.
less is the angle they make with the surface of the earth
resulting in slant sun rays. The area covered by vertical rays 33. Solution: (c)
is always less than the slant rays. If more area is covered, Exp) Option c is correct.
the energy gets distributed and the net energy received Option a is incorrect. In the mesosphere, temperature
per unit area decreases. Thus, the amount of insolation decreases with the increase in altitude. In this layer, once
received decreases while moving form equator towards again, temperature starts decreasing with the increase in
poles. altitude and reaches up to minus 100° C at the height of
80 km.
30. Solution: (c)
Option b is incorrect. The ionosphere is located
Exp) Option c is correct.
between 80 and 400 km above the mesopause. It contains
Option 1 is correct – When two different air masses electrically charged particles known as ions, and hence, it
meet, the boundary zone between them is called a is known as ionosphere. Radio waves transmitted from
front. The process of formation of the fronts is known as the earth are reflected back to the earth by this layer.
frontogenesis.
Option c is correct. The stratosphere is found above the
Option 2 is correct – The fronts are more prominent tropopause and extends up to a height of 50 km. One
in middle latitudes. They are uncommon in lower and important feature of the stratosphere is that it contains
polar latitudes. the ozone layer. This layer absorbs ultra-violet radiation
Option 3 is correct – The fronts are characterized by and shields life on the earth from intense, harmful form
a steep gradient in temperature. They bring abrupt of energy.
changes in temperature and cause the air to rise to form Option d is incorrect. The zone separating the
clouds and cause precipitation. troposphere from stratosphere is known as the
Option 4 is incorrect – When a sudden change in tropopause. The temperature here is nearly constant, and
temperature occurs through a front, the pressure also hence, it is called the tropopause.
changes steeply.
Option 5 is correct – A front experiences wind shift 34. Solution: (b)
since the wind motion is a function of pressure gradient Exp) Option b is correct.
and Coriolis force. Statement 1 is correct. High relative humidity is
necessary for the formation of dew. It means the current
31. Solution: (b)
volume of air is near to saturation.
Exp) Option b is correct.
Statement 2 is incorrect. Calm air is necessary for
Statement 1 is incorrect. Relief rain, also known as dew formation as it helps to maintain the volume of
orographic rain, results when the saturated air mass air constant in temperature and moisture, with less
comes across a mountain, it is forced to ascend and as it disturbances by outside air flow.
rises, it expands; the temperature falls, and the moisture
is condensed. The chief characteristic of this sort of rain Statement 3 is correct. Cold and long nights help to
is that the windward slopes receive greater rainfall. The reduce the temperature and clear sky results in fast
area situated on the leeward side gets less rainfall and is convection of warm air from surface to the upper
known as the rain-shadow area. atmospheres. This leads to cold and moisture laden air to
subside near the earth surface.
Statement 2 is correct. After giving rain on the windward
side, when these winds reach the other slope, they Statement 4 is incorrect. For the formation of dew, the
descend, and their temperature rises. This results in dew point needs to be above the freezing point. If the
increased capacity of winds to take in moisture and dew point is at or below the freezing point then it results
hence, these leeward slopes remain rainless and dry. in formation of frost.

111 Workbook
GEOGRAPHY

35. Solution: (a) the entirety of Antarctica. At lower latitudes, permafrost


Exp) Option a is correct. is found at high altitude locations such as the Alps and
the Tibetan plateau. Thus, in Northern Hemisphere,
Statement 1 is correct. In regions where a pronounced
Permafrost is not confined to higher latitudes only.
low-level inversion is present, convective clouds cannot
grow high enough to produce showers. And, at the Statement 4 is correct - When permafrost thaws,
same time, visibility may be greatly reduced below microbes start decomposing this carbon matter,
the inversion by the accumulation of dust and smoke releasing greenhouse gases like methane and carbon
particles. dioxide. Researchers have estimated that for every
1-degree Celsius rise in average temperature, permafrost
Statement 2 is incorrect. Convection clouds cannot move
regions could release greenhouse gases to the tune of
high upwards to produce showers. So, there is less rainfall
4-6 years of emissions from coal, oil, and natural gas —
during temperature inversion.
becoming a major factor of climate change in themselves.
Statement 3 is incorrect. The air trapped under the
inversion includes the moisture, smoke and pollutants 39. Solution: (d)
released into the air mass. The air quality under Exp) Option d is correct.
an inversion layer deteriorates as the pollutants
accumulate. Statement 1 is incorrect. When two different air masses
meet, the boundary zone between them is called a front.
36. Solution: (b) When the cold air moves towards the warm air mass, its
Exp) Option b is correct. contact zone is called the cold front, whereas if the warm
air mass moves towards the cold air mass, the contact
Westerlies are called as Roaring Forties, Furious Fifties, zone is a warm front.
and Shrieking sixties in the southern hemisphere because
of the degree of storminess in the latitudes in which they Statement 2 is incorrect. If an air mass is fully lifted
flow. They flow with storminess because large expanse above the land surface, it is called the occluded front.
of oceans does not obstruct their flow and so, they When the front remains stationary, it is called a stationary
flow with a much greater force and regularity in the front.
Southern hemisphere. Westerlies are more variable in the 40. Solution: (d)
northern hemisphere.
Exp) Option d is correct.
37. Solution: (c) Statement 1 is incorrect. If we increase the temperature
Exp) Option c is correct. of the given volume of air, it will result in evaporation.
Statement 1 is incorrect. Anticyclones can be formed Evaporation is a process by which water is transformed
both in summer and winter. from liquid to gaseous state.
Statement 2 is correct. Winds in Anti-cyclone blow Statement 2 is correct. If we add more moisture to
outwards and also subjected to deflection but they blow the given volume of air, through evaporation, it will
clockwise in the northern hemisphere and anticlockwise lead to increase in density and pressure resulting in
in southern hemisphere due to high pressure at the center. condensation.
Statement 3 is correct. Anticyclone normally herald fine Statement 3 is correct. Condensation takes place when
weather because of gentle pressure gradients. Skies are either the temperature is reduced to dew point, with
clear, the air is calm and temperatures are high in summer its volume remaining constant or reducing the volume
but cold in winter. of air, so relative humidity increases. So, if we reduce
both volume of air and its temperature, then it will lead to
38. Solution: (b) condensation of the water vapour.
Exp) Option b is correct. Statement 4 is correct. Condensation also takes place
Statement 1 is correct – Permafrost is a ground that when the moist air comes in contact with some colder
remains completely frozen at 0 degrees Celsius or below object, because it will result in lowering the temperature
for at least two years. It is defined solely based on of the given volume of air and temperature of the air gets
temperature and duration. closer to the dew point.
Statement 2 is correct – Beneath its surface, permafrost 41. Solution: (c)
contains large quantities of organic leftover from
thousands of years prior — dead remains of plants, Exp) Option c is correct.
animals, and microorganisms that got frozen before they Statement 1 is correct: The Inter Tropical Convergence
could rot. It also holds a massive trove of pathogens. Zone (ITCZ) is a low-pressure zone located at the equator
Statement 3 is incorrect - These grounds are known to be where trade winds from two hemispheres converge, and
below 22 per cent of the land surface on Earth, mostly in so, it is a zone where air tends to ascend.
polar zones and regions with high mountains. They are Statement 2 is correct: This convergence zone lies more
spread across 55 per cent of the landmass in Russia and or less parallel to the equator but moves north or south
Canada, 85 per cent in the US state of Alaska, and possibly with the apparent movement of the sun.

Workbook 112
GEOGRAPHY

42. Solution: (d) approximately 80% of the mass of the atmosphere of


Exp) Option d is correct. the earth.
Statement 1 is correct. The temperature of a place Statement 3 is correct. The majority of atmospheric
directly depends on the insolation received. Insolation water vapour or moisture is found in the troposphere.
varies according to the latitude and temperature of So, it is the wettest layer of the atmosphere.
a place directly depends on the insolation received. Statement 4 is correct. Its average height is 13km while
Higher latitude receives slant sun rays; thus, energy gets it is 8km near the poles and about 18km at the equator
distributed and the net energy received per unit area due to the transportation of heat to the greatest height by
decreases compared to vertical rays in lower latitude. strong convectional current.
Statement 2 is correct. Passage of air masses also 45. Solution: (c)
affects the temperature. The places, which come under
Exp) Option c is correct.
the influence of warm air-masses experience higher
temperature and vice-versa. Similarly, the places located Statement 1 is correct. In this layer temperature rises as
on the coast where the warm ocean currents flow record one move upwards through the stratosphere. Because of
higher temperature than the places located on the coast this temperature stratification, there is little convection
where the cold currents flow. and mixing in the stratosphere, so the layers of air there
are quite stable. So due to lack of vertical convection in
Statement 3 is correct. Atmosphere is indirectly heated the stratosphere, material like CFCs, aerosols etc. can
by terrestrial radiation from below. Therefore, the stay there for long times.
places near the sea-level record higher temperature than
the places situated at higher elevations. It can also be Statement 2 is correct. Ozone is relatively abundant in
understood with the phenomena of normal lapse rate the upper stratosphere. It heats this layer as it absorbs
(temperature decreases with increase in elevation). energy from the incoming ultra-violet radiation from
the sun. Thus, the temperature rises as one move
Statement 4 is correct. Compared to land, the sea gets upwards through the Stratosphere.
heated slowly and loses heat slowly. Land heats up
and cools down quickly. Therefore, the variation in 46. Solution: (a)
temperature over the sea is less as compared to land. Exp) Option a is correct.
Also, there is moderating effect of land-sea breeze of
places which are situated near the sea which affects the Statement 1 is correct. La Niña is characterized by lower-
temperature. than-normal air pressure over the western Pacific. During
La Nina events, trade winds are even stronger than usual,
Statement 5 is correct. Temperature is also dependent on pushing more warm water toward Asia. Off the west
local factors. Such as lake in an arid region cools off the coast of the Americas, upwelling increases, bringing cold,
heat and similarly temperature on the equator ward side nutrient-rich water to the surface. These cold waters in
of the mountains is generally higher as compared to their the Pacific push the jet stream northward. This tends to
pole ward side. lead to drought in the southern U.S. and heavy rains and
43. Solution: (b) flooding in Canada.
Exp) Option b is correct. Statement 2 is incorrect. La Nina Modoki is associated
with colder central tropical Pacific and warming in
Statement 1 is incorrect. Water vapour decreases from the eastern and western Pacific. La Nina Modoki is the
equator towards poles and also decreases with altitude. counter part of El Nino Modoki
Thus, in warm and wet tropics, it may account for 4% of
air by volume and while in dry and cold areas of desert 47. Solution: (c)
and polar regions, it may be less than 1% of the air. Exp) Option c is correct.
Statement 2 is correct. The concentration of dust Statement 1 is correct. MJO can be defined as an
particles is high in subtropical and temperate regions eastward moving ‘pulse’ of clouds, rainfall, winds and
due to dry winds in comparison to equatorial and polar pressure near the equator that typically recurs every
regions. 30 to 60 days. It’s a traversing phenomenon and is most
Statement 3 is incorrect. Nitrogen constitutes the prominent over the Indian and Pacific Oceans.
maximum amount of gas followed by Oxygen, Argon, Statement 2 is incorrect. When MJO lies over the Indian
Carbon dioxide etc. Ocean during the Monsoon season, it brings good rainfall
44. Solution: (d) over the Indian subcontinent. On the other hand, when
it witnesses a longer cycle and stays over the Pacific
Exp) Option d is correct. Ocean, MJO brings bad news for the Indian Monsoon.
Statement 1 is correct. Commercial jet aircraft fly in Statement 3 is correct. As it recurs every 30 to 60 days,
the stratosphere to avoid the turbulence present in the there can be multiple MJO events within a season, and
troposphere as it contains lots of water vapour . so the MJO is best described as intraseasonal tropical
Statement 2 is correct. The troposphere contains climate variability (i.e., varies on a week-to-week basis).

113 Workbook
GEOGRAPHY

48. Solution: (d) 51. Solution: (c)


Exp) Option d is correct. Exp) Option c is correct.
Statement 1 is incorrect: Heat budget is not constant over Pair 1 is correctly matched: Conduction takes place
the entire earth. It depends on many factors and based when two bodies of unequal temperature are in contact
on those there are areas of energy surplus and energy with one another there is a flow of energy from the warmer
deficit. This difference in heat budget gives rise to various to cooler body. The transfer of heat continues until both
climatic phenomena like ocean current, atmospheric the bodies attain the same temperature or the contact is
circulations, etc. broken.
Statement 2 is incorrect: Atmosphere is heated up by Pair 2 is incorrectly matched: The transfer of heat
4 methods. Of the 65% of incident solar energy 14% through horizontal movement of air is called advection.
are directly absorbed by atmosphere. This is in the In tropical regions particularly in northern India during
form of short-wave radiation. On the contrary, the summer season local winds called ‘loo’ is the outcome of
terrestrial radiation heats the lower atmosphere by advection process.
long wave radiation and it accounts for 6%. Other two Pair 3 is incorrectly matched: The process of vertical
important components of atmospheric heat budget are heating of the atmosphere is known as convection. Air
convection and evaporation which accounts for 9% and in contact with the earth rises vertically on heating in the
19% respectively. Thus, along with short wave and long form of currents and further transmits the heat of the
wave radiation atmosphere is also heated by conduction, atmosphere. It is confined only to the troposphere.
convection and advection.
52. Solution: (c)
49. Solution: (d) Exp) Option c is correct.
Exp) Option d is correct. Statement 1 is correct: Net radiation is the difference
It is observed that normally temperature decreases with between all incoming solar energy and outgoing
increase in elevation. This is known as normal lapse rate terrestrial energy. It is an important measure for
which averages around 6.5 Celsius every 1 kilometer. categorization of earth on the basis of heat energy.
But in some case this temperature starts to increase, this Statement 2 is correct: The surplus heat energy from the
is called as inversion of temperature. tropics is redistributed poleward through atmospheric
Statement 1 is correct. Smoke and dust is accumulated circulation and ocean currents. Thus, sensible heat gets
in lower strata of the atmosphere below the inversion transferred from energy surplus to energy deficit areas.
layer. This creates problem of air pollution as they spread 53. Solution: (c)
horizontally and fills the lower strata of the atmosphere.
Exp) Option c is correct.
Statement 2 is correct. Creation of dense fog especially
Statement 1 is incorrect – The temperature decreases
during winter is also a common phenomenon. This
with altitude till Troposphere. It remains nearly constant
inversion last only for a few hours as the sun begins to
in Tropopause. Then it starts increasing in stratosphere.
warm the earth. It is a potential environment hazard as
Similarly, temperature decreases with altitude in
it harms crops and decreases visibility.
Mesosphere and decreases in Thermosphere.
Statement 3 is correct. An inversion acts as a cap on Statement 2 is incorrect – The air temperature at the
the upward movement of air from below thus creating tropopause is about minus 80oC over the equator and
conditions of stability in the atmosphere. Also due to cap about minus 45oC over the poles.
on upward movement moisture supply remains restricted
because of which also we see stable climatic conditions. 54. Solution: (a)
50. Solution: (a) Exp) Option a is correct.
Exp) Option a is correct Statement 1 is correct – The topographical variations are
the major factors modifying the distribution of insolation.
Statement 1 is correct. Farther from the equator, water Variability in elevation, surface orientation (slope and
vapor concentrations are high in the hemisphere aspect), and obstruction by surrounding topographic
experiencing summer and low in the one experiencing features create strong local gradients of insolation. With
winter because warm air is less dense than cold air, thus regards to factors affecting temperature in mountain
there is more room for water vapour in warm summer as ranges, the steep slope experiences a much more rapid
compared with cold winter air. change in temperature than a gentle slope.
Statement 2 is incorrect. Water vapor amounts over Statement 2 is incorrect – Those mountain ranges which
land areas decrease more in winter months than adjacent have an east-west alignment like the Alps show higher
ocean areas do. This is largely because air temperatures temperatures on the south facing slope, the sunny slope.
over land drop more in the winter than temperatures The sheltered north-facing slope has lower temperature.
over the ocean. Water vapor condenses more rapidly in The greater insolation of the south facing slope has a more
colder air. flourishing vegetative cover.

Workbook 114
GEOGRAPHY

55. Solution: (c) Statement 1 is incorrect. Westerlies form the part of


Exp) Option c is correct. Ferrel cell circulation. They are prevailing winds that
blow from the west at mid-latitudes. In the Ferrel cell,
Statement 1 is correct. Light soils reflect more heat than air flows poleward and eastward near the surface and
darker ones. Such differences give rise to slight variations equatorward and westward at higher altitudes; this
in the temperature of the region. Regions of light-coloured movement is the reverse of the airflow in the Hadley cell.
soil are thus comparatively at lower temperature than
Statement 2 is incorrect. Westerlies are on-shore winds on
those of darker soils.
the western coasts. Thus, they bring much precipitation
Statement 2 is correct. Dry soils like sands are very to the western coasts of the continents.
sensitive to temperature. They heat up and cool down
Statement 3 is incorrect. In the southern hemisphere
rapidly, hence the extreme temperature variations where there is a large expanse of ocean and few
in desert areas. Wet soils like clay retain much more landmasses to slow winds, from 40oS to 60oS, Westerlies
moisture, thus they do not warm up or cool down rapidly blow with much greater force and regularity throughout
due to the higher specific heat capacity of water. the year. The strongest westerlies are also referred to as
56. Solution: (b) Roaring forties, Furious fifties and Shrieking sixties.
Exp) Option b is correct. 59. Solution: (d)
Statement 1 is incorrect. Ferrel’s Law of Deflection tells Exp) Option d is correct.
us that the effect of rotation of the earth tends to deflect Statement 1 is incorrect. Land and sea breezes are
the direction of winds. caused by differential heating of land and sea. These are
Statement 2 is correct. In the Northern hemisphere diurnal (daily) rhythms, while Monsoon is a seasonal
winds are deflected to the right and in the southern rhythm.
hemisphere to their left. For this reason, winds Statement 2 is incorrect. Land breeze blows out from
blowing out from the subtropical high-pressure belt in land to sea. Fishermen in the tropics often take advantage
the Northern hemisphere towards the equatorial low of the land breeze and sail out with it. They return the
become north east trade winds and those in the southern next morning with the in-coming sea breeze.
hemisphere become the south east trade winds.
Statement 3 is incorrect. The Coriolis force is absent 60. Solution: (b)
along the equator but increases progressively towards Exp) Option b is correct.
the Poles. As the latitude at which horizontally and freely Statement 1 is correct. In mountainous regions, during
moving objects are located decreases, the twisting of the the night the slopes get cooled and the dense air
underlying Earth’s surface due to the planet’s rotation descends into the valley as the mountain wind. The cool
decreases. That is, the Coriolis effect decreases as the air, of the high plateaus and ice fields draining into the
latitude decreases. It is maximum at the poles and valley is called katabatic wind.
absent at the equator. While, during the day the slopes get heated up and air
57. Solution: (a) moves upslope and to fill the resulting gap the air from
the valley blows up the valley. This wind is known as the
Exp) Option a is correct valley breeze.
The pattern of planetary winds largely depends on Statement 2 is incorrect. Katabatic winds occur during
1. latitudinal variation of atmospheric heating. the night when the slopes get cooled.
Statement 1 is correct.
Statement 3 is correct. When a katabatic wind is warmed
2. emergence of pressure belts; by compression during its descent into denser air, it is
3. the migration of belts following apparent path of the called a fohn. While descending, the wind experiences
sun. Statement 2 is correct. an increase in pressure and temperature. The air is
4. the distribution of continents and oceans; compressed and gets warm. It may raise the temperature
5. the rotation of earth. in small time.
Statement 3 is incorrect. Revolution of earth is linked to Fohn is experienced in the Alps valley particularly in
emergence of day and night phenomena. Statement 4 is Switzerland.
incorrect. The pattern of the movement of the planetary
61. Solution: (c)
winds is called the general circulation of the atmosphere.
The general circulation of the atmosphere affects ocean Exp) Option c is correct.
water circulation and sets in motion the ocean water Statement 1 is correct. The air with distinctive
circulation which influences the earth’s climate. Pattern characteristics in terms of temperature and humidity
of Planetary wind is not dependent on ocean currents. is called an air mass. It is defined as a large body of air
having little horizontal variation in temperature and
58. Solution: (d) moisture. Air masses have fairly uniform temperature
Exp) Option d is correct. and moisture content.

115 Workbook
GEOGRAPHY

Statement 2 is correct. When the air remains over Option b is correct. Orographic/Relief rainfall-When
a homogenous area for a sufficiently longer time, it the saturated air mass comes across a mountain,
acquires the characteristics of the area. The homogenous it is forced to ascend and as it rises, it expands; the
regions can be the vast ocean surface or vast plains. The temperature falls, and the moisture is condensed. The
homogenous surfaces, over which air masses form, are chief characteristic of this sort of rain is that the windward
called the source regions. Such source regions determine slopes receive greater rainfall and leeward slopes remain
the properties of the air masses. rainless and dry.
Option c and d are incorrect. Cyclonic rainfall is
62. Solution: (a) independent of relief or convection. It is purely associated
Exp) Option a is correct. with cyclonic activity whether in the temperate regions or
Pair 1 is correct. The ideal conditions for formation of tropical regions. It is due to convergence of two different
air masses with different temperatures and other physical
dew are clear sky, calm air, high relative humidity, and cold
properties.
and long nights. For the formation of dew, it is necessary
that the dew point is above the freezing point. 65. Solution: (a)
Pair 2 is correct. Frost forms on cold surfaces when Exp) Option a is correct
condensation takes place below freezing point i.e., the Tropical cyclones originate and intensify over warm
dew point is at or below the freezing point. The ideal tropical oceans. The conditions favourable for the
conditions for the formation of white frost are the same formation and intensification of tropical storms are:
as those for the formation of dew, except that the air 1. Large sea surface with temperature higher than 27° C;
temperature must be at or below the freezing point. 2. Presence of the Coriolis force;
Pair 3 is incorrect. When the temperature of an air mass 3. Small variations in the vertical wind speed;
containing a large quantity of water vapour falls all of a 4. A pre-existing weak low-pressure area or low-level-
sudden, condensation takes place within itself on fine cyclonic circulation;
dust particles. The fog is a cloud with its base at or very 5. Upper divergence above the sea level system.
near to the ground. 6. A high humidity level in the middle troposphere
63. Solution: (b) from 3 to 6 km (1.8 to 3.7 miles) in height is more
conducive to the production of deep cumulonimbus
Exp) Option b is correct. convection and therefore to stronger vertical coupling
Option a is incorrect. Cirrus cloud look fibrous and in the troposphere.
appears like wisps in the blue sky. It is often called ‘mares
tails’. It indicates fair weather and often give brilliant 66. Solution: (c)
sunset. Exp) Option c is correct.
Option b is correct. It is a very low cloud, uniformly grey Statement 1 is incorrect: If a storm is dubbed a
and thick, which appears like a low ceiling or highland bomb cyclone or “bombogenesis”, to refer to a rapidly
fog. It brings dull weather with light drizzle. It reduces intensifying area of low pressure.
the visibility of aircraft and is thus a danger. The term bombogenesis comes from the merging of two
Option c is incorrect. Cumulus cloud is a vertical cloud words: bomb and cyclogenesis. All storms are cyclones,
with a rounded top and horizontal base, typical of humid and genesis means the creation or beginning. In this case,
bomb refers to explosive development. Altogether the
tropical regions, associated with up-rising convectional
term means explosive storm strengthening.
currents. It is great white globular mass, might look grey
against the sun. But it is a fair-weather cloud. Statement 2 is correct: Temperate Cyclones are
depressions with low pressure at the centre. These are
Option d is incorrect. Cumulonimbus cloud is an generally steered by deep westerly winds in a general west
overgrown cumulus cloud, extending from a base of 2000 to east motion across both the Northern and Southern
feet to over 30,000 feet. It is a black and white globular hemispheres of the Earth.
mass. It is also referred to as a thunder cloud and brings
Statement 3 is incorrect: During a cyclone, winds blow
convectional rain accompanied by lightning and inwards into regions of low pressure in the centre. Winds
thunder. circulate in the anticlockwise direction in the northern
64. Solution: (b) hemisphere and clockwise in the southern hemisphere.
Exp) Option b is correct. 67. Solution: (b)
Option a is incorrect Convectional rainfall. The air on Exp) Option b is correct.
being heated, becomes light and rises up in convection Statement 1 is incorrect – Cyclones are depressions, they
currents. As it rises, it expands and loses heat and are areas with low pressure at the centre. The approach
consequently, condensation takes place and cumulous of the cyclone is characterised by a fall in barometric
clouds are formed. With thunder and lightning, heavy reading (i.e. low pressure), dull sky, oppressive air and
rainfall takes place but this does not last long. strong winds.

Workbook 116
GEOGRAPHY

Statement 2 is correct – Anticyclones are areas with high humidity is called an airmass. It is defined as a large body
pressure centres. Pressure gradient is gentle and winds are of air having little horizontal variation in temperature
light. They are characterized by clear skies and calm air. and moisture.
The temperature is high in summer and cool in winter. Statement 2 is correct. When two different air masses
68. Solution: (b) meet, the boundary zone between them is called a front.
When the cold air movestowards the warm air mass, its
Exp) Option b is correct. contact zone is called the cold front, whereas if the warm
Sirocco is a hot. dry dusty wind which originates in air mass moves towards the cold air mass, the contact
the Sahara Desert. Though it may occur at any time zone is a warm front.
of the year, it is most frequent in spring and normally Statement 3 is Incorrect. The fronts occur in middle
lasts for only a few days. The Sirocco blows outwards in latitudes and are characterised by steep gradient in
a southerly direction from the desert interiors into the temperature and pressure. They bring abrupt changes in
cooler Mediterranean Sea. It is usually associated with temperature and cause the air to rise to form clouds and
depressions from the Atlantic passing from the coast cause precipitation.
eastwards inland. After crossing the Mediterranean Sea,
the Sirocco is slightly cooled by the absorption of the 71. Solution: (b)
water vapour. Even then, it is still hot and dry with a Exp) Option b is correct.
temperature of over 105°F. Its scorching breath withers
vegetation and crops. Statement 1 is correct. Tropical cyclones originate and
intensify over warm tropical oceans. The conditions
Option a is Incorrect. The Fohn wind is experienced in the favourable for the formation and intensification of
valleys of the northern Alps, particularly in Switzerland in tropical storms are: (i) Large sea surface with temperature
spring. It is dry winds experienced on the leeward side higher than 27° C; (ii) Presence of the Coriolis force; (iii)
of mountains when descending air becomes compressed Small variations in the vertical wind speed; (iv) A pre-
with increased pressure. existing weak low-pressure area or low-level-cyclonic
Option c is Incorrect. Mistral is a cold wind from the circulation; (v) Upper divergence above the sea level
north, rushing down the Rhone valley in violent gusts system.
between 40 and 80 miles per hour.In winter when the Statement 2 is Incorrect. The place where a tropical
Mistral is most frequent the temperature of the wind may cyclone crosses the coast is called the landfall of the
be below freezing-point, though the sky may be clear and cyclone. On reaching the land the moisture supply is cut
cloudless. off and the storm dissipates. The energy that intensifies
Option d is Incorrect. On the eastern slopes of the the storm, comes from the condensation process in the
Rockies in Canada and U.S.A. a local wind, similar to the towering cumulonimbus clouds, surrounding the centre
Fohn in Switzerland, called the Chinook, comes in a south of the storm. With continuous supply of moisture from
westerly direction to the Prairies and has a considerable the sea, the storm is further strengthened.
effect on the local pastures. Statement 3 is correct. A mature tropical cyclone is
characterised by the strong spirally circulating wind
69. Solution: (b) around the centre, called the eye. The diameter of the
Exp) Option b is correct. circulating system can vary between 150 and 250 km.
Statement 1 is Incorrect. During the day the land heats 72. Solution: (b)
up faster and becomes warmer than the sea. Therefore,
over the land the air rises giving rise to a low-pressure Exp) Option b is correct.
area, whereas the sea is relatively cool and the pressure Statement 1 is correct. Thunderstorms are of short
over sea is relatively high. Thus, pressure gradient from duration, occurring over a small area but are violent.
sea to land is created and the wind blows from the sea to Thunderstorms are caused by intense convection on moist
the land as the sea breeze. hot days.
Statement 2 is correct. During the night the slopes get Statement 2 is Incorrect. A thunderstorm is a well-grown
cooled and the dense air descends into the valley as the cumulonimbus cloud producing thunder and lightning.
mountain wind. The cool air, of the high plateaus and ice When the clouds extend to heights where sub-zero
fields draining into the valley is called katabatic wind. temperature prevails, hails are formed and they come
down as hailstorm.
70. Solution: (a) Statement 3 is correct. From severe thunderstorms
Exp) Option a is correct. sometimes spiralling wind descends like a trunk of an
Statement 1 is correct. When the air remains over a elephant with great force, with very low pressure at the
homogenous area for a sufficiently longer time, it acquires centre, causing massive destruction on its way. Such a
the characteristics of the area. The homogenous regions phenomenon is called a tornado.
can be the vast ocean surface or vast plains. The air with Statement 4 is Incorrect. Tornadoes generally occur in
distinctive characteristics in terms of temperature and middle latitudes.Most tornadoes are found in the Great

117 Workbook
GEOGRAPHY

Plains of the central United States – an ideal environment


for the formation of severe thunderstorms

73. Solution: (c)


Exp) Option c is correct.
Pair 1 is correct – The, air on being heated, becomes light
and rises up in convection currents. As it rises, it expands
and loses heat and consequently,condensation takes place
and cumulousclouds are formed. With thunder and
lightning, heavy rainfall takes place but this does not last
long. Such rain is common in the summer or in the hotter
part of the day. It is very commonin the equatorial regions
Statement 2 is incorrect: Revolution of Earth around
and interior parts of the continents, particularly in the Sun is responsible for the change in seasons, not for the
northern hemisphere. phenomena of Twilight.
Pair2 is Incorrect –Orographic precipitation occurs 75. Solution: (a)
when warm, humid air strikes an orographic barrier (a Exp) Option a is correct
mountain range).When the saturated air mass comes
The rate of decrease of temperature with height is
across a barrier, it is forced to ascend and as it rises, it termed as the normal lapse rate. It is 6.5°C per 1,000 m.
expands;leading to adiabatic cooling. Sothe temperature The lapse rate of non-rising air – commonly referred to
falls, and themoisture is condensed. The surplus moisture as the normal, or Environmental, Lapse Rate (ELR) – is
falls as orographic rainfall along the windward slopes. highly variable, being affected by radiation, convection,
and condensation.
Pair 3 is correct – Cyclonic Rainfall is convectional
Statement 1 is correct. The insolation received by the
rainfall on a large scale. The precipitation in a tropical earth is in short waves forms and heats up its surface. The
cyclone is of convectional type while that in a temperate earth after being heated itself becomes a radiating body
cyclone is because of frontal activity. In tropical cyclone, the and it radiates energy to the atmosphere in long wave
zone of rising winds leads to formation of cumulonimbus form. This energy heats up the atmosphere from below.
This process is known as terrestrial radiation. The long
clouds and bring torrential rains. Whereas in temperate
wave radiation is absorbed by the atmospheric gases
areas, because of meeting of fronts, precipitation occurs. particularly by carbon dioxide and the other greenhouse
gases. Thus, the atmosphere is indirectly heated by the
74. Solution: (c) earth’s radiation.
Exp) Option c is correct. Statement 2 is correct. With increase in elevation, the
Twilight is a time between day and night when there is atmospheric pressure falls. Fall in pressure implies that
the temperature also falls as the pressure is directly
light outside, but the Sun is below the horizon. Morning proportional to temperature and vice versa.
twilight is called Dawn, and evening twilight is called
Statement 3 is incorrect. With increase in elevation, the
Dusk. concentration of greenhouse gases decreases and not
Statements 1, 3 and 4 are correct: Twilight occurs when increases (Water vapour and carbon dioxide fall sharply
the sun rays fall at angle below the horizon, and the light with elevation). Hence the heat absorption capacity of
atmosphere will also decrease.
hits the upper layers of the atmosphere. The atmosphere
in turn reflects and scatter this light in a diffuse manner 76. Solution: (b)
onto the lower layers and the Earth surface. For this to Exp) Option b is correct
happen, the Earth has to rotate on its axis, in order for The amount of Water Vapour present in the air is known as
the sun’s rays to hit at different angles at different times humidity. It is expressed quantitatively in different ways.
to create the phenomenon of day and night. For example, Pair 1 is correct. The actual amount of the water vapour
in the mornings and evenings they hit at an angle below present in the atmosphere is known as the absolute
humidity. It is the weight of water vapour per unit volume
the horizon, whereas at midday they are directly above the
of air and is expressed in terms of grams per cubic metre.
head. The tilt of Earth on its axis, intensifies the angle of
Pair 2 is incorrect. Relative humidity is the ratio of the
inclination of the sun’s rays. The Spherical shape of the amount of water vapor actually in a volume occupied by
earth also ensures that the sun’s rays fall at an angle on air to the amount the space could contain at saturation.
the earth’s surface. The percentage of moisture present in the atmosphere as

Workbook 118
GEOGRAPHY

compared to its full capacity at a given temperature is hot with continuous insolation, while one half would be
known as the relative humidity. The relative humidity of frozen due to no insolation will occur only if the earth
an air at saturation point is hundred percent. stopped rotating on its axis (not because of lack of tilted
Pair 3 is correct. Dew point is the temperature at which axis). If the earth stopped rotating, there would be no
the air becomes fully saturated (100 percent relative day and night for all places on earth. Then one half of the
humidity). It is dependent on only the amount of earth will continuously face the sun, and one half will be
moisture in the air. in perpetual darkness.
Statement b is incorrect: All the places on different
77. Solution: (c) latitudes will receive the same amount of insolation
Exp) Option c is correct. only when the earth is flat and no longer spherical in
An ice age is a long period of reduction in the temperature shape (not because of lack of tilt of the axis). As long
of Earth’s surface and atmosphere, resulting in the as the earth is spherical in shape, the sun rays will have
presence or expansion of continental and polar ice sheets to travel varying lengths to reach them and the level of
and alpine glaciers. their directness and concentration upon unit area will
vary, leading to uneven distribution of insolation.
Statement 1 is correct. Temperate and tropical zones do
become restricted to the lower equatorial latitudes during The tilt of the axis only enhances the slant of the sun
ice ages. This is because ice ages change the Earth’s rays with an increase in latitude, so only the intensity
climatic belts. During the most recent glacial maximum of difference in insolation will change if the earth is no
(stadial, 23,000 to 11,000 years ago), the average global longer tilted.
temperature was about 8 degree Celsius, with polar Statement c is incorrect: Hot places becoming cold due
regions experiencing average temperatures of -2 degree to reduced insolation and vice versa will never happen
Celsius. as places in a more direct path to sun’s rays will always
Statement 2 is correct. Ice ages are not uniformly cold. receive more insolation and will always be hotter than
There can be colder and warmer periods during the places on higher latitude.
overall ice-age period. Colder periods lead to more Statement d is correct: The main effect of the tilt of earth
extensive areas of continental ice sheets, valley glaciers on its axis, as it moves along its orbit around the sun, is
and sea ice, while warmer periods lead to reduced areas that it brings one half of the earth marginally closer to
of ice. the sun for half the year and makes one half marginally
farther from the sun for half the year. This causes the
78. Solution: (b) temperatures to change seasonally in a place. Days are
Exp) Option b is correct longer, there is more insolation and temperatures are
hotter in the half tilted towards the sun and vice versa.
A heat budget refers to the balance between incoming
heat absorbed by earth and outgoing heat escaping it in 80. Solution: (c)
the form of radiation. Exp) Option c is correct.
Option b is correct. The Earth’s temperature remains There are different ways of heating and cooling of the
stable over long periods of time. This is because the earth atmosphere like conduction, convection and advection.
receives almost all of its energy from the sun and in turn The earth after being heated by insolation transmits the
radiates back to space the energy received from the heat to the atmospheric layers near to the earth in long
sun. This is termed as heat budget or heat balance of the wave form.
earth. Thus, the amount of heat received by different parts
of the earth is not the same. This variation causes pressure Statement 1 is correct: Advection refers to the horizontal
differences in the atmosphere. This leads to transfer of movement of air, parallel to the surface of the earth.
heat from one region to the other by winds. Advection is the main mechanism of heat redistribution
in the middle latitudes.
The oceans do absorb most of the excess heat from
These winds blow from a region of high pressure (cooler)
greenhouse gas emissions. However, this is not the
to low pressure (hotter). This helps dissipate some of the
correct reason for earth to maintain the heat balance. The
heat and affects the diurnal variation of temperature.
absorption of greenhouse gases by oceans has led to rise
This effect of advection on diurnal temperature ranges is
in ocean temperatures.
most pronounced in the mid latitudes.
79. Solution: (d) Statement 2 is correct: Heat within the earth’s atmosphere
Exp) Option d is correct. is also transferred through the process of convection.
In this air, over intensely heated areas expand and
The energy received by the earth’s surface in the form of
becomes less dense and rises up, thus transferring heat
short waves is termed as Incoming Solar Radiation or
from lower layers to the upper layers which are cooler.
Insolation.
However, these convective currents are seen only within
The axis of Earth is tilted towards the ecliptic of the Sun at the first layer of the atmosphere, i.e., the Troposphere.
an angle of approximately 23.5 degree. The Tropopause (last limit of troposphere, about 10-12
Statement a is incorrect: One half of the Earth being too km above earth surface) acts as a lid and prevents any

119 Workbook
GEOGRAPHY

convective currents from going beyond to the next layer 83. Solution: (c)
of atmosphere, i.e., Stratosphere. This happens because Exp) Option c is correct.
of the temperature inversion that occurs at tropopause.
A circumpolar vortex, or simply polar vortex, is a large
81. Solution: (b) region of cold, rotating air that encircles both of Earth’s
polar regions.
Exp) Option b is correct.
Statement 1 is incorrect. Polar vortex is described as a
Mercury, Venus, Earth and Mars are called as the inner whirling cone of low pressure over the poles.
planets of the Solar System as they lie between sun and
the belt of the asteroids. They are also called as Terrestrial Statement 2 is incorrect. Normally, when the vortex is
Planets (earth like). The rest four are called as Jovian or strong and healthy, it helps to keep a current of air (the jet
stream) travelling around the globe in almost a circular
Gas Giant planets. Jovian means Jupiter like.
path. This current keeps the cold air up north and warm
Statement 1 is incorrect: Mars is tilted at an angle of air down south. When the vortex becomes weak there is
almost 25 degrees. This leads to the phenomenon of a lack of a strong low-pressure system, resulting in jet
seasons as it completes its revolution around the Sun. stream losing the hold to keep it in line, and becoming
On the other hand, Venus has a tilt of barely 2 or 3 wavy and all of a sudden, a river of cold air is pushed
degrees, so it does not experience seasons. down south.
Statement 2 is correct: Venus has an atmosphere Statement 3 is correct. The term vortex refers to
dominated by carbon dioxide as well as sulphuric counterclockwise flow of air that keeps the colder air
clouds, which traps all the heat received from the sun. near the poles.
This leads to very hot temperatures (around 450 degrees 84. Solution: (d)
celsius) as well as very high pressures (as much as 90
times that on Earth) on Venus. Exp) Option d is correct.
A front is a weather system that is the boundary separating
82. Solution: (d) two different types of air. One type of air is usually denser
Exp) Option d is correct. than the other, with different temperatures and different
levels of humidity.
The ocean is a significant influence on Earth’s weather and
climate. The ocean covers 70% of the global surface. This Statement 1 is correct. A cold front is defined as the
great reservoir continuously exchanges heat, moisture, transition zone where a cold air mass is replacing a warmer
and carbon with the atmosphere, driving our weather air mass. Cold fronts often come with thunderstorms
patterns and influencing the slow, subtle changes in our or other types of extreme weather. They usually move
climate. from west to east. Cold fronts move faster than warm
fronts because cold air is denser, meaning there are more
Statement 1 is correct. The oceans influence climate by molecules of material in cold air than in warm air.
absorbing solar radiation and releasing heat needed to
drive the atmospheric circulation. It releases aerosols Statement 2 is correct. Stationary fronts will either
dissipate after several days or devolve into shear lines. But
that influence cloud cover, by emitting most of the water
can change into a cold or warm front if conditions aloft
that falls on land as rain, by absorbing carbon dioxide
change causing the stronger of the two fronts to win out.
from the atmosphere and storing it for years to millions
Else, the stationary front will be pushed in one direction
of years.
by another stronger front pushing behind it.
Statement 2 is correct. The tropics are particularly rainy Statement 3 is correct. An occluded front is formed
because heat absorption, and thus ocean evaporation, is during the process of cyclolgenesis when a cold front
highest in this area. Hence, it helps to distribute heat overtakes a warm front. When this occurs, the warm
around the globe air is separated (occluded) from the cyclone centre at the
When water molecules are heated, they exchange freely earth’s surface. While frontogenesis is the atmospheric
with the air in a process called evaporation. Ocean water process that leads to the formation or intensification of
is constantly evaporating, increasing the temperature fronts is Galled frontogenesis
and humidity of the surrounding air to form rain and
storms that are then carried by trade winds. In fact, almost 85. Solution: (b)
all rain that falls on land starts off in the ocean. Exp) Option b is correct.
Statement 3 is correct. Carbon dioxide is also transferred An air mass is a large volume of air in the atmosphere that
through the air-sea interface. Deep water of the ocean is mostly uniform in temperature and moisture. A source
can store carbon dioxide for centuries. Carbon dioxide region of air masses must have fairly homogeneous
dissolves in cold water at high latitudes, and is subducted surface conditions of temperature and moisture content
with the water. It stays in the deeper ocean for years to i.e., possessing homogeneous isolation throughout to
centuries before the water is mixed back to the surface impart homogeneity to the air mass.
and warmed by the sun. The warm water releases carbon Statement a is incorrect Air masses have fairly uniform
dioxide back to the atmosphere. temperature and moisture content in horizontal

Workbook 120
GEOGRAPHY

direction. Air masses form over large surfaces with 88. Solution: (a)
uniform temperatures and humidity, called source Exp) Option a is correct.
regions. There are four categories for air masses: arctic,
The Earth’s axis of rotation is tilted at an angle of 23.5° to
tropical, polar and equatorial.
its orbital plane. This tilt — combined with factors such as
Statement b is correct. Air masses can extend thousands Earth’s spin and orbit — leads to variations in the duration
of kilometers in any direction, and can reach from ground of sunlight that any location on the planet receives on
level to the stratosphere up to 16 kilometers into the different days of the year.
atmosphere.
Amount of daylight hours depends on our latitude. These
Statement c is incorrect. Air masses are formed in the latitudes, are a measure of a location’s distance from the
polar region. There are four categories for air masses: Equator.
arctic, tropical, polar and equatorial. Arctic air masses
The following table talk about the latitude and approximate
form in the Arctic region and are very cold. Polar air
length of the day.
masses take shape in high-latitude regions and are cold.
Statement d is incorrect. Air masses move from one
region to another under the influence of pressure
gradients and upper-level wind patterns. They are
sometimes pushed or blocked by high level jet streams.
Air masses form an integral part of the global planetary
Option a is correct. Paramaribo is the capital city of
wind system. Therefore, they are associated with one or
Suriname on the banks of the Suriname River. It is near to
other wind belt.
Equator and its latitude is 5.8° N.
86. Solution: (b) San Salvador is the capital and the largest city of El
Exp) Option b is correct Salvador. It is the country’s political, cultural, educational
and financial center. It is near to Equator and its latitude
The process of changing/turning vapour [Gases] directly
is 13.7° N.
into the ice [Solid] is called desublimation or Deposition.
Malta is an archipelago in the central Mediterranean
Option b is correct. Deposition is the phase transition between Sicily and the North African coast. Its latitude is
process in which gas (vapour) transforms into solid 35.9° N.
without passing through the liquid phase transition.
Deposition is a thermodynamic process. The reverse Lisbon is hilly, coastal capital city of Portugal. Its latitude
of deposition is sublimation in which the solid directly is 38.7° N.
transforms into gas(vapour) without passing through the Hence the duration of day in these cities will be in this
liquid phase and hence sometimes deposition is called order- Paramaribo>San>Salvador>Malta > Lisbon
desublimation.
89. Solution: (b)
Sublimation is most often used to describe the process of Exp) Option b is correct
snow and ice changing into water vapor in the air without
first melting into water. The air in motion is called wind. Wind is moving air
and is caused by differences in air pressure within our
87. Solution: (b) atmosphere. Air under high pressure moves toward areas
Exp) Option b is correct of low pressure. The greater the difference in pressure, the
faster the air flows.
Noctilucent clouds are the highest clouds in the sky,
Statement 1 is incorrect. The horizontal winds near the
however, they are not associated with weather like the rest
earth surface is the result of the combined effect of three
of the clouds.
forces – the pressure gradient force, the frictional force
Option b is correct. Noctilucent clouds are formed due and the Coriolis force. In addition, the gravitational force
to the cold temperature of the mesosphere, where water also acts downward. The wind at the surface experiences
vapour is frozen, forming ice clouds. Noctilucent clouds friction which is responsible for limiting the speed
are the highest clouds in the Earth’s atmosphere, located in of the wind. Along with that gravitational force is also
the mesosphere at altitudes of around 76 to 85 kilometers. responsible for wind direction, which acts downwards.
They are normally too faint to be seen, and are visible only Rotation of the earth also affects the wind movement.
when illuminated by sunlight from below the horizon The force exerted by the rotation of the earth is known as
while the lower layers of the atmosphere are in the Earth’s the Coriolis force. In the Northern Hemisphere, wind and
shadow. Noctilucent clouds are not fully understood and currents are deflected toward the right, in the Southern
are a recently discovered meteorological phenomenon. Hemisphere they are deflected to the left.
In the early 21st century, because of increases in methane Statement 2 is correct. The friction force on the wind
concentrations in the upper atmosphere, noctilucent is more at the land surface than on the sea surface.
clouds appeared with increasing frequency in lower Frictional Force is greatest at the surface and its influence
latitudes. generally extends up to an elevation of 1 – 3 km. Over the

121 Workbook
GEOGRAPHY

sea surface the friction is minimal. Over uneven terrain, the thunderstorm. This continues till the pellets become
however, due to high friction, the wind direction makes too heavy to be lifted by the air currents and fall to the
high angles with, isobars and the speed gets retarded. ground as hailstone
90. Solution: (a) 92. Solution: (c)
Exp) Option a is correct. Exp) Option c is correct.
An anticyclone is a weather phenomenon defined as a Titan is the largest moon of Saturn and the second-largest
large-scale circulation of winds around a central region of natural satellite in the Solar System. It is the only moon
high atmospheric pressure. known to have a dense atmosphere.
Statement 1 is correct. Anticyclones are regions of Option c is correct.
relatively high pressure on horizontal surfaces around Earth and Titan are the only bodies in the Solar System
which air circulates clockwise in the Northern Hemisphere where liquid rains on a solid surface - though on Titan,
and counterclockwise in the Southern Hemisphere. the rain is methane rather than water. On Titan, liquid
Statement 2 is incorrect. Anti-cyclones are indicative of hydrocarbons take the place of water as rainfall. On
dry weather which mostly remain rainless. Anticyclones Titan, where the surface temperature averages -179C,
are largely rainless. The sky is free of clouds because of it rains methane.
the fact that descending air in the centre of anticyclone is NASA’s Cassini orbiter captured the occurrence of rainfall
warmed up at dry adiabatic rate due to subsidence. This is on north pole of Titan, looking like a wet sidewalk after a
why anticyclones are indicative of dry weather. This does bit of rain. This rainfall also signifies a change in season
not mean that anticyclones are always rainless. While on the Titan.
passing over oceans sometimes they pick up moisture
and yield light rains or drizzles with moderate clouds. 93. Solution: (b)
Statement 3 is correct. In summer, the clear settled Exp) Option b is correct.
conditions associated with anticyclones allow the Sun’s Temperate grasslands are found in the regions with
light to warm the ground. This is why anticyclones are temperate and semi-arid to semi-humid climates. The
more frequent in summers. This can bring long sunny Veldts of South Africa, the Puszta of Hungary, the Pampas
days and warm temperatures. The weather is normally of Argentina and Uruguay, the Steppes, and the plains
dry, although occasionally, very hot temperatures can and Prairies of Central North America are Temperate
trigger localized thunderstorms. Grasslands
91. Solution: (d) Statement 1 is incorrect. In the northern hemisphere, the
climate of Temperate grasslands is continental with a high
Exp) Option d is correct. range of annual temperature. Temperate grassland in the
Sleet are small ice particles that form from the freezing northern hemisphere is extensive and continental.
of liquid water drops, such as raindrops. Hail is frozen In the southern hemisphere, the climate is never severe.
precipitation that can grow to very large sizes through the
It is mainly due to the narrowness of the temperate
collection of water that freezes onto the hailstone’s surface.
portions of the southern continents. Here, the annual
Statement 1 is correct. Sleet is formed under the range of temperature is very low. Temperate grassland in
conditions of temperature inversion. When the falling the southern hemisphere is restricted and less continental
snow reaches the layer of warm air, it melts, then it hits the i.e., their annual range of temperature is very low.
layer of cold air just above Earth’s surface and refreezes.
This results into creation of tiny ice pellets called as sleet. Statement 2 is correct. The tropical savanna climate is
also called the tropical wet and dry climate. It is the climate
Statement 2 is correct. Sleet forms in winter storms, experienced in savanna or tropical grassland regions of
while hail is a warm-season type of precipitation. Sleet the world. These places are located near the equator, and
forms when snow melts in a warm layer and then refreezes they lie between the Southern and the Northern Tropics.
into ice pellets as it falls though a cold layer. Hail, however,
forms in spring, summer or fall thunderstorms. Savanna (or Sudan) type of climate has alternate hot,
rainy season and cool, dry seasons but has considerably
Statement 3 is correct. Sleet pellets are normally smaller
less annual rainfall. It is confined within the tropics and
than hail. Due to repeated upliftment of frozen pellets by
is best developed in Sudan, hence its name the Sudan
the upward air current of the air, the hailstones are made
Climate. It is a transitional type of climate found between
of many layers of ice which increases their size. Hence,
hailstones are larger pellets than the pellets that make up the equatorial rainforests and hot deserts. Other features
sleet. of this climate are:
Hailstones are clumps of layered ice that form in updraughts Mean annual temperature is greater than 18° C.
(rising air) within thunderstorms. Hailstones form when Highest temperatures do not coincide with the period of
an upward current of air generated by thunderstorms the highest sun (e.g. June in the northern hemisphere)
lift water droplets high in the troposphere. When the but occur just before the onset of the rainy season, i.e.
raindrops freeze in the upper atmosphere and fall down April in Northern Hemisphere and October in Southern
then they are again lifted by the upward air currents of Hemisphere.

Workbook 122
GEOGRAPHY

This extreme diurnal range of temperature is another


characteristic feature of this climate with days being hot
and nights being cold.
The prevailing winds of the region are the Trade Winds,
which bring rain to the coastal districts.
94. Solution: (a)
Exp) Option a is correct. Statement 2 is correct. Geostrophic winds, flow at great
Albedo is the portion of solar energy reflected from the speeds due to low friction and are subjected to greater
surface of the Earth back into space. Coriolis force. Upper-air winds are faster than surface
It is a reflection coefficient and has a value of less than winds because friction is greatly reduced aloft.
one.
Option a is correct.
When solar radiation passes through the atmosphere, a
certain amount of it is scattered, reflected and absorbed.
The reflected sum of radiation is called the albedo of the
earth.
Albedo is an important concept in climatology, astronomy,
and environmental management.
It plays a major role in the energy balance of the earth’s
surface, as it defines the rate of the absorbed portion of Statement 3 is correct. The Jet Stream is a geostrophic
the incident solar radiation wind blowing horizontally through the upper layers of the
Different surfaces have different values. troposphere, generally from west to east, at an altitude of
20,000 - 50,000 feet.
Albedo is higher in Snow or Ice.
Jet Streams develop where air masses of differing
temperatures meet. So, usually surface temperatures
determine where the Jet Stream will form.
96. Solution: (d)
Exp) Option d is correct.
Coriolis force is an apparent force caused by the earth’s
rotation. The magnitude of the Coriolis force depends on
the speed of the object and its latitude.
Statement a is correct. The Coriolis force is responsible
for deflecting winds towards the right in the northern
hemisphere and towards the left in the southern
hemisphere. This is also known as ‘Ferrel’s Law’.
95. Solution: (c) Statement b is correct. Coriolis force is absent at the
Exp) Option c is correct equator and highest at the poles. The Coriolis force acts
perpendicular to the pressure gradient force. The pressure
Geostrophic wind is a wind whose direction and speed are gradient force is perpendicular to an isobar. At the equator
determined by a balance of the pressure-gradient force the wind blows perpendicular to the isobars. Thus, the
and the force due to the earth’s rotation. The Coriolis Coriolis force is zero at equator and it increases moving
force is a result of the rotation of the Earth on its axis. towards poles.
Statement 1 is correct. In geostrophic wind the Coriolis Statement c is correct. Where a cyclone forms, the
force is balanced by horizontal pressure gradient force. direction it spins and the general path it takes are
An air parcel initially at rest will move from high pressure determined by a range of factors including the Coriolis
to low pressure because of the pressure gradient force force. Cyclones are formed due to the presence of a low-
(PGF). However, as that air parcel begins to move, it is pressure zone. As the low pressure intensifies over a period
deflected by the Coriolis force to the right in the northern of time (depending on various factors) the surrounding
hemisphere (to the left on the southern hemisphere). As air rushes in to fill it. As the air is rushing in it will rotate
the wind gains speed, the deflection increases until the according to the Ferrel’s law and will cause cyclones to
Coriolis force equals the pressure gradient force. At this rotate anticlockwise in the northern hemisphere and
point, the wind will be blowing parallel to the isobars. clockwise in the southern hemisphere.
When this happens, the wind is referred to as geostrophic. Statement d is incorrect. The Coriolis effect bends the

123 Workbook
GEOGRAPHY

direction of surface currents to the right in the Northern off-shore trades and a concentration of rainfall in winter
Hemisphere and left in the Southern Hemisphere. This is with on-shore westerlies.
true for the deep-water circulation as well.
100. Solution: (c)
97. Solution: (a) Exp) Option c is correct.
Exp) Option a is correct. The Cool Temperate Western Margin climate is also
Fog is a visible aerosol comprising tiny water droplets known as the British type of climate. These are the regions
or ice crystals suspended in the air at or near the Earth’s which remains under the permanent influence of the
surface. Westerlies all-round the year. They are also regions of
Stateent 1 is correct: Fogs are drier compared to mists. much cyclonic activity. From Britain, the climatic belt
Both are ground level clouds, formed when water stretches far inland into the lowlands of North-West
condenses around nuclei in a large mass of air near the Europe, including northern and western France, Belgium,
the Netherlands, Denmark, western Norway and also
surface of earth. However, mist contains much more
northwestern Iberian Peninsula.
moisture, as in mist each nuclei contains a thicker layer
of moisture around it, compared to a fog. Statement 1 is correct. The British type of climate has
adequate rainfall throughout the year with a tendency
Statement 2 is incorrect: Fogs are generally prevalent
towards a slight winter or autumn maximum from
where warm currents of air come in contact with cold
cyclonic sources. Since the rain-bearing winds come
currents. On the other hand, mists are generally formed from the west, the western margins have the heaviest
on mountain as the rising warm air up the slopes meet a rainfall.
cold surface.
The amount decreases eastwards with increasing
98. Solution: (b) distance from the sea.
Exp) Option b is correct. Statement 2 is incorrect. As in other temperate regions
Tropical evergreen forests are found in warm and humid there are four distinct seasons in the British climate
areas. Tropical evergreen forests are well stratified, with type. Light snowfalls can be expected in the winter
layers closer to the ground and are covered with shrubs months normally only of short duration because of the
and creepers, with short structured trees followed by tall comparatively mild weather.
variety of trees. Statement 3 is correct. The open nature of the forests
in British type of climate supports and promotes
Statement 1 is correct. The tropical evergreen forests
Lumbering industry. Unlike the equatorial forests,
usually occur in areas receiving more than 200 cm of
the deciduous trees here occur in pure stands and have
rainfall. Tropical evergreen forests of India are found in
greater lumbering value. The open nature of the forests
the western slope of the Western Ghats, hills of the north-
with sparse undergrowth is useful in logging operations.
eastern region and the Andaman and Nicobar Islands.
Easy penetration means much cost can be saved in the
Statement 2 is incorrect. The areas having a temperature movement of the logs. The deciduous hardwoods are
of 15 to 30 degrees Celsius are conducive for the growth excellent for both fuel and industrial purposes.
of tropical evergreen forests. They occupy about seven
per cent of the earth’s land surface and habours more than 101. Solution: (a)
half of the world’s plants and animals. They are found Exp) Option a is correct
mostly near the equator. The ozone layer is a thin part of the Earth’s atmosphere
Statement 3 is correct. Tropical Evergreen Forests are that absorbs almost all of the sun’s harmful ultraviolet
dense and multi-layered. They harbour many types of light. The ozone layer is found in the lower stratosphere
plants and animals. The trees are evergreen as there is no at a height of roughly 20-40km.
period of drought. They are mostly tall and hardwood Statement 1 is correct. Within days of the ozone layer’s
type. Leaves are broad and give out excess water through disappearance, the intensity of the sun’s radiation
evapo-transpiration. would make photosynthesis — a process by which
plants convert light energy into chemical energy to fuel
99. Solution: (c)
their growth — an impossibility for all. And even these
Exp) Option c is correct. holdouts, primarily massive trees, would eventually die,
Warm Temperate Western Margin climate, aka too. Without plants, the food chain would collapse.
Mediterranean type climate is found in few areas of Statement 2 is incorrect. Mesosphere (and not ozone
the world. It is entirely confined to the western portion layer) takes care of extra-terrestrial objects like meteors
of continental masses, between 30° and 40° north and which get burnt up while passing through this layer due
south of equator. The basic cause of this type of climate is to friction.
shifting of wind belts. Statements 3 and 4 are correct. Stratospheric ozone is
The Mediterranean type climate is characterised by very a naturally occurring gas that filters the sun’s ultraviolet
distinctive climatic features – a dry and hot summer with (UV) radiation. A diminished ozone layer allows more

Workbook 124
GEOGRAPHY

UV radiation to reach the Earth’s surface. For people, Niño has a warming influence on global temperatures,
overexposure to UV rays can lead to skin cancer, cataracts while La Niña has the opposite effect.
in eyes, and weakened immune systems. Increased UV Statement 2 is incorrect. La Nina (not El Niño)
can also lead to reduced crop yield and disruptions in the events represent periods of below-average sea surface
marine food chain. temperatures across the east-central Equatorial Pacific.
102. Solution: (c) It is indicated by sea-surface temperature decreased by
more than 0.9 for at least five successive three-month
Exp) Option c is correct seasons.
Carbon dioxide (CO2) is an important greenhouse gas.
It is naturally available and also released through human
activities such as deforestation and burning fossil.
Statement 1 is incorrect. Carbon dioxide and water
vapour are found only up to 90 km from the surface of
the earth (and not all layers) as the proportion of gases
changes in the higher layers of the atmosphere. Similarly,
oxygen will be almost in negligible quantity at the height
of 120 km.
Statement 2 is correct. Carbon dioxide is the most
important of Earth’s long-lived greenhouse gases.
Carbon dioxide is not destroyed over time, but instead 3.3. Climate Phenomena
moves among different parts of the ocean–atmosphere–
land system. Some of the excess carbon dioxide is absorbed 104. Solution: (d)
quickly (for example, by the ocean surface), but some will Exp) Option d is correct.
remain in the atmosphere for thousands of years. Statement 1 is correct. The moraine creates topographic
Statement 3 is correct. The concentration of carbon depression in which the melt water is generally
dioxide in Earth’s atmosphere has been increasing since accumulated leading to formation of glacial lake. Such
the beginning of Industrial Age. It is currently at nearly moraine-dammed lakes appear to be the most common
412 parts per million (ppm) and rising. This represents a type of glacial lakes. When this lake is watertight, melt
47 percent increase since the beginning of the Industrial waters will accumulate in the basin until seepage or
Age, when the concentration was near 280 ppm, and an overflow limits the lake level. The impoundment of
11 percent increase since 2000, when it was near 370 ppm. the melt may sometimes be unstable, leading to sudden
Statement 4 is correct. The Copernicus Anthropogenic release of large quantities of stored water.
Carbon Dioxide Monitoring mission, or CO2M is one A moraine is any accumulation of unconsolidated debris
of the missions of European Space Agency (ESA). The (regolith and rock) that occurs in both currently and
aim is to measure atmospheric carbon dioxide at high formerly glaciated regions, and that has been previously
spatial resolution. These measurements will be used to carried along by a glacier or ice sheet.
eventually reduce uncertainties in estimates of emissions
Statement 2 is correct. The high temporal and high
of carbon dioxide from the combustion of fossil fuel at
spatial remote sensing imagery from satellites supported
local, national and regional scales.
by field surveys will allow decision makers to better assess
103. Solution: (d) the possible risk of future outbursts in this region.
Exp) Option d is correct. Statement 3 is correct. Through extensive research,
Normally when the tropical eastern south Pacific Ocean CDAC has deployed India’s first GLOF Early Warning
experiences high pressure, the tropical eastern Indian System in Sikkim to predict GLOFs, which will help the
Ocean experiences low pressure. But in certain years, Government authorities in case of an impending GLOF
there is a reversal in the pressure conditions and the event.
eastern Pacific has lower pressure in comparison to the 105. Solution: (d)
eastern Indian Ocean. This periodic change in pressure
conditions is known as the Southern Oscillation (SO). Exp) Option d is correct.
The difference in pressure over Tahiti (Pacific Ocean, The Fujiwhara effect is a phenomenon that occurs when
18°S/149°W) and Darwin in northern Australia (Indian two nearby cyclonic vortices move around each other.
Ocean, 12°30’S/131°E) is computed to predict the When cyclones are in proximity of one another, they start
intensity of the monsoons. rotating about a point between the two systems due to
Statement 1 is incorrect. ENSO is a periodic fluctuation their cyclonic wind circulations.
in sea surface temperatures and the air pressure of If one cyclone (hurricane) is a lot stronger than the other,
the overlying atmosphere across the equatorial Pacific the smaller one will orbit it and eventually crash into it.
Ocean. ENSO has a major influence on weather and If both are comparable in size than the two vortices will
climate patterns such as heavy rains, floods and drought. El be attracted to each other, and eventually spiral into the

125 Workbook
GEOGRAPHY

center point and merge, or merely spin each other around Statement 3 is correct. Due to repeated upliftment
for a while before shooting off on their paths. of frozen pellets by the upward air current of the air,
Often the effect is additive with the two cyclones the hailstones are made of many layers of ice which
(hurricanes) come together, and we usually end up with increases their size. Hence, hailstones are larger pellets
one massive cyclone instead of two smaller ones. than the pellets that make up sleet.

106. Solution: (c) 110. Solution: (a)


Exp) Option c is correct. Exp) Option a is correct.
The High temperature recorded in Death Valley Statement 1 is correct. In the interior parts of the
(california) is a result of ‘Heat dome’. Heat dome is continents large landmass is available to get heated,
the phenomenon in which high pressure circulation resulting into hot air on a huge scale, which helps in the
in the atmosphere acts like a dome or cap which traps convectional rainfall.
hot ocean air at the surface and favours the formation Statement 2 is incorrect. High temperature and
of a heat wave. Heat dome happens when strong, high- intense heating are needed for the air to get heated and
pressure atmospheric conditions combine with influences ultimately rise, which condenses in the upper atmosphere.
from La Niña, creating vast areas of sweltering heat that Therefore, such rain is common in the summer or during
gets trapped under the high-pressure “dome.” the hotter part of the day.
107. Solution: (c) 111. Solution: (c)
Exp) Option c is correct. Exp) Option c is correct.
Option a is incorrect. Heat waves occur when actual Statement 1 is incorrect. Haze (not fog) is formed due to
maximum temperature is ≥45°C. unequal refraction of light in air of different densities in
Option b is incorrect. Heat waves is a period of abnormally the lower atmosphere. Fog occurs when water condenses
high temperatures that occurs during the summer season. on dust and other particles like smoke.
In India, heat waves typically occur between March and Statement 2 is correct. Dense fogs are more likely to
June, and in some rare cases even extend till July. occur in the high and middle latitudes rather than
Option c is correct. Severe heat waves occur when tropics.
temperature departure from normal is greater than 6.4°C. Statement 3 is correct. Fog only occurs in the lower
Option d is incorrect. To declare a heat wave, the strata of atmosphere as a sort of dense ‘ground cloud’.
set criteria should be met at least in 2 stations in a
Meteorological subdivision for at least two consecutive 112. Solution: (a)
days and it will be declared on the second day. Exp) Option a is correct
Statement 1 is correct. Positive IOD brings good
108. Solution: (c)
monsoon rainfall in India and Indian Ocean and poor
Exp) Option c is correct. rainfall in eastern parts of Indian Ocean.
El Nino Modoki is associated with strong anomalous Statement 2 is Incorrect. Strong El Nino events contribute
warming in the central tropical Pacific and cooling in to weaker monsoons and even droughts in India.
the eastern and western tropical Pacific. During El Nino
modoki event, warming is observed more in the central Statement 3 is Incorrect.La Nina means Little Girl in
part with cooling on its both sides that is in the eastern Spanish. La Nina has the opposite effect of El Nino.
and western Pacific Ocean. During La Nina events, trade winds are even stronger
than usual, pushing more warm water toward Asia. Off
109. Solution: (d) the west coast of the Americas, upwelling increases,
Exp) Option d is correct. bringing cold, nutrient-rich water to the surface.
Statement 1 is correct. Under conditions of temperature 113. Solution: (d)
inversion, when the falling snow reaches the layer of warm Exp) Option d is correct.
air, it melts, then it hits the layer of cold air just above
Earth’s surface and refreezes. This results into creation of Statement 1 is incorrect. Trade winds between latitudes
tiny ice pellets called as sleet. 35-40 degrees North and South of the equator, brings
plentiful rainfall to the eastern coast of the continents and
Statement 2 is correct. Hailstones are clumps of goes on decreasing towards the west.
layered ice that form in updraughts (rising air) within
thunderstorms. Hailstones form when an upward current Statement 2 is incorrect. Westerlies between 45-65
of air generated by thunderstorms lift water droplets degrees North and South of the equator, the rainfall is first
high in the troposphere. When the raindrops freeze in received on the western margins of continents and it goes
the upper atmosphere and fall down then they are again on decreasing towards the east.
lifted by the upward air currents of the thunderstorm.
114. Solution: (a)
This continues till the pellets become too heavy to be lifted
by the air currents and fall to the ground as hailstones. Exp) Option a is incorrect.

Workbook 126
GEOGRAPHY

Statement 1 is incorrect. The rate of change of pressure content and vice versa.
in regard to distance is the pressure gradient. The vertical Statement 3 is correct. Relative humidity is greater over
pressure gradient force is much larger than that of the oceans and least over the continents. It is due to
the horizontal pressure gradient force. amount of availability of water vapor over the oceans is
Statement 2 is correct. The difference between vertical greater than over the land.
and horizontal pressure gradient force is balanced by the
118. Solution: (d)
gravitational force. Hence, we do not experience strong
upward winds. Exp) Option d is correct
Statement 1 is correct. Earth’s axis makes an angle of
115. Solution: (d)
66 ½ with the plane of its orbit round the sun which
Exp) Option d is correct. influences the amount of insolation received at different
Option 1 is correct. The surrounding atmosphere latitudes. Thus, rotation of earth on its axis plays a major
exerts buoyant force on low-pressure cells and hence role in controlling insolation.
the air within a low pressure cell rises. On the other Statement 2 is correct: The angle of inclination of the
hand, the air within a high-pressure cell sinks as it is rays depends on the latitude of a place. The higher the
denser than the surrounding atmosphere. Rising air latitude the less is the angle they make with the surface of
is associated with convergence and unstable weather
the earth resulting in slant sun rays. Vertical ray covers
(cyclonic conditions) whereas the sinking (subsiding)
less area which means the energy remains concentrated
air is associated with divergence and stable conditions
(anticyclonic conditions). in a small area. Due to inclination slant rays has to pass
through greater depth of the atmosphere resulting in
Option 2 is correct. Frictional Force affects the speed more absorption, scattering and diffusion.
of the wind. It is greatest at the surface and its influence
generally extends up to an elevation of 1 - 3 km. Over the Statement 3 is correct: Length of the day has a direct
sea surface the friction is minimal. relation with the amount of insolation. For instance the
amount of insolation received in winter by middle and
Option 3 is correct. Coriolis Force deflects the wind to
higher latitudes is less than what is received in summer.
the right direction in the northern hemisphere and to
the left in the southern hemisphere. It is directly linked to the length of the day which is shorter
in winter in high and mid latitudes.
Option 4 is correct. Centripetal acceleration creates a
force directed at right angles to the wind movement Statement 4 is correct: The atmosphere is largely
and inwards towards the centres of rotation. This force transparent to short wave solar radiation. The incoming
produces a circular pattern of flow (vortex) around centres solar radiation passes through the atmosphere before
of high and low pressure. striking the earth’s surface. If the atmosphere is not
transparent then the light will get scattered in the
116. Solution: (c) atmosphere and back into space.
Exp) Option c is correct.
119. Solution: (d)
Statement 1 is correct. When isobars are straight, and
when there is no friction, the pressure gradient force is Exp) Option d is correct.
balanced by the Coriolis force, and the resultant wind Option a is correct: Terrestrial radiation is the amount
blows parallel to the isobar (deflection of the wind is of energy which is radiated out by the earth’s surface. It
maximum). This wind is known as the geostrophic wind. is also called as effective radiation as it helps in heating
Statement 2 is correct. The Jet Stream is a geostrophic the lower portion of the atmosphere.
wind blowing horizontally through the upper layers of the Option b is correct: In the terrestrial heat budget of the
troposphere, generally from west to east, at an altitude of
earth total amount of energy received is 51% of the total
20,000 - 50,000 feet.
insolation. Out of this 23% of the energy is lost in the
117. Solution: (d) form of terrestrial radiations.
Exp) Option d is correct. Option c is correct: Greenhouse gases trap the outgoing
Statement 1 is incorrect. Relative humidity could be terrestrial radiation which results in generating heat in
defined as the percentage of moisture present in the the atmosphere in the night in absence of the sun. This
atmosphere as compared to its full capacity at a given is called as greenhouse effect, where the energy from the
temperature. terrestrial radiations acts as a warm blanket for the earth.
Statement 2 is correct. The ability of the air to hold water Option d is incorrect: Terrestrial radiations are long
vapour depends on temperature along with other factors. wave radiations which are reflected back to earth by
With the change of air temperature, the capacity to greenhouse gases. This reflection is only possible because
retain moisture increases or decreases and the relative of long wave nature of terrestrial radiation otherwise it
humidity is also affected. Relative humidity increases would have simply crossed through them just like the
with the decrease in temperature of the water vapour incoming short wave radiations.

127 Workbook
GEOGRAPHY

Two of the important factors determining plant growth


3.4. Different forms of Vegetation (World) are temperature and precipitation.
120. Solution: (c) Pair 1 is correct – Teak is a plant belonging to tropical
Exp) Option c is correct. monsoon climate.
Statement 1 is correct. Amazon rainforests covers 6% of Pair 2 is correct – Olive belongs to Mediterranean type
climate, that is, warm temperate western margin.
world’s area and is home for large number of flora and
fauna and thus it is known as lungs of planet earth. Pair 3 is correct – Cactus is well adapted for desert
climates.
Statement 2 is incorrect. The Prairies (not Amazon)
were home of other tribes also like the Apache, the Pair 4 is incorrect – Spruce grows naturally in Siberian
Crow, the Cree and the Pawnee. Kawahiva, Korubo, type climate or Cool temperate continental climate.
Piripkura are some famous tribes of Amazon rainforests. 124. Solution: (c)
Statement 3 is correct. Amazon Basin stretches directly Exp) Option c is correct.
on the equator and is characterized by hot and wet
Pair 1 is incorrect – Savannah Type climate has onshore
climate throughout the year. Both day and nights are
winds in summer and off-shore in winter. It is characterised
almost equally hot and humid and it usually rains almost by an alternate hot, rainy season and cool, dry season. The
every day. annual rainfall here is mostly concentrated in summer.
121. Solution: (b) Precipitation with Winter maxima is in Mediterranean
climate.
Exp) Option b is correct.
Pair 2 is correct – Equatorial climate witnesses’ heavy
Statement 1 is incorrect – The Littoral and Swamp forests precipitation which is well distributed throughout the
can survive and grow both in fresh as well as brackish year. There is no month without the rain. A distinct dry
water. Brackish water is a mixture of seawater and fresh season like those of the Savanna or the tropical monsoon
water and its salinity can range from 0.5 to 35 ppt. climate is absent. Maximum rainfall occurs shortly after
Statement 2 is correct – The Littoral and Swamp forests the equinox.
provide hard and durable timber which is used for Pair 3 is correct – Tropical Monsoon type have three
construction and building purposes as well as for distinct seasons –
making boats. 1. Cool, dry season from October to February
Statement 3 is incorrect – Littoral and swamp forests 2. Hot dry season from March to mid-June
occur in and around the deltas, estuaries and creeks 3. Rainy season from mid-June to September
prone to tidal influences. They occur in the deltas of the
Ganga, the Mahanadi, the Godavari, the Krishna and 125. Solution: (d)
the Cauvery. Sunderbans in the Ganga delta is the most Exp) Option d is correct.
pronounced location of such types of forests. Statement 1 is incorrect – two periods of maximum
122. Solution: (b) rainfall in April and October, in equatorial region. These
two peaks are observed shortly after the equinoxes which
Exp) Option b is correct. occur around March 21st and September 23rd every year.
Statement 1 is incorrect. A megadiverse country must This double rainfall peak is a unique feature of equatorial
have: climate.
1. at least 5,000 species of endemic plants and Statement 2 is incorrect – Tropical vegetation has
2. must border marine ecosystems. multiple layers, the top layer has thick canopy of foliage,
Statement 2 is correct. Conservation International smaller trees form the next layer and the third layer occurs
at ground having ferns and herbaceous plants. Unlike the
identified 17 megadiverse countries in 1998.
temperate forest where only few species occur in particular
Statement 3 is correct. In 2002, Mexico formed a area, the trees of tropical rain forest are not found in pure
separate organization focusing on Like-Minded stands of single species instead it has multiple species.
Megadiverse Countries, consisting of countries rich in
Statement 3 is incorrect. The equatorial region is
biological diversity and associated traditional knowledge. generally sparsely populated. The high temperature, high
This was formed in Cancun and hence called as Cancun humidity makes one feel sticky and uncomfortable. The
initiative. monotonous climate, oppressive and enervating taxes
Statement 4 is incorrect. Like-Minded Megadiverse one’s mental alertness and physical ability. Dangers of sun
Countries Organization does not include all the stroke, diseases like malaria and yellow fever etc. and also
megadiverse countries as identified by Conservation prevalence of bacteria and insect pests are some of the
International. reasons for low density of population.
123. Solution: (d) 126. Solution: (c)
Exp) Option d is correct. Exp) Option c is correct.

Workbook 128
GEOGRAPHY

Statement 1 is correct – Monthly temperatures in Pair 3 is incorrect – Gobi Desert is a rain shadow desert,
the region of grasslands can range between 21 to 32 formed by the Tibetan Plateau blocking precipitation
degrees centigrade for lowlands, but the range increases from the Indian Ocean reaching the Gobi territory. It is
as one moves away from the equator. Annual average rainless because of continentality.
temperature is around 18 degrees centigrade. Sky is Pair 4 is correct – Sahara Desert is linked with Canaries
clear during the night leading to rapid radiation loss current which is a cold current.
and even in the hot season, the temperatures drop to
Figure: Hot and mid-latitude deserts of the world
below 10 degrees centigrade. Thus, a typical characteristic
feature of Savanna climate is the extreme diurnal range of 129. Solution: (b)
temperature.
Exp) Option b is correct.
Statement 2 is incorrect – The savanna landscape is Pair 1 is incorrect – Indian tribes are found in the
typified by tall grass and short trees. Amazon rainforests. They collect wild rubber. In the
Trees with small wide leaves and absence of shade is the Congo basin, Pygmy tribe is found.
distinctive feature of Mediterranean climate. Pair 2 is correct – Orang Asli tribes roam the jungles of
Statement 3 is correct –In South America, there are two Malaysia. They are known for making and selling all sorts
distinct regions of savanna north and south of equator, of cane products.
namely the Llanos and the Campos (Brazil). Pair 3 is correct – Bindibu or Aborigines are the nomadic
In Australia savanna is located south of the monsoon strip hunters and food gatherers of Australia.
running from west to east north of tropic of Capricorn. Pair 4 is incorrect – The Bushmen are the nomadic
127. Solution: (d) hunters and food gatherers. They roam the Kalahari
Desert with their bows and poisoned arrows, spears, traps
Exp) Option d is correct. and snares.
Statement 1 is correct – Deserts are regions of scanty
rainfall which may be hot like the hot deserts of the 130. Solution: (b)
Saharan type; or temperate as are the mid-latitude deserts Exp) Option b is correct.
like the Gobi. The Aridity of hot deserts is mainly due to Statement 1 is incorrect – In the recent years, Temperate
effects of off-shore trade winds. grasslands have been ploughed up for extensive,
Statement 2 is correct – Plants that exist in deserts have mechanized wheat cultivation and are now the granaries
highly specialized means of adapting themselves to arid of the world. Besides wheat, maize is also increasingly
environment. Most desert shrubs have long roots and are cultivated.
well spaced out and search for groundwater. Plants have Statement 2 is correct – Temperate grasslands are found
few or no leaves and their foliage is either waxy leathery, in mid-latitudinal regions and interior parts of countries,
hairy and needle shaped to reduce the loss of water and the grass there is short and nutritious like the lucerne
through transpiration. Some of them are entirely leafless
or alfalfa grass. They are very productive areas for cattle
with pricks and thorns. Others like Cacti have thick and
and sheep rearing and are leading ranching regions of the
succulent stems to store water for long droughts.
globe.
Statement 3 is correct – The Deserts have rich mineral
resources. The lure of mineral wealth has attracted many 131. Solution: (a)
immigrants into the desert. Exp) Option a is correct
1. Gold brought immigrants to the Kalgoorlie and Pair 1 is correct – Pustaz are found in Hungary. Steppes
Coolgardie towns of the Great Australian Desert.
of Europe are interrupted by highlands. Pustaz are the
2. The Kalahari Desert is rich in diamond and copper. isolated sections of these steppes.
3. The most arid Atacama Desert (in Northern Chile)
Pair 2 is incorrect – Velds are found in South Africa.
is famous for mining of caliches (cemented gravels)
from which sodium nitrate is prepared. Beside They are sandwiched between Drakensberg and Kalahari
nitrates, copper is also mined. Desert.
4. Desert of North America - Silver is mined in Mexico, Pair 3 is correct – Pampas are found in Argentina and
Uranium in Utah and Copper in Nevada. Uruguay.
5. Saharan and Arabian deserts are today supplying Oil Pair 4 is incorrect – Prairies are the extensive grasslands
all over the globe. found in North America. They lie in the foothills of the
Rockies and great lakes.
128. Solution: (c)
Exp) Option c is correct. 132. Solution: (d)
Pair 1 is correct – Namib Desert is linked with Benguela Exp) Option d is correct.
current which is a cold current. Statement 1 is incorrect – Warm Temperate Eastern
Pair 2 is correct – Mohave Desert is linked with margin climate is found on the eastern margins of the
Californian current which is a cold current. continents in warm temperate latitudes, just outside the

129 Workbook
GEOGRAPHY

tropics. It is typified by warm moist summer and a cool, Statement 3 is incorrect. Between the latitudes 35 degree
dry winter. The climate here is strongly influenced by and 40-degree N and S of the equator, the rain is heavier
maritime influence. on the eastern coasts and goes on decreasing towards the
Statement 2 is incorrect – The eastern margins of warm west.
temperate latitudes have a much heavier rainfall than 136. Solution: (b)
either the western margins or the continental interiors
and thus have luxuriant vegetation. Exp) Option b is correct.
Pair 1 is incorrect – Sirocco is a hot, dry, dusty wind
Statement 3 is incorrect – Gulf type of climate is a
which originates from the Sahara. It is most frequent in
subtype of Warm Temperate Eastern Margin climate. It
the springs and normally last for only a few days. The
is experienced in the south-eastern parts of the USA Sirocco blows outward from the desert interiors into
bordering the Gulf of Mexico and not in middle east. the cooler Mediterranean Sea where it absorbs some
133. Solution: (c) moisture.
Exp) Option c is correct. Pair 2 is correct – Buran is a cold, northerly polar wind
of Eurasia, blowing violently at 50 m.p.h or more and at
Statement a is correct. Laurentian climate is an a temperature of 50oF below freezing point. The powdery
intermediate type of climate between the British and the snow-flakes are blown around in the lower atmosphere
Siberian type of climate. It has features of both maritime and visibility is greatly reduced.
and continental climates.
Pair 3 is incorrect – Mistral is a cold wind from north
Statement b is correct. The predominant vegetation is rushing down the Rhone valley in France. The velocity
cool temperate forests. of the Mistral is intensified by the funneling effect in the
Statement c is incorrect. Lumbering and its associated valley between the Alps and the Central Massif.
timber, paper and pulp industries are the most
137. Solution: (a)
important economic undertaking. Agriculture is less
important in view of the severity of the winter and its Exp) Option a is correct.
long duration. Statement 1 is correct. Mangroves are the distinctive
Statement d is correct. The rainfall regime of the ecosystems found along the shallow coasts 30o N or S and
laurentian climate in Asiatic region is similar to the the equator.The sediment in which mangrove trees live
tropical monsoon type in India. The laurentian type must be covered with brackish or salt-water for part or
all of the day.
here is often described as cool temperate monsoon type
climate. Statement 2 is correct. Mangroves are highly productive
ecosystems which are capable of exporting energy and
134. Solution: (b) materials to adjacent communities. They support a
Exp) Option b is correct. diverse heterotrophic food chain, act as nurseries in the
life cycle of some organisms, and offer some protection
Statement 1 is incorrect – Cool temperate western margin against coastal erosion and storm surge attack
climate is also known as British type climate. Natal type
of climate belongs to Warm temperate eastern margin. T Statement 3 is Incorrect. Mangroves for Future is not
an intergovernmental organization. Mangroves for
Statement 2 is incorrect – This type of climate is most the Future (MFF) is a unique partner-led initiative to
pronounced in and around Britain. In North America, promote investment in coastal ecosystem conservation
if is mainly confined to coastlands of British Columbia. for sustainable development. Co-chaired by IUCN and
In the southern hemisphere, the climate is experienced UNDP, MFF provides a platform for collaboration among
in Southern Chile, Tasmania and most parts of New the many different agencies, sectors and countries which
Zealand. are addressing challenges to coastal ecosystem and
Statement 3 is correct – The natural vegetation of this livelihood issues.
climatic type is deciduous forest (trees shed their leaves 138. Solution: (c)
in the cold season). This is an adaptation for protecting
themselves against the winter snow and frost. Exp) Option c is correct.
Statement 1 is correct.The distribution of coral reefs and
135. Solution: (a) coral formations are controlled by environmental factors,
Exp) Option a is correct notably water temperature, purity of water, and salinity.
Most of reef-forming corals prefer sea temperatures
Statement 1 is correct. As we proceed from the equator
between 18o C to 32o C, salinity between 30 and 38 parts
towards the pole’s rainfall goes on decreasing steadily.
per thousand, and clear water. Light is also important, and
Statement 2 is correct. The coastal areas of the world coral growth is usually restricted to the upper 25 or 30
receive greater amounts of rainfall than the interior of metres of the sea surface. Because of these factors, coral
the continents. Also, the rainfall is more over the oceans reefs are mainly found between latitudes 30o N and S
than on the landmasses of the world because of being on mud free coastlines, particularly in western parts of
great sources of water. Pacific, Indian and Atlantic Oceans.

Workbook 130
GEOGRAPHY

Statement 2 is correct. An increase of ammonia and 142. Solution: (b)


nitrate in the water increases the process of eutrophication. Exp) Option b is correct.
The excessive eutrophication damage the corals and their
Statement a is correct. A mangrove is a shrub or small
formations.
tree that grows in coastal saline or brackish water. These
139. Solution: (d) forests mainly grow at tropical and subtropical latitudes
near the equator because they cannot withstand freezing
Exp) Option d is correct. temperatures.
All these factors are deterrents to the growth of much Statement b is incorrect. Viviparous mode of
undergrowth. reproduction is common in mangrove species. In this
Statement 1 is correct – The Podzolized soils of the method of reproduction, seeds germinate and develop into
coniferous forests are poor. They are excessively leached seedlings while the seeds are still attached to the parent
and are very acidic. Undergrowth is negligible because of tree. These seedlings are normally called as propagules
the poor soil conditions. and they photosynthesize while still attached to the
Statement 2 is correct – The evergreen leaves provide mother tree. The parent tree supplies water and necessary
nutrients. They are buoyant and float in the water for some
little leaf-fall for humus formation. There is no annual
time before rooting themselves on suitable soil.
replacement of new leaves as in deciduous trees. The same
leaf remains on the tree for as long as five years. Statement c is correct. Mangroves are salt-tolerant trees,
also called halophytes, and are adapted to life in harsh
Statement 3 is correct – The rate of decomposition of coastal conditions. They contain a complex salt filtration
the leathery ‘needles’ in a region of such low temperature system e.g., Leaves of many mangrove species have salt
is slow. secreting glands and complex root system to cope with
Statement 4 is correct – Absence of direct sunlight and salt water immersion and wave action.
the short duration of summer are other contributory Statement d is correct. Top five mangrove forests
factors to a sparse undergrowth. in India are Sundarbans (West Bengal), Pichavaram
Mangroves (Tamil Nadu), Godavari – Krishna Mangroves
140. Solution: (a)
(Andhra Pradesh), Bhitarkanika Mangroves (Odisha) and
Exp) Option a is correct Baratang Island Mangroves (Andamans and Nicobar).
Statement 1 is correct – Cool Temperate Eastern Margin
143. Solution: (a)
Climate also known as Laurentian type of climate is found
in Eastern Canada and north-east USA and Eastern Exp) Option a is correct.
coastlands of Asia – North China, Korea and Japan. The Statement 1 is correct. Coral reefs are the counterpart to
region near the Newfoundland and the Japan coast are the tropical rain forest in terms of species diversity and
one of the best fishing areas of the world. biological productivity in the Ocean. They play a crucial
Statement 2 is incorrect – The predominant vegetation of role in supporting the flora and fauna in the marine
Laurentian type climate is cool temperate forest. Forest ecosystem. They are also described as ‘underwater tropical
tends to be coniferous north of the 50° N latitude. South rainforest, fairy land under water, biologist’s paradise,
of 50° N latitude, deciduous forest is found. These forests magnificent repository of resources, genetic garden,
submerged meadows and treasure house of wealth.’
have great value and the region is famous for lumbering.
Statement 2 is correct. Zooxanthellae, the algae present
Equatorial climates predominant vegetation is
in the corals, contain a pigment called chlorophyll
mahogany, ebony, rosewood etc.
which gives the coral its green or brown color. Corals
141. Solution: (d) may change their color (like to blue, violet, red) to adapt
to different light conditions and water temperature.
Exp) Option d is correct.
Statement 3 is incorrect. India’s coral reef areas are Gulf of
Statement 1 is correct – The greatest inhibiting factor in Mannar, Andaman and Nicobar Islands, Lakshadweep
the growth of the polar climatic vegetation is deficiency islands, the Gulf of Kutch and Malvan.
in heat. Thus, few plants survive in such adverse
environment. Malvan coral reef is located along the Konkan coast in
Maharashtra.
Statement 2 is correct – Convectional rainfall is
generally absent in these regions because of the low rate 144. Solution: (b)
of evaporation and the lack of moisture in the cold polar Exp) Option b is correct.
air.
Vernal pools are temporary or ephemeral depressional
Statement 3 is correct – With a growing season of less wetlands in forests or fields. They vary in size from
than three months and warmest month not exceeding one square meter to over two acres in size. They may
50oF, there are no trees in the tundra. Such adverse be isolated, connected to larger wetlands, or located in
environment can only support lowest form of vegetation, floodplains along rivers. These seasonal pond forms from
mosses, lichens and sedges. high groundwater or from rain and snowmelt in late fall,

131 Workbook
GEOGRAPHY

winter, and early spring. They are often dry by summer’s Option b is Incorrect. Central Asian Steppe is found in
end. There are no inlets or outflows of water from the Ukraine where temperate conditions with low rainfall
pools. Because vernal pools are seasonal and are not wet are found and grasses dominate the vegetation. The
all year, they are not inhabited by fish. fauna include large herbivores like bison, antelope, cattle,
Option a is incorrect – Magma chamber is a large pool of rodents, prairie dog, wolves, and a rich and diverse array
liquid rock beneath the surface of the Earth. of ground nesting bird.
Option c is incorrect – Hot Springs or geothermal Option c is Incorrect. Boreal forest lies in cold regions
springs are produced by the emergence of geothermally with high rainfall, strong seasonal climates with long
heated groundwater that rises from the Earth’s crust.
winters and short summers. These are characterised by
Option d is incorrect –A subglacial lake is a lake that is evergreen plant species such as Spruce, fir and pine trees,
found under a glacier, typically beneath an ice cap or ice etc and by animals such as the lynx, wolf, bear, red fox,
sheet. Subglacial lakes form at the boundary between ice
porcupine, squirrel, and amphibians like Hyla, Rana, etc
and the underlying bedrock.
145. Solution: (d) 146. Solution: (b)
Exp) Option d is correct Exp) Option b is correct
In Siberian Tundra tree growth is hindered by low Statement 1 is Incorrect. Desert plants have well
temperatures and short growing seasons. It is composed developed roots that spread over large area for absorbing
of dwarf shrubs, sedges and grasses, mosses, and lichens. water.
The typical animals are reindeer, lemming and ptarmigan. Statement 2 is correct. Leaves and stem are succulent
Reptiles and amphibians are almost absent here.
and water storing. In some plants even the stem contains
Option a is Incorrect. In African Savannah grasses chlorophyll for photosynthesis.
with scattered trees and fire resisting thorny shrubs are
found. The fauna include a great diversity of grazers and Statement 3 is correct. Ephemeral lifestyle is found in
browsers such as antelopes, buffaloes, zebras, elephants desert ecosystem. Desert ephemerals are plants which are
and rhinoceros; the carnivores include lion, cheetah, adapted to take advantage of the short wet periods in arid
hyena; and mongoose, and many rodents. climates.

Workbook 132
GEOGRAPHY

GEOGRAPHY
INDIA PHYSICAL ENVIRONMENT
5. Consider the following statements:
4.1. Previous Years Questions [UPSC CSE Pre. 2012]
1. The Brahmaputra, Irrawady and Mekong rivers 1. The duration of the monsoon decreases from
originate in Tibet and flow through narrow and southern India to northern India.
parallel mountain ranges in their upper reaches. 2. The amount of annual rainfall in the northern
Of these rivers, Brahmaputra makes a “U” turn in plains of India decreases from east to west.
its course to flow into India. This “U” turn is due Which of the statements given above is/are correct?
to [UPSC CSE Pre. 2011]
(a) 1 only
(a) Uplift of folded Himalayan series (b) 2 only
(b) Syntaxial bending of geologically young (c) Both 1 and 2
Himalayas (d) Neither 1 nor 2
(c) Geo-Tectonic disturbance in the tertiary folded
mountain chains 6. Consider the following statements:
(d) Both (a) and (b) above [UPSC CSE Pre. 2012]
2. A state in India has the following characteristics: 1. The duration of the monsoon decreases from
[UPSC CSE Pre. 2011] southern India to northern India.
1. Its northern part is arid and semi-arid. 2. The amount of annual rainfall in the northern
plains of India decreases from east to west.
2. Its central part produces cotton.
3. Cultivation of cash crops is predominant over Which of the statements given above is/are correct?
food crops. (a) 1 only
Which one of the following states has all of the above (b) 2 only
characteristics? (c) Both 1 and 2
(d) Neither 1 nor 2
(a) Andhra Pradesh
(b) Gujarat 7. When you travel in Himalayas, you will see the
(c) Karnataka following: [UPSC CSE Pre. 2012]
(d) Tamil Nadu 1. Deep gorges
3. Two important rivers- one with its source in 2. U-turn river courses
Jharkhand (and known by a different name in 3. Parallel mountain ranges
Odisha), and another, with its source in Odisha- 4. Steep gradients causing land-sliding
merge at a place only a short distance from the Which of the above can be said to be the evidences
coast of Bay of Bengal before flowing into the for the Himalayas being young fold mountains?
sea. This is an important site of wildlife and
(a) 1 and 2 only
biodiversity and a protected area. Which one of
(b) 1, 2 and 4 only
the following could be this?
(c) 3 and 4 only
[UPSC CSE Pre. 2011] (d) 1, 2, 3 and 4
(a) Bhitarkanika
(b) Chandipur-on-sea 8. A particular State in India has the following
(c) Gopalpur-on-sea characteristics: [UPSC CSE Pre. 2012]
(d) Simlipal 1. It is located on the same latitude which passes
through northern Rajasthan.
4. Among the following States, which one has
2. It has over 80% of its area under forest cover.
the most suitable climatic conditions for the
cultivation of a large variety of orchids with 3. Over 12% of the forest over constitutes Protected
minimum cost of production, and can develop an Area Network in this State.
export-oriented industry in this field? Which one among the following States has all the
[UPSC CSE Pre. 2011] above characteristics?
(a) Andhra Pradesh (a) Arunachal Pradesh
(b) Arunachal Pradesh (b) Assam
(c) Madhya Pradesh (c) Himachal Pradesh
(d) Uttar Pradesh (d) Uttarakhand

133 Workbook
GEOGRAPHY

9. The Narmada river flows to the west, while most 3. Subansiri


other large peninsular rivers flow to the east. Which of the above flows/flow through Arunachal
Why? [UPSC CSE Pre. 2013] Pradesh?
1. It occupies a linear rift valley.
(a) 1 only
2. It flows between the Vindhyas and the Satpuras. (b) 2 and 3 only
3. The land slopes to the west from Central India. (c) 1 and 3 only
Select the correct answer using the codes given (d) 1, 2 and 3
below.
(a) 1 only 15. With reference to ‘Changpa’ community of India,
(b) 2 and 3 consider the following statements:
(c) 1 and 3 [UPSC CSE Pre. 2014]
(d) None 1. They live mainly in the State of Uttarakhand
10. Consider the following pairs: 2. They rear the Pashmina goats that yield a fine
[UPSC CSE Pre. 2013] wool.
3. They are kept in the category of Scheduled
Tribe State Tribes.
1. Limboo (Limbu) Sikkim Which of the statements given above is/are correct?
2. Karbi Himachal Pradesh (a) 1 only
(b) 2 and 3 only
3. Dongaria Odisha
(c) 3 only
4. Bonda Tamil Nadu (d) 1, 2 and 3
Which of the above pairs are correctly matched? 16. Consider the following pairs:
(a) 1 and 3 only
[UPSC CSE Pre. 2014]
(b) 2 and 4 only
(c) 1, 3 and 4 only Hills Region
(d) 1, 2, 3 and 4
1. Cardamom Hills Coromandel Coast
11. Which one of the following pairs of islands is
separated from each other by the ‘Ten Degree 2. Kaimur Hills Konkan Coast
Channel’? [UPSC CSE Pre. 2014] 3. Mahadeo Hills Central India
(a) Andaman and Nicobar 4. Mikir Hills North-East India
(b) Nicobar and Sumatra
(c) Maldives and Lakshadweep Which of the above pairs are correctly matched?
(d) Sumatra and Java (a) 1 and 2 only
(b) 2 and 3 only
12. The seasonal reversal of winds is the typical (c) 3 and 4 only
characteristic of [UPSC CSE Pre. 2014] (d) 2 and 4 only
(a) Equatorial climate
(b) Mediterranean climate 17. In India, in which one of the following types of
(c) Monsoon climate forests is teak a dominant tree species?
(d) All of the above climates [UPSC CSE Pre. 2015]
13. If you travel through the Himalayas, you are likely (a) Tropical moist deciduous forest
to see which of the following plants naturally (b) Tropical rain forest
growing there? [UPSC CSE Pre. 2014] (c) Tropical thorn scrub forest
1. Oak (d) Temperate forest with grasslands
2. Rhododendron 18. Consider the following rivers:
3. Sandalwood [UPSC CSE Pre. 2015]
Select the correct answer using the code given below. 1. Vamsadhara
(a) 1 and 2 only 2. Indravati
(b) 3 only 3. Pranahita
(c) 1 and 3 only 4. Pennar
(d) 1, 2 and 3
Which of the above are tributaries of Godavari?
14. Consider the following rivers: (a) 1, 2 and 3 only
[UPSC CSE Pre. 2014] (b) 2, 3 and 4 only
1. Barak (c) 1, 2 and 4 only
2. Lohit (d) 2 and 3 only

Workbook 134
GEOGRAPHY

19. Consider the following States: (a) Andhra Pradesh


[UPSC CSE Pre. 2015] (b) Chhattisgarh
1. Arunachal Pradesh (c) Karnataka
2. Himachal Pradesh (d) Rajasthan
3. Mizoram 25. Recently, linking of which of the following rivers
In which of the above States do ‘Tropical Wet was undertaken? [UPSC CSE Pre. 2016]
Evergreen Forests’ occur? (a) Cauvery and Tungabhadra
(a) 1 only (b) Godavari and Krishna
(b) 2 and 3 only (c) Mahanadi and Son
(c) 1 and 3 only (d) Narmada and Tapti
(d) 1, 2 and 3 26. Which of the following is/are tributary/ tributaries
20. Which one of the following pairs of States of India of Brahmaputra? [UPSC CSE Pre. 2016]
indicates the easternmost and westernmost State 1. Dibang
[UPSC CSE Pre. 2015] 2. Kameng
(a) Assam and Rajasthan 3. Lohit
(b) Arunachal Pradesh and Rajasthan Select the correct answer using the code given below.
(c) Assam and Gujarat (a) 1 only
(d) Arunachal Pradesh and Gujarat (b) 2 and 3 only
21. Consider the following pairs: (c) 1 and 3 only
[UPSC CSE Pre. 2015] (d) 1, 2 and 3

Place of Pilgrimage Location 27. Consider the following statements:


[UPSC CSE Pre. 2017]
1. Srisailam Nallamala Hills 1. In India, the Himalayas are spread over five
2. Omakareshwar Satmala Hills states only.
2. Western Ghats are spread over five states only.
3. Pushkar Mahadeo Hills
3. Pulicat Lake is spread over two States only.
Which of the above pairs is/are correctly matched ?
Which of the statements given above is/are correct?
(a) 1 only
(b) 2 and 3 only (a) 1 and 2 only
(c) 1 and 3 only (b) 3 only
(d) 1, 2 and 3 (c) 2 and 3 only
(d) 1 and 3 only
22. In a particular region in India, the local people
train the roots of living trees into robust bridges 28. If you travel by road from Kohima to Kottayam,
across the streams. As the time passes, these what is the minimum number of States within
bridges become stronger. These unique ‘living India through which you can travel, including the
root bridges’ are found in [UPSC CSE Pre. 2015] origin and the destination? [UPSC CSE Pre. 2017]
(a) Meghalaya (a) 6
(b) Himachal Pradesh (b) 7
(c) Jharkhand (c) 8
(d) Tamil Nadu (d) 9

23. In which of the following regions of India are shale 29. At one of the place in India, if you stand on the
gas resources found? [UPSC CSE Pre. 2016] seashore and watch the sea, ‘you will find that
the sea water recedes from the shore line a few
1. Cambay Basin
kilometres and comes back to the shore, twice
2. Cauvery Basin a day, and you can actually walk on the sea floor
3. Krishna-Godavari Basin when the water recedes. This unique phenomenon
Select the correct answer using the code given below. is seen at [UPSC CSE Pre. 2017]
(a) 1 and 2 only (a) Bhavnagar
(b) 3 only (b) Bheemunipatnam
(c) 2 and 3 only (c) Chandipur
(d) 1, 2 and 3 (d) Nagapattinam
24. Recently, which of the following States has explored 30. With reference to river Teesta, consider the
the possibility of constructing an artificial inland following statements: [UPSC CSE Pre. 2017]
port to be connected to sea by a long navigational 1. The source of river Teesta is the same as that of
channel? [UPSC CSE Pre. 2016] Brahmaputra but it flows through Sikkim.

135 Workbook
GEOGRAPHY

2. River Rangeet originates in Sikkim and it is a Famous Place River


tributary of river Teesta.
3. River Teesta flows into Bay of Bengal on the 1. Pandharpur Chandrabhaga
border of India and Bangladesh. 2. Tiruchirappalli Cauvery
Which of the statements given above is/are correct? 3. Hampi Malaprabha
(a) 1 and 3 only
Which of the pairs given above are correctly
(b) 2 only
matched?
(c) 2 and 3 only
(d) 1, 2 and 3 (a) 1 and 2 only
(b) 2 and 3 only
31. With reference to ‘Indian Ocean Dipole (IOD)’ (c) 1 and 3 only
sometimes mentioned in the news while (d) 1, 2 and 3
forecasting Indian monsoon, which of the
36. What is common to the places known as Aliyar,
following statements is/are correct?
Isapur and Kangsabati? [UPSC CSE Pre. 2019]
[UPSC CSE Pre. 2017]
(a) Recently discovered uranium deposits
1. IOD phenomenon is characterised by a (b) Tropical rain forests
difference in sea surface temperature between (c) Underground cave systems
tropical Western Indian Ocean and tropical (d) Water reservoirs
Eastern Pacific Ocean.
2. An IOD phenomenon can influence an El Nino’s 37. Consider the following pairs:
impact on the monsoon. [UPSC CSE Pre. 2019]
Select the correct answer using the code given below: Glacier River
(a) 1 only
(b) 2 only 1. Bandarpunch Yamuna
(c) Both 1 and 2 2. Bara Shigri Chenab
(d) Neither 1 nor 2
3. Milam Mandakini
32. Which one of the following is an artificial lake? 4. Siachen Nubra
[UPSC CSE Pre. 2018]
(a) Kodaikanal (Tamil Nadu) 5. Zemu Manas
(b) Kolleru (Andhra Pradesh) Which of the pairs given above are correctly
(c) Nainital (Uttarakhand) matched?
(d) Renuka (Himachal Pradesh) (a) 1, 2 and 4 only
(b) 1, 3 and 4 only
33. Consider the following statements:
(c) 2 and 5 only
[UPSC CSE Pre. 2018]
(d) 3 and 5 only
1. The Barren Island volcano is an active volcano
located in the Indian territory. 38. With reference to Ocean Mean Temperature
2. Barren Island lies about 140 km east of Great (OMT), which of the following statements is/are
Nicobar. correct? [UPSC CSE Pre. 2020]
3. The last time the Barren Island volcano erupted 1. OMT is measured up to a depth of 26°C isotherm
was in 1991 and it has remained inactive since which is 129 meters in the south -western Indian
then. Ocean during January–March.
2. OMT collected during January –March can be
Which of the statements given above is/are correct?
used in assessing whether the amount of rainfall
(a) 1 only in monsoon will be less or more than a certain
(b) 2 and 3 only long -term mean.
(c) 3 only
(d) 1 and 3 only Select the correct using the code given below:
(a) 1 only
34. Among the following cities, which one lies on a (b) 2 only
longitude closest to that of Delhi? (c) Both 1 and 2
[UPSC CSE Pre. 2018] (d) Neither 1 nor 2
(a) Bengaluru
39. Siachen Glacier is situated to the
(b) Hyderabad
(c) Nagpur [UPSC CSE Pre. 2020]
(d) Pune (a) East of Aksai Chin
(b) East of Leh
35. Consider the following pairs: (c) North of Gilgit
[UPSC CSE Pre. 2019] (d) North of Nubra Valley

Workbook 136
GEOGRAPHY

40. Gandikota canyon of South India was created by 44. Consider the following pairs of local wind and
which one of the following rivers? their associated features.
[UPSC CSE Pre. 2022]
Local wind Feature
(a) Manjira
(b) Pennar 1. Mango Shower Post monsoon showers
(c) Cauvery famous in Uttar Pradesh.
(d) Tungabhadra
2. Loo Localized thunderstorm
41. Consider the following pairs: during very hot summer in
[UPSC CSE Pre. 2022] northern plains.
3. Kaal Baisakhi Evening thunderstorms
Peak Mountains with torrential downpours
1. Namcha Barwa Garhwal Himalaya common in Bengal
2. Nanda Devi Kumaon Himalaya and Assam
3. Nokrek Sikkim Himalaya Which of the pairs given above is/are correct?
(a) 1 only
Which of the pairs given above is/are correctly
(b) 2 only
matched?
(c) 3 only
(a) 1 and 2 (d) 1, 2 and 3
(b) 2 only
(c) 1 and 3 45. Which of the following can be the reasons behind
(d) 3 only the number of earthquakes in Delhi in recent
times?
42. Consider the following pairs
1. Closeness to main boundary thrust fault.
[UPSC CSE Pre. 2022]
2. Movement of the Indian tectonic plate.
Wetland/Lake Location 3. Loose soil in the Ganga Alluvial Plain.
1. Hokera Wetland Punjab 4. Presence of many weak zones and faults.
Select the correct answer using the code given below:
2. Renuka Wetland Himachal Pradesh
(a) 1, 2 and 3 only
3. Rudrasagar Lake Tripura (b) 2, 3 and 4 only
4. Sasthamkotta Lake Tamil Nadu (c) 1, 3 and 4 only
(d) 1, 2 and 4 only
How many pairs given above are correctly matched?
(a) Only one pair 46. Consider the following statements:
(b) Only two pairs 1. The snowline in western Himalayas is at a higher
(c) Only three pairs altitude than the eastern Himalayas.
(d) All four pairs 2. The average height of western Himalayas is more
43. Consider the following pairs: than the eastern Himalayas.
[UPSC CSE Pre. 2022] 3. Eastern Himalayas have different orogeny
compared to western Himalayas.
Reservoirs States Which of the statements given above is/are correct?
(a) 2 only
1. Ghataprabha Telangana
(b) 1 and 2 only
2. Ghandhi Sagar Madhya Pradesh (c) 2 and 3 only
3. Indira Sagar Andhra Pradesh (d) 1, 2 and 3

4. Maithon Chhattisgarh 47. Which of the following regions are separated by


How many pairs given above ate not correctly Malda fault?
matched? (a) Kachchh mainland and Rann of Kachchh.
(a) Only one pair (b) Satpura and Vindhya ranges.
(b) Only two pair (c) Karbi Anglong plateau and Chota Nagpur
(c) Only three pair plateau.
(d) All four pair (d) Nilgiri hills and Anaimalai hills.

Practice Questions 48. With reference to Indian physical features, which


of the statements given below is/are correct?
1. Karewas are the thick deposits of glacial clay
4.2. Location and Physiography which are useful for the cultivation of Zafran.

137 Workbook
GEOGRAPHY

2. Marusthali is the region in Rajasthan which has Type Feature


arid climate with low vegetation cover.
Bhabar Sand bars, meanders,
Select the correct answer using the code given below:
ox-bow lakes
(a) 1 only
(b) 2 only Tarai Marshy and swampy
(c) Both 1 and 2 conditions
(d) Neither 1 nor 2 Khadar Deposits of heavy
49. Consider the following statements with reference materials like rocks and
boulders
to the Peninsular block of India:
1. The Thar Desert is part of the Peninsular block. Which of the pairs given above is/are correctly
matched?
2. The river valleys in Peninsular block are deep
with high gradients. (a) 1 only
(b) 2 only
Which of the statements given above is/are correct? (c) 3 only
(a) 1 only (d) 1 and 2 only
(b) 2 only
(c) 1 and 2 54. With reference to the Peninsular plateau, which of
the statements given below is incorrect?
(d) Neithr 1 and 2
(a) The Peninsular region is made up of a series of
50. Consider the following statements with reference Patland plateaus.
to the features of Himalayas: (b) The general elevation of the plateau is from east
1. These mountains are tectonic in origin. to west.
2. These are young, weak and flexible unlike the (c) The north-western part of the plateau has relief
Peninsular block. features like ravines and gorges.
(d) Tors are one of the features of the Peninsular
Which of the statements given above is/are correct? plateau.
(a) 1 only
(b) 2 only 55. Consider the following statements regarding the
(c) Both 1 and 2 Great Indian Desert:
(d) Neither 1 nor 2 1. The prevailing southwest monsoon winds tend
to bypass the Thar to the east.
51. Which of the following statements are correct 2. The Soils in this region are predominantly
regarding Himalayan Mountain range? coarse-textured, well-drained, and calcareous.
1. The Shiwalik formations are absent in Arunachal Which of the statements given above is/are correct?
Himalayas. (a) 1 only
2. Dun formations is a feature of Sikkim Himalayas. (b) 2 only
3. Bugyals are summer grasslands in the Himalayan (c) Both 1 and 2
region of Uttarakhand. (d) Neither 1 nor 2
Select the correct answer using the code given below: 56. With reference to the western coast of India,
(a) 1 and 2 only consider the following statements:
(b) 2 and 3 only 1. It provides natural conditions for the
(c) 1 and 3 only development of ports and harbours.
(d) 1, 2 and 3 2. The rivers flowing through this coastal plain do
52. Arrange the following North-Eastern hills in not form any delta.
order starting from North to South: 3. The western coastal plain is broader than the
eastern coast.
1. Lushai Hills
2. Manipur Hills Which of the statements given above is/are correct?
3. Patkai Bum (a) 1 only
(b) 1 and 2 only
4. Naga Hills (c) 2 and 3 only
Select the correct answer using the code given below: (d) 1, 2 and 3
(a) 3-2-1-4 57. In the context of Andaman and Nicobar Islands,
(b) 1-3-4-2 which of the statements given below is/are correct?
(c) 3-4-2-1
1. Saddle peak is the highest mountain peak of
(d) 1-3-2-4 these Islands.
53. Consider the following pairs in the context of the 2. These islands receive convectional rainfall and
Northern plains in India: have an equatorial type of vegetation.

Workbook 138
GEOGRAPHY

Select the correct answer using the code given below: 62. Consider the following events in the geological
(a) 1 only history of India:
(b) 2 only 1. Formation of Gondwana coal
(c) Both 1 and 2 2. Formation of Himalayas
(d) Neither 1 nor 2 3. Marine transgression in western India
58. These are longitudinal valleys formed as a result 4. Formation of Deccan traps
of folding. Overtime these have been filled with What is the correct chronological order of the above
alluvium brought down by Himalayan rivers. events, starting from the earliest time?
The region is rich in biodiversity and is also very (a) 4 – 3 – 1 – 2
fertile. The economy of this region is based on tea, (b) 3 – 1 – 4 – 2
timber, and tourism. (c) 1 – 3 – 4 – 2
On the basis of information given above, identify the (d) 1 – 4 – 2 – 3
geographical region: 63. Consider the following rock systems that make up
(a) Shiwaliks the Indian landmass:
(b) Duars 1. Aryan
(c) Karewas
2. Dravidian
(d) Kayals
3. Archean
59. Consider the following statements about the Great 4. Dharwar
Northern Plains:
What is the correct chronological order of the
1. Rivers often disappear in the Bhabar zones due formation of the above rock systems, starting from
to deposition of rocks and boulders. the earliest time?
2. Bhangar is the old alluvial deposits often (a) 2 – 3 – 1 – 4
impregnated with calcareous concretions. (b) 3 – 2 – 1 – 4
3. Khadar involves the deposition of new layer of (c) 4 – 3 – 2 – 1
alluvium by river flood almost every year. (d) 3 – 4 – 2 – 1
Which of the statements given above are correct?
64. With reference to the salient features of the
(a) 1 and 3 only different rock systems of India, consider the
(b) 2 and 3 only following pairs:
(c) 1, 2 and 3
(d) 1 and 2 only Rock system Feature
60. Consider the following statements about Malwa 1. Dravidian system Found in extra-
Plateau: peninsular region
1. Vindhyan Hills lie at the southern edge of the 2. Dharwar system Valuable minerals
plateau.
2. The plateau has drainage systems, both, towards 3. Gondwana system Coal with highest
the Arabian sea and the Bay of Bengal. carbon content
3. The plateau is covered with black soils. 4. Deccan Traps Black cotton soil
Which of the statements given above are correct? Which of the pairs given above is/are correctly
(a) 1 and 2 only matched?
(b) 2 and 3 only (a) 2 and 4 only
(c) 1 and 3 only (b) 1 and 3 only
(d) 1, 2 and 3 (c) 1, 2 and 4 only
61. Which of the following physiographical features (d) 1, 2, 3 and 4
were formed when the Indo-Australian plate and 65. In the context of the process of formation of
the Eurasian plate converged? Himalayas, consider the following statements:
1. Trans-Himalayas 1. The Himalayas are formed due to intense
2. Purvanchal volcanic outpouring.
3. Lakshadweep Islands 2. The Himalayas are composed of consolidated
4. Arakan Yoma range marine sediments.
Select the correct answer using the code given below: 3. The Indo-Australian plate subducted under the
(a) 1 and 2 only Eurasian plate.
(b) 3 and 4 only Which of the statements given above is/are correct?
(c) 1, 2 and 4 only (a) 1 and 2 only
(d) 1, 2, 3 and 4 (b) 1 and 3 only

139 Workbook
GEOGRAPHY

(c) 2 and 3 only 70. With reference to ‘Duns’, consider the following
(d) 1, 2 and 3 statements:
1. Duns are elongated ridges found in between
66. Which of the following statement(s) support
Lesser Himalayas and Shiwaliks.
the hypothesis that the Himalayan region is still
tectonically active? 2. They are found only in the western part of the
Himalayan range.
1. The Himalayas are continuing to rise in height
every year. Which of the statements given above is/are correct?
2. Earthquakes occur frequently in the region. (a) 1 only
(b) 2 only
3. The region frequently witnesses intense volcanic
(c) Both 1 and 2
activity.
(d) Neither 1 nor 2
4. The Himalayan rivers have been rejuvenated in
recent times. 71. Consider the following pairs:
Select the correct answer using the code given below: Mountain range Region
(a) 1, 2 and 3 only
(b) 1, 2 and 4 only 1. Dhaula Dhar Sikkim
(c) 2, 3 and 4 only 2. Mahabharat Nepal
(d) 1, 3 and 4 only
3. Kailash Range Arunachal Pradesh
67. Which of the following is the correct sequence of Which of the pairs given above is/are correctly
the mountain ranges of Himalayas when arranged matched?
from north to south? (a) 1 and 2 only
(a) Greater Himalayas – Ladakh range – Zanskar (b) 2 only
range – Pir Panjal (c) 1 and 3 only
(b) Ladakh range – Zanskar range – Pir Panjal – (d) 1, 2 and 3
Greater Himalayas
72. Consider the following pairs:
(c) Ladakh range – Greater Himalayas – Zanskar
range – Pir Panjal Mountain Pass Connects
(d) Ladakh range – Zanskar range – Greater
Himalayas – Pir Panjal 1. Zoji La Kashmir valley and
Ladakh
68. Consider the following events in the formation of
‘Karewas’: 2. Baralacha La Lahaul and Leh
1. Formation of a lake 3. Lipu Lekh Kinnaur and Tibet
2. Deposition of glacial clay 4. Shipki La Uttarakhand and Tibet
3. Draining of the lake Which of the pairs given above is/are correctly
4. Formation of a gorge matched?
What is the correct chronological order of the above (a) 1 and 2 only
events, starting from the earliest time? (b) 2 and 4 only
(a) 4 – 1 – 3 – 2 (c) 1 and 3 only
(b) 1 – 4 – 3 – 2 (d) 3 and 4 only
(c) 1 – 2 – 4 – 3 73. Arrange the following mountain ranges from
(d) 4 – 1 – 2 – 3 North to South.
69. Which of the following is/are correct about 1. Vindhya Range
Shiwalik Range of Himalayas? 2. Ajanta range
1. It is predominantly found in the regions of duar 3. Satpura range
formations. 4. Nallamala Hills
2. Southern slopes are gentle while the northern Choose the correct answer using the code given
slopes are steep. below.
3. They are heavily forested in the east, but forest (a) 1 – 3 – 4 – 2
cover decreases in the west. (b) 1 – 3 – 2 – 4
(c) 2 – 1 – 4 – 3
Select the correct answer using the code given below: (d) 2 – 1 – 3 – 4
(a) 1 and 2 only
(b) 3 only 74. With reference to ‘Terai’ region of the Indo-
Gangetic plains, consider the following statements:
(c) 2 and 3 only
(d) 1, 2 and 3 1. It is a narrow tract to the south of Bhangar
region.

Workbook 140
GEOGRAPHY

2. It is an ill-drained and heavily forested region. (c) 3 only


3. It is extensive in the western region and (d) 2 and 3 only
completely absent in the eastern region. 79. Consider the following statements:
Which of the statements given above is/are correct? 1. Andaman and Nicobar Islands formed due to
(a) 1 and 2 only collision between lithospheric plates.
(b) 2 only 2. Hotspot volcanism contributed to the formation
(c) 1 and 3 only of Lakshadweep Islands.
(d) 2 and 3 only
Which of the statements given above is/are correct?
75. Consider the following plateaus of central India: (a) 1 only
1. Baghelkhand (b) 2 only
2. Bundelkhand (c) Both 1 and 2
3. Malwa Plateau (d) Neither 1 nor 2
4. Chotanagpur Plateau 80. Consider the following statements:
5. Marwar Plateau 1. Lakshadweep Islands are characterized by hilly
Which of the following is the correct sequence of the terrain and lush forests.
plateaus when arranged from west to east? 2. Coral reefs are found along the coasts of both
(a) 5 – 3 – 2 – 1 – 4 Lakshadweep and Andaman and Nicobar.
(b) 5 – 3 – 1 – 2 – 4 3. India’s only active volcano is found in Andaman
(c) 3 – 5 – 4 – 2 – 1 and Nicobar.
(d) 3 – 2 – 5 – 1 – 4 Which of the statements given above is/are correct?
76. Consider the following statements: (a) 1 and 2 only
1. Aravali Range is an old fold mountain range (b) 1 and 3 only
aligned in north-east to south-west direction. (c) 2 and 3 only
2. Vindhya Range runs in east-west direction (d) 1, 2 and 3
between the Narmada and the Tapi. 81. With reference to the islands of India, consider the
3. Aravali Range forms the watershed between following pairs:
Chambal river and the Narmada river.
Island Location
Which of the statements given above is/are correct?
(a) 1 only 1. Kiltan Island Lakshadweep
(b) 1 and 2 only 2. Ghoramara Island Gulf of Mannar
(c) 1 and 3 only
(d) 2 and 3 only 3. Pirotan Island Gulf of Kutch
77. With reference to the ‘Western Ghats’, consider the 4. Baratang Island Odisha coast
following statements: Which of the pairs given above is/are correctly
1. They are discontinuous and irregular, dissected matched?
by many rivers. (a) 1, 2 and 3 only
2. They are spread across a total of six states. (b) 2 and 4 only
Which of the statements given above is/are correct? (c) 1 and 3 only
(a) 1 only (d) 1, 2, 3 and 4
(b) 2 only 82. With reference to the coastlines of India, consider
(c) Both 1 and 2 the following statements:
(d) Neither 1 nor 2 1. They formed due to the faulting of
78. With reference to ‘Eastern Ghats’, consider the Gondwanaland.
following statements: 2. The western coast is more indented than the
1. They are a continuous chain of hills running eastern coast.
parallel to the eastern coast of India. Which of the statements given above is/are incorrect?
2. They leave narrow plains between the base and (a) 1 only
the coast. (b) 2 only
3. The hill ranges of the Eastern Ghats do not have (c) Both 1 and 2
structural unity. (d) Neither 1 nor 2
Which of the statements given above is/are correct? 83. Which of the following pair of mountain passes
(a) 1 and 3 only and the states/UTs they are located in, are correctly
(b) 2 only matched?

141 Workbook
GEOGRAPHY

Mountain Pass State/UT 88. Consider the following pairs:

1. Nathu La Sikkim Geothermal Spring State


2. Shipki La Himachal Pradesh 1. Panamik Sikkim
3. Jelep La Sikkim 2. Kheer Ganga Himachal Pradesh
4. Lanak La Jammu and Kashmir 3. Gaurikund Uttarakhand
5. Lipu Lekh pass Uttarakhand Which of the pairs given above is/are correctly
matched?
Select the correct answer using the code given below:
(a) 1 only
(a) 1, 2, 3 and 4 only (b) 2 and 3 only
(b) 2, 3, 4 and 5 only (c) 2 only
(c) 1, 2, 3 and 5 only (d) 1, 2 and 3
(d) 1, 2, 3, 4 and 5
89. Arrange the following Islands of Andaman and
84. Among the following cities, which one lies on a Nicobar from North to South:
latitude closest to tropic of Cancer? 1. Little Andaman
(a) Bhopal 2. Car Nicobar
(b) Jaipur 3. South Andaman
(c) Raipur
4. Great Nicobar
(d) Kolkata
Select the correct answer from the codes given
85. Consider the following pairs: below:
(a) 1-3-4-2
Mountain Pass Location
(b) 3-1-2-4
1. Thal Ghat Mumbai and Pune (c) 1-3-2-4
(d) 3-1-4-2
2. Bhor Ghat Mumbai and Nasik
3. Pal Ghat Coimbatore and Palakkad 90. Consider the following cities of India:
1. Lucknow
4. Chorla Ghat Intersection of Goa, 2. New Delhi
Maharashtra and Karnataka
3. Kolkata
Which of the pairs given above are correctly 4. Shimla
matched? 5. Chandigarh
(a) 1 and 2 only
With reference to the Coriolis force experienced
(b) 1 and 3 only
by the cities mentioned above, which one of the
(c) 2 and 4 only
following is the correct ascending order?
(d) 3 and 4 only
(a) 3-1-2-5-4
86. With reference to Rann of Kutch, consider the (b) 3-2-1-4-5
following statements: (c) 1-3-2-4-5
1. It is famous for wide expanse of saline soil. (d) 1-3-2-5-4
2. Sir Creek, a disputed region located in Rann of 91. With reference to the latitudinal and longitudinal
Kachchh. extent of India, consider the following statements:
3. Mahi is an important river which flows through 1. Ten States in India does not share any
Rann of Kachchh. international borders or a coastline.
Which of the statements given above are correct? 2. North-south extent of Indian mainland is greater
(a) 1 and 2 only than its east- west extent.
(b) 2 and 3 only Which of the statements given above is/are correct?
(c) 1 and 3 only (a) 1 only
(d) 1, 2 and 3 (b) 2 only
(c) Both 1 and 2
87. Which of the following is incorrect regarding
(d) Neither 1 nor 2
Lakshadweep islands?
(a) Entire group of islands is built of coral deposits. 92. With reference to the standard meridian of India,
(b) Eleven-degree channel separates Minicoy from consider the following statements:
the main Lakshadweep archipelago. 1. The longitude 82º 30’E is the standard meridian
(c) Lakshadweep has zero percent of forest area. of India.
(d) It is located north of Maldives and Chagos group 2. It is ahead of the Coordinated Universal Time
of Islands. (UTC) by 6 hours 30 minutes.

Workbook 142
GEOGRAPHY

Which of the statements given above is/are correct? observed in the region.
(a) 1 only 3. It has led to the formation of Deccan traps.
(b) 2 only Select the correct answer using the code given below:
(c) Both 1 and 2 (a) 1 and 3 only
(d) Neither 1 nor 2
(b) 2 and 3 only
93. The standard meridian of India passes through (c) 1 and 2 only
which of the following states? (d) 1, 2 and 3
1. Jammu and Kashmir 98. Which of the following statements is incorrect
2. Madhya Pradesh with reference to Malda fault?
3. Chhattisgarh (a) The northeastern states of India are separated by
4. Odisha the Malda fault from the Chotanagpur Plateau.
5. Andhra Pradesh (b) The down-thrusting of the Malda gap area
6. Maharashtra during the mid-pleistocene period, diverted the
Brahmaputra River system to flow towards the
Select the correct answer using the code given below:
Bay of Bengal.
(a) 1, 2, 5 and 6 only (c) Malda gap was filled by sediment deposits of the
(b) 2, 3 and 5 only Ganga and its tributaries.
(c) 2, 3, 4 and 5 only (d) It is a major fault along the southern boundary
(d) 1, 2 and 6 only of the Shillong plateau.
94. The Tropic of Cancer passes through which of the 99. Which of the following pairs are correctly
following states in India? matched?
1. Rajasthan
2. Chhattisgarh List I List II
3. Manipur Passes Connects
4. Bihar 1. Nathu La Sikkim to Tibet
Select the correct answer using the code given below:
2. Shipki La Himachal Pradesh with
(a) 1 and 2 only Tibet
(b) 1, 3 and 4 only
(c) 2 and 4 only 3. Zoji La Manali and Leh
(d) 1, 2, 3 and 4 4. Khardung La Leh and Siachin
95. Which one of the following countries shares glaciers
border with the least number of states/UTs of Select the correct answer using the code given below:
India? (a) 1, 2, 3 and 4
(a) China (b) 2, 3, and 4 only
(b) Nepal (c) 1, 2, and 4 only
(c) Bangladesh (d) 1 and 3 only
(d) Myanmar
100. Which of the following is incorrect with reference
96. If you travel through the shola forests of western to Peaty soil?
Ghats, you are likely to see which of the following (a) These soils are found in the areas of heavy
features? rainfall and high humidity.
1. Epiphytes like Lichens (b) Peaty soils are rich in humus and organic
2. Ferns and bryophytes grown on trees content.
3. Red-coloured young leaves turning into different (c) The phosphate content of these soils is as high.
colours on maturity (d) These soils are not common in floodplains due
4. Rolling grasslands to the periodic scouring that occurs during flood
events.
Select the correct answer using the code given below:
(a) 1, 2 and 3 only 101. Which of the following rivers in India are western
(b) 2, 3 and 4 only flowing?
(c) 1, 3 and 4 only 1. Shetruniji river
(d) 1, 2, 3 and 4 2. Mahi river
97. Which of the following is/are the Consequences of 3. Vaitarna river
the northward movement of the Indian plate? 4. Sharavati river
1. It is causing rise in the height of the Himalayas. Select the correct answer using the code given below:
2. Frequent minor and major earthquakes are (a) 1 and 2 only

143 Workbook
GEOGRAPHY

(b) 2 and 4 only 106. Recently, a small village known as Susta was in
(c) 1 and 3 only news for which of the following reason?
(d) 1, 2, 3 and 4 (a) Disputed village between India and Nepal due to
changing course of Kosi River.
102. With reference to the Himalayas, consider the
(b) Disputed village between India and China due to
following statements:
changing course of Kosi River.
1. The Western Himalayas rise almost abruptly (c) Disputed village between India and Nepal due to
from the plains located at its foothills. changing course of Gandak River.
2. Higher peaks of Western Himalayas are found to (d) Disputed village between India and China due to
be at greater distance from the plains. changing course of Gandak River.
Which of the statements given above is/are correct?
107. With reference to the Sundarbans delta, consider
(a) 1 only the following statements:
(b) 2 only
1. Sundarbans delta is formed by the floodplain of
(c) Both 1 and 2
Ganges, the Brahmaputra and Meghna River.
(d) Neither 1 and 2
2. More than 50% of the Sundarbans delta is
103. Consider the following pairs: located in Bangladesh.
3. It is sparsely to moderately populated region in
City Located along River the world.
1. Nasik Narmada Which of the statements given above is/are correct?
2. Vijayawada Krishna (a) 1 and 2 Only
(b) 2 and 3 Only
3. Cuttack Mahanadi (c) 1 and 3 Only
4. Ferozepur Sutlej (d) 1, 2 and 3
Which of the pairs given above is/are correct? 108. With reference to the Brahmaputra River which
(a) 1, 2 and 3 only drains most of the part of North-Eastern India,
(b) 2, 3 and 4 only Consider the following statements:
(c) 1, 3 and 4 only 1. Lohit is the right bank tributary of Brahmaputra
(d) 1, 2 and 4 only River.
104. With reference to the physiographic features in 2. More than 50% of the Brahmaputra basin is
the Himalayan region, consider the following covered with water bodies.
statements: 3. Jaldhaka river joins it in the plains of Bangladesh.
1. River Indus flows between Zaskar ranges and Which of the statements given above is/are correct?
Ladakh ranges. (a) 1 and 2 only
2. K-2, the second highest peak in the world is (b) 2 and 3 only
located in the Ladakh Ranges. (c) 3 only
3. Kailash Range is an extension of Trans- (d) 1, 2 and 3
Himalayas. 109. Which of the following river forms Inland
Which of the statements given above is/are correct? drainage basin?
(a) 1 and 2 Only 1. Kantu river
(b) 2 and 3 Only 2. Sukri River
(c) 1 and 3 Only 3. Sabarmati River
(d) 1, 2 and 3
4. Ghaggar river
105. Consider the following pairs 5. Kakni river
River Cultural Significance Select the correct answer using the code given below:
(a) 1 and 5 Only
1. Cauvery River Aadi Perukku Festival (b) 2, 3 and 5 Only
2. Pamba River Mandala- (c) 1, 2, 3 and 4 Only
Makaravilakku Festival (d) 1, 2, 4 and 5 Only
3. Shipra River - Kumbh Mela 110. Arrange the following west flowing rivers from
Which of the statements given above is/are correct? north to south.
(a) 1 and 2 Only 1. Kalinadi River
(b) 2 and 3 Only 2. Mandovi River
(c) 1 and 3 Only 3. Kajavi River
(d) 1, 2 and 3 4. Ambika River

Workbook 144
GEOGRAPHY

Which of the following sequence of rivers is correct? 1. Moderating influence of the sea
(a) 3-4-1-2 2. Higher atmospheric pressure over the peninsular
(b) 2-3-4-1 region
(c) 4-3-2-1 3. Proximity of the peninsular landmass to equator
(d) 3-4-2-1 Select the correct answer using the code given below:
111. With reference to Indian Climate, consider the (a) 1 only
following statements: (b) 2 and 3 only
1. The northeast trade winds blow from land to sea (c) 1 and 3 only
in North India during the winter season. (d) 1, 2 and 3
2. There is no counter-equatorial current in the 116. Which of the following regions lie in the rain
Indian ocean during winters. shadow zone of the South west monsoon?
Which of the statements given above is/are correct? 1. Interior parts of Maharashtra
(a) 1 only 2. Madhya Pradesh
(b) 2 only 3. Telangana
(c) Both 1 and 2 4. Western Himalayan region
(d) Neither 1 nor 2
Select the correct answer using the code given below:
112. Consider the following statements with reference (a) 1 and 3 only
to tropical easterly jet streams: (b) 2, 3 and 4 only
1. It causes the reversal in upper air circulation. (c) 2 and 4 only
2. The advent of easterly jet stream is held (d) 1, 2, 3 and 4
responsible for the burst of the monsoon in 117. Which of the following is/are the reasons behind
India. the occurrence of dry spells of monsoons?
3. It does not extend beyond 15°N latitude in the 1. Less frequency of rain-bearing storms along the
upper atmosphere. Inter Tropical Convergence Zone
Which of the statements given above is/are correct? 2. Blowing of the monsoon winds parallel to the
(a) 1 and 2 only west coast
(b) 2 only 3. Southward shifting of the monsoon trough
(c) 1 and 3 only 4. Pressure gradient at Gangetic plains weakens
(d) 1, 2 and 3
Select the correct answer using the code given below:
113. Which of the following is the effect of El Nino (a) 1 and 2 only
Southern Oscillation (ENSO) on Indian climate? (b) 2, 3 and 4 only
(a) The El Nino effect makes winter weather (c) 1, 3 and 4 only
conditions harsher over the Northern plains. (d) 1, 2, 3 and 4
(b) It has no impact on the development of cyclones
in Bay of Bengal region. 118. Consider the following pairs:
(c) It determines the strength of southwest Region Favourable Crop
monsoons in India.
(d) It is the major cause for the seasonal reversal of 1. Eastern Rajasthan Barley
winds in the Indian ocean. 2. West Coastal Plains Jute
114. Consider the following statements with respect to 3. Coast of Paddy
the jet streams: Maharashtra
1. The subtropical westerly jet streams blowing Which of the pairs given above is/are correct?
over India in winter intensifies the northeast (a) 1 only
monsoons. (b) 2 and 3 only
2. The jet streams impact the time of onset of (c) 2 only
southwest monsoons. (d) 1 and 3 only
Which of the statements given above is/are correct?
119. Which of the following statements is/are correct
(a) 1 only regarding the impact of climate change?
(b) 2 only
1. Sea-surface temperature of Indian Ocean may
(c) Both 1 and 2
increase by 1 to 2°C leading to intensification of
(d) Neither 1 nor 2
cyclones.
115. Which of the following is/are the reasons for 2. There are likely to be 300 days in a year with
the absence of well-defined winter season in maximum temperatures above 35°C in case of
Peninsular India? 1.5 to 2°C global warming.

145 Workbook
GEOGRAPHY

3. El Nino–Southern Oscillation will lead to Which of the following statements given above is/are
rainfall variability and compounded events in correct?
coming decades. (a) 1 and 2 only
Select the correct answer using the code given below: (b) 2 and 3 only
(a) 1 and 2 only (c) 1 and 3 only
(b) 2 only (d) 1, 2 and 3
(c) 1 and 3 only 124. Consider the following statements with respect to
(d) 3 only the Black soil:
120. Which of the following is/are the features of wet 1. It is highly retentive of moisture.
tropical evergreen vegetation? 2. They are rich in phosphorous, nitrogen and
1. Dense Forest with thick spread of grass. organic matter.
2. Low vegetal species per unit area. 3. Black soils of uplands are of low fertility but they
3. Found in north-eastern hills in India. are fertile in the valleys.
4. Plants found are Mahogony, Cinchona and 4. Cotton, millets and sugarcane are some of the
Bamboos. important crops grown in this soil.
Select the correct answer using the code given below: Which of the statements given above are correct?
(a) 1, 2 and 4 only (a) 1, 3 and 4 only
(b) 3 and 4 only (b) 1, 2 and 3 only
(c) 1, 2 and 3 only (c) 1 and 3 only
(d) 1, 2, 3 and 4 (d) 4 only
121. If you pass through this region, you are most likely 125. Consider the following statements with respect to
to find tall grasslands, scrub savannah, sal forests the Mountain Soils:
clay rich swamps and high biological diversity. 1. It is heterogeneous in nature and its character
Due to the very high humidity, this region is changes with parent rocks.
very marshy. Most of the region lies especially in 2. They are rich in humus but are deficient in
Punjab, Uttar Pradesh and Uttarakhand, and has potash.
been turned into agricultural land which gives
good crops of sugarcane, rice and wheat. You are Which of the statements given above is/are correct?
likely in the belt of: (a) 1 only
(a) Bhabar (b) 2 only
(b) Khadar (c) Both 1 and 2
(c) Terai (d) Neither 1 nor 2
(d) Bhangar 126. With respect to the Land Degradation and
122. With reference to the Khejri Tree, consider the Desertification in India, consider the following
following statements: statements.
1. It covers about two-thirds of the total 1. India witnessed an increase in the level of
geographical area of Rajasthan. desertification in majority of the states and
2. The fruit of this tree is used to make the very Union territories in the last decade.
popular local dish ‘Sangri’. 2. Vegetation degradation is found to be responsible
3. The tree has inspired the Chipko Movement. for about 50% of desertification in the country.
4. It is the state tree of Rajasthan. 3. Water erosion resulting in Badland Topography
is an initial stage of desertification.
Which of the statements given above is/are correct?
4. Sand encroachment by wind, reduces fertility
(a) 1,2 and 4 only
(b) 2 and 3 only of the soil making the land susceptible to
(c) 1,3 and 4 only desertification.
(d) 1, 2, 3 and 4 Which of the following statements given above is/are
correct?
123. With reference to the geomorphology of the Great
(a) 1, 2 and 3 only
Plains of North India, consider the following
(b) 1, 3 and 4 only
statements.
(c) 2, 3 and 4 only
1. Alluvial soil is rich in potash and poor in (d) 1, 2 and 4 only
phosphorous.
2. Khadar is a new alluvium whereas Bhangar is an
old alluvium. 4.3. Drainage
3. Unlike Khadar, Bhangar contains calcareous 127. Consider the following bridges constructed on
concretions. various rivers:

Workbook 146
GEOGRAPHY

Important Bridges River (a) 1 and 3 only


(b) 1 and 4 only
1. Bhupen Hazarika Dibang (c) 2, 3 and 4 only
2. Bogibeel Brahmaputra (d) 1, 2 and 4 only

3. Mahatma Gandhi Ganga 132. Consider the following statements regarding the
Setu river Indus:
Which of the pairs given above are correctly 1. The Indus drainage is an example of antecedent
matched? drainage.
(a) 1 and 2 only 2. River Galwan is the right bank tributary of river
(b) 1 and 3 only Indus.
(c) 2 and 3 only 3. In India, the river flows only through Union
(d) 1, 2 and 3 Territory of Ladakh and Jammu and Kashmir.
Which of the statements given above is/are correct?
128. “This lake is India’s largest inland Salt Lake.
(a) 1 only
The lake, a shallow, elliptical wetland, has been
(b) 1 and 2 only
designated a Ramsar site because of the staggering
(c) 2 and 3 only
diversity of birds it attracts. It is best known for
(d) 1 and 3 only
the thousands of flamingos that migrate here every
year. Recently, the Lake came under the spotlight 133. Consider the following pairs:
due to Avian botulism, the deadly disease that
killed thousands of birds”. Indian Rivers Origin
The lake described in the above paragraph is: 1. Narmada Amarkantak Plateau
(a) Pulikat Lake 2. Godavari Sahyadri Hills
(b) Sambhar Lake
(c) Rudrasagar Lake 3. Krishna Trimbakeshwar Plateau
(d) Lonar Lake 4. Kaveri Brahmagiri Hills
129. In the context of drainage system of river Sutlej, Which of the pairs given above are NOT correctly
which of the following statement is correct? matched?
(a) It originates from a glacier near Bokhar Chu in (a) 1 and 2 only
the Tibetan region. (b) 1 and 4 only
(b) It flows through the Kullu valley. (c) 2 and 3 only
(c) It is an antecedent river feeding the canal system (d) 3 and 4 only
of the Bhakra Nangal project. 134. Which of the following are reasons for frequent
(d) The river is still in its youth stage and yet it forms floods in regions surrounding Assam?
meanders.
1. Brahmaputra River unloads huge sediment in
130. Consider the following statements: the region brought from the uplands.
1. Chota Nagpur plateau presents a radial drainage 2. Floodplains of the region are characterized by
pattern. unplanned urbanization.
2. Karbi-Anglong plateau lies in the west to the 3. Assam is an earthquake-prone region.
Meghalaya Plateau. Select the correct answer using the code given below:
Which of the statements given above is/are correct? (a) 1 and 2 only
(a) 1 only (b) 2 and 3 only
(b) 2 only (c) 1 and 3 only
(c) 1 and 2 (d) 1, 2 and 3
(d) Neither 1 and 2
135. With reference to river Tapi, consider the following
131. Which of the following are the reasons for lack of statements:
formation of delta by peninsular rivers falling into 1. It originates from Amarkantak plateau.
the Arabian sea? 2. Purna and Panjhra are important tributaries of
1. Such rivers cover larger distance from their river Tapi.
point of origin to sea. 3. It flows through a rift valley.
2. Presence of Hard rocks. 4. It flows exclusively in Madhya Pradesh and
3. Presence of Steep slopes. Gujarat.
4. Flowing through faults. Which of the statements given above are correct?
Select the correct answer using the codes given (a) 1 and 2 only
below: (b) 1, 2 and 4 only

147 Workbook
GEOGRAPHY

(c) 2 and 3 only Which of the pairs given above are correctly
(d) 2, 3 and 4 only matched?
136. Consider the following pairs: (a) 1 and 2 only
(b) 1 and 4 only
National Waterways Route (c) 1, 2 and 3 only
1. National Sadiya-Dhubri stretch (d) 1, 3 and 4 only
waterway-2 of Brahmaputra River
140. Which of the following hills/range depict the
2. National Lakhipur-Bhanga radial form of drainage pattern?
waterway-3 stretch of River Barak (a) Nilgiri Hills
3. National Kollam-Kottapuram (b) Kumaon Hills
waterway-6 stretch of West coast (c) Sahyadari Hills
canal (d) Amarkantak range
Which of the pair(s) given above is/are not correctly
matched? 141. Consider the following statements:
(a) 1 only 1. In an antecedent drainage pattern, the river cuts
(b) 1 and 3 only through uplifted portions of land.
(c) 1 and 2 only 2. In a superimposed drainage pattern, the river
(d) 2 and 3 only flows in relation to the topology of the area.
137. Consider the following statements about Lonar 3. In a concordant drainage pattern, the river cuts
lake: through the exposed rock strata and maintains
1. It is a salt water lake located in peninsular India. its initial path.
2. The lake was formed due to meteorite hit the Which of the statements given above is/are correct?
earth. (a) 1 only
3. This has been identified as a National Geo- (b) 1 and 2 only
Heritage Monument and as a Ramsar site. (c) 2 and 3 only
Which of the statements given above is/are correct? (d) 1, 2 and 3
(a) 1 and 2 only 142. Consider the following pairs:
(b) 1 and 3 only
(c) 2 and 3 only River Tributary
(d) 1, 2 and 3
1. Ganga Son
138. With Reference to the river Yamuna, which of the 2. Yamuna Chambal
following statements are correct?
1. The river has its source in the Satopanth Glacier. 3. Godavari Bhima
2. It is the western most tributary of River Ganga. 4. Krishna Pennar
3. River Sind, Chambal, Betwa and Ken join on its Which of the pairs given above is/are correctly
left bank. matched?
4. Hathni Kund Barrage is located on River (a) 1 and 2 only
Yamuna. (b) 1, 2 and 4 only
Select the correct answer using the codes given (c) 2 and 3 only
below: (d) 1 and 4 only
(a) 1 and 4 only
(b) 1 and 3 only 143. The drainage basin of Godavari River is distributed
(c) 2 and 3 only across which of the following states?
(d) 2 and 4 only 1. Madhya Pradesh
139. Consider the following pairs: 2. Orissa
3. Chhattisgarh
National Park Rivers Flowing 4. Karnataka
1. Panna Ken Select the correct answer using the code given below:
2. Manas Brahmaputra (a) 1, 2 and 4 only
3. Simlipal Mahanadi (b) 2, 3 and 4 only
(c) 1 and 3 only
4. Nagarhole Kabini (d) 1, 2, 3 and 4

Workbook 148
GEOGRAPHY

144. Consider the following pairs: 148. Which of the following is/are characteristics of the
Peninsular Drainage System?
River Origin 1. Senile topography
1. Chambal Vindhyas 2. Antecedent drainage
2. Son Amarkantak Plateau 3. Flow through shallow valleys
3. Banas Mahadeo hills Select the correct answer using the code given below:
(a) 1 and 3 only
4. Damodar Satpuras (b) 1 and 2 only
Which of the pairs given above is/are correctly (c) 2 and 3 only
matched? (d) 1, 2 and 3
(a) 1 only 149. Arrange the following left bank tributaries of
(b) 1 and 2 only Ganga from west to east:
(c) 2, 3 and 4 only
1. Burhi Gandak
(d) 1, 2, 3 and 4
2. Gomti
145. Which of the following rivers flow into the 3. Ramganga
Arabian sea? 4. Ghaghara
1. Mahi Select the correct answer using the code given below:
2. Vaigai (a) 2-3-1-4
3. Luni (b) 2-4-1-3
4. Mandovi (c) 3-2-4-1
Select the correct answer using the code given below: (d) 3-4-2-1
(a) 1 and 3 only 150. Consider the following statements with reference
(b) 2 and 4 only to “river regimes”:
(c) 1 and 4 only 1. It is the volume of water flowing in a river
(d) 1, 3 and 4 only measured over time.
146. Consider the following pairs: 2. Ganges has the minimum flow in January due to
its monsoonal regime.
Multipurpose Project River
Which of the statements given above is/are incorrect?
1. Hirakud Dam Mahanadi (a) 1 only
2. Bhakra Nangal Sutlej (b) 2 only
Dam (c) Both 1 and 2
(d) Neither 1 nor 2
3. Gandhi Sagar Dam Banas
4. Nagarjuna Sagar Godavari 151. Consider the following pairs:
Dam 1. National Waterway 2 ‘Brahmaputra River’
Which of the pairs given above is/are correctly
2. National Waterway 3 ‘West Coast Canal’
matched?
(a) 1 and 2 only 3. National Waterway 4 ‘Barak River’
(b) 2 and 3 only Which of the pairs given above are correctly
(c) 1 and 3 only matched?
(d) 1, 2 and 4 only (a) 1 and 2 only
147. Consider the following statements with reference (b) 1 and 3 only
to peninsular rivers: (c) 2 and 3 only
(d) 1, 2 and 3
1. The Peninsular drainage system is older than the
Himalayan one.
2. Peninsular rivers are generally characterized
4.4. Indian Climate
by fixed course, absence of meanders and non- 152. With reference to Inter Tropical Convergence
perennial flow of water. Zone (ITCZ), consider the following statements:
Which of the statements given above is/are correct? 1. By the month of July, the ITCZ shifts northwards
(a) 1 only roughly parallel to 20°-25° N latitude.
(b) 2 only 2. Northern shift of ITCZ is related with the
(c) Both 1 and 2 withdrawal of the tropical easterly jet stream
(d) Neither 1 nor 2 from the north Indian plain.

149 Workbook
GEOGRAPHY

3. ITCZ attracts the maritime tropical air mass 1. Entire southern India receives its major portion
from the southern hemisphere. of rainfall from retreating monsoon.
Which of the above statements is/are correct? 2. As we move from east to west the intensity of
(a) 1 only rainfall decreases in northern plains.
(b) 2 and 3 only Which of the statements given above is/are correct?
(c) 1 and 3 only (a) 1 only
(d) 1, 2 and 3 (b) 2 only
153. Which of the following statements is/are correct (c) Both 1 and 2
about the retreating monsoon season? (d) Neither 1 nor 2
1. Clear skies 158. With reference to Indian climate, which of the
2. Shifting of monsoon trough southwards following statements is/are incorrect?
3. Rise in diurnal range of temperature 1. A positive Indian Ocean Dipole can neutralize
Select the correct answer using the code given below: the effect of an El Nino year.
(a) 1 and 2 only 2. Tropical cyclones originate more commonly in
(b) 1 and 3 only the Bay of Bengal than in the Arabian Sea.
(c) 1 only 3. India receives western disturbances only during
(d) 1, 2 and 3 the month of December and January.
Select the correct the answer using the code given
154. Consider the following statements:
below:
1. The duration of the monsoon decreases from
(a) 1 only
southern India to northern India.
(b) 1 and 2 only
2. The amount of annual rainfall in the northern (c) 2 only
plains of India decreases from east to west. (d) 3 only
Which of the statements given above is/are correct?
(a) 1 only 159. “These are dreaded evening thunderstorms in
(b) 2 only Bengal and Assam. Their notorious nature can
(c) Both 1 and 2 be understood from the local nomenclature of
(d) Neither 1 nor 2 ‘Kalbaisakhi’, a calamity of the month of Baisakh.
These showers are useful for tea, jute and rice
155. Which of the following statements is/are correct cultivation. In Assam, these storms are also known
regarding the variability of rainfall? as “Bardoli Chheerha”.
1. The variability of rainfall in western Ghats is The above paragraph refers to which among the
more than that in western part of Rajasthan. following?
2. The Variability of rainfall is lowest at the place (a) Mango Shower
with lowest average annual rainfall. (b) Blossom Shower
Select the correct answer using the code given below: (c) Norwesters
(a) 1 only (d) Loo
(b) 2 only 160. Consider the following statements:
(c) Both 1 and 2 1. Hydrological Drought results when the
(d) Neither 1 nor 2 availability of water in different storages falls
156. With reference to the climatic regions of India below what the precipitation can replenish.
according to koeppan’s scheme, which of the 2. Agricultural drought is when the productivity
following statements is/are correct? of a natural ecosystem fails due to shortage of
1. The climate is of cool temperate type if mean water.
temperature of the coldest month is over minus Which of the statements given above is/are incorrect?
3°C. (a) 1 only
2. The climate is of tropical type when mean (b) 2 only
monthly temperature throughout the year is (c) Both 1 and 2
over 18°C. (d) Neither 1 nor 2
Select the correct answer using the code given below: 161. With reference to the winter season, which of the
(a) 1 only following statements is/are correct?
(b) 2 only 1. The tropical cyclones originated during winter
(c) Both 1 and 2 season strikes Tamil Nadu, Andhra Pradesh,
(d) Neither 1 nor 2 Orissa and West Bengal.
157. With reference to monsoon in India, consider the 2. The western cyclonic disturbances in this season
following statements: are brought by sub-tropical westerly Jet streams.

Workbook 150
GEOGRAPHY

Select the correct answer using the code given below: (a) 4 and 5 only
(a) 1 only (b) 2, 3 and 4 only
(b) 2 only (c) 1, 2 and 4 only
(c) Both 1 and 2 (d) 1, 2, 3, 4 and 5
(d) Neither 1 nor 2 166. Consider the following statements:
162. Which of the following are associated with the 1. Thar Desert experiences the highest diurnal
break in monsoon? range of temperature in India.
1. Weakening of tropical easterly jet stream in the 2. Both diurnal and mean annual temperature
upper troposphere. ranges are low in coastal regions.
2. Replacing of south-easterlies wind over northern Which of the statements given above is/are correct?
India by westerly winds flowing over the Indian (a) 1 only
plains. (b) 2 only
3. Winds blow parallel to the eastern coast. (c) Both 1 and 2
(d) Neither 1 nor 2
Select the correct answer using the code given below:
(a) 1 and 2 only 167. Which of the following is/are correct with respect
(b) 3 only to the weather functioning in winter?
(c) 1 and 3 only 1. High pressure prevails over North India due to
(d) 1, 2 and 3 low temperature.
163. Which of the following statements given below is/ 2. Himalayan belt receives snowfall because of the
arrival of western cyclonic disturbances.
are correct?
1. During Winter, the rainfall goes on increasing Select the correct answer using the code given below:
from west to east in peninsular regions and goes (a) 1 only
on decreasing from west to east in the extra (b) 2 only
peninsular regions. (c) Both 1 and 2
2. During Summer, the rainfall goes on decreasing (d) Neither 1 nor 2
from west to east in the peninsular region and 168. Consider the following terms associated with the
goes on increasing from west to east in extra Indian weather conditions:
peninsular region. 1. Coriolis force deflects winds towards the left in
Select the correct answer using the code given below: the northern hemisphere and towards right in
(a) 1 only the southern hemisphere.
(b) 2 only 2. Jet Streams develop where air masses of differing
(c) Both 1 and 2 temperatures meet.
(d) Neither 1 nor 2 3. Western cyclonic disturbances influence the
weather of the north-western India.
164. Which of the following statements is/are correct
regarding the Bay of Bengal branch of summer Which of the statements given above is/are correct?
season? (a) 3 only
1. The Chotanagpur plateau gets its entire rainfall (b) 2 and 3 only
(c) 2 only
from the Bay of Bengal branch of the monsoon
(d) 1, 2 and 3
winds.
2. The entire Bay of Bengal branch of summer 169. With reference to the Intertropical convergence
season moves over India. zone (ITCZ), consider the following statements:
Select the correct answer using the code given below: 1. It is a belt of converging trade winds and
(a) 1 only descending air that encircles the Earth near the
equator.
(b) 2 only
(c) Both 1 and 2 2. This convergence zone moves north or south
(d) Neither 1 nor 2 with the apparent movement of the sun.
3. It is known as doldrums.
165. Which of the following factors is/are responsible Which of the statements given above is/are correct?
for changes in Indian climate?
(a) 1 only
1. Continentality (b) 2 and 3 only
2. Western Disturbances (c) 2 only
3. Upper air circulation (d) 1, 2 and 3
4. Altitude
170. El Nino is the unusual warming of surface waters
5. Relief in the eastern tropical Pacific Ocean. Which of the
Select the correct answer using the code given below: following is/are the effects of the El Nino?

151 Workbook
GEOGRAPHY

1. Adversely affects monsoon rainfall in India Select the correct answer using the code given below:
2. Good for cyclogenesis in the Bay of Bengal (a) 1 only
3. Suppresses cyclogenesis in the Arabian Sea (b) 2 only
Select the correct answer using the code given below: (c) Both 1 and 2
(d) Neither 1 nor 2
(a) 1 only
(b) 2 and 3 only 175. In the context of ‘break’ monsoon period, consider
(c) 2 only the following statements:
(d) 1, 2 and 3 1. Heavy rainfall occurs during the monsoon break
171. Consider the following statements: over the sub-Himalayan regions.
1. Heating of the Tibetan plateau in summers 2. Monsoon breaks are caused by the southward
results in the replacement of westerly jet stream shifting of the monsoon trough.
by easterly jet stream. 3. Tropical cyclones originating in the Bay of
2. The rainfall pattern of the south-west monsoon Bengal can trigger monsoon break.
can be determined by the frequency of tropical Which of the statements given above is/are correct?
depression. (a) 1 and 3 only
Which of the statements given above is/are incorrect? (b) 2 and 3 only
(a) 1 only (c) 3 only
(b) 2 only (d) 1, 2 and 3
(c) Both 1 and 2
(d) Neither 1 nor 2 176. Consider the following statements regarding the
retreating monsoon:
172. In India, winter monsoon contributes only 11% to 1. The phenomenon is marked by absence of
India’s annual rainfall of 1,187 mm, compared to tropical cyclones in the Bay of Bengal.
about 75% in the summer monsoon season. Which
2. The October heat phenomena is observed during
of the following is/are the probable reasons?
this monsoon.
1. The winter monsoon winds carry little humidity
as they move from land to sea. Which of the statements given above is/are correct?
2. The rainfall reduces due to anti cyclonic (a) 1 only
circulation on land. (b) 2 only
(c) Both 1 and 2
Select the incorrect answer using the code given
(d) Neither 1 nor 2
below
(a) 1 only 177. Consider the following pairs:
(b) 2 only
(c) Both 1 and 2 Koeppen Climate Type Location
(d) Neither 1 nor 2 1. Monsoon with West Coast of India
173. Which of the following factors is/are responsible short dry season
for south-west monsoons in India? 2. Semi-arid steppe Western Rajasthan
1. Low pressure on the landmass and high pressure climate
around seas
3. Monnsoon with dry Jammu & Kashmir
2. Presence of the low-pressure area east of winter
Madagascar
3. Heating of the Tibetan Plateau Which of the statements given above is/are correct?
4. Movement of the westerly jet stream to the north (a) 1 and 2 only
of the Himalayas (b) 2 only
(c) 2 and 3 only
Select the correct answer using the code given below: (d) 1, 2 and 3
(a) 2, 3 and 4 only
(b) 1, 2 and 3 only 178. With the reference to tropical cyclones consider
(c) 1, 3 and 4 only the following statements:
(d) 1, 2, 3 and 4 1. Tropical cyclones are violent storms that
originate over oceans in tropical areas.
174. The Tamil Nadu coast remains relatively dry
during the south-west monsoon period. Which of 2. A mature tropical cyclone is characterised by
the following factors is/are responsible for it? the strong spirally circulating wind around the
1. The coast lies in the rain shadow effect of the centre, called the eye.
Arabian Sea branch of the monsoon. 3. Tropical cyclones are less destructive in nature
2. Bay of Bengal branch of the monsoon flows than extra-tropical cyclones.
parallel to the Tamil Nadu coast. Which of the statements given above is/are correct?

Workbook 152
GEOGRAPHY

(a) 1 and 2 only (b) 2 and 3 only


(b) 1 and 3 only (c) 3 only
(c) 2 and 3 only (d) 1, 2 and 3
(d) 1, 2 and 3
183. Consider the following statements with respect to
179. With the reference to Flash floods consider the the black soils:
following statements: 1. It is found in parts of Tamil Nadu.
1. There are sudden surges in water levels during or 2. It is clayey, deep and impermeable.
following an intense spell of rain. 3. It is rich in phosphorous, nitrogen and organic
2. They are usually associated with cloud bursts or matter.
storms. Which of the statements given above is/are correct?
Which of the following statements given above is/are (a) 2 only
correct? (b) 1 and 2 only
(a) 1 only (c) 2 and 3 only
(b) 2 only (d) 1, 2 and 3
(c) Both 1 and 2 184. Consider the following statements with respect to
(d) Neither 1 nor 2 the red soils:
1. Red soil form on crystalline igneous rocks in
4.5. Soils areas of low rainfall.
180. Which of the following is/are the factors 2. The reddish colour of the soil is due to diffusion
responsible for the formation of soil? of iron in metamorphic rocks.
3. They are highly rich in nitrogen and humus.
1. Topography
2. Parent Material Which of the statements given above is/are correct?
3. Human Activity (a) 1 and 3 only
(b) 2 only
4. Biological Activity (c) 1 and 2 only
5. Time (d) 2 and 3 only
Select the correct answer using the code given below:
185. These soils range from red to brown in colour.
(a) 1, 2 and 4 only They are generally sandy in structure and saline
(b) 1, 2 and 5 only in nature. These soils are developed in western
(c) 1, 2, 4 and 5 only Rajasthan. They contain little humus and organic
(d) 1, 2, 3, 4 and 5 matter.
181. Which of the following practices are considered Which soil has been described above?
appropriate for soil conservation in India? (a) Saline soil
1. Planting of Shelterbelts (b) Arid soil
2. Contour bunding (c) Forest soil
3. Sand fences. (d) Peaty soil
Which of the statements given above is/are correct? 186. With reference to the saline soils, consider the
(a) 1 and 2 only following statements:
(b) 1 and 3 only 1. They occur in both waterlogged and swampy
(c) 2 and 3 only areas.
(d) 1, 2 and 3 2. The salinity in soil can be caused due to dry
climate and poor drainage.
182. Consider the following pairs:
3. Adding urea can help in leaching salt from the
Factors Influence on soil soil.
formation Which of the statements given above is/are correct?
1. Parent rock Determines soil (a) 2 and 3 only
permeability (b) 1 and 3 only
(c) 1 and 2 only
2. Relied Determines (d) 1, 2 and 3
accumulatio of soil
187. Consider the following statements about the peaty
3. Micro-organism Affects the rate of soils:
humas formation 1. These soils are formed in the areas of heavy
Which of the pairs given above is/are correctly rainfall and high humidity.
matched? 2. They are largely found on the coasts of Goa and
(a) 1 only Maharashtra.

153 Workbook
GEOGRAPHY

Which of the statements given above is/are correct? (a) 1 only


(a) 1 only (b) 1 and 2 only
(b) 2 only (c) 2 and 3 only
(c) Both 1 and 2 (d) 3 only
(d) Neither 1 nor 2
188. Consider the following pairs:
4.6. Natural Vegetation
192. Match the following pairs of Tiger reserves in
Soils in India Favourable for India and their characteristics
cultivation of
1. Laterite soils Wheat Tiger Reserves Characteristics

2. Alluvial soils Maize 1. Nagarahole A. endangered species


of Mouse deer was re-
3. Black soils Cotto introduced here.
Which of the pairs given above is/are correctly 2. Amrabad B. Three large cats-
matched? Bengal tiger, Indian
(a) 2 and 3 only leopard and clouded
(b) 3 only leopard are found.
(c) 1 and 3 only
3. Pakhui C. home to 45 tribal
(d) 1, 2 and 3
settlements locally
189. Which of the following methods can help in known as “Haadis”.
preventing soil erosion? Which of the following pairs is/are correctly
1. Contour bunding matched?
2. Cover cropping (a) 1-C, 2-B, 3-A
3. Gully plugging (b) 1-C, 2-A, 3-B
4. Crop rotation (c) 1-A, 2-B, 3-C
(d) 1-A, 2-C, 3-B
Select the correct answer using the code given below:
(a) 1, 2 and 3 only 193. Which of the following statement is incorrect in
(b) 2, 3 and 4 only reference to National Parks?
(c) 1, 2 and 4 only (a) National Parks are declared under the Wildlife
(d) 1, 2, 3 and 4 (Protection) Act, 1972.
(b) National Parks can be declared both by State
190. With reference to Soil types in India, consider the government and Central government.
following pairs: (c) No human interference in any form is allowed in
National Parks.
Soil Type Region Where (d) All the States in India have atleast one or more
Commonly found national park.
1. Alluvial Soil Gangetic Plains 194. Consider the following pairs of species and their
2. Black Soil Deccan Plateau locations.
3. Red soil Deltas of Eastern coast Species Location
4. Arid Soil Western Rajasthan 1. Kolar Leaf Nosed A. Rajasthan
Which of the pairs given above are correctly Bat
matched? 2. Caracal B. Ladakh
(a) 1 and 4 only
3. Seabuckthorn C. Karnataka
(b) 1 and 3 only
(c) 3 and 4 only Which of the pairs given above is/ are correctly
(d) 1, 2 and 4 only matched?
(a) 1-C, 2-A, 3-B
191. Consider the following statements about Laterite (b) 1-A, 2-B, 3-C
soil: (c) 1-B, 2-C, 3-A
1. The laterite soil develops under tropical and (d) 1-A, 2-C, 3-B
subtropical climates.
195. Which of the following statements is/are incorrect
2. This soil is the result of intense leaching. about Fishing Cats?
3. Lateritic soils are mostly deep to very deep and 1. Fishing cat is endemic to India and is mostly
are basic (pH<8.0) in nature. found in floodplains, deltas and coastal wetlands
Which of the statements given above is/are incorrect? of eastern India.

Workbook 154
GEOGRAPHY

2. It has been listed as ‘endangered’ under the Red (d) The highest density of Greater one-horned
List of IUCN. rhinos in the world is found in Kaziranga
3. The practice of intensive aquaculture in wetlands National Park.
led to an increase in their population in the past 200. Consider the following statements regarding
few decades. Himalayan Mountain Forests:
Select the correct answer using the code give below: 1. The vegetation in the Himalayan regions changes
(a) 1 and 2 only with changes in altitude.
(b) 1 and 3 only 2. The northern slopes of the Himalayas carry a
(c) 2 and 3 only thicker vegetation cover than the south-facing
(d) 1, 2 and 3 slopes.
196. Consider the following statement regarding 3. The forests are dominated by coniferous species.
Bamboo: 4. There is a complete absence of temperate
1. It is found only in peninsular regions in India. grasslands.
2. Bamboos grown in private land also require Which of the statements given above are correct?
felling permission from the state forest (a) 1 and 3 only
department. (b) 2 and 4 only
3. It is also known by the name Green Gold. (c) 1, 2 and 3 only
(d) 2, 3 and 4 only
Which of the statements given above is/are incorrect?
(a) 1 and 2 only 201. “It is an ecologically sensitive area known for its
(b) 2 and 3 only lilies and biodiversity. It is home to number of rare
(c) 1 only flora and fauna species like Blyth’s Tragopan, Asian
(d) 1, 2 and 3 golden cat and Hoolock gibbon. It is dominated by
dwarf bamboo and are prone to forest fires.”
197. Which of the following statements is/are correct In the above paragraph which valley is being best
regarding Snow leopards? described?
1. Global Snow Leopard and Eco-system protection (a) Yumthang valley
Program is a project started by UNEP to ensure (b) Dzukou valley
its long-term survival. (c) Neora valley
2. Its habitat in India is limited to the western (d) Araku valley
Himalayas.
202. It has been upgraded to National Park from
Select the correct answer using the code given below: Wildlife Sanctuary recently to protect the sanctity
(a) 1 only of area from coal and oil mining exploration.
(b) 2 only It is also known as Jeypore forest having fauna
(c) Both 1 and 2 like Malayan sun bear and marbled cat. It is also
(d) Neither 1 nor 2 referred as The Amazon of East and is home to
198. It is a phenomenon where a wave appears blue. It Asiatic Elephants. The national park mentioned
occurs when bioluminescent phytoplankton called here is:
dinoflagellates, give a fluorescent glow, through (a) Dehing Patkai National Park
chemical reactions in proteins. This phenomenon (b) Dibru-Saikhowa National Park
in the last few years has become an annual (c) Nameri National Park
occurrence along the west coast of India. (d) Orang National Park
Which of the following best describes the above 203. Recently this beach has been accorded ‘Blue Flag’
phenomenon? tag by Foundation for Environment Education
(a) Breakwater (FEE) for having grey water treatment plants and
(b) Red Tide solid waste management plants. It is situated in
(c) Blue Tide Karnataka and is famous for the Dakkebali ritual
(d) Tombolo held once in two years. The beach mentioned here
is:
199. With reference to the Greater one-horned rhinos,
which of the following statement is correct? (a) Rushikonda Beach
(b) Radhanagar Beach
(a) These are highly interactive animals and often (c) Kappad Beach
hangout with each other in large herds. (d) Padubidri Beach
(b) The average gestation period can be 15 to 16
months. 204. Which of the following statements is/are correct
(c) Sexual maturity reaches early for males as regarding Gadgil committee report on Western
compared to females. Ghats?

155 Workbook
GEOGRAPHY

1. 1. It provided for the distinction between natural is famous for horticultural products like Saffron
landscape and cultural landscape. and contributes the largest share in India’s Apple
2. 2. It recommended the constitution of a Western production.
Ghats Ecology Authority (WGEA), as a statutory Which of the following best describes the above
authority under MOEF&CC. mentioned agro-climatic region of India?
(a) 1 only (a) Eastern plateau and hill region
(b) 2 only (b) Western plateau and hill region
(c) Both 1 and 2 (c) Eastern Himalayan region
(d) Neither 1 nor 2 (d) Western Himalayan region
205. This is a unique forest found only in the narrow 209. Consider the following states
coastal strip of Southern India. Most of the 1. Arunachal Pradesh
rainfall in these types of forests occurs in winters. 2. Himachal Pradesh
Mean annual temperature is about 28°C. The
3. Mizoram
growth of evergreen forests in areas of such low
rainfall arouses botanical interest. Bamboos are In which of the above states do ‘Tropical Wet
rare and grasses not conspicuous. The important Evergreen Forests’ occur?
species are jamun, tamarind, neem, etc. Most of (a) 1 only
the land under these forests has been cleared for (b) 2 and 3 only
agriculture. The type of forest described above is: (c) 1 and 3 only
(a) Tropical Wet Evergreen (d) 1, 2 and 3
(b) Tropical Semi-Evergreen 210. In terms of the forest cover as percentage of total
(c) Tropical Dry Evergreen geographical area, which of the following states
(d) Sub-tropical Dry Evergreen are in the ascending order?
206. Which of the following statements is incorrect 1. Arunachal Pradesh
about Eravikulam national park? 2. Himachal Pradesh
(a) It is home to the highest density and largest 3. Chattisgarh
surviving population of Nilgiri tahr. 4. Manipur
(b) The highest peak of Western Ghat, Anaimudi is Select the correct code using the option given below
located in the Eravikulam National Park. (a) 2-3-4-1
(c) River Kaveri flows through this park. (b) 2-3-1-4
(d) It encompasses three major types of plant
(c) 3-2-1-4
communities i.e. grasslands, shrub Land and
(d) 3-2-4-1
shola Forests.
211. With reference to the Forest conservation in India,
207. This is a waterbody located on the eastern coast
of India. This is directly connected to the Bay of consider the following statements:
Bengal by a channel. Some important aspects 1. Cultivation of spices, rubber and mulberry
about its biodiversity are: a habitat for largest are allowed in reserve forest under the Forest
congregation of waterfowls in India, arribada, conservation Act, 1980.
Irrawaddy dolphins and Barakudia limbless 2. Provisions to check deforestation of non-forest
skink. The waterbody encompasses Nalbana areas are incorporated in the Forest conservation
wildlife sanctuary within itself. The biodiversity Act 1980.
in the waterbody is under the threat from illegal Which of the statements given above is/are incorrect?
farming activities. In the above paragraph which
(a) 1 only
of the following waterbody has been described?
(b) 2 only
(a) Kolleru Lake (c) Both 1 and 2
(b) Pulicat Lake (d) Neither 1 nor 2
(c) Chilika Lake
(d) Vembanad Lake 212. Consider the following statements about a forest
vegetation in India:
208. Topography and temperatures of this agro-
climatic region show great variation. Average These forests are found in the western slope of the
temperature in July ranges between 5°C and 30 °C, Western Ghats and the Andaman and Nicobar
while in January it ranges between 5 °C and -5 °C. Islands. Forests are well stratified covered with
Mean annual rainfall varies between 75 cm to 150 shrubs and creepers. These forests contain several
cm, with some of the areas experiences a very low tree species of great commercial significance e.g.
rainfall of 30 cm too. This agro-climatic region Indian rosewood , Malabar Kino, teak and Indian
is made up of fragile eco system prone to soil laurel. Which type of vegetation has been described
erosion and has low land productivity. This region above?

Workbook 156
GEOGRAPHY

(a) Tropical thorn forests 1. The total forest cover of the country is around
(b) Montane forests 22% of the geographical area.
(c) Moist deciduous forests 2. Chhattisgarh has highest forest cover in the
(d) Tropical evergreen forests country in terms of area.
213. With reference to the tropical deciduous forest, 3. West Bengal has the largest area of wetlands
consider the following statements: within recorded forest area in the country.
1. They are called monsoon forests. Which of the statements given above is/are correct?
2. The moist deciduous forests are found largely in (a) 3 only
the plains of Uttar Pradesh and Bihar. (b) 2 and 3 only
Which of the statements given above is/are correct? (c) 1 only
(a) 1 only (d) 1, 2 and 3
(b) 2 only 217. Which of the following is the correct sequence of
(c) Both 1 and 2 states/UTs as per area under mangrove forest in
(d) Neither 1 nor 2 descending order?
214. These forests provide commercially viable wood (a) Gujarat, West Bengal, Andaman & Nicobar
like Teak. They are marked by high degree of Islands, Andhra Pradesh
gregariousness. Amla and sandalwood are also (b) Gujarat, West Bengal, Andhra Pradesh,
found in these. Which forests are we talking about? Andaman & Nicobar Islands
(a) Moist deciduous forests (c) West Bengal, Gujarat, Andhra Pradesh,
(b) Semi evergreen forests Andaman & Nicobar Islands
(c) Montane forests (d) West Bengal, Gujarat, Andaman & Nicobar
(d) Evergreen forests Islands, Andhra Pradesh
215. Consider the following statements: 218. Which of the following is/are component of the
1. The mountain forests in India are only found in forest policy of Government of India?
Himalayan ranges and in the hilly areas of West 1. Bringing 33% of the geographical areas under
Bengal and Uttaranchal. forest cover.
2. Tropical thorn forests are found in semi-arid 2. Reduction of floods & droughts.
areas of Madhya Pradesh and Uttar Pradesh. 3. Creating a peoples movement involving women
3. Littoral forests can survive both in fresh as well to encourage planting of trees.
as brackish water. 4. Creating an additional carbon sink of 2.5 to 3
Which of the statements given above is/are correct? billion tonnes of carbon dioxide equivalent by
(a) 1 and 2 only the year 2030.
(b) 2 only Select the correct answer using the code given below:
(c) 3 only
(a) 1, 3 and 4 only
(d) 2 and 3 only
(b) 1, 2 and 3 only
216. Consider the following statements about the forest (c) 2, 3 and 4 only
cover in India: (d) 1, 2, 3 and 4

157 Workbook
GEOGRAPHY

SOLUTIONS

west in northern India. The Arabian Sea branch does not


4.1. Previous Years Questions contribute much and exhaust most of its moisture when it
1. Solution: (b) hit Western Ghats.
Exp) Option b is correct. 6. Solution: (c)
Syntaxial bending is the primary reason for the sudden Exp) Option c is correct.
“U” turn of Brahamaputra when it enters India. Statement 1 is correct. Southern part of India, being near
2. Solution: (b) to sea, gets rainfall early and for a longer duration than
in northern part. In peninsular region, India’s rainfall
Exp) Option b is correct. decreases from coast to interior parts.
The climate of North of Gujarat is arid and semi-arid Statement 2 is correct. As the Bay of Bengal branch of
whereas the southern Gujarat comes under the influence Southwest monsoon moves towards north east carrying
of monsoon winds. Gujarat is the leading producer of more moisture and they give heavy rainfall in this region.
cotton. Gujarat’s agricultural economy mainly produces The quantity of rainfall decreases as rain-bearing winds
cash crops like peanut, cotton, tobacco. move towards West over the Northern plains, this is
3. Solution: (a) because of loss of moisture in the winds.
Exp) Option a is correct. 7. Solution: (d)
Bhitarkanika Wildlife Sanctuary is located in the estuarial Exp) Option d is correct.
region of Bramhani-Baitarani, in the north-eastern place
The Himalayas began forming in the Upper Cretaceous
of Kendrapara district of Odisha (Orissa).
period (60 million years ago). Young fold mountains
4. Solution: (b) have not yet undergone extensive erosional process of
exogenic forces, thus they have deep gorges, U-turn river
Exp) Option b is correct.
courses, Parallel mountain ranges and steep gradients.
Orchids require cold temperature and moist environment
These features are generally absent in old fold mountains
with indirect sunlight to grow. In Arunachal Pradesh, the
which are characterized by lower altitude, rounded peaks,
richest concentration of orchids is believed to be in Ziro
gentle slopes and rolling hills.
Valley. Arunachal Pradesh alone accounts for 52% of total
species of Orchids in the country. 8. Solution: (a)
Government of India has recognized orchids as the Exp) Option a is correct.
prime plant group for promoting export oriented
Arunachal Pradesh and Northern Rajasthan lie on same
activity in NE region.
latitude at around 28°N. Approximately, 82 percent area of
5. Solution: (c) Arunachal Pradesh is under forest cover and over 12% of
the forest over constitutes Protected Area Network.
Exp) Option c is correct.
Statement 1 is correct. 9. Solution: (a)
The word monsoon is derived from the Arabic word Exp) option a is correct
‘Mausim’ which means season. Monsoon refers to the Statement 1 is correct. The Narmada and the Tapi
seasonal reversal in the wind direction during a year. The flow through the rift valley. The Narmada and The
Southern part of India, being near to the sea, gets rainfall Tapti flow in trough faults. They along with many
early and for a longer duration than in the northern small rivers discharge their waters in the Arabian Sea.
part. As rain-bearing winds cross western ghats humidity Statement 2 is incorrect. Narmada flows between
decreases and so does rainfall in the northern part of Vindhyas in north and the Satpuras in south but this is
India. The Southern part of India receives rainfall from not the reason for flowing towards West.
the southwest monsoon and the retreating southwest
monsoon (northeast monsoon). Statement 3 is incorrect. The Peninsular plateau consists
of two broad divisions, namely, the Central Highlands
Statement 2 is correct. and the Deccan Plateau. The part of the peninsular
The Bay of Bengal is the main branch of the monsoon plateau lying to the north of the Narmada river, covering a
winds which moves from northeast and then hits major area of the Malwa plateau, is known as the Central
Himalayas to return westwards by covering the northern Highlands. The flow of the rivers draining this region,
plains. As the winds move westward, their moisture namely the Chambal, the Sind, the Betwa and the Ken is
contains tends to reduce as they cause rains along Indian from southwest to northeast, thus indicating the slope.
subcontinent. So that the rainfall decreases from east to Thus, the slope is towards northeast.

Workbook 158
GEOGRAPHY

10. Solution: (a) During winter, there is a high-pressure area north of


Exp) Option a is correct. the Himalayas. Cold dry winds blow from this region
to the low-pressure areas over the oceans to the south.
Pair 1 is correctly matched. Limboo tribe occupy In summer, a low-pressure area develops over interior
the region in eastern Nepal and western Sikkim. Asia, as well as, over north-western India. Thus, wind in
Pair 2 is incorrectly matched. Karbi people are the main India blows from the northeast during cooler months and
tribe of Karbi Anglong and West Karbi Anglong districts reverses direction to blow from the southwest during the
of Assam. warmest months of the year.
The scheduled tribes in Himachal Pradesh includes
Gaddis, Gujjars, Kinnara or Kinnauras, Lahaulas and 13. Solution: (a)
Pangwalas apart from some other smaller tribes groups Exp) Option a is correct.
like Bhot/Bodh, Beda, Jad/Lamba/ Khampa, and Swangla. Option 1 and 2 are correct. The Eastern Himalayan
Pair 3 is correctly matched. The Niyamgiri hill broadleaf forests are diverse and species-rich, with a
range in Odisha state, eastern India, is home to great diversity of oaks and rhododendrons in particular.
the Dongria Kondh tribe. The Dongria have The ecoregion has two broad forest types: evergreen and
repeatedly expressed opposition to efforts to mine deciduous. Evergreen forests are characterized by oaks
in Niyamgiri, recently against Vedanta Resources. together with Rhododendrons. In Western Himalayas, the
Pair 4 is incorrectly matched. Bondas are a Particularly evergreen broadleaf forests is dominated by oaks.
Vulnerable Tribal Groups (PVTGs) found in Odisha. Option 3 is incorrect. Sandalwood is an evergreen tree
They live in the isolated hill regions of the Malkangiri which generally grows in the tropical dry, deciduous
district of southwestern Odisha near the junction of the forests of Karnataka, Tamil Nadu, Kerala and Andhra
three states of Odisha, Chhattisgarh, and Andhra Pradesh. Pradesh.

11. Solution: (a) 14. Solution: (b)


Exp) Option a is correct. Exp) Option b is correct.
Ten degrees Channel which lies between Little Andaman Option 1 is incorrect. Barak originates in Manipur, flows
and Nicobar Islands. through Mizoram and Assam and drains into Bangladesh.
The Channel is approximately 150 km wide, running Option 2 and 3 are correct. Lohit and Subansiri are a
essentially along an east-west orientation. It is so named tributarries of the Brahmaputra River. They originate in
as it lies on the 10-degree line of latitude, north of the Tibet and flow through Arunachal Pradesh, before joining
equator. Brahmaputra in Assam.
15. Solution: (b)
Exp) Option b is correct.
Statement 1 is incorrect. The Changpa are a semi-
nomadic Tibetan people. They are mainly found in
the Changtang, a high plateau that stretches across
the cold desert of Ladakh, Jammu and Kashmir.
Statement 2 is correct. They usually stay in one place for a
few months in a row, near pastures where their sheep, yaks
and Pashmina goats can graze.
Statement 3 is correct. In 1989 the Changpa were granted
official status in India as a Scheduled Tribe.
16. Solution: (c)
Exp) Option c is correct.
Pair 1 is incorrectly matched. Coromandel Coast lies in
eastern Tamil Nadu whereas, Cardamom Hills lie in
Kerala in Western Ghats region.
Pair 2 is incorrectly matched. Konkan extends
throughout the western coasts of Maharashtra, Goa and
Karnataka. Kaimur hills in Jabalpur district of Madhya
12. Solution: (c) Pradesh form the eastern portion of Vindhyan range.
Exp) Option c is correct Pair 3 is correctly matched. Mahadeo hills are located in
A monsoon is a seasonal reversal in wind patterns over a the northern part of Satpura range in Madhya Pradesh in
region. The seasonal wind shift is usually accompanied by Central India.
a dramatic change in precipitation. The phenomenon is Pair 4 is correctly matched. Mikir hills are spread over
best developed over the Indian subcontinent. Assam and Meghalaya region in North Eastern India.

159 Workbook
GEOGRAPHY

district of Karnataka and flows towards east eventually


draining into the Bay of Bengal.
19. Solution: (c)
Exp) Option c is correct.
The tropical wet evergreen forest in India is usually found
in areas receiving more than 200 cm of rainfall and having
a temperature of 15-30 degrees Celsius. These forests are
found in the plains, on low hills and gentle slopes of the
foothills of the mountains, at elevations ranging from
almost sea level up to about 1000–1200 m. They occur in
the states of Maharashtra, Karnataka, Tamil Nadu, Kerala,
Andaman and Nicobar islands in the south and in the
entire north-eastern region including the sub- mountain
division of West Bengal.
The forest of Himachal Pradesh can be classifies into
nine types: Dry alpine forests, Moist alpine scrub forests,
Fig. Depicting location of various mountain ranges in
Subalpine forests, Temperate forests, Wet temperate forest,
India.
Subtropical Pine forest, Tropical dry dry deciduous forests
17. Solution: (a) and Tropical thorny forests. Thus. it is clear that tropic wet
evergreen forests are not found in Himachal Pradesh.
Exp) Option a is correct.
Teak is the most dominant species of Tropical Moist 20. Solution: (d)
Deciduous Forest. Tropical Deciduous Forest are the Exp) Option d is correct.
most widespread forests of India. They are also called the
Arunachal Pradesh and Gujarat are the easternmost
monsoon forests and spread over the region receiving
and westernmost States of India. Kibithu in Arunachal
rainfall between 200 cm and 70 cm. Trees of this forest Pradesh is the Easternmost point of India. Ghuar Mota
type shed their leaves for about six to eight weeks in in the Kutch region of Gujarat is the Westernmost point
dry summer. On the basis of the availability of water, of India. It is located near the disputed Sir Creek and the
these forests are further divided into moist and dry Koteshwar temple, at 23.713°N 68.032°E.
deciduous. The former is found in areas receiving rainfall
between 200 and 100 cm. These forests exist, therefore, 21. Solution: (a)
mostly in the eastern part of the country — northeastern Exp) Option a is correct.
states, along the foothills of the Himalayas, Jharkhand,
West Odisha and Chhattisgarh, and on the eastern slopes Pair 1 is correctly matched. Nallamala Hills
of the Western Ghats. Teak is the most dominant species located in eastern Andhra Pradesh are a part of
of this forest. Bamboos, sal, shisham, sandalwood, khair, Eastern Ghats. Srisailam on the River Krishna
kusum, arjun and mulberry are other commercially is an ancient temple dedicated to Lord Shiva.
important species. Pair 2 is incorrectly matched. Omkareshwar is a temple
dedicated to Lord Shiva located in Madhya Pradesh, on
18. Solution: (d) Madhata island in Narmada river. Whereas, Satmala is a
Exp) Option d is correct. mountain range in Nashik District of Maharashtra.
Option 1 is incorrect. Vamsadhara originates in Kalahandi Pair 3 is incorrectly matched. Pushkar in Ajmer district
district of Odisha. It finally joins the Bay of Bengal at of Rajasthan has a temple dedicated to Lord Brahma.
Kalingapatnam, Andhra Pradesh. Water sharing of this Pushkar town is situated in Aravalli hills. Mahadeo hills
river is a source of dispute between Andhra Pradesh and form the northern section of Satpura ranges in Madhya
Pradesh.
Odisha.
Option 2 and 3 are correct. The river Godavari is the 22. Solution: (a)
largest of the peninsular rivers. Pranhita is the largest Exp) Option a is correct.
tributary of Godavari.
The jing kieng jri or living root bridges — aerial bridges
Left Bank Tributaries of Godavari: built by weaving and manipulating the roots of the Indian
Banganga, Kadva, Shivana, Purna, Kadam, Pranahita, rubber tree — have been serving as connectors for
Indravati, Taliperu, Sabari, Dharna generations in Meghalaya. Spanning between 15 and
Right Bank Tributaries: 250 feet and built over centuries, the bridges, primarily
a means to cross streams and rivers, have also become
Nasardi, Pravara, Sindphana, Manjira, Manair, world-famous tourist attractions.
Kinnerasani
Option 4 is incorrect. The Pennar river rises in the Chenna 23. Solution: (d)
Kasava hill of the Nandidurg range, in Chikkaballapura Exp) Option d is correct.

Workbook 160
GEOGRAPHY

India has identified six basins as areas for shale 28. Solution: (b)
gas exploration: Cambay (Gujarat), Assam- Exp) option b is correct
Arakan (North East), Gondwana (Central
India), Krishna Godavari onshore (East Coast), If we follow the shortest route, the minimum number of
Cauvery onshore, and Indo-Gangetic basin. states through which we travel from Kohima (Nagaland)
As per the US EIA 2015 report, India has got technically and Kottayam (Kerala):
recoverable shale gas of 96 trillion cubic feet. The Nagaland – Assam – West Bengal – Odisha – Andhra
recoverable reserves are identified in Cambay, Krishna Pradesh – Tamil Nadu or Karnataka – Kerala.
– Godavari, Cauvery, Damodar Valley, Upper Assam,
Pranahita – Godavari, Rajasthan and Vindhya Basins.
24. Solution: (d)
Exp) Option d is correct.
To develop inland shipping port, Rajasthan is developing
a channel along the Kutch Creek at Jalore which would
be connected to the Arabian sea. If implemented,
Rajasthan will not remain a landlocked state.
25. Solution: (b)
Exp) Option b is correct.
The Pattiseema lift irrigation project is a river interlinking
project which connects Godavari river to Krishna river.
Pattiseema scheme will help revive Krishna delta.
The first pump of this Godavari-Krishna linking project
was installed and switched on in September 2015. It will
benefit the people especially farmers in the Krishna delta
in terms of timely availability and adequate quantities of
water.
Fig. Route from Kohima (Nagaland) to Kottayam (Kerala).
Under the National Perspective Plan (NPP) prepared
by Ministry of Water Resources, NWDA has already 29. Solution: (c)
identified 14 links under Himalayan Rivers Component Exp) Option c is correct.
and 16 links under Peninsular Rivers Component for inter
basin transfer of water Chandipur is a beach located on the shore of
Bay of Bengal in Balasore District of Odisha.
26. Solution: (d) Chandipur beach in Odisha has a unique distinction on
Exp) Option d is correct. its own. Unlike other beaches, the sea water here recedes
away from the shore line about 5 km twice a day, an
All of the given rivers are tributaries of Brahmaputra. unusual phenomenon, rarely found anywhere.
Option 1 and 3 are correct. Lohit originates in Tibet, flows
through Arunachal Pradesh, before it is joined by Dibang 30. Solution: (b)
in Assam where it flows into Brahmaputra. The Bhupen Exp) Option b is correct.
Hazarika Setu is constructed on River Lohit. Statement 1 is incorrect. Teesta River originates from the
Option 2 is correct. Kameng river also called as Bharali is Pahunri (or Teesta Kangse) glacier and flows southward
a right bank tributrary of Brahmaputra. It originates near through the Sikkim Himalaya. where the Rangpo River
Indo-Tibet border in Tawang and meets Brahmaputra joins, and where it forms the border between Sikkim
near Tezpur in Assam. and West Bengal. Whereas, The Brahmaputra river rises
from Chemayundung glacier of the Kailash range near
27. Solution: (b)
the Mansarovar Lake to the north of the Himalayas in the
Exp) Option b is correct. southwest Tibet region.
Statement 1 is incorrect. In India, the Himalayas Statement 2 is correct. The Rangeet is a tributary of the
are spread over Jammu & Kashmir, Haryana, Teesta river, which forms a boundary between Sikkim and
Punjab, Himachal Pradesh, Uttarakhand, Sikkim, Darjeeling. The Rangeet river originates in the Himalayan
Assam, West Bengal and Arunachal Pradesh. mountains in West Sikkim district.
Statement 2 is incorrect. Western Ghats are spread over
Gujarat, Maharashtra, Goa, Karnataka, Tamil Nadu, Statement 3 is incorrect. Teesta flows through Sikkim,
Kerala. West Bengal, and joins Jamuna river at Fulhhari in
Bangladesh.
Statement 3 is correct. Pulicat Lake is located at border of
Andhra Pradesh and Tamil Nadu. 31. Solution: (b)

161 Workbook
GEOGRAPHY

Exp) Option b is correct. Statement 2 is incorrect. It lies about 140km northeast


Statement 1 is incorrect. The Indian Ocean Dipole - of Port Blair, the capital of Andaman and Nicobar Islands.
often called the “Indian Niño” because of its similarity It lies north of Great Nicobar. It is a tourist destination
to its Pacific equivalent - refers to the difference in sea- and surrounded by waters ideal for scuba diving and is
surface temperatures in opposite parts of the Indian home to a wide variety of aquatic life.
Ocean. Temperatures in the eastern part of the Indian Statement 3 is incorrect. Historically, the first record of
Ocean oscillate between warm and cold compared with the volcano’s eruption dates back to 1787. It was known
the western part, cycling through phases referred to as to have erupted at least five times over the next 100 years.
“positive”, “neutral” and “negative”. Then there was silence for a century. In 1991, it spewed so
The dipole’s positive phase this year means warmer sea massively that smoke billowed out for about six months.
temperatures in the western Indian Ocean region, with Ever since, there have been eruptions every two-three
the opposite in the east. The result of this may be higher- year. In the recent past, it erupted in February 2016 and
than-average rainfall and floods in eastern Africa and in January 2017.
droughts in south-east Asia and Australia.
34. Solution: (a)
During a negative IOD, the opposite happens, that is, the
Exp) Option a is correct.
western Indian Ocean becomes cooler with higher air
pressures resulting in westerly winds blowing toward the Longitude is measured by imaginary lines that run
Indian subcontinent. around the Earth vertically (up and down) and meet
Statement 2 is correct. The Indian Ocean Dipole (IOD) is at the North and South Poles. These lines are known
the Indian Ocean counterpart of the Pacific El Niño and as meridians. They are used to define the East-West
La Niña. position of a location on the planet.
Like ENSO, IOD is a coupled ocean-atmosphere Longitude of Delhi- 77.1025° E
phenomenon. The shifting pools of warm/cool water Longitude of Bengaluru- 77.5946°E
contribute to variations in rainfall and storm activities of Longitude of Hyderabad- 78.4867° E
many countries surrounding the Indian Ocean.
Longitude of Nagpur- 79.0882° E
Positive IODs are often associated with El Niño and
Longitude of Pune- 73.8567° E
negative IODs with La Niña. A positive IOD leads to
greater monsoon rainfall and more active (above normal
rainfall) monsoon days while negative IOD leads to less
rainfall and more monsoon break days (no rainfall).
32. Solution: (a)
Exp) Option a is correct.
Kodaikanal Lake is a manmade lake located in the
Kodaikanal city in Dindigul district in Tamil Nadu, India.
Sir Vere Henry Levinge, the then Collector of Madurai,
was instrumental in creating the lake in 1863, amidst the
Kodaikanal town which was developed by the British and
early missionaries from USA.
Option b is incorrect. Kolleru Lake located in Andhra
Pradeshis a Ramsar site. It is located between Krishna and
Godavari river systems. 35. Solution: (a)
Option c is incorrect. Nainital lake is a natural fresh Exp) Option a is correct.
water lake which is tectonic in origin.
All the sites given above are famous for temple structures
Option d is incorrect. Renuka Lake is a designated and being pilgrimage towns.
Ramsar site. It is the largest fresh water lake of Himachal
Pradesh. Pair 1 is correctly matched. Pandharpur is a pilgrimage
town in Solapur district of Maharashtra. The famous
33. Solution: (a) Vitobha temple is located here.
Exp) Option a is correct. Pair 2 is correctly matched. Tiruchirappalli is situated at
Statement 1 is correct. Barren Island is an active the head of Cauvery delta, where the river seperates into
volcano located in the Andaman Sea in the Union two streams. Tiruchirappalli or Trichy is home to many
territory of Andaman and Nicobar Islands. It is the only famous temples such as Ranganathaswamy temple,
confirmed active volcano of India and South Asia. It is Erumbeeswarar Temple and the Jambukeswarar temple.
a stratovolcano composed of lava, rock fragments, and Pair 3 is incorrectly matched. Classified as a World
volcanic ash. It lies over the fault whose movement caused Heritage Site by UNESCO, Hampi (Karnataka) situated
the tsunami on December 26, 2004. along the banks of the Tungabhadra River, was the

Workbook 162
GEOGRAPHY

capital city of the Vijayanagara Empire. It is a famous 39. Solution: (d)


pilgrimage place. Vijaya Vittala Temple and Virupaksha Exp) Option d is correct.
Temple are situated in Hampi, The Famous stone chariot
The Siachen in Jammu and Kashmir (18,000ft.) near the
dedicated to Gadura is also located here. Both Malaprabha
Karakoram Pass is the longest glacier in the Indian Himalayas.
and Tungabhadra are tributaries of Krisha river.
It is situated at the North of Nubra Valley which acts as
36. Solution: (d) the gateway to the Siachen Glacier and Karakoram pass.
It is situated West of Aksai Chin, North of Leh and to the
Exp) Option d is correct answer.
East of Gilgit.
All the three are water reservoirs.
Aliyar reservoir (Aliyar river, Tamil Nadu)
Isapur dam (Penganga river, Maharashtra)
Kangsabati reservoir (Kangsabati river, West Bengal)
37. Solution: (a)
Exp) Option a is correct.
Pair 1 is correctly matched. Banderpoonch Glacier
perched in the Garhwal division of the Himalayas. The
Banderpoonch Glacier is highly significant as it is the
prime source of fresh water for the Yamuna basin.
Pair 2 is correctly matched. The Bhara Shigiri glacier in
the Chandra Valley of Lahaul in Himachal Pradesh feeds
Fig. Location of Siachen Glacier
the Chenab river.
Pair 3 is incorrectly matched. Milam is a major glacier in 40. Solution: (c)
the state of Uttarakhand. Milam is situated in the Kumaon Exp) Option c is the correct answer.
Himalayas, originating from the eastern slopes of the
Gorges are formed when rivers erode through gigantic
Trishuli. The river Gori Ganga has its source here.
rock formations over thousands of years.
Pair 4 is correctly matched. The Siachen in Jammu and
Gandikota is a small village in the Kadapa district of
Kashmir (18,000ft.) near the Karakoram Pass is the
Andhra Pradesh.
longest glacier in the Indian Himalayas. It is 78km long
and the source for the Nubra River that waters the Nubra The swift flowing waters of River Pennar cut through
Valley near Leh in the Trans Himalayas. The waters of the the granite rocks near the village of Gandikota, naturally
Nubra drain into the Shyok River and then into the Indus. eroding the rock and forming a deep valley. Gandikota got
its name from ‘gandi’ the Telugu word for ‘gorge’. Pennar
Pair 5 is incorrectly matched. The Zemu glacier is the River streams from the Erramala hills.
largest in the Eastern Himalayas in Sikkim. It is at the
base of the Kanchendzonga and is one of the sources The spectacular gorge is famously adjudged as the Grand
for the Teesta. Canyon of India.

38. Solution: (b) 41. Solution: (b)


Exp) Option b is correct. Exp) Option b is the correct answer.
Statement 1 is incorrect. Ocean Mean Temperature Pair 1 is incorrectly matched. Namcha Barwa is not
is measured up to a depth of 26 degree C isotherm. situated in Garhwal Himalaya.
The 26 degree C isotherm is seen at depths varying Namcha Barwa is considered as the Eastern point of the
from 50–100 metres. During January–March, Himalayas. It is located on the extreme east of the Indian
the mean 26 degree C isotherm depth in the state of Arunachal Pradesh. The height of Namcha Barwa
Southwestern Indian Ocean is 59 metres. It is more is about 7782 m. Brahmaputra river takes a U turn on
stable and consistent, and the spatial spread is also less. reaching Namcha Barwa.
Statement 2 is correct. Ocean mean temperature (OMT) Pair 2 is correctly matched. The part of
can be used for predicting whether the total amount of the Himalayas lying between Satluj and
rainfall that India receives during the monsoon season Kali rivers is known as Kumaon Himalayas.
will be less or more than the long-term mean of 887.5 mm. Nanda Devi is the second highest mountain in India and
Using OMT, the information on whether the amount of the highest entirely within the country (Kangchenjunga
monsoon rainfall will be more or less than the long-term being on the border of India and Nepal). It is part of
mean will be available by beginning of April, two months the Kumaon Himalayas, and is located in the state of
before the southwest monsoon can set in. This is because Uttarakhand, between the Rishiganga valley on the west
OMT is analysed by measuring the ocean thermal energy and the Goriganga valley on the east. The surrounding
during the period from January to March. Southwest Nanda Devi National Park was declared a UNESCO
monsoon sets in around June 1 each year in Kerala. World Heritage Site in 1988.

163 Workbook
GEOGRAPHY

of Godavari Water Disputes Tribunal (GWDT). The


project implements Godavari-Krishna link under
Interlinking of rivers project. Indira Sagar (Polavaram)
project is located on river Godavari near Ramayyapet
village of Polavaram Mandal of West Godavari district in
Andhra Pradesh.
Pair 4 is incorrectly matched. Maithon, dam is located
on the banks of river Barakar. The Maithon Dam is
located about 48 kms from the Coal City of Dhanbad,
Jharkhand. The dam with an underground power station
is one of its kinds in the whole of South East Asia. The lake
on which it is built is spread over 65 square kilometers.
This was developed by the Damodar Valley Corporation
Pair 3 is incorrectly matched. Nokrek is not situated in (DVC Ltd) .
Sikkim Himalaya. Nokrek is the highest peak in West
Garo Hills of Meghalaya.
4.2. Location and Physiography
42. Solution: (b)
44. Solution: (c)
Exp) Option b is the correct answer.
Exp) Option c is correct.
Pair 1 is incorrectly matched. Hokera Wetland is
located in Jammu and Kashmir, not Punjab. Hokera Pair 1 is incorrect. Mango showers are the local winds
wetland is about 10 km from Srinagar. It is a natural towards the close of the summer season and are pre-
perennial wetland contiguous to the Jhelum basin. monsoon showers common in especially Kerala and
Karnataka. They help in the early ripening of mangoes.
Pair 2 is correctly matched. Renuka Wetland is the
largest natural lake in Himachal Pradesh.. Pair 2 is incorrect. Loo is a local afternoon dust storms
are common during summers. They are very hot and dry
It is a natural wetland with freshwater springs and inland winds blow over North Indian plains. Exposure to theses
subterranean karst formations, fed by a small stream hot winds may cause heat or sun stroke.
flowing from the lower Himalayan out to the Giri river.
Pair 3 is correct. Nor Westers/Kalbaisakhi/ Bardoli
Pair 3 is correctly matched. Rudrasagar Lake, also known Chheerha season for localized thunderstorms, associated
as Rudijala, is a lake located in Melaghar, Tripura. with violent winds, torrential downpours, often
The lake is designated as a Ramsar site. A lowland accompanied by hail. In West Bengal, these storms are
sedimentation reservoir in the northeast hills, fed by three known as the ‘Kaal Baisakhi’. In Assam, these storms are
perennial streams discharging to the River Gomti. known as “Bardoli Chheerha”.
Pair 4 is incorrectly matched. Sasthamkotta is located 45. Solution: (d)
in the Kollam district of Kerala, not Tamil Nadu. It is the
largest freshwater lake in Kerala and has always been a Exp) Option d is correct.
centre of tourist activity. Option 1 is correct. Delhi is located within a distance
of 200-400 km from different locations of the main
43. Solution: (c) boundary thrust fault. The main boundary thrust fault of
Exp) Option c is the correct answer. the Himalayas, which runs from Kashmir to Arunachal
Only one pair is correctly matched. Pradesh, is highly active.
Pair 1 is incorrectly matched. The Ghataprabha Option 2 is correct. Earthquakes in Delhi-NCR were
Reservoir is located in the Belgavi district of the state of due to the “release of stress” accumulated from the
Karnataka. The reservoir has storage of about 659 million movement of the Indian tectonic plate and its collision
cubic meters, providing irrigation to a total extent of with the Eurasian tectonic plate.
1,396,000 hectares of land. The Ghataprabha River is a Option 3 is incorrect. The “large sediment thickness
major tributary of the River Krishna. (loose soil) in the Ganga Alluvial Plains” to the north
Pair 2 is correctly matched. The Gandhi Sagar Dam is of Delhi tends to amplify the impact of earthquakes.
one of the four major dams built on India’s Chambal River It is not directly responsible for the generation of
and is located in the Mandsaur & Neemuch district of earthquakes.
Madhya Pradesh. It is a masonry gravity dam. Option 4 is correct. There are so many weak zones
Pair 3 is incorrectly matched. The Indira Sagar Dam and faults in the Delhi-NCR: - Delhi-Haridwar ridge,
on Narmada river is a multipurpose project, located Mahendragarh-Dehradun subsurface fault, Moradabad
at Narmada Nagar, Punasa in the Khandwa district of fault, Sohna fault, Great boundary fault, Delhi-Sargodha
Madhya Pradesh. ridge, Yamuna River lineament, Ganga River lineament
etc.
Indirasagar Polavaram, an interstate project on river
Godavari has been conceived as a part of recommendations 46. Solution: (a)

Workbook 164
GEOGRAPHY

Exp) Option a is correct. Peninsular block are older than the Himalayan river
Statement 1 is incorrect. The snowline in eastern valleys. The Peninsular rivers are smaller and have
Himalayas is at a higher altitude due to the lower height a fixed course. They have almost reached their base
of the eastern Himalayas and the moderating influence of levels. Therefore, the river valleys are shallow with low
the South west monsoon winds. gradients.
Statement 2 is correct. The average height of western 50. Solution: (c)
Himalayas is higher compared to eastern Himalayas as
Exp) Option c is correct
in case of western Himalayas. The Indo plate is pushing
towards the Eurasian plate and causing the height to Statement 1 is correct. The Himalayas are tectonic
increase more rapidly. in origin. They have formed as a result of the collision
between the Indian Plate and Eurasian Plate which
Statement 3 is incorrect. Eastern Himalayas have same
began 50 million years ago and continues today. Earlier,
orogeny as western Himalayas however their scale of relief
and morphology is different. India was a large island situated off the Australian coast
and separated from Asia by the Tethys Ocean. The
47. Solution: (c) Himalayas are still rising by more than 1 cm per year
Exp) Option c is correct. as India continues to move northwards into Asia, which
explains the occurrence of shallow focus earthquakes in
Malda Fault was observed at a place called Malda in West the region today.
Bengal. This facture was due to the formation of fold
mountains (Himalayas). This fault separates Chota Nagpur Statement 2 is correct. The Himalayas along with other
Plateau and North-Eastern Himalayan mountains. This Peninsular mountains are young, weak and flexible in
Malda gap is also called the Garo-Rajmahal gap. their geological structure unlike the Peninsular block
which is rigid and stable. Because of this, the Himalayas
Option a is incorrect – Kachchh mainland and Rann of are still subject to the effects of exogenic and endogenic
Kachchh are separated by the Kachchh Mainland fault
forces, resulting in the development of faults, folds and
(KMF).
thrust plains.
Option b is incorrect – Fault that runs parallel to Satpura
and Vindhya ranges forms a rift valley in which Narmada 51. Solution: (c)
River flows. Exp) Option c is correct.
Option d is incorrect – The Palakkad gap or Palghat Gap Statement 1 is correct. The Himalayas in Arunachal and
forms a major break in the Western Ghats Mountain Sikkim are characterized by the absence of the Shiwalik
range, in south-western India. Located between the formations, unlike other sections of Himalayas. Instead
Nilgiri Hills to the north and the Anaimalai Hills to the of this, duar formations are found here. They are used
south, it serves as a major communication route between extensively for tea plantation.
Kerala and Tamil Nadu.
Statement 2 is incorrect. Duar formations is a feature
48. Solution: (c) of Sikkim Himalayas. In Uttarakhand, there are Dun
Exp) Option c is correct. formations. Duns are longitudinal valleys formed as a
result of folding when Eurasian plate and Indian plate
Statement 1 is correct. Karewas are the thick deposits collided. They are formed between Lesser Himalayas and
of glacial clay and other materials embedded with Shiwaliks.
moraines. The Kashmir Himalayas are famous for
Karewa formations, which are useful for the cultivation Statement 3 is correct. Bugyals are summer grasslands
of Zafran, a local variety of saffron. in the higher reaches of the Great Himalayas in the
Uttarakhand region. The nomadic groups like Bhotiyas,
Statement 2 is correct. Great Indian Desert, in
migrate to Bugyals during summer months and return to
Rajasthan, is dotted with longitudinal dunes and
valleys during winters.
barchans. This region receives low rainfall below 150 mm
per year; hence, it has arid climate with low vegetation 52. Solution: (c)
cover. It is because of these characteristic features that this
Exp) Option c is correct
is also known as Marusthali.
The correct order of the hills starting from North to South
49. Solution: (a) is as follows –
Exp) Option a is correct. • Patkai Bum – Patkai Range is part of the Arakan
Statement 1 is correct. The boundary of the Peninsular system along the Myanmar border located in the
block is from Kachchh along the western flank of the Indian state of Arunachal Pradesh and extend to
Aravali Range near Delhi and then roughly parallel to the Nagaland.
Yamuna and the Ganga as far as the Rajmahal Hills and • Naga Hills - Naga Hills are part of the complex
the Ganga delta. Along with these, the Peninsula also mountain barrier on the border of India and
includes Rajasthan and its desert. Myanmar (Burma). They are located in the Indian
Statement 2 is incorrect. The river valleys of the state of Nagaland.

165 Workbook
GEOGRAPHY

• Manipur Hills - Manipur hills are enclosed by Option c is correct. The spatial variations have brought in
Nagaland in the north, Mizoram in the south, elements of diversity in the relief of the Peninsular plateau.
Myanmar in the east and Cachar of Assam in the west. The north-western part of the plateau has a complex
• Lushai Hills - Lushai Hills (or Mizo Hills) are a relief of ravines and gorges. The ravines of Chambal,
mountain range in Mizoram and Tripura. Bhind and Morena are some of the well-known examples.
Option d is correct. Tors are landforms created by the
erosion and weathering of rock; most commonly granites,
but also schists, dacites, dolerites, ignimbrites, coarse
sandstones and others. Tors are mostly less than 5 meters
high. It is one of the features of the Peninsular plateau.
55. Solution: (c)
Exp) Option c is correct.
Statement 1 is correct. There is persistent high pressure
and subsidence due to subtropical climate in the Thar
desert because of which the prevailing southwest winds
that bring rain to much of the subcontinent bypass Thar
desert.
Statement 2 is correct. The soils in this region consist
of several main groups—desert soils, red desertic soils,
sierozems (brownish grey soils), the red and yellow
soils of the foothills, the saline soils of the depressions,
and the lithosols (shallow weathered soils) and regosols
(soft loose soils) found in the hills. All those soils are
53. Solution: (b) predominantly coarse-textured, well-drained, and
Exp) Option b is correct. calcareous (calcium-bearing). A thick accumulation of
Pair 1 is incorrectly matched. The alluvial plains of the lime often occurs at varying depths.
northern India have a Bhabar region which is a narrow 56. Solution: (b)
belt parallel to Shiwalik foothills at the break-up of the
slope. This results in, deposition of heavy materials like Exp) Option b is correct.
rocks and boulders by the streams and rivers coming Statement 1 is correct. The western coastal plains are
from the mountains. At times, the streams disappear in an example of submerged coastal plain. Because of
this zone. this submergence, the sea is deeper in west coast which
Pair 2 is correctly matched. South of the Bhabar is provides favourable conditions for the development of
Tarai belt, where most of the streams and rivers reemerge ports and harbours.
without having any properly demarcated channel. This Statement 2 is correct. The western coastal plains are
creates marshy and swampy conditions in this region. narrow in the middle and gets broader towards north and
Therefore, Tarai belt has a luxurious growth of natural south. The rivers flowing through this coastal plain do not
vegetation and houses a varied wild life. form any delta.
Pair 3 is incorrectly matched. Khadar is a new alluvial Statement 3 is incorrect. As compared to the eastern
deposit region south of the Tarai and then the Bhangar coastal plain, the western coastal plain is narrower and
(old alluvial deposits) region. Khadar and Bhangar have is an example of a submergent coast. Hence, there are
characteristic features of mature stage of fluvial erosional well developed deltas on the eastern plains, formed by the
and depositional landforms such as sand bars, meanders, rivers flowing eastward in to the Bay of Bengal.
ox-bow lakes and braided channels.
57. Solution: (c)
54. Solution: (b) Exp) Option c is correct.
Exp) Option b is correct. Statement 1 is correct. Saddle peak is the highest
Option a is correct. Patland are high-level plateau mountain peak of the Andaman and Nicobar Islands.
found in the Peninsular region. The Peninsular region Saddle peak is located in North Andaman with an
is made up of a series of Patland plateaus (consists elevation of 738 m.
of series of step like sub-plateaus) such as Hazaribagh Statement 2 is correct. Andaman and Nicobar Islands
plateau, Palamu plateau, Malwa plateau, Ranchi plateau, consist of about 572 islands. These are situated roughly
Karnataka plateau, Coimbtore plateau. between 6°N-14°N and 92°E -94°E. Due to this location,
Option b is incorrect. The general elevation of the these islands receive convectional rainfall and have an
plateau is from the west to the east, which is also proved equatorial type of vegetation as they lie close to Hot, wet
by the pattern of the flow of rivers. Equatorial Climatic type.

Workbook 166
GEOGRAPHY

58. Solution: (b) 62. Solution: (c)


Exp) Option b is correct. Exp) Option c is correct.
Option b is correct. Duars are alluvial floodplains found Some of the important events in the recent geological
in Darjeeling and Sikkim HImalayas. They are the result history of India from earliest to latest time are:
of folding during the formation of the Himalayas. The 1. Formation of ‘basement complex’ (Pre-Cambrian) –
moderate slope, thick soil cover with high organic content, formation of the stable shield like region that forms
uniform year-round rainfall with mild winters paved way the base of Indian land mass.
for their development into tea plantations by the British.
They are known for their scenic beauty and are dotted with
tourist destinations such as Darjeeling and Siliguri. They
are home to many endangered species like one horned
rhino and tiger. National parks and wildlife sanctuaries
are found in large numbers such as Manas, Jaldpara, Buxa
National Parks and Mahananda Wildlife Sanctuary.
59. Solution: (c)
Exp) Option c is correct.
Statement 1 is correct – Bhabar is in east-west direction
along the foothills of the Shivalik at the break-up of the
slope. As a result of this, the streams and rivers coming
from the mountains deposit heavy materials of rocks
and boulders, and at times, disappear in this zone.
Statement 2 is correct – Bhangar is the old alluvial
deposits often impregnated with calcareous concretions
known as ‘Kankar’.
Statement 3 is correct – The Khadar is composed of
newer alluvium and forms the flood plains along the
river banks. A new layer of alluvium is deposited by
river flood almost every year. This makes them the most
fertile soils of the Ganges. 2. Formation of Gondwana rock system
3. Marine transgression in western India (Jurassic) –
60. Solution: (d) It led to deposition of thick series of shallow water
Exp) Option d is correct. deposits in Rajasthan and Gujarat.
Statement 1 is correct – Vindhyan hills form the 4. Formation of Deccan traps (Cretaceous) – Series
southern edge of the Malwan plateau. The plateau is of basaltic lava flows took place when Indian plate
bounded by the Aravali Range in the west and Madhya passed over a volcanic hotspot.
Bharat Pathar to the north and Bundelkhand to the east. 5. Formation of Himalayas (Tertiary) due to collision of
Statement 2 is correct – This plateau has two systems of Indian plate with Eurasian plate.
drainage; one towards the Arabian sea (The Narmada, 6. Formation of Indo-Gangetic plains (Quaternary)
the Tapi and the Mahi), and the other towards the Bay of
Bengal (Chambal and Betwa, joining the Yamuna). 63. Solution: (d)
Statement 3 is correct – The plateau is composed of Exp) Option d is correct.
extensive lava flow and is covered with black soils. The correct chronological order is:
1. Archean rock system (4 billion years ago)– These
61. Solution: (c) rocks were formed before the Cambrian period
Exp) Option c is correct. making them the oldest rocks of the Indian land mass.
The convergence between the Indo-Australian plate and They are mostly gneisses and schists that formed due
the Eurasian plate gave rise to many geomorphological to the solidification of molten magma.
features including Himalayas. These include: 2. Dharwar (4 – 1 billion years ago)- They are highly
1. Trans-Himalayas – They lie to the north of the Great metamorphosed sedimentary rocks (metamorphosis
Himalayan range. Includes ranges such as Kailash of sedimentary rocks of Archean rocks).
range, Ladakh range, Karakoram range etc. 3. Dravidian (600 – 300 million years ago) – They are
2. Purvanchal mountain range – They were formed on found in the extra-peninsular region (Himalayas
the eastern margin of the continental convergence, and Ganga plain) and are rare in Peninsular India.
aligned in a north-south direction. They include They are rich in fossils.
ranges such as Patkai Bum, Naga hills, Mizo hills etc. 4. Aryan (Upper Carboniferous to present) – They
3. Arakan Yoma range in Myanmar are found in river valleys of Damodar, Mahanadi and

167 Workbook
GEOGRAPHY

Godavari, among other places. They are rich in coal 67. Solution: (d)
reserves. Exp) Option d is correct.
64. Solution: (c) Ladakh range and Zanskar range are part of Trans-
Himalayas. So, they are located to the north of the Great
Exp) Option c is correct.
Himalayas.
Pair 1 is correct: Dravidian rock system is found
The Pir Panjal Range is considered as part of the Lesser
predominantly in the extra-peninsular region (Himalayas Himalayas of Kashmir region. So, they are located to the
and northern plains) and is rare in the peninsular region. south of the Great Himalayas.
The term does not refer to south India.
The correct order is – Ladakh Range, Zanskar Range,
Pair 2 is correct: Dharwar system of rocks is rich in Great Himalayas and Pir Panjal.
metallic minerals like iron ore, manganese, copper, lead,
gold etc. 68. Solution: (c)
Pair 3 is incorrect: The coal that occurs in Gondwana Exp) Option c is correct.
rock system is low in carbon content. It constitutes 98% of The Karewas are a unique geomorphological feature of the
the total coal reserves of India. Kashmir Valley. They are extremely fertile regions and are
Pair 4 is correct: The erosion of the igneous rock used for the cultivation of saffron, almond, walnut, apples
formations of the deccan traps has led to the formation of etc. They were formed through a series of events
black cotton soil (regur). 1. Formation of a lake – During the early Pleistocene
Period, the entire Kashmir Valley was under water
65. Solution: (c) (lake).
Exp) Option c is correct. 2. Deposition of glacial clay – The lake was filled with
Statement 1 is incorrect: Himalayas formed due to the unconsolidated gravel and mud.
intense folding and uplifting of land that followed the 3. Creation of a gorge – Endogenetic forces led to the
collision between the Indo-Australian plate and the creation of a gorge called Baramulla Gorge.
Eurasian plate. They are not volcanic mountains. 4. Draining of the lake - The lake was drained through
Statement 2 is correct: When the Indo-Australian plate this gorge.
started converging on to the Eurasian plate, the existing 5. Formation of Karewas – The deposits left in the
marine sediments of the Tethys Sea were folded and process are called Karewas.
raised, becoming a part of the Himalayan Mountain 69. Solution: (b)
range. This accounts for the presence of oceanic
sedimentary rocks in the rock structure of the Himalayas. Exp) Option b is correct.
Statement 3 is correct: After the collision between the Statement 1 is incorrect: As compared to the other
two plates, the Indo-Australian plate continued to push sections of the Himalayas, Sikkim Himalayas are
forward and ultimately subducted under the Eurasian conspicuous by the absence of the Shiwalik formations.
plate. This led to the upliftment of the Tibetan plateau to In place of the Shiwalik’s here, the ‘duar formations’
are important, which have also been used for the
its present levels of elevation.
development of tea gardens.
66. Solution: (b) Statement 2 is incorrect: The southern slopes are steep
Exp) Option b is correct. while the northern slopes are gentle.
Statement 1 is correct: The Himalayas continue to rise Statement 3 is correct: The Shiwaliks are thickly forested
in height year after year. This indicates that the region from North-east India up to Nepal, and it decreases from
is a tectonically active zone, with the plates continuing Nepal towards west. This is on account of the decrease in
to push against each other, thereby causing folding and precipitation from east to west.
uplifting of land. 70. Solution: (d)
Statement 2 is correct: As the plates push against each Exp) Option d is correct.
other, sudden releases of energy are manifested in the
Statement 1 is incorrect: Duns are longitudinal valleys
form of earthquakes.
that are formed in between the mountain ranges of the
Statement 3 is incorrect: Volcanic activity is notably Himalayas.
insignificant in the Himalayan region. There are no active
Statement 2 is incorrect: They are found in both western
volcanos. So, even though volcanic activity can be an and eastern parts of the Himalayan range. They are
indicator of a tectonically active region, such activity is called as ‘doons’ in the west and ‘duars’ in the east. Duars
absent in the Himalayan region. are alluvial flood plains that lie to the south of the outer
Statement 4 is correct: The Himalayan rivers are in their foothills of the Himalayas.
youthful stage and have been rejuvenated in recent times.
This shows that the Himalayan landmass is rising and 71. Solution: (b)
keeping the rivers in youthful stage since a long time. Exp) Option b is correct.

Workbook 168
GEOGRAPHY

Option a is incorrectly matched. Dhauladhar range Statement 2 is correct: The re-appearing streams flow
is part of a lesser Himalayan chain of mountains in in multiple directions without a well-defined drainage
Himachal Pradesh. pattern, making the region ill-drained and marshy. The
Option b is correctly matched. Mahabharat Lekh is part soil is fertile, and water is abundantly available, and as
a result the region is home to lush vegetation and thick
of the lesser Himalayas located in Nepal.
forests.
Option c is incorrectly matched. Kailash Range is located
Statement 3 is incorrect: The Terai region is more marked
in Tibet region. in the eastern part than in the west because the eastern
72. Solution: (a) parts receive comparatively higher amount of rainfall.
Exp) Option a is correct. 75. Solution: (a)
Pair 1 is correctly matched: Zoji La connects Kashmir Exp) Option a is correct.
valley with Ladakh. The Srinagar-Leh Highway traverses Peninsular India is a complex amalgamation of plateaus
the pass. and hills.
Pair 2 is correctly matched: Baralacha La is a high From west to east, the major plateau regions of central
mountain pass in the Zanskar range. It connects Lahaul India can be arranged as follows –
district in Himachal Pradesh to Leh district in Ladakh. Marwar Plateau (Rajasthan)-Malwa Plateau- Madhya
Pair 3 is incorrectly matched: Lipu Lekh pass is located Bharat Plateau- Bundelkhand-Baghelkhand and
on the tri-junction of Uttarakhand, Tibet, and Nepal. It Chotanagpur Plateau.
connects Uttarakhand with Tibet.
76. Solution: (a)
Pair 4 is incorrectly matched: Shipki La is an important
border post between India and China. It is located in Exp) Option a is correct.
Kinnaur district of HP and Tibet. Statement 1 is correct: Aravali Range is the oldest fold
mountain range in India. It runs across the states of
73. Solution: (b) Delhi, Haryana, Rajasthan, and Gujarat in a northeast to
Exp) Option b is correct. southwest direction.
Statement 2 is incorrect: Vindhya Range is located to the
north of Narmada River. Satpura Range runs between the
Narmada and the Tapi.
Statement 3 is incorrect: The Vindhya-Satpura Range
forms the watershed between the Ganga system and the
rivers of south India. Chambal is a river of Ganga system.
The Narmada River, traditionally considered to be the
boundary between north and south India, lies to the south
of the Vindhyan hills.
77. Solution: (b)
Exp) Option b is correct.
Statement 1 is incorrect: Western Ghats lie parallel to the
western coast. They are continuous and can be crossed
through passes only. The Eastern Ghats are discontinuous
and irregular and dissected by rivers draining into the Bay
of Bengal
Statement 2 is correct: Western Ghats are spread across
the states of Gujarat, Maharashtra, Goa, Karnataka, Tamil
Nadu, and Kerala.
78. Solution: (c)
Exp) Option c is correct.
74. Solution: (b) Statement 1 is incorrect: Eastern Ghats are a highly
broken and discontinuous chain of hill ranges running
Exp) Option b is correct. parallel to the eastern coast of India. From south to north
Statement 1 is incorrect: Terai region lies immediately the hill ranges are – Shevroy Hills, Javadi Hills, Palkonda
to the south of the Bhabar belt. The rivers which Range, Nallamala Range, Madugula Konda Range and
disappear underground in the Bhabar belt re-emerge in Maliya Range.
the Terai region, as the deposits are finer and more clayey. Statement 2 is incorrect: Eastern Ghats leave broad
This creates a marshy topography. plains between the base of the Ghats and the eastern coast.

169 Workbook
GEOGRAPHY

Statement 3 is correct: The hill ranges of the Eastern conditions for the development of ports and harbours.
Ghats do not structural unity and continuity like the Kandla, Mazagaon, JLN port Navha Sheva, Marmagao,
Himalayas or the Western Ghats. They seem to be formed Mangalore, Cochin, etc. are
due to different geomorphological processes. The hill
ranges are, hence, treated as independent ranges. 83. Solution: (c)
Exp) Option c is correct.
79. Solution: (c)
Pair 1 is correctly matched. Nathu La Pass - It is located
Exp) Option c is correct.
in the state of Sikkim. This famous pass is located in the
Statement 1 is correct: Andaman and Nicobar Islands India- China border and was reopened in 2006. It forms a
were formed when the Indo-Australian Plate collided part of an offshoot of the ancient silk route. It is one of the
with the Eurasian Plate in the Tertiary Period. They are border trading posts between India and China.
considered the southward extension of the Arakan Yoma
Range in Myanmar, but underwater. Pair 2 is correctly matched. Shipki La Pass - It is located
through Sutlej Gorge. It connects Himachal Pradesh
Statement 2 is correct: Lakshadweep Islands were with Tibet. It is India’s third border post for trade with
formed when corals accumulated atop volcanic China after Lipu Lekh and Nathula Pass.
landforms giving rise to atolls. These volcanic landforms
(submerged under sea) were formed when the Indian plate Pair 3 is correctly matched. Jelep La Pass - This pass
passed through the Reunion hotpot during its northward passes through the Chumbi valley. It connects Sikkim
journey after splitting up from Gondwanaland. with Lhasa, the capital of Tibet.
Pair 4 is incorrectly matched. Lanak La Pass - This is
80. Solution: (c) located in the Aksai Chin in the Ladakh Union Territory
Exp) Option c is correct. and not in Jammu and Kashmir. It connects Ladakh and
Statement 1 is incorrect: Lakshadweep Islands are tiny Lhasa.
islands characterized by a flat and featureless topography. Pair 5 is correctly matched. Lipu Lekh Pass – It is located
Statement 2 is correct: Coral reefs are found in both in Uttarakhand. It connects Uttarakhand with Tibet. This
island groups. Lakshadweep Islands themselves have pass is an important border post for trade with China. The
a coral origin (atolls). The islands are surrounded by pilgrims for Manasarovar travel through this pass. It was
fringing reefs. A&N Islands are also home to fringing in news due to Nepal redrawing its political map.
reefs.
84. Solution: (a)
Statement 3 is correct: India’s only active volcano
is found on the Barren Island of the A&N group of Exp) Option a is correct
Islands. There are no active volcanoes on the mainland. • Bhopal is 23.2599° N.
81. Solution: (c) • Jaipur is 26.9124° N
Exp) Option c is correct. • Raipur is 21.2514° N
Pair 1 is correctly matched: Kiltan Island is a coral • Kolkata is 22.5726° N.
island belonging to the Amindivi Subgroup of islands Bhopal is the nearest city to tropic of cancer (23.5° N)
of Lakshadweep. among the given options having latitude of 23.2599° N.
Pair 2 is incorrectly matched: Ghoramara Island is in
85. Solution: (d)
the Sundarban Delta of the Bay of Bengal. The island
is small, roughly five square kilometers in area, and is Exp) Option d is correct.
quickly disappearing due to erosion and sea level rise. Pair 1 is incorrectly matched– Thal Ghat (also called as
Pair 3 is correctly matched: Pirotan Island is an Arabian Thul Ghat or Kasara Ghat) is a ghat section (mountain
Sea island in the Marine National Park, Jamnagar incline or slope) in the Western Ghats near the town of
District of Gujarat. Kasara in Maharashtra. The Thal Ghat is located on the
Pair 4 is incorrectly matched: Baratang Island is a part busy Mumbai–Nashik route.
of the Andaman and Nicobar Island off the coast. It is Pair 2 is incorrectly matched – It connects Mumbai to
located between South and Middle Andaman. Pune via Khopoli. Bhor ghat is a mountain pass which is
located near Khandala, Maharashtra which is the famous
82. Solution: (d) hill station of Sahyadri range.
Exp) Option d is correct. Pair 3 is correctly matched – The most prominent break
Statement 1 is correct: The Indian coastline emerged as in the Western Ghats is the Palghat gap (Palakkad Gap),
a result of the faulting of the Gondwanaland during the which is about 40 km wide. The mountain pass is located
Cretaceous Period. between Nilgiri Hills in the north and Anaimalai Hills
Statement 2 is correct: The western coast is more towards the south and connects Coimbatore in Tamil
indented than the eastern coast, which provides natural Nadu with Palakkad in Kerala.

Workbook 170
GEOGRAPHY

90.33% (according to the ISFR 2019) while the forest area


is 0% there.
Option d is correct – Lakshadweep islands is located
north of Maldives and Chagos group of Islands.

Pair 4 is correctly matched – Chorla Ghat is located very


interestingly in the intersection of three states — Goa, 88. Solution: (b)
Maharashtra and Karnataka, just about some 50
Exp) Option b is correct.
kilometers from Panjim, Goa. It is a part of the Western
Ghats in the Sahyadri mountain range, and is at an Pair 1 is incorrectly matched. Panamik is situated
elevation of 800 metres. close to Siachen glacier, which is famous for its curative
86. Solution: (a) hot springs. The spring water contains high amount of
sulphur and it is believed best for curing rheumatism and
Exp) Option a is correct
other ailments. It is located in close proximity to Nubra
Statement 1 is correct – Rann of Kutch is the largest salt
desert in the world. In the Rann of Kutch, the Southwest valley in Ladakh.
Monsoon brings salt particles and deposits there as a Pair 2 is correctly matched. Kheer ganga is located at
crust. Statement 2 is correct – Sir Creek is the disputed Akhara Bazar, Kullu, Himachal Pradesh which is an
region between Indian and Pakistan. It is located in
important site for hot spring.
Rann of Kutch. Sir Creek opens up in the Arabian Sea and
roughly divides the Kutch region of Gujarat from the Pair 3 is correctly matched. Gaurikund is considered
Sindh Province of Pakistan. the most sacred hot spring and is a major attraction for
Statement 3 is incorrect – Luni is an important river several pilgrimages located in Uttarakhand. Like any
that flows into Rann of Kutch. Luni rises from Aravali other hot spring, the hot water which comes deep from
ranges and ends in the marshy lands of Rann of Kutch. the earth core carries away lot of minerals mainly sulfur.
Mahi river rises in Madhya Pradesh and flows through
Rajasthan and Gujarat and into the Gulf of Khambat. 89. Solution: (b)
87. Solution: (b) Exp) Option b is correct.
Exp) Option b is correct answer. South Andaman Island is the southernmost island of
Option a is correct – Entire group consists of tiny islands the Great Andaman and is home to the majority of the
of coral origin (coral depositions on atolls) and are population of the Andaman Islands. The Capital Port
surrounded by fringing reefs. Blair is located on this island.
Option b is incorrect. Nine-degree channel (and Little Andaman is separated from Great Andaman by
not eleven degree) separates Minicoy from the main
Lakshadweep archipelago. 50 Km wide Duncan Passage. It is also a part of South
Andamans District. (It is southern to South Andaman)
Option c is correct – The forest area is the area notified
and recorded as the forest land irrespective of the Car Nicobar is the northernmost among the Nicobar
existence of trees, while the actual forest cover is the Islands. (It is southern to Little Andaman)
area occupied by forests with canopy. The former is based
on the records of the State Revenue Department, while Great Nicobar is the largest and the southernmost island
the latter is based on aerial photographs and satellite among the Nicobar Islands. It is very close to Sumatra
imageries. The forest cover of Lakshadweep Islands is Island of Indonesia. (It is southern to Car Nicobar)

171 Workbook
GEOGRAPHY

regulates time and clocks. It is the successor to Greenwich


Mean Time (GMT). UTC and GMT are almost the
same, and most people would not notice any practical
differences between the two.
93. Solution: (c)
Exp) Option c is correct.
The Standard Meridian of India (82º 30’E) passes
through Uttar Pradesh (Mirzapur), Madhya Pradesh,
Chhattisgarh, Odisha and Andhra Pradesh (5 states).

90. Solution: (a)


Exp) Option a is correct
At the equator, the Coriolis force is zero. It increases as
we move away from the equator.
The city of Kolkata lies at 22.5726° N, Lucknow is 94. Solution: (a)
at 26.8467° N, then come to New Delhi which is at Exp) Option a is correct.
28.6139° N. Chandigarh comes next at 30.7333° N and
Tropic of Cancer passes through – Gujarat, Rajasthan,
northernmost capital out of the given cities is Shimla
Madhya Pradesh, Chhattisgarh, Jharkhand, West
which is at 31.1048° N.
Bengal, Tripura, and Mizoram.
Hence, the correct ascending order of Coriolis force for
the given cities is 3-1-2-5-4.
91. Solution: (b)
Exp) Option b is correct.
Statement 1 is incorrect: All states except Madhya
Pradesh, Chhattisgarh, Jharkhand, Haryana and
Telangana (5 states) in the country have one or more
The Orient BlackSwan School Atlas, 7th Edition, Pg. No.
international borders or a coastline.
11
Statement 2 is correct: Even though the latitudinal
and longitudinal extents of the India mainland are both 95. Solution: (d)
equal to about 30º, there is a significant difference in the Exp) Option d is correct.
distances between the two sets of extremities. While the
distance between north-south extremity is around 3214 Option a is incorrect: China shares border with Ladakh,
km, the distance between east-west extremity is only Himachal Pradesh, Uttarakhand, Sikkim, and Arunachal
2933 km. Pradesh (5 states/UTs).
Option b is incorrect: Nepal shares border with
92. Solution: (a) Uttarakhand, Uttar Pradesh, Bihar, West Bengal, and
Exp) Option a is correct. Sikkim (5 states/UTs)
Statement 1 is correct: Standard meridian of India is the Option c is incorrect: Bangladesh shares border with
longitude located 82º 30’ to the east of the Prime Meridian West Bengal, Assam, Meghalaya, Tripura, and Mizoram
that is considered as the standard time for the whole (5 states/UTs).
country. Option d is correct: Myanmar shares border with
Statement 2 is incorrect: It is ahead of the UTC by 5h Arunachal Pradesh, Nagaland, Manipur, and Mizoram (4
30m. UTC is the primary standard by which the world states/UTs).

Workbook 172
GEOGRAPHY

96. Solution: (d) northeastern parts are separated by the Malda fault in
Exp) Option d is correct West Bengal from the Chotanagpur plateau.
The Shola forests of Western Ghats are found in the upper Statement b is correct.The Peninsular plateau has been
reaches of the Nilgiris, Anamalais, Palni hills, Kalakadu, separated from the peninsular block by a wide gap known
Mundanthurai and Kanyakumari in the states of Tamil as the Garo-Rajmahal Gap (Malda Gap). The down-
Nadu and Kerala. These forests are found sheltered in thrusting of the Malda gap area between the Rajmahal hills
valleys with sufficient moisture and proper drainage, at an and the Meghalaya plateau during the mid-pleistocene
altitude of more than 1,500 metres. The upper reaches are period, diverted the Ganga and the Brahmaputra systems
covered with grasslands, known as Shola grasslands. to flow towards the Bay of Bengal.
Option d is correct. Statement c is correct. Malda gap was later filled by
Features of Shola forests: sediment deposits of the Ganga and its tributaries. The
plateau is largely formed by Archaean quartzites, shales
The vegetation that grows in Shola forests is evergreen. and schists with granite intrusions and some basic silts
The trees are stunted and have many branches. Generally, similar to the peninsular region.
the leaves are small in size and leathery. Red-coloured
young leaves turning into different colours on maturity Statement d is incorrect. The Dauki fault (and not malda
is a prominent characteristic of the Shola forests. fault) is a major fault along the southern boundary of
Epiphytes like lichens, ferns and bryophytes usually the Shillong plateau that may be a source of destructive
grow on the trees. seismic hazards for the adjoining areas, including
northeastern Bangladesh.
The rolling grasslands are found on top of the Western
Ghats. Shola forests and grasslands are found in a ratio
of 1:5.
97. Solution: (d)
Exp) Option d is correct.
India was a large island situated off the Australian coast,
in a vast ocean. The Tethys Sea separated it from the
Asian continent till about 225 million years ago. India is
supposed to have started her northward journey about
200 million years ago at the time when Pangaea broke.
Statement 1 is correct. India collided with Asia about 40-
50 million years ago causing rapid uplift of the Himalayas.
The subcontinent was still close to the equator. From 40
million years ago and thereafter, the event of formation
of the Himalayas took place. Scientists believe that
the process is still continuing and the height of the
Himalayas is rising even to this date. 99. Solution: (c)
Statement 2 is correct. The past earthquakes and possible Exp) Option c is correct.
future earthquakes along the plate interior depend either
upon the internal lithospheric stress or on the stress from Pair 1 is correct. Nathu La is located in the state of
the plate boundary. The northward movement of the Sikkim. This famous pass is located in the India- China
Indian tectonic plate pushing slowly against the Asiatic border and was reopened in 2006. It forms a part of an
plate is evident by the frequent earthquakes in the region. offshoot of the ancient silk route. It is one of the trading
borders posts between India and China. Nathu La Pass
Statement 3 is correct. The two major plates were connects Sikkim to China’s Tibet Autonomous Region.
separated by the Tethys Sea and the Tibetan block was
It was sealed for almost 4 decades after the People’s
closer to the Asiatic landmass. During the movement of
Republic of China suppressed a Tibetan uprising in 1959.
the Indian plate towards the Eurasian plate, a major event
that occurred was the outpouring of lava and formation Pair 2 is correct. Shipki La is located through Sutlej
of the Deccan Traps. Gorge. It connects Himachal Pradesh with Tibet. It is
India’s third border post for trade with China after Lipu
98. Solution: (d) Lekh and Nathu La Pass.
Exp) Option d is incorrect. Pair 3 is incorrect. Zoji La connects Srinagar with
A fault is a fracture or zone of fractures between two Kargil and Leh. Rohtang Pass and Baralacha La, connects
blocks of rock. Faults allow the blocks to move relative to to Leh with Manali.
each other. Pair 4 is correct. Khardung La is the second-highest
Statement a is correct.The Karbi Anglong and the motorable pass in the country. It connects Leh and
Meghalaya Plateau in the northeast and Rajasthan in the Siachen glaciers. This pass remains closed during the
west are also extensions of Indian peninsular block. The winter.

173 Workbook
GEOGRAPHY

rivers of India and a major part of the river basin lies in


the Western Ghats.
102. Solution: (b)
Exp) Option b is correct
The Himalayas is the highest mountain system of the
world. It extends in arc shape for a distance of about 2500
km from west to east along the northern boundary of
India. The breadth of the Himalayas ranges between 400
km in the west to 150 km in the east.
Statement 1 is incorrect: the Eastern Himalayas rise
nearly abruptly from the plains of West Bengal and Assam.
100. Solution: (c) This is the reason why we find highest peaks of Himalayas
Exp) Option c is incorrect. in the east. For example, two of the highest peaks of
Peaty Soils are found in the areas of heavy rainfall, where Himalayas, i.e. Mt. Everest (in Nepal) and Kanchenjunga
there is a good growth of vegetation. It occurs widely in are in the east and they are not very far from the plains.
the northern part of Bihar, southern part of Uttaranchal On the other hand, the western part of Himalayas rises
and the coastal areas of West Bengal, Orissa and Tamil rather gradually from the plains.
Nadu. Statement 2 is correct: As Western Himalayas rises
Statement a is correct. They are found in the areas of gradually from the plains, therefore, the higher peaks in
heavy rainfall and high humidity, where there is a good this part are farther from the plains and a number of
growth of vegetation. ranges lie between the plains and high peak, for example,
Shivaliks, Lesser Himalayan Ranges, etc. Therefore, the
Statement b is correct. Large quantity of dead organic high peaks of this part such as Nanga Parbat, Nanda Devi
matter accumulates in these areas, and this gives a rich and Badrinath are very far from the plains.
humus and organic content to the soil. Organic matter
in these soils may go even up to 40-50 per cent. These 103. Solution: (b)
soils are normally heavy and black in colour. At many Exp) Option b is correct
places, they are alkaline also. It occurs widely in the
northern part of Bihar, southern part of Uttarakhand and Pair 1 is incorrect.
the coastal areas of West Bengal, Odisha and Tamil Nadu. Nasik
Statement c is incorrect. They are deficient in phosphate Located on bank of Godavari River. It is best known for
and potash. Arid desert soils have high phosphate content. being one of Hindu pilgrimage sites, that of Kumbh Mela
Statement d is correct. Peaty soils have a thick layer which is held every 12 years
(>40 cm) of organic matter accumulation, representing Pair 2 is correct.
the long periods of saturation on an annual basis. These
soils typically occur in depressional settings and are not Vijayawada
common in floodplains due to the periodic scouring Located on bank of Krishna River in Krishna district of
that occurs during flood events. south coastal Andhra. Vijayawada is a major commercial
hub.
101. Solution: (d)
Pair 3 is correct.
Exp) Option d is correct
Cuttack
All the rivers given above are western flowing.
Located on bank of Mahanadi River. It is the second largest
The Shetruniji river which rises near Dalkahwa in Amreli city in the state and a hub for commercial activities, such
district drains in Arabian sea. progress has earned the city the tag of ‘Millenium City’
Mahi river is the famous rivers of Gujarat which drains on of Odisha.
west coast of India. It rises in Madhya Pradesh and, after
Pair 4 is correct.
flowing through the Vagad region of Rajasthan, enters
Gujarat and flows into the Arabian Sea. Ferozepur
The Vaitarna rises from the Trimbak hills in Nasik The boundary of the present Ferozepur District on the
district at an elevation of 670 m. Vaitarna has a confluence east runs along the Moga, Faridkot, Mukatsar and Fazilka
with the Tanasa just before it enters the Arabian Sea. The districts. On the North-east, the River Sutlej generally
Vaitarna river is a river in Nashik and Palghar district of separates it from the Jalandhar and Kapurthala districts.
Maharashtra. Upper stretches of Vaitarna are clean but in The united Stream of the Sutlej and Beas generally
lower stretches it’s polluted due to untreated industrial separates it from the Taran district in the north-west and
and civic waste. farther down from Pakistan, with exception of some areas
on each side of river.
Sharavati river originates and flows entirely within the
state of Karnataka. It is one of the few westward flowing 104. Solution: (c)

Workbook 174
GEOGRAPHY

Exp) Option c is correct the international boundary between India and Nepal. In
The Himalayas is the highest mountain system of the this, Gandak River also forms the international boundary
world. It extends in arc shape for a distance of about 2500 between Nepal and India. But, Susta became the disputed
km from west to east along the northern boundary of territory because of shifting course of the river Gandak.
India. Earlier, Susta was on the right bank of the Gandak, which
falls in Nepal. But, with course of time, the river has
Statement 1 is correct: The river Indus flows towards shifted its route and as a result, Susta now falls on the left
northwest between Zaskar range and Ladakh range. bank of the Gandak, which is controlled by India.
A Himalaya range which is located to the north and
runs almost parallel of the Himadri/Great Himalayas is
Zaskar range. Further, to the north of Zaskar range is the
Ladakh range. Further, North of the Ladakh range lie
the Karakoram. Many scholars treat Zaskar and Ladakh
ranges as parts of the Great Himalayas and include them
in Kashmir Himalayas.
Statement 2 is incorrect. K2 (8611m) is the second
highest peak of the world, next only to Mt. Everest. It is
also the highest peak of the Karakoram Mountains.
Statement 3 is correct. The three major ranges of the
Himalayas are the Great Himalayan Ranges/Himadri, 107. Solution: (a)
Lesser Himalayan Ranges/the Himachal and the Siwalik
ranges. Further, to the north of Great Himalayas, The Exp) Option a is correct
Zaskar, the Ladakh and the Karakoram ranges along with Sundarbans is a mangrove area in the delta formed by
their eastern extension, Kailash in Tibet are considered the confluence of the Ganges, Brahmaputra and Meghna
as the trans Himalayan ranges. Rivers in the Bay of Bengal. It spans the area in West
Bengal and Bangladesh.
105. Solution: (d) Statement 1 is correct: The Ganges Brahmaputra Delta,
Exp) Option d is correct also named Ganges Delta, Sunderban Delta or Bengal
Statement 1 is correct: Aadi Perukku is a distinctive Delta is located in Asia which is the floodplain of three
event and a remarkable festival of Tamil Nadu. In this great rivers: the Ganges, the Brahmaputra and the
Mother Nature is worshipped by the people of state in the Megna. Together, these three rivers drain a catchment
form of Amman deities. They also express gratitude to of about 1.72 million km2, at the southern side of the
the river Kaveri during this time. It is to thank nature for Himalaya.
showering its grace, peace and prosperity. In this, people Statement 2 is correct: The distance from the confluence
worship lakes and all perennial river sources of the state. of the Brahmaputra and the Ganges to the coast is
Statement 2 is correct: Kerala’s famous Sabarimala temple around 250 km. Along the coast, the width of the delta
is well known for the Mandala-Makaravilakku (Makar is measured to be approximately 350km. It is considered
Sankranti) festival, which lasts for almost two months. It to be the world’s largest Delta, having a surface area of
is celebrated in The Lord Ayyappa temple at Sabarimala. around 100.000 km2. Approximately two-thirds of the
Sabarimala temple dedicated to Lord Ayyappa is delta is in Bangladesh, the rest constitutes the state of
located on the banks of the river Pamba. West Bengal, India.
Statement 3 is correct; Kumbh Mela is the largest Statement 3 is incorrect: The Ganges Delta is one the most
Hindu fair held in India. It is held every fourth year at fertile part of the world. It has approximately 130 million
four different places. - the banks of the Godavari River inhabitants, which makes the Ganges delta belonging to
in Nashik, the Ganga River in Haridwar, the Shipra the most densely populated areas in the world.
(Kshipra) River in Ujjain, and at confluence of the
108. Solution: (c)
Ganges, Yamuna and Saraswati in Allahabad (now known
as Prayagraj). Exp) Option c is correct
The Brahmaputra River is also known as Yarlung Zangbo
106. Solution: (c) in China. It originates from the glacier mass of the Kailash
Exp) Option c is correct Range of the Himalayas.
The Gandak river is originated at the Nhubine Himal Statement 1 is incorrect: Its principal right bank
glacier in Nepal. Thereafter, it flows across Nepal and tributaries are the Kameng River, Manas River, Beki River,
enters India near the Valmiki Tiger Reserve in Bihar. Raidak River, Jaldhaka River, Teesta River, Subansiri River
Option c is correct. Susta is a disputed village which is whereas the principal left bank tributaries are Lhasa River,
claimed by both India and Nepal. It is mainly because Nyang River, Parlung Zangbo, Lohit River, Dhansiri River,
of the changing course of the Gandak river, known Kolong River.
as Narayani in Nepal. The Treaty of Sugauli which was Statement 2 is incorrect: The major part of basin is
signed between India and Nepal led to the demarcation of covered with forest areas which accounts to 55.48% of

175 Workbook
GEOGRAPHY

the total area whereas 5.79% of the basin is covered by


water bodies.
Statement 3 is correct: Six tributaries of Brahmaputra
River are namely the Teesta, the Sankosh, the Raidak-I,
the Raidak-II, the Torsa and the Jaldhaka which flows
through the northern part of West Bengal. Jaldhaka join
the main stream of Brahmaputra but, in the plains of
Bangladesh. 2. The Kajavi River is a west flowing river which rises in
Jaldhaka River’ one of the trans-boundary rivers, the Vishalghat region of Sahyadri hills and flows West
originates from the Himalayas of southeast Sikkim and ward and joins the Arabian Sea near Ratnagiri port,
Maharashtra. It forms a 10 km. long creak named Bhatya
flows over Jalpaiguri and Kuch Bihar. The river enters
Creak has been formed.
Bangladesh through Lalmonirhat district. After entering
3. The Mandovi River is one of the main West Flowing
Bangladesh, the river joins with the Dharla and flows as
Rivers of Goa State. The river runs in the North- East
Dharla before debouching into the Brahmaputra near direction for about 5 km and then follows in the West
Kurigram. ward direction.
109. Solution: (d) 4. The Kali River or Kali Nadi is a west flowing river. The
river originates near Kushavali, a small village in Uttar
Exp) Option d is correct Kannada district. It further flows through Uttara Kannada
Inland drainage basin is formed by rivers pouring their district of Karnataka state.
waters in a lake or an inland sea. Inland Drainage Basin
Rivers do not enter a sea or ocean. They are found in
different parts of North and South India.
Statements 1, 2 and 5 are correct: Area of Inland drainage
in Rajasthan extends over states of Haryana and Rajasthan.
Small rivers which form inland drainage are named as the
Kantu, the Kakni, the Ghugri and the Sukri.
Statement 3 is incorrect: The Sabarmati River which
is originated in Rajasthan and flows through Gujarat
State before entering into the Gulf of Khambhat in the
Arabian Sea. It is not an inland drainage basin. It is Luni
River which is located in western Rajasthan. It starts its
journey from the Pushkar valley of the Aravalli ranges
and moves through Thar desert to disappears into the
111. Solution: (a)
marshy lands of Rann of Kutch, Gujarat.
Exp) Option a is correct
Statement 4 is correct: Ghaggar River is also an inland
The cold weather season i.e. winter season in India begins
drainage basin river. It originates from the Shivalik from mid-November and stays till February.
hills of Himachal Pradesh at an elevation of about 1,927 Statement 1 is correct.
meters above mean sea level. It flows through Indian states
During the Winter season, the northeast trade winds
of Punjab and Haryana to at last enter into Rajasthan. prevail over the country. The northeast trade winds blow
110. Solution: (c) from land to sea in most of North India during the winter
season. They blow from land to sea and hence, for most
Exp) Option c is correct part of the country, it is a dry season.
The correct sequence of the rivers from North-South is However, in some parts in India it blows from sea to land.
given below Some amount of rainfall occurs on the Tamil Nadu coast
from these winds as, here they blow from sea to land.
1. Ambika River is one of the important west flowing
rivers with its catchment in Gujarat and Maharashtra. It Statement 2 is incorrect. The influence of easterly trade
winds creates a counter-equatorial current which flows
originates from Saputara Hill ranges near village Kotambi between the north equatorial current and the south
of Surgana taluka in the Nasik district of Maharashtra. equatorial current in west-east direction. This raises the
After flowing for a length of 136 km it drains in to the level of western Indian (south-east of horn of Africa)
Arabian Sea. ocean by few centimetres.

Workbook 176
GEOGRAPHY

There is no counter-equatorial current in summer. Due For the northern parts of India, it is likely that the La
to the effects of the strong south-west monsoon and the Nina weather phenomenon may result in extreme cold
absence of the north-east trades in summer, a strong from December 2021 to February 2022. While La Niña
current flow from west to east, which also completely is referred to as the cold phase of ENSO, El Niño is the
obliterates the north equatorial current. Hence, the warm phase of ENSO. Both La Niña and El Niño are
absence of counter-equatorial current. deviations from the normal surface temperatures of the
Pacific Ocean.
Option b is incorrect. The tropical cyclones develop in
the Bay of Bengal due to vast low pressure created by the
higher sea surface temperature. They also develop under
the influence of western disturbances. The frequency and
direction of these cyclones influence weather conditions
along the eastern coast during October, November and
December.
Research finds that changes in the Bay’s cyclone
environment were caused by decades-long changes in
the El Niño Southern Oscillation, or ENSO, a regular
pattern variation of winds and sea surface temperatures
affecting the tropics and sub-tropics. The ENSO triggers
112. Solution: (a)
an atmospheric wave that helps instigate the May-June
Exp) Option a is correct monsoon circulation over the Bay of Bengal, which then
Tropical Easterly Jet Stream occurs near the tropopause becomes responsible for initiating a north-south dipole of
over India, and Africa during summer. It indicates a deep tropical cyclone activity.
layer of warm air to the north of the jet and colder air to Option c is correct. The southern oscillation is a pattern
the south over the Indian Ocean. of meteorological changes which are often observed
Statement 1 is correct. During summer, due to the between Indian and Pacific oceans. Normally when the
apparent shift of the sun in northern hemisphere, the tropical eastern South Pacific Ocean experiences high
reversal in upper air circulation takes place due to pressure, the tropical eastern Indian Ocean experiences
easterly jet stream. low pressure. But in certain years, there is a reversal in
Statement 2 is correct. The shift in the position of the the pressure conditions and the eastern Pacific has lower
Intertropical Convergence Zone (ITCZ) is related to pressure in comparison to the eastern Indian Ocean.
the phenomenon of the withdrawal of the westerly jet This periodic change in pressure conditions is known as
stream from its position over the north Indian plain, the Southern Oscillation or SO. It has been noticed that
south of the Himalayas. The westerly stream is replaced when the pressure is high over the Pacific Ocean and low
by easterly jet stream which owes its origin to the heating over Indian Ocean, the Southwest monsoons in India tend
of the Tibetan plateau. This leads to the development of to be weaker. In the reverse case, the monsoons are most
an easterly cold jet stream centered around 15°N latitude likely to be stronger.
and blowing over peninsular India only after the western Option d is incorrect. As per the traditional theory,
jet stream has withdrawn itself from the region. This seasonal reversal of winds is caused by the differential
easterly jet stream is held responsible for the burst of the heating of land and sea (and not El Nino Southern
monsoon in India. Oscillation (ENSO)). Due to a higher temperature over
Statement 3 is incorrect. An easterly jet stream flows the land in summer, a low-pressure area develops over the
over the southern part of the Peninsula in June, and has continents and the winds blow from neighbouring oceans
a maximum speed of 90 km per hour. In August, it is towards the land. These winds are of maritime origin and
confined to 15°N latitude, and in September up to 22°N hence cause ample rainfall in summer.
latitudes. The easterlies normally do not extend to the However, German meteorologist Flohn argues that the
north of 30°N latitude (and not 15°N latitude) in the origin of the monsoon is caused by the seasonal shift of
upper atmosphere. the pressure and wind belts under the influence of the
shift of the vertical rays of the sun.
113. Solution: (c)
Exp) Option c is correct 114. Solution: (c)
The El Nino Southern Oscillation (ENSO) phenomenon Exp) Option c is correct
involves fluctuating ocean temperatures in the central and The Jet Stream is a geostrophic wind blowing horizontally
eastern equatorial Pacific, coupled with changes in the through the upper layers of the troposphere, generally
atmosphere. from west to east, at an altitude of 20,000 - 50,000 feet.
Option a is incorrect. The recent IMD forecast warns of Statement 1 is correct. The subtropical westerly jet
an impending La Nina effect, (not El Nino), that will streams blowing over India in winter cause a high
lead to a major dip in temperature across north India. pressure over northern India. It thus intensifies the

177 Workbook
GEOGRAPHY

northeast monsoons. This jet stream shifts northwards Option 1 is correct. In northern India rains are likely
beyond India in summer season and tropical easterly jets to fail if the rain-bearing storms are not very frequent
develop over India in this season. along the monsoon trough or the Inter Tropical
Statement 2 is correct. The behaviour of jet streams Convergence Zone (ITCZ) over this region. This would
(subtropical westerly and tropical easterly jet streams) is result in a drop in humidity levels in northern plains
partly responsible for the variations in the time of onset and the surface winds start blowing from the northwest
of southwest monsoons over India and the strength of the direction. This would reduce rainfall considerably.
monsoon. Timing of the shift of the subtropical westerly Option 2 is correct. Over the west coast the dry spells are
jet at the beginning of summer is critical to the onset of associated with days when monsoon winds blow parallel
the southwest monsoon over India. If the shift is delayed, to the coast.
so is the southwest monsoon. The strength of the easterly Option 3 is incorrect. The monsoon breaks are brought
tropical jet over central India determines the strength of about by the northward shifting of the monsoon trough
the southwest monsoon. (minimum low-pressure cell in ITCZ). The axis of the
A strong easterly tropical jet results in a strong trough lies at the foothills of the Himalayas during the
southwest monsoon and a weak jet result in a weak break period. Consequently, a southward position of
monsoon. the monsoon trough is usually an indication of well-
distributed rain over central India and the Indo-Gangetic
115. Solution: (c) plains.
Exp) Option c is correct Option 4 is incorrect. The Monsoon trough normally
The Peninsular region of India does not have any well- slopes southward with height because there is a drop in
defined cold weather season. Also, there is hardly temperature. The pressure gradient at surface levels over
any seasonal change in the distribution pattern of the the Peninsular India weakens, while it becomes more
temperature in coastal areas. over the Gangetic plains. Normally, the reverse happens
during the four month-long Monsoon season.
Option c is correct. The Peninsular region of India does
not have any well-defined cold weather season because of 118. Solution: (d)
moderating influence of the sea and the proximity to Exp) Option d is correct
equator. For example, the mean maximum temperature
Pair 1 is correct. The Eastern part of Rajasthan receives
for January at Thiruvananthapuram is as high as 31°C,
more rainfall and is drained by a number of rivers
and for June, it is 29.5°C. The air pressure is lower in
and their tributaries. Thus, it has been favourable for
these regions. Due to a low-pressure gradient, the light
agriculture. Cultivation crops like barley, gram, wheat,
winds blow outwards of the region.
mustard are grown in the rabi season.
116. Solution: (a) Pair 2 is incorrect. Jute grows well on well-drained
Exp) Option a is correct fertile soils in the flood plains where soils are renewed
every year. High temperature is required during the time
A rain shadow area is an area of dry land that lies on of growth. West Bengal, Bihar, Assam, Odisha and
the leeward (or downwind) side of a mountain. High Meghalaya are the major jute producing states. The west
mountains act as barriers for rain bearing winds. coastal plains are narrow and the flat river valleys that
Option a is correct. Arabian Sea branch of Southwest receive more than 200 cm. of annual rainfall and have
monsoons strikes the western coast of India and causes loamy soils. Rice is the main food crop grown in west
heavy rains on the western slopes of the Western Ghats. coast regions besides, plantations of bananas, coconut,
After crossing the Western Ghats, these winds cause less arecanut, cashewnut and rubber.
rainfall on the eastern slopes as they gain temperature Pair 3 is correct. With Sahyadri ranges running along the
while descending. This area is, therefore, known as coast of Maharashtra, the state is geographically divided
rain shadow zone. This explains why interior parts of into two regions; the Konkan coastal plains which are
Maharashtra, Karnataka and Telangana get meagre paddy fields and coconut gardens and the great river
rains from these winds. Southwest monsoons striking basins formed by the rivers flowing out from the Western
along the coast of Saurastra and Kuchchh and pass over Ghats (Sahyadri). Agriculture is the mainstay of the state
Madhya Pradesh and beyond to meet the Bay of Bengal of Maharashtra. The main food crops of Maharashtra are
branch. These winds cause widespread rain in western mangoes, grapes, bananas, oranges, wheat, rice, jowar,
Himalayan region also. bajra, and pulses. Cash crops include groundnut, cotton,
sugarcane, turmeric, and tobacco.
117. Solution: (a)
Exp) Option a is correct 119. Solution: (c)
During the south-west monsoon period after having rains Exp) Option c is correct
for a few days, if rain fails to occur for one or more weeks, According to the Intergovernmental Panel on Climate
it is known as break in the monsoon. These dry spells are Change (IPCC) report titled ‘Climate Change 2021: The
quite common during the rainy season. Physical Science Basis’, glacial retreat in the Hindu Kush

Workbook 178
GEOGRAPHY

Himalayas; compounding effects of sea-level rise and Alluvial soil can be classified into two groups on the basis
intense tropical cyclones leading to flooding; an erratic of its age - the khaddar and the bhangar. The former is
monsoon; and intense heat stress are likely to impact light in colour and is made up of newer deposits. The latter
India in recent years. is the older alluvium and is composed of lime nodules or
Option 1 is correct. The Indian Ocean, which includes kanker and its composition is clayey.
the Arabian Sea and Bay of Bengal, has warmed faster Statement a is incorrect. The Bhabar is a narrow belt
than the global average. The oceans factsheet released by about 8-16 km wide running in east-west direction along
IPCC indicates that sea surface temperature over Indian the foot of the Shiwalik. Rivers descending from the
ocean is likely to increase by 1 to 2 degrees C (°C) when Himalayas deposit their load along the foothills in the
there is 1.5°C to 2°C global warming. The rapid warming form of alluvial fans.
of Indian Ocean would lead to powerful and intense Therefore, the area is marked by dry river courses except
cyclones in the region. in the rainy season. The Bhabar belt is comparatively
Option 2 is incorrect. As per the IPCC report, heat narrow in the east and extensive in the western and
extremes have increased while cold extremes have north-western hilly region. The area is not suitable for
decreased, and these trends will continue over the coming agriculture and only big trees with large roots thrive in
decades over Asia. There are likely to be 90 to 120 days in this belt.
a year with maximum temperatures above 35°C in case of Statement b is incorrect. The Khadar is composed of
1.5 to 2°C global warming and over 180 days in case of newer alluvium and forms the flood plains along the river
4°C warming. banks. A new layer of alluvium is deposited by river flood
Option 3 is correct. Rainfall variability related to the El almost every year. These deposits are normally confined
Niño–Southern Oscillation is projected to be amplified by to the vicinity of the present river channels. The clays
the second half of the 21st century by the IPCC report. 2°C have less kankar, and the organic remains entombed in
global warming has serious implications due to possible them belong to still living species. .
increases in extreme weather events like heat waves, Statement c is correct. The Terai is a 15-30 km wide
heavy precipitation, intensification of tropical cyclones marshy tract in the south of Bhabar running parallel to
etc. Many regions are projected to experience an increase it. It is marked by the re-emergence of the underground
in the probability of compound events (i.e., different types streams of the Bhabar belt. The re- emerged water
of extreme events happening at the same time or one after converts large areas along the rivers into ill-drained
the other). For example, drought followed by heat waves. marshy lands of excessive dampness covered with thick
Dry becomes drier and wet becomes wetter. This can be forests giving shelter to a variety of wild life.
particularly alarming for India.
The Tarai is more marked in the eastern part than in the
120. Solution: (b) west because the eastern parts of the plain receive higher
amount of rainfall than their western counterparts.
Exp) Option b is correct
Most of the Tarai land, especially in Punjab, Uttar
The Wet Tropical Evergreen Vegetation is found in Pradesh and Uttarakhand, has been reclaimed and
regions of very high annual rainfall exceeding 300 cm turned into agricultural land which gives good crops of
with a very brief dry season. Southern parts of Western sugarcane, rice and wheat.
Ghats of Kerala and Karnataka are very wet. It resembles
Statement d is incorrect. The Bhangar (or Bangar) is
the equatorial vegetation.
composed of old alluvium of the Middle Pleistocene
Option b is correct. Features of the Wet Tropical Evergreen age and forms the alluvial terrace above the level of
Vegetation: flood plains. It is often impregnated with calcareous
Though these forests are dense and have lofty evergreen concretions known as kankar.
trees, often as high as 60 metres and above, grass is almost Remnants of the Bhangar are eroded by every change
absent. in the direction of river channels, and are being levelled
Undergrowth is very dense and thick. down by their meandering tendencies. It mostly occupies
The wood of these trees is very hard and heavy to work the Pleistocene terrace dating back to Middle and Upper
with. Pleistocene periods.
The number of vegetal species per unit area is too large 122. Solution: (d)
to exploit them commercially. Exp) Option d is correct.
Mahogony, cinchona, bamboos and palms are typical Khejri is an important tree of dry lands. Khejri tree plays
species of plants found in these forests. a vital role in maintaining the ecosystem of the dreary
North-eastern Hills are known for this type of vegetation. Thar region because of its ability to survive in such tough
This type of vegetal cover has been badly depleted due to conditions. Apart from being a source of firewood and
over cutting of trees. fodder, the Khejri also helps in sustaining the nutrient
value of the soil and ensuring a good yield.
121. Solution: (c) Statement 1 is correct. Khejri (Prosopis cineraria) covers
Exp) Option c is correct. about two-thirds of the total geographical area of the

179 Workbook
GEOGRAPHY

State and is of immense significance culturally and Statement 2 is incorrect. Chemically, the black soils
economically. are rich in lime, iron, magnesia and alumina. They also
Statement 2 is correct. The fruit is used to make the very contain potash. But they lack in phosphorous, nitrogen
popular local dish ‘Sangri’ and can fetch up to Rs 300 and organic matter. The colour of the soil ranges from
per kilo for the farmers. “Khejri bark is used medicinally deep black to grey.
for a range of ailments and can be ground up and made Statement 3 is correct. In all regur soils in general,
into flour, and famously, saved thousands of lives during and in those derived from ferromagnesian schists in
the Great Rajputana Famine of 1868. The flowers make an particular, there is a layer rich in kankar nodules formed
amazing bee-fodder. by segregation of calcium carbonate at lower depths. As a
Statement 3 is correct. In 1730 AD, around 363 people general rule, black soils of uplands are of low fertility
sacrificed their lives in an attempt to save the trees by but they are darker, deeper and richer in the valleys.
hugging them while the then king’s men chopped their Statement 4 is correct. Because of their high fertility
bodies with axes. The trees which they were protecting and retentivity of moisture, the black soils are widely
were ‘Khejri’. In 1970s, this sacrifice became the used for producing several important crops. Some of
inspiration behind the Chipko Movement. the major crops grown on the black soils are cotton,
Statement 4 is correct. It is the state tree of Rajasthan. wheat, jowar, linseed, Virginia tobacco, castor, sunflower
However, the tree is said to be dying vis-a-vis losing its and millets. Rice and sugarcane are equally important
natural cover. The root cause of decline in the Khejri cover where irrigation facilities are available. Large varieties of
is its excessive lopping (cutting of branches), which all vegetables and fruits are also successfully grown on the
farm owners do annually to procure its fruit, pods, leaves, black soils.
branches and twigs.
125. Solution: (c)
123. Solution: (a) Exp) Option c is correct.
Exp) Option a is correct. Mountain Soils are mainly heterogeneous soils found
Alluvial soils are widespread in the northern plains and mostly on the hill slopes covered by forests. The formation
the river valleys. These soils cover about 40 per cent of of these soils is mainly governed by the characteristic
the total area of the country. They are depositional soils, deposition of organic matter derived from forests and
transported and deposited by rivers and streams. their character changes with parent rocks, ground-
Statement 1 is correct. The alluvial soils vary in nature configuration and climate.
from sandy loam to clay. They are generally rich in Statement 1 is correct. The mountain soils are complex
potash but poor in phosphorous. and extremely varied. The soils vary from deep alluvium
Statement 2 is correct. In the Upper and Middle Ganga in the river basins and lower slopes to highly immature
plain, two different types of alluvial soils have developed, residual gravelly on higher altitudes. Because of complex
viz. Khadar and Bhangar. Khadar is the new alluvium topographic, geologic, vegetation and climatic conditions,
and is deposited by floods annually, which enriches the no large areas of homogenous soil groups are found.These
soil by depositing fine silts. Bhangar represents a system soils are heterogeneous in nature and their character
of older alluvium, deposited away from the flood plains. changes with parent rocks, ground-configuration and
climate. Consequently, they differ greatly even if they
Statement 3 is incorrect. Both the Khadar and Bhangar
occur in close proximity to one another.
soils contain calcareous concretions (Kankars). These
soils are loamier and more clayey in the lower and middle Statement 2 is correct. These soils are very rich in humus
Ganga plain and the Brahmaputra valley. The sand content but are deficient in potash, phosphorus and lime.
decreases from the west to east. Therefore, they require good deal of fertilizers for high
yields.
124. Solution: (a)
Exp) Option a is correct. 126. Solution: (b)
The Black soils are generally clayey, deep and impermeable. Exp) Option b is correct.
Black soil covers most of the Deccan Plateau which Land degradation refers to the deterioration or loss
includes parts of Maharashtra, Madhya Pradesh, Gujarat, of the productive capacity of the soils for present and
Andhra Pradesh and some parts of Tamil Nadu. future.
Statement 1 is correct. Black Soil swell and become Statement 1 is correct. Recently, a document published
sticky when wet and shrink when dried. So, during the by ISRO (Indian Space Research Organization) named
dry season, these soils develop wide cracks. Thus, there Desertification and Land Degradation Atlas shows that
occurs a kind of ‘self-ploughing’. Because of this character Land Degradation and Desertification has increased
of slow absorption and loss of moisture, the black soil significantly in recent years. India witnessed an increase
retains the moisture for a very long time, which helps the in the level of desertification in 28 of 31 states and Union
crops, especially, the rain fed ones, to sustain even during territories between 2011-13 and 2018-19, a closer look at
the dry season. data in the atlas showed.

Workbook 180
GEOGRAPHY

Around 23.79% of the area undergoing desertification designated to be of international importance — because
/ land degradation with respect to TGA of the country of the staggering diversity of birds it attracts. The lake
was contributed by Rajasthan, Maharashtra, Gujarat, supports flamingos and migratory birds from as far away
Karnataka, Ladakh, Jharkhand, Odisha, Madhya Pradesh as Siberia that feeds on the algae and micro-organisms
and Telangana. found in the saline waters.
Statement 2 is incorrect. Vegetation degradation is In late 2019, the lake bed had turned into a mass
defined as, “the temporary or permanent reduction in the graveyard for migratory birds due to Avian botulism.
density, structure, species composition or productivity of An estimated 25,000 birds dropped dead: Kentish plovers,
vegetation cover”. It is found to be responsible for 9.15% of tufted ducks, northern shovelers, pied avocets, little ringed
desertification in the country. plovers, stilts and gadwalls, among 36 species.
Statement 3 is correct. Badlands are a type of dry
129. Solution: (c)
terrain where softer sedimentary rocks and clay-
rich soils have been extensively eroded. It results in Exp) Option c is correct.
Badland Topography which itself is an initial stage of Option a is incorrect. Indus river also known as the
desertification. In 2011-13, water erosion was responsible Sindhu, is the westernmost of the Himalayan rivers in
for 10.98% of desertification in the country. India. It originates from a glacier near Bokhar Chu in
Statement 4 is correct. Wind erosion was found to be the Tibetan region in the Kailash Mountain range. The
responsible for 5.46% of the desertification in India. Sutlej originates in the ‘Raksas Tal’ near Manasarovar in
Sand encroachment by wind reduces fertility of the soil Tibet where it is known as Langchen Khambab.
making the land susceptible to desertification. Option b is incorrect. Beas river flows through the
Kullu valley and forms gorges at Kati and Largi in the
4.3. Drainage Dhaoladhar range. Sutlej river flows almost parallel to the
Indus for about 400 km before entering India and comes
127. Solution: (c) out of a gorge at Rupar. It passes through the Shipki La on
Exp) Option c is correct. the Himalayan ranges and enters the Punjab plains.
Option c is correct. Sutlej is an antecedent river
(continued flowing in its course despite the upliftment
of area as Himalayas). It is a very important tributary as
it feeds the canal system of the Bhakra Nangal project.

Pair 1 is incorrectly matched – The Dhola–Sadiya


Bridge is also referred to as the Bhupen Hazarika Setu.
The bridge spans the Lohit River, a major tributary of the
Brahmaputra River, from the village of Dhola in the south
to Sadiya to the north.
Pair 2 is correctly matched – A 4.94 km long bridge,
Bogibeel is the country’s longest rail-cum-road river
bridge. The bridge will be over the river Brahmaputra Indus River system
and will connect Dibrugarh in Assam to Pasighat in Option d is incorrect. Jhelum river in the valley of
Arunachal Pradesh. It has two railway lines at its lower Kashmir is still in its youth stage and yet forms meanders
deck and a three-lane road bridge on the top. – a typical feature associated with the mature stage in the
Pair 3 is correctly matched – Mahatma Gandhi Setu evolution of fluvial land form.
bridge is over river Ganga in the state of Bihar. The 5.75
km long bridge between Patna and Hajipur, over the river 130. Solution: (a)
Ganga, is the next on the list of river bridges. Exp) Option a is correct
Statement 1 is correct – The Chota Nagpur plateau is
128. Solution: (b)
drained by numerous rivers and streams in different
Exp) Option b is correct. directions and presents a radial drainage pattern
Sambhar Lake spread over the 230 sq. km. area is (When the rivers originate from a hill and flow in all the
shallow, elliptical wetland straddling across the districts directions, the drainage pattern is ‘radial’).
of Jaipur, Nagaur and Ajmer of Rajasthan. Statement 2 is incorrect – The Karbi-Anglong plateau
Sambhar has been designated a Ramsar site — a wetland lies in the east of the Meghalaya Plateau.

181 Workbook
GEOGRAPHY

the south and the Vindhyan range in the north, it forms


a picturesque gorge in marble rocks and Dhuandhar
waterfall near Jabalpur.
Pair 2 is incorrect – The Godavari rises from
Triambakeshwar Plateau in the Nasik district of
Maharashtra and discharges its water into the Bay of
Bengal. It is the largest Peninsular river system. It is also
called the Dakshin Ganga.
Pair 3 is incorrect – The Krishna is the second largest east
flowing Peninsular river which rises near
Mahabaleshwar in Sahyadri hills. Its total length is 1,401
km.
Pair 4 is correct – The Kaveri rises in Brahmagiri hills
(1,341m) of Kogadu district in Karnataka. Its
length is 800 km and it drains an area of 81,155 sq. km.
131. Solution: (c)
134. Solution: (d)
Exp) Option c is correct.
Exp) Option d is correct.
Option 1 is incorrect. As the western flowing rivers travel
smaller distance over a more inclined terrain, they don’t Statement 1 is correct – The Brahmaputra features
get enough time to form deltas. For the formation of delta, among the world’s top five rivers in terms of discharge
the length of the river from the point of origin to the sea as well as the sediment it brings. As the river comes from
should be long enough. a high slope to a flat plain, its velocity decreases suddenly
and this results in the river unloading the sediment,
Option 2 is correct. Presence of Hard rocks is a reason leading to floods.
behind non-formation of deltas as the west flowing
rivers, especially the Narmada and the Tapi, flow through Statement 2 is correct – Unplanned settlement of people
in floodplains restricts the space the river has to flow. It
hard rocks and hence do not carry any good amount of
leads to destructive floods when the rainfall is heavy.
silt.
Statement 3 is correct – Because of the earthquake-
Option 3 is correct. As west flowing rivers rapidly from
prone nature of the Assam region, the river has not
steep slope and merges into Arabian sea so they form
been able to acquire a stable character which aggravates
estuaries and not deltas. flooding. Following the devastating earthquake of 1950,
Option 4 is correct. The West flowing rivers flow the level of the Brahmaputra rose by two metres in the
through faults (linear rift, rift valley, trough) so much Dibrugarh area in eastern Assam.
of the silt gets deposited there itself. Thus, less alluvium
gives no opportunity in the formation of delta. 135. Solution: (c)
Exp) Option c is correct.
132. Solution: (d)
Statement 1 is incorrect. River Tapi originates near Multai
Exp) Option d is correct. reserve forest in Betul district of Madhya Pradesh.
Statement 1 is correct – The Himalayan Rivers existed Statement 2 is correct. The important tributaries of Tapi
even before the formation of Himalayas and cut their river are the Suki, the Gomai, the Arunavati and the Aner
courses southward by making gorges in the mountains. which joins it from right, and those joining from left are
Therefore, the river Indus is an antecedent river. the Vaghur, the Amravati, the Buray, the Panjhra, the Bori,
Statement 2 is incorrect – The Galwan river is not right- the Girna, the Purna, the Mona and the Sipna.
bank tributary of the Indus. The Galwan is the tributary Statement 3 is correct. River Tapi flows through rift
of river Shyok which is Indus’s right bank tributary. valley. It also flows from west to east. (River Narmada and
Statement 3 is correct – The Indus flows in India through Tapi both flow through the rift valley)
the Leh and Kargil districts in Ladakh. Then for around Statement 4 is incorrect. For the first 282 Km, the river
200 KM the river flows through the Pakistan Occupied flows in Madhya Pradesh, out of which 54 Km forms
Kashmir. Thus, officially along with Union Territory of the common boundary with Maharashtra State. It flows
Ladakh, river Indus also flows through Union Territory for 228 Km in Maharashtra draining Khandesh and east
of Jammu and Kashmir. Vidarbha regions in the northwest corner of the Deccan
Plateau before entering Gujarat.
133. Solution: (c)
Exp) Option c is correct. 136. Solution: (d)
Pair 1 is correct – The Narmada originates on the western Exp) Option d is correct.
flank of the Amarkantak plateau at a height of about Pair 1 is correctly matched: National Waterway 2 (NW
1,057 m. Flowing in a rift valley between the Satpura in 2) lies on a section of the Brahmaputra River and has

Workbook 182
GEOGRAPHY

a length of 891 km. It connects Dhubri and Sadiya in Statement 3 is correct. The lake was declared a notified
Assam. National Geo-Heritage Monument in 1979 and had been
Pair 2 is incorrectly matched: National Waterway 3 added to the list of recognized Ramsar sites in 2020.
(NW 3) or the West Coast Canal is located in Kerala. It 138. Solution: (d)
runs from Kollam to Kottapuram.
Exp) Option d is correct.
Pair 3 is incorrectly matched: National Waterway 6 is
proposed on the river Barak. It will connect Lakhipur Statement 1 is incorrect. River Yamuna has its source
and Bhanga. in the Yamunotri glacier on the western slopes of
Banderpunch range (6,316 km) in the Garhwal region
137. Solution: (d) in Uttarakhand.
Exp) Option d is correct. The Alaknanda has its source in the Satopanth glacier
Statement 1 is correct. Lonar Lake is located in above Badrinath.
Maharashtra on Deccan plateau. The lake has saline Statement 2 is correct. The Yamuna is the western most
water with pH of 10.5 as it is enclosed from all sides, a and the longest tributary of the Ganga. It joins the Ganga
continuous cycle of precipitation and evaporation has at Prayag (Allahabad).
made its water saline and alkaline. Statement 3 is incorrect. River Yamuna is joined by the
Statement 2 is correct. The oval-shaped Lonar lake was Chambal, the Sind, the Betwa and the Ken on its right
formed after a meteorite hit the earth some 50,000 years bank which originates from the Peninsular plateau; while
ago and it is a popular tourist hub. the Hindan, the Rind, the Sengar, the Varuna, etc. join it

183 Workbook
GEOGRAPHY

on its left bank. examples of antecedent rivers.


Statement 4 is correct. The Hathni Kund is a concrete 2. Superimposed – The river originally flows on a soft
barrage located on the Yamuna River in Yamuna Nagar rock bed and over time it exposes an underlying
district of Haryana, India. The barrage is infamous for region of hard rock. The river sticks to its original
flooding in the parts of Delhi in case of excess rainfall in path by cutting through the exposed landform.
higher reaches of the river. Examples include The Damodar, The Subarnarekha,
The Chambal etc.
139. Solution: (b)
Concordant – The river flows in accordance with the
Exp) Option b is correct. topography and geology of the drainage region. The
Relevance: Conservation reserves including Nationals path of the river is highly dependent on the slope, the
Parks are very much important for Prelims. arrangement of rocks, the hardness/softness of rocks etc.
Pair 1 is correct – Ken River passes through the Panna Peninsular rivers are examples of concordant drainage
National Park. Recently due to Ken-Betwa River link pattern.
project, the core tiger area of Panna is coming under
submergence. 142. Solution: (a)
Pair 2 is incorrect – The river Manas flows into the Exp) Option a is correct.
Manas national Park from the gorges of Bhutan and split Pair 1 is correctly matched: Son is a right bank tributary
into two major streams of which the main water course of Ganga. It rises near Amarkantak Hill (MP) and merges
comes out of the National Park about 30 km downstream with Ganga near Patna, Bihar. It flows along the Kaimur
is known as ‘Beki”. Hills of Central India. It passes through states of MP, UP,
Brahmaputra river flows through Kaziranga National Jharkhand and Bihar.
Park. Pair 2 is correctly matched: Chambal is a major right
Pair 3 is incorrect – The rivers Budhabalang, Baitarani bank tributary of Yamuna river. It rises in the highlands
and Subarnarekha flow through Simlipal National Park. of Janapao Hills in the Vindhyan range. It joins the
Simlipal Biosphere Reserve is also designated under Yamuna in Etawah district of UP.
World Network of Biosphere Reserves (WNBR) under Pair 3 is incorrectly matched: Bhima river is a major
UNESCO’s Man and the Biosphere (MAB) Programme. left bank tributary of Krishna river. It flows through the
River Mahanadi passes through Satkosia Tiger Reserve. states of Maharashtra, Karnataka, and Telangana before
Pair 4 is correct – Nagarhole National Park is drained entering the Krishna. It flows through the Bhimashankar
by the rivers Kabini, Lakshmana Teertha and Nagarahole Wildlife Sanctuary in Maharashtra.
rivers. The Kabini River is one of the major tributaries of Pair 4 is incorrectly matched: Pennar is east-flowing
the river Cauvery in southern India. river in south India. It rises in the Nandi hills in
Chikkaballapur district of Karnataka and runs across
140. Solution: (d) the states of Karnataka and AP to empty into the Bay of
Exp) Option d is correct. Bengal.
Option a and b are incorrect. Kumaon and Nilgiri hills 143. Solution: (d)
show annular drainage pattern. In such a pattern, the
radial streams develop subsequent tributaries which try Exp) Option d is correct.
to follow a circular drainage around the summit. Godavari river rises in Trimbakeshwar in the Nashik
Option c is incorrect. Sahyadari hills show dendritic district of Maharashtra and makes a huge delta along the
drainage pattern. River Krishna originates from these Bay of Bengal coast in Andhra Pradesh. The main channel
hills. flows through the states of Maharashtra, Telangana, and
Andhra Pradesh. However, Godavari receives waters from
Option d is correct. When the rivers originate from a
hill and flow in all directions, the drainage pattern is numerous tributaries whose basins extend over the states
known as ‘radial’. Rivers like Narmada, Son and Mahanadi of Madhya Pradesh (Pench, Wainganga), Chhattisgarh
originating from Amarkantak range flow in different (Indravati), Odisha (Indravati, Sabari) and Karnataka
directions and are good examples of radial pattern. (Manjira).
This pattern is also found in the Girnar Hills of Gujarat 144. Solution: (b)
and Mikir Hills of Assam. Exp) Option b is correct.
141. Solution: (a) Pair 1 is correct: Chambal rises in the Janapao Hills of the
Exp) Option a is correct. Vindhyan range.
1. Antecedent – Even when a part of the river slope Pair 2 is correct: Son rises near Amarkantak hill in
gets uplifted, the river sticks to its original slope Madhya Pradesh.
by cutting through the uplifted land. The river has Pair 3 is incorrect: Banas River rises in the Aravali
strong capacity for vertical erosion. Himalayan rivers Range. It lies entirely in Rajasthan. It is a major tributary
such as The Indus, The Ganga, The Brahmaputra are of Chambal River.

Workbook 184
GEOGRAPHY

Pair 4 is incorrect: Damodar river rises in the Palamau Statement 2 is incorrect: These rivers were not present
hills of Chotanagpur Plateau and flows through a rift before the consequent landforms. The present landform
valley. has played an important role in shaping the current
drainage pattern. This is called as consequent drainage.
145. Solution: (c) Whereas in Himalayan river the drainage pattern is
Exp) Option c is correct. antecedent which means it was present even before the
Statement 1 is correct: Mahi river rises from the northern creation of contemporary landforms.
slopes of Vindhyas in Madhya Pradesh. It flows through Statement 3 is correct: Unlike Himalayan rivers which
the states of MP, Rajasthan, and Gujarat. It drains into the flow through deep V-shaped valley (gorges), Peninsular
Arabian sea through the Gulf of Khambat. rivers flow in a comparatively shallow valleys, which are
Statement 2 is incorrect: Vaigai is a minor peninsular more or less completely graded.
river that drains into the Bay of Bengal. 149. Solution: (c)
Statement 3 is incorrect: Luni is the only major river Exp) Option c is correct.
in the arid region of Western Rajasthan. It rises in the
Aravalis and drains into Rann of Kutch, without getting Ganga river basin has the largest basin area in India. It
a chance to reach the Arabian sea. is a Himalayan river with a comprehensive network of
tributaries and distributaries. It is formed by two head
Statement 4 is correct: Mandovi is a fast-flowing stream streams, Alaknanda and Bhagirathi which joins at
that rises in the Western ghats and flows westwards into Devprayag. Its left bank tributaries from west to east are:
Arabian sea. It is a major river of the state of Goa.
Ramganga
146. Solution: (a) Gomti
Exp) Option a is correct. Ghaghara
Pair 1 is correctly matched: Hirakud Dam is constructed Gandak
on Mahanadi river, in Odisha. It is one of the first
Burhi Gandak
multipurpose river valley projects started after India’s
independence. Kosi
Pair 2 is correctly matched: Bhakra Nangal Dam is built Its major right bank tributaries are Yamuna and Son.
on Sutlej river, in Himachal Pradesh. It forms the Gobind
150. Solution: (c)
Sagar reservoir.
Exp) Option c is correct.
Pair 3 is incorrectly matched: Gandhi Sagar Dam is
one of the four major dams built on Chambal river. It is Statement 1 is incorrect: River regime is the pattern of
located in Madhya Pradesh. flow of water in a river channel over a year. In simple
terms it is the seasonal fluctuation in the volume of
Pair 4 is incorrectly matched: Nagajuna Sagar Dam is
water in a river.
built on Krishna river at Nagarjuna Sagar, which is located
on the boundary between Andhra Pradesh and Telangana. The volume of water flowing in the river over a time is
called discharge and it is measured in cusecs (cubic feet
147. Solution: (c) per second).
Exp) Option c is correct. Statement 2 is incorrect: Ganga has its peak during the
Statement 1 is correct. The Peninsular drainage system is monsoon period; hence it has a monsoonal regime. But
older than the Himalayan one. This is evident from the its minimum flow is not in January but in April and
broad, largely-graded shallow valleys, and the maturity of May. As Ganga has a large catchment area in its upper
the rivers. The Western Ghats running close to the western reaches it receives rainfall in winters due to Western
coast act as the water divide between the major Peninsular Disturbances. As a result the water flow in January is
Rivers, discharging their water in the Bay of Bengal and as considerably higher than that in April or May.
small rivulets joining the Arabian Sea.
151. Solution: (a)
Statement 2 is correct. Peninsular rivers are
Exp) Option a is correct
characterized by fixed course, absence of meanders
and non-perennial flow of water. The Narmada and Statement 1 is correct: National Waterway 2 is
the Tapi which flow through the rift valley are, however, constructed on Brahmaputra river from Sadiya to
exceptions. Dhubri, stretching 891 km.
Statement 2 is correct: The West Coast Canal from
148. Solution: (a) Kollam to Kottapuram was declared as National
Exp) Option a is correct. Waterway 3 in 1993. This canal has a navigable length of
Statement 1 is correct: Peninsular rivers have been 205 km. It comprises of natural lakes, back-waters, river
flowing for a long period. They have senile topography sections and man-made canal sections.
and show features of mature stage. They have almost Statement 3 is incorrect: Kakinada-Puducherry canal
reached the base level of erosion. stretch along the Godavari River stretch and Krishna

185 Workbook
GEOGRAPHY

River stretch is termed as NW-4 with a total length of 155. Solution: (d)
1095 km. Exp) Option d is correct
National Waterways 6 is a waterway between Lakhipur Statement 1 and 2 both are incorrect. The variability
and Bhanga of the Barak River. of rainfall is measured by the values of co-efficient of
variation which shows the changes from the mean values
4.4. Indian Climate of the rainfall. A variability of less than 25% exits on the
152. Solution: (c) western Ghats, western coasts, north eastern peninsula,
the eastern plains of the Ganga, north-eastern India,
Exp) Option c is correct. Uttarakhand and Himachal Pradesh and south-western
Statement 1 is correct – It is a broad trough of low part of Jammu and Kashmir etc. These areas have an
pressure in equatorial latitudes. In the month of July, annual rainfall of over 100cm. While the variability of
because of intense heating of Indian peninsula, the ITCZ over 50% exists in the western part of the Rajasthan,
shifts northwards, roughly parallel to Himalayas between northern part of Jammu and Kashmir and interior parts
20°-25° N latitude. of Deccan plateau. This shows the variability of rainfall
Statement 2 is incorrect – The shift in the position of the is highest at the place where average annual rainfall is
ITCZ is related to the phenomenon of the withdrawal of lowest.
the westerly jet stream from its position over the north 156. Solution: (b)
Indian plain, south of the Himalayas. Once the Westerly
jet stream has withdrawn, the easterly Jet Stream (Somali Exp) Option b is correct
Jet) sets in along 15°N latitude only after the western jet Statement 1 is incorrect. The climate is of cool temperate
stream has withdrawn itself from the region. type if mean temperature of the warmest month is over
Statement 3 is correct – The ITCZ being a zone of 10°C and mean temperature of the coldest month is
low pressure, attracts inflow of winds from different under minus 3°C.
directions. The maritime tropical air mass from the Statement 2 is correct. The climate is of tropical type
southern hemisphere, after crossing the equator, rushes when mean monthly temperature throughout the year is
to the low-pressure area in the general south-westerly over 18°C.
direction. It is this moist air current which is popularly
157. Solution: (b)
known as the southwest monsoon.
Exp) Option b is correct
153. Solution: (d) Statement 1 is incorrect: Monsoonal rainfall in southern
Exp) Option d is correct India is due to two different phenomena. Malabar Coast
Statement 1 is correct. The retreating south-west receives its major portion of rainfall from south-west
monsoon is marked by clear skies and rise in temperature. monsoon. Whereas the Coromandel Coast receives
major part of its rainfall due to the retreating north-east
Statement 2 is correct. As the monsoons retreat,
monsoon.
the monsoon trough weakens and gradually shifts
southward. Consequently, the pressure gradient is low. Statement 2 is correct: Intensity of rainfall goes on
decreasing with increase in distance from the sea. As
Statement 3 is correct. With retreat of the monsoons, the the moisture-bearing winds of the Bay of Bengal branch
clouds disappear and the sky becomes clear The diurnal of the southwest monsoon move inland, the moisture
range of temperature increases due to lack of cloud gradually decreases and results in low rainfall.
cover.
158. Solution: (d)
154. Solution: (c)
Exp) Option d is correct.
Exp) Option c is correct.
Statement 1 is correct: In a positive Indian Ocean
Statement 1 is correct. The duration of the monsoon Dipole the temperature in western Indian Ocean is
decreases from southern India to northern India. The comparatively higher than eastern Indian Ocean.
reasons behind this pattern is that since the rainfall in This high temperature creates a low pressure area thus
the Indian subcontinent is caused by moisture bearing strengthens the monsoon winds. On the other hand, in
south-west monsoon winds, the duration of the monsoon an El Nino year monsoon winds are weak. So a positive
is highest nearby coastal regions and decreases onwards. IOD can compensate for an El Nino year.
This happens due to fast loosening of moisture in these
Statement 2 is correct: Tropical cyclones originate
winds.
mostly in October and November in the Bay of Bengal.
Statement 2 is correct. The amount of annual rainfall Reasons for this are:
in the northern plains of India decreases from east to
Large fresh water influx
west. As the moisture bearing winds of the Bay of Bengal
branch of the south west monsoon move further inland, Comparatively warmer than Arabian Sea
the moisture gradually decreases and results in low rainfall Low pressure circulations brought here by trade winds
in the western states. from China Sea

Workbook 186
GEOGRAPHY

Statement 3 is incorrect: Western Disturbances are 162. Solution: (a)


low pressure depressions which originate in the Exp) Option a is correct.
Mediterranean Sea. They are brought to India by Tropical
Statement I is correct. In summer season during active
Westerly Jet Streams and responsible for winter rainfall
phase the tropical easterly jet stream remains very
in northern India. India receives the western disturbances strong in the upper troposphere indicating very strong
mostly between the months of December and March. convection and latent heating. But when the maximum
159. Solution: (c) cloudiness remains locked up in the foothills of Himalayas
and the monsoon rainfall zone moves in this direction,
Exp) Option c is correct. subsidence occurs to produce a weak easterly flow in the
Option a is incorrect. Mango Shower are pre-monsoon upper troposphere. This creates the condition of break
showers which are a common phenomenon in Kerala and in monsoon.
coastal areas of Karnataka towards the end of summer. Statement 2 is correct. In break monsoon condition,
Locally, they are known as mango showers since they help there is general rise of pressure (as well as temperature)
in the early ripening of mangoes. over the country. Due to the shifting of ITCZ over the
Option b is incorrect. Blossom Shower are common in Himalayas, the cloudiness decreases and south-easterlies
Kerala and nearby areas. With this shower, coffee flowers at the surface level over northern India are replaced
blossom. by hot westerly air which blows over the Indian plains.
Thus, rainfall practically ceases over the country south of
Option c is correct. Norwesters are dreaded evening the Himalayas region leading to high floods in the plain
thunderstorms in Bengal and Assam. Their notorious of these Himalayas rivers. Thus, there is no rain over the
nature can be understood from the local nomenclature of plains.
‘Kalbaisakhi’, a calamity of the month of Baisakh. These
Statement 3 is incorrect. Dry spells occur along the west
showers are useful for tea, jute and rice cultivation. In
coast as wind blows parallel to the western coast, not
Assam, these storms are known as “Bardoli Chheerha”.
the eastern coast.
Option d is incorrect. Loo is hot, dry and oppressing
winds blowing in the Northern plains from Punjab to 163. Solution: (c)
Bihar with higher intensity between Delhi and Patna. Exp) Option c is correct.
160. Solution: (b) Statement 1 is correct. During winter in peninsular
regions, the north east monsoon while crossing over the
Exp) Option b is correct. Bay of Bengal picks up moistures and causes torrential
Statement 1 is correct. The term ‘drought’ is applied to an rainfall over the Coromandel coast and goes on decreasing
extended period when there is a of water availability due from east to west. In extra peninsular regions due to some
to inadequate precipitation, excessive rate of evaporation weak temperate cyclones from the Mediterranean Sea
and over-utilization of water from the reservoirs and cause rainfall in Punjab, Haryana, Delhi etc and starts
other storages, including the ground water. Hydrological decreasing from west to east as it moves forwards.
drought results when the availability of water in different Statement 2 is correct. During summer, the Arabian sea
storages and reservoirs like aquifers, lakes, reservoirs, etc. branch brings heavy rainfall on the windward side of the
falls below what the precipitation can replenish. Sahaydris and as it moves forwards, these winds descend
Statement 2 is incorrect. Agricultural drought, also and gets heated up and reduce humidity in winds and
called as soil moisture drought, characterized by low soil thus the rainfall decreases from west to east in peninsular
moisture that is necessary to support the crops, thereby regions. While in extra peninsular regions, the Bay of
Bengal branch brings widespread rainfall due to the hill
resulting in crop failures. Moreover, if an area has more
ranges in North-East India and eastern Himalayas and as
than 30 per cent of its gross cropped area under irrigation,
it moves from east to west in these regions rainfall goes
the area is excluded from the drought-prone category.
on decreasing. Also, the Arabian sea branch brings scanty
161. Solution: (b) rainfall over the Western Rajasthan due to parallel ranges
of Aravalis.
Exp) Option b is correct.
Statement 1 is incorrect. During winter season, the 164. Solution: (d)
tropical cyclones originate over Bay of Bengal and the Exp) Option d is correct
Indian Ocean. These tropical cyclones have very high Statement 1 is incorrect. The Chotanagpur plateau gets
wind velocity and heavy rainfall and hit Tamil Nadu, 15 cm rainfall from Arabian Sea branch. And the Bay of
Andhra Pradesh and Orissa coast, not West Bengal. Bengal moving from south-east and south enters west
Statement 2 is correct. The western cyclonic disturbances Bengal and also brings rainfall over chhotanagpur.
which enter the Indian subcontinent from the west and Statement 2 is incorrect. The Bay of Bengal enter West
the northwest during the winter months originate over Bengal and Bangladesh from south-east and south. From
Mediterranean Sea and are brought into India by the here this branch splits into 2 under the influence of the
Subtropical westerly Jet streams. Himalayas. It’s one branch moves westward along the

187 Workbook
GEOGRAPHY

Ganga plains reaching as far as the Punjab plains. The Statement 2 is correct. The Jet Stream is a geostrophic
other branch moves up the Brahmaputra valley in the wind blowing horizontally through the upper layers of the
north and the north east. troposphere, generally from west to east, at an altitude of
20,000 - 50,000 feet. They develop where air masses of
165. Solution: (d) differing temperatures meet.
Exp) Option d is correct Statement 3 is correct. The western cyclonic disturbances
The Indian climate is controlled by a number of factors are weather phenomena of the winter months brought in
which can be broadly divided into two groups — factors by the westerly flow from the Mediterranean region.
related to location and relief, and factors related to air They enter the Indian subcontinent from the west and the
pressure and winds. Some of these factors are: northwest and usually influence the weather of the north
• Latitudinal location and north-western regions of India.
• Distance from the Sea (Continentality) These disturbances also affect the coastal regions of the
country.
• Altitude
• Himalayas 169. Solution: (b)
• Physiography Exp) Option b is correct.
• Monsoon Winds Statement 1 is incorrect. Intertropical convergence zone
• Upper Air Circulation (ITCZ), also called equatorial convergence zone, is a low-
pressure zone located at the equator where trade winds
• El Nino and La Nina converge, and so, it is a zone where air tends to ascend.
• Tropical Cyclones and Western Disturbances Statement 2 is correct. The Inter Tropical Convergence
166. Solution: (c) Zone (ITCZ) lies more or less parallel to the equator but
moves north or south with the apparent movement of the
Exp) Option c is correct. sun.
Statement 1 is correct. The difference between the daily Statement 3 is correct. The rising air produces high
maximum and minimum temperature is called Diurnal cloudiness, frequent thunderstorms, and heavy rainfall;
range of temperature. Highest diurnal temperature the doldrums, oceanic regions of calm surface air, occur
ranges occur in the Thar desert, where if the day within the zone. This is why it is known to sailors around
temperature is around 50°C, at night, it may drop the world as the doldrums.
down considerably upto 15°-20°C. The highest annual
temperature ranges are recorded in the Himalayan region. 170. Solution: (a)
Statement 2 is correct. In general, coastal areas Exp) Option a is correct
experience less contrasts in temperature conditions. Option a is correct. EI-Nino involves oceanic and
Seasonal contrasts are more in the interior of the country. atmospheric phenomena with the appearance of warm
Both diurnal and mean annual temperature ranges are currents off the coast of Peru in the Eastern Pacific and
least in coastal regions. affects weather in India. Cyclogenesis is the development
167. Solution: (c) or strengthening of an area of low pressure in the
atmosphere, resulting in the formation of a cyclone.
Exp) Option c is correct
Effect of El-Nino:
Statement 1 is correct. During winter, high air pressure
prevails over large parts of north-west India due to low It adversely affects monsoon rainfall and cyclogenesis
temperatures coupled with divergence induced by the in the Bay of Bengal. Droughts are common during El
ridge of the Sub-tropical Jetstream (STJ). Pressure is Nino events due to less monsoonal and cyclonic rainfall.
comparatively lower in south India. The winds start La Nina, on the other hand, is good for monsoons and
blowing from high-pressure area of the north-west to low- cyclogenesis in the Bay of Bengal.
pressure area of the south-east. El Nino is good for cyclogenesis in the Arabian Sea,
Statement 2 is correct. The western cyclonic disturbances whereas, La Nina suppresses cyclogenesis in the Arabian
are brought into India by the westerly jet stream. These Sea.
disturbances cause light rain in the Indus-Ganga plains
and snowfall in the Himalayan belt. 171. Solution: (d)
Exp) Option d is correct
168. Solution: (b)
Statement 1 is correct. During winter season the upper
Exp) Option b is correct air westerly jet streams are positioned in Asia. . The
Statement 1 is incorrect. The Coriolis force is an reversal in upper air circulation takes place in summer
apparent force caused by the earth’s rotation. It is due to the apparent shift of the sun’s vertical rays in the
responsible for deflecting winds towards the right in northern hemisphere. The westerly jet stream is replaced
the northern hemisphere and towards the left in the by the easterly jet stream which owes its origin to the
southern hemisphere. This is also known as ‘Ferrel’s Law’ heating of the Tibet plateau.

Workbook 188
GEOGRAPHY

Statement 2 is correct. The easterly jet stream steers Nadu coast remains dry during this season is that it is
the tropical depressions into India. These depressions situated parallel to the Bay of Bengal branch of southwest
play a significant role in the distribution of monsoon monsoon.
rainfall over the Indian subcontinent. The tracks of these
depressions are the areas of highest rainfall in India. The 175. Solution: (a)
frequency at which these depressions visit India, their Exp) Option a is correct.
direction and intensity, all go a long way in determining
the rainfall pattern during the southwest monsoon period. Statement 1 is correct. During July and August, there
are certain periods when the monsoons become weak.
172. Solution: (d) During the break period, there is a sharp decrease in
Exp) Option d is correct rainfall over most parts of the country but heavy rainfall
Both statements are incorrect. Winter monsoons do occurs over the sub-Himalayan regions and the southern
not cause rainfall as they move from land to the sea. It slopes of the Himalayas.
contributes only 11% to India’s annual rainfall of 1,187
Statement 2 is incorrect. The monsoon break are believed
mm, compared to about 75% in the summer monsoon
season. In the higher reaches of Jammu and Kashmir, to be brought about by the northward shifting of the
Himachal Pradesh, and Uttarakhand, the precipitation monsoon trough (minimum low-pressure cells in ITCZ)
is often in the form of snow. The reason for no rainfall to the foothills of Himalayas.
is because firstly, they have little humidity and secondly, Statement 3 is correct. The breaks can occur due to
due to anti cyclonic circulation on land, the possibility of tropical cyclones which originate in the Bay of Bengal.
rainfall from them reduces. So, most parts of India do not
Over the west coast the dry spells are associated with days
have rainfall in the winter season.
when winds blow parallel to the coast.
173. Solution: (c)
176. Solution: (b)
Exp) Option c is correct.
Following factors are responsible for the formation of the Exp) Option b is correct
south-west monsoons: Statement 1 is incorrect. The months of October and
Statement 1 is correct. Differential heating and cooling November are known for retreating monsoons. This
of land and water creates low pressure on the landmass shift is associated with the occurrence of the most
of India while the seas around experience comparatively severe and devastating tropical cyclones in the Indian
high pressure. seas especially in the Bay of Bengal. The area’s most
Shift of the position of Inter Tropical Convergence Zone vulnerable to these storms include the coastal belts of
(ITCZ) in summer, over the Ganga plain. Tamil Nadu, Andhra Pradesh, Odisha and West Bengal.
Statement 2 is incorrect. Presence of the high-pressure Statement 2 is correct. The months of October-November
area (and not low-pressure), east of Madagascar, form a period of transition from hot rainy season to
approximately at 20°S over the Indian Ocean. The dry winter conditions. The retreat of the monsoon is
intensity and position of this high-pressure area affects
marked by clear skies and rise in temperature. While day
the Indian Monsoon.
temperatures are high, nights are cool and pleasant.
Statement 3 is correct. The Tibetan plateau gets intensely
The land is still moist. Owing to the conditions of high
heated during summer, which results in strong vertical
air currents and the formation of low pressure over the temperature and humidity, the weather becomes rather
plateau at about 9 km above sea level. oppressive during the day. This is commonly known as
Statement 4 is correct. Movement of the westerly jet ‘October heat’.
stream to the north of the Himalayas and the presence of 177. Solution: (a)
the tropical easterly jet stream over the Indian peninsula
during summer. Exp) Option a is correct
Changes in the pressure conditions over the southern Koeppen based his scheme of climatic classification on
oceans also affect the monsoons. monthly values of temperature and precipitation. He
identified five major climatic types, namely: Tropical
174. Solution: (c)
climates, Dry climates, Warm temperate climates, Cool
Exp) Option c is correct temperate climates, Ice climates.
Statement 1 is correct. The Tamil Nadu coast remains
Koppen divided India into nine climatic regions making
relatively dry during the south-west monsoon period.
There are two factors responsible for it: the Tamil Nadu use of the above scheme.
coast lies in the rain shadow area of the Arabian Sea Climatic Regions of India according to Koeppen’s scheme:
branch of the south-west monsoon.
Statement 2 is correct. The second reason why the Tamil Type Qi Climate Areas

189 Workbook
GEOGRAPHY

Amw Monsoon with short West coast of India south a result of decay, it is also the medium for growth.
dry season of Goa Option 1 is correct. The influence of topography is felt
As - Monsoon with dry Coromandel coast of through the amount of exposure of a surface covered by
parent materials to sunlight and the amount of surface
summer Tamil Nadu
and sub-surface drainage over and through the parent
Aw - Tripical savannah Most of the Peninsular materials.
Bwhw - Semi-arid steppe plateaus, south of the
Option 2 is correct: Parent material is a passive control
climate Tropic of Cancer
factor in soil formation. Soil formation depends upon
Bwhw - Hot desert North-western Gujarat, the texture (sizes of debris) and structure (disposition of
some parts of western individual grains/particles of debris) as well as the mineral
Cwg - Monsoon with dry Rajasthan and
winter and chemical composition of the rock debris/deposits.
Punjab Option 3 is correct. Anthropogenic factors play an
Die - Cold humid winter
with short summer Extreme western important role in the formation of soils.
Rajasthan Option 4 is correct. Dead plants provide humus, the finely
E - Polar type
Ganges plain, eastern divided organic matter of the soil. Some organic acids
Rajasthan, northern which form during humification aid in decomposing the
Madhya Pradesh, most of minerals of the soil parent materials.
North-east India Option 5 is correct. The length of time the soil forming
Arunachal Pradesh processes operate, determines maturation of soils and
profile development.
Jammu and Kashmir,
Himachal Pradesh and 181. Solution: (d)
Uttarakhand
Exp) option d is correct.
178. Solution: (a) Practices considered appropriate for soil conservation are:
Exp) option a is correct. Planting of shelterbelts-In the coastal and dry regions,
Statement 1 is correct. Tropical cyclones are violent rows of trees are planted to check the wind movement
storms that originate over oceans in tropical areas and to protect soil cover. These shelter belts have contributed
move over to the coastal areas bringing about large-scale significantly to the stabilisation of sand dunes and in
destruction caused by violent winds, very heavy rainfall stabilising the desert in western India.
and storm surges. Contour bunding-It involves the placement of lines of
Statement 2 is correct. The eye is a region of calm with stones along the natural rises of a landscape. It helps to
subsiding air. Around the eye is the eye wall, where there is capture and hold rainfall before it can become runoff.
a strong spiralling ascent of air to greater height reaching Sand fences-Sand fences are barriers made of small,
the tropopause and results in torrential rain. evenly spaced wooden slats or fabric. They are erected
Statement 3 is incorrect. Tropical cyclones unlike extra- to reduce wind velocity and to trap blowing sand. Sand
tropical cyclones are less common but more violent in fences can be used as perimeter controls around open
nature when compared to extra-tropical cyclones which construction sites to keep sediments from being blown off
are static and not violent in nature. site by the wind.
In terms of temporal distribution, they always occur 182. Solution: (d)
either before or after the monsoon. Exp) Option d is correct
179. Solution: (c) Pair 1 is correctly matched. Parent rock determines
Exp) option c is correct colour, texture, chemical properties mineral,
Statement 1 is correct. Flash floods are the floods which content, permeability.
occur within six hours of the beginning of intense spell of Pair 2 is correctly matched. Relief features, altitude and
rain. It is a highly localized event of short duration. slope determine accumulation of soil.
Statement 2 is correct. They are usually associated with Pair 3 is correctly matched. Flora, fauna and micro-
cloud bursts, storms and cyclones requiring rapid organisms affect the rate of humus formation.
localized warnings and immediate response to reduce
damage 183. Solution: (b)
Exp) Option b is correct
4.5. Soils Statement 1 is correct. Black soil covers most of the
180. Solution: (d) Deccan Plateau which includes parts of Maharashtra,
Madhya Pradesh, Gujarat, Andhra Pradesh and some
Exp) Option d is correct. parts of Tamil Nadu. In the upper reaches of the Godavari
Soil is a dynamic medium in which many chemical, and the Krishna, and the north western part of the Deccan
physical and biological activities go on constantly. Soil is Plateau, the black soil is very deep.

Workbook 190
GEOGRAPHY

Statement 2 is correct. The black soils are generally 187. Solution: (a)
clayey, deep and impermeable. They swell and become Exp) Option a is correct
sticky when wet and shrink when dried.
Statement 1 is correct. Peaty Soils are found in the areas
Statement 3 is incorrect. The black soils are rich in lime, of heavy rainfall and high humidity, where there is a good
iron, magnesia and alumina. They also contain potash. But growth of vegetation. Thus, large quantity of dead organic
they lack in phosphorous, nitrogen and organic matter. matter accumulates in these areas, and this gives a rich
humus and organic content to the soil.
184. Solution: (c)
Statement 2 is incorrect. It occurs widely in the northern
Exp) Option c is correct part of Bihar, southern part of Uttaranchal and the coastal
Statement 1 is correct. Red soil develops on crystalline areas of West Bengal, Orissa and Tamil Nadu (and not on
igneous rocks in areas of low rainfall in the eastern the coast of Goa and Maharashtra).
and southern part of the Deccan Plateau. Along the
piedmont zone of the Western Ghats, long stretch of area 188. Solution: (a)
is occupied by red loamy soil. Yellow and red soils are also Exp) Option a is correct
found in parts of Odisha and Chhattisgarh and in the Pair 1 is incorrectly matched. Laterite soils develop in
southern parts of the middle Ganga plain. areas with high temperature and high rainfall. These are
Statement 2 is correct. The soil develops a reddish the result of intense leaching due to tropical rains. With
colour due to a wide diffusion of iron in crystalline and rain, lime and silica are leached away, and soils rich in iron
metamorphic rocks. It looks yellow when it occurs in a oxide and aluminium compound are left behind. Hence,
hydrated form. laterites are not suitable for cultivation.
Statement 3 is incorrect. The fine-grained red and yellow Pair 2 is correctly matched. Alluvial soils are mostly flat
soils are normally fertile, whereas coarse-grained soils and regular soils and are best suited for agriculture. These
found in dry upland areas are poor in fertility. They are soils are intensively cultivated – yield crops of rice,
generally poor in nitrogen, phosphorous and humus. wheat, sugarcane, tobacco, cotton, jute, maize, oilseeds,
etc.
185. Solution: (b)
Pair 3 is correctly matched. Black soils are best suited
Exp) Option b is correct for cotton crop. Hence these soils are also known as the
Arid soils range from red to brown in colour. They are ‘Regur Soil’ or the ‘Black Cotton Soil’. Other crops grown
generally sandy in structure and saline in nature. Due on these soils include wheat, jowar, linseed, virginia
to the dry climate, high temperature and accelerated tobacco, castor, sunflower and millets.
evaporation, they lack moisture and
189. Solution: (d)
humus. Nitrogen is insufficient and the phosphate content
Exp) Option d is correct.
is normal. Lower horizons of the soil are occupied by
‘kankar’ layers because of the increasing calcium content Contour bunding, Contour terracing, regulated forestry,
downwards. Arid soils are characteristically developed controlled grazing, cover cropping, mixed farming
in western Rajasthan, which exhibit characteristic arid and crop rotation are some of the remedial measures
topography. which are often adopted to reduce soil erosion. In bigger
gullies, the erosive velocity of water may be reduced by
186. Solution: (c) constructing a series of check dams. This can be done by
Exp) Option c is correct gully plugging, terracing or by planting cover vegetation.
Statement 1 is correct. Saline soils occur in arid and 190. Solution: (d)
semi-arid regions, and in waterlogged and swampy Exp) Option d is correct.
areas. Saline soils are more widespread in western
Gujarat, deltas of the eastern coast and in Sunderbans Pair 1 is correctly matched: Alluvial soils are widespread
areas of West Bengal. in the Gangetic plains and the river valleys. They are
depositional soils, transported and deposited by rivers
Statement 2 is correct. They have more salts, largely and streams.
because of dry climate and poor drainage. Seawater
intrusions in the deltas promote the occurrence of saline Pair 2 is correctly matched: Black soil covers most of
soils. In the areas of intensive cultivation with excessive the Deccan Plateau which include parts of Maharashtra,
Madhya Pradesh, Gujarat, Andhra Pradesh and some
use of irrigation, especially in areas of green revolution,
parts of Tamil Nadu.
the fertile alluvial soils are becoming saline.
Pair 3 is incorrectly matched: Red soil develops on
Statement 3 is incorrect. Excessive irrigation with dry
crystalline igneous rocks in areas of low rainfall in the
climatic conditions promotes capillary action, which
eastern and southern parts of the Deccan Plateau.
results in the deposition of salt on the top layer of the
soil. In such areas, especially in Punjab and Haryana, Saline soil is found in the Deltas of the Eastern coast.
farmers are advised to add gypsum (not urea) to solve Pair 4 is correctly matched: Arid soils range from red
the problem of salinity in the soil. to brown in colour. Arid soils are characteristically

191 Workbook
GEOGRAPHY

developed in western Rajasthan, which exhibits Pair 1-C is correctly matched.


characteristic arid topography. These soils are poor and Kolar Leaf Nosed Bat is endemic to India
contain little humus and organic matter.
It is found only in one cave in Hanumanahalli village in
191. Solution: (d) Kolar district, Karnataka.
Exp) option d is the correct answer. It is critically endangered as per IUCN red list.
Statement 1 is correct. The laterite soil develops under Pair 2-A is correctly matched.
tropical and subtropical climate that is in areas with high Caracal is a medium-sized wildcat found in parts of
temperature and high rainfall. Rajasthan and Gujarat.
Statement 2 is correct. Due to tropical rain, laterite soil The National Board for Wildlife and Union Ministry of
suffers intense leaching and lime, silica gets leached away Environment, Forest and Climate Change included the
and soils rich in iron oxide and aluminium compounds caracal in the list of endangered species under the scheme
are left behind. ‘the recovery programme for critically endangered species’.
Statement 3 is incorrect. Laterite soil deficient in organic Pair 3-B is correctly matched.
matter and due to intense leaching- lime and silica is also Seabuckthorn is a shrub which produces an orange-yellow
leached away which makes laterite soil acidic in nature coloured edible berry.
(pH <6).
In India, it is found above the tree line in the Himalayan
region, generally in dry areas such as the cold deserts of
4.6. Natural Vegetation Ladakh and Spiti.
192. Solution: (b) In Himachal Pradesh, it is locally called chharma and
grows in the wild in Lahaul and Spiti and parts of Kinnaur.
Exp) Option b is correct.
Pair 1-C: Nagarahole TR in Karnataka is a home to 195. Solution: (b)
45 tribal settlements locally known as “Haadis” having Exp) Option b is correct.
1703 families with a population of 6579 nos. These are
Statement 1 is incorrect. Fishing cat is a symbolic species
indigenous Dravidian adivasi (Tribals) belonging to
of floodplains, deltas and coastal wetlands of South and
Jenukuruba, Bettakuruba, Yerava, Paniya and soliga
Southeast Asia and not endemic to India.
communities.
Statement 2 is correct. The fishing cat is listed as
Pair 2-A: Amrabad tiger reserve- endangered species of
‘endangered’ on the International Union for Conservation
Mouse deer was re-introduced here for the first time in a
of Nature (IUCN)’s Red List.
country.
Statement 3 is incorrect. Big pisciculture farms
Pair 3-B: Pakhui tiger reserve- Three large cats - the
replacing natural riparian vegetation resulted in the
tiger, leopard and clouded leopard, share space with
habitat loss of fishing cat. As a result, the fishing cat
two canids – the wild dog and Asiatic jackal in this tiger
has disappeared from the East Kolkata Wetlands due
reserve.
to intensive aquaculture. Shrimp farming is another
193. Solution: (d) growing threat to mangrove habitats of the Fishing Cat.
Exp) Option d is correct.
Option a and b are correct. The Wild Life (Protection)
Act (WPA) of 1972 provided for the declaration of
National Parks by the State Government.
The Central Government may also declare, Wild Life
Sanctuary and National Park under certain conditions.
Thus, both State government and Central Government
can both declare National parks.
Option c is correct. National parks enjoy a greater
degree of protection than wildlife sanctuaries. Unlike
wildlife sanctuaries, no human interference in any form
of harvesting of timber, collecting minor forest products 196. Solution: (a)
and private ownership rights is allowed. Exp) Option a is correct
Option d is incorrect. The State of Punjab does not have Statement 1 is incorrect – Bamboo is found almost
any National Park. everywhere in India and is not restricted to peninsular
National Capital Territory of Delhi and Union Territory of regions.
Chandigarh don’t have any National Park. Statement 2 is incorrect – Initially Bamboo was
considered as a tree under the Indian Forest Act, 1927.
194. Solution: (a) However, after amending Section 2(7) of Indian Forest
Exp) Option a is correct. Act, 1927 in 2017, bamboo is no longer a tree in non-

Workbook 192
GEOGRAPHY

forest areas and felled bamboo too is not timber. So, any nearing adulthood gather at wallows or to graze. Rhinos
bamboo grown in private or homestead land by millions don’t often hang out with each other. They are not
of farmers does not require felling permission or transit interactive animals.
permission from any state forest department. Statement b is correct. Breeding occurs throughout the
Statement 3 is correct – Bamboo is also known as Green year, with a gestation period of 15-16 months.
Gold or poor man’s timber. Statement c is incorrect. Females become sexually mature
197. Solution: (d) at 5-7 years old, while males mature at about 10. So sexual
maturity reached early in females as compared to males.
Exp) Option d is correct.
Statement d is incorrect. The highest density of Indian
Statement 1 is incorrect. GSLEP, the Global Snow rhinos in the world is found in Pobitora wildlife
Leopard & Ecosystem Protection Program, is an alliance sanctuary.
of 12 snow leopard range countries. It has participation
of non-governmental organizations, multi-lateral 200. Solution: (a)
institutions, scientists and local communities. It is not Exp) Option a is correct
started by UNEP.
Statement 1 is correct – The Himalayan ranges show a
Statement 2 is incorrect. In India, they are found in a succession of vegetation from the tropical to the tundra,
large part of the western Himalayas including the states which changes with altitude. .
of Jammu and Kashmir, Himachal Pradesh, Uttarakhand
and Sikkim and Arunachal Pradesh in the eastern Statement 2 is incorrect – The southern slopes of the
Himalayas. Himalayas carry a thicker vegetation cover because of
relatively higher precipitation than the drier north-facing
198. Solution: (c) slopes.
Exp) Option c is correct. Statement 3 is correct – Moist Temperate type Forests are
The phenomenon is called ‘blue tide’ and the waves extensively found in Himalayan forests. The forests are
appear blue. It appears when luminescent marine mainly composed of coniferous species.
life makes the sea appear a deep shade of blue. The Statement 4 is incorrect – At many places in this zone,
spectacle occurs when phytoplankton (microscopic temperate grasslands are also found. But in the higher
marine plants), commonly known as dinoflagellates, reaches, there is a transition to Alpine forests and pastures.
produce light through chemical reactions in proteins,
said researchers. Waves disturb these unicellular 201. Solution: (b)
microorganisms and make them release blue light. Exp) Option b is correct.
Dzukou valley is located behind Nagaland’s japfu peak in
North east India. It is an ecologically sensitive area and
home to Dzuko lily, a rare species of lily found only in
the Dzukou valley. It is known by different names like
Angamis call them silent valley. The valley is famous for
its rare flora and fauna like Blyth’s Tragopan (Nagaland’s
State bird), Asian golden cat, Hoolock gibbon etc. As
large-scale fire recurs over the years the greatest loss
is that of endangered and vulnerable plant life, wildlife
and newly discovered species of birds for whom the valley
acts as a crucial habitat.
202. Solution: (a)
Bioluminescence has been an annual occurrence Exp) Option a is correct.
along the west coast since 2016 during November and Option a is correct. Dehing Patkai has been upgraded to
December. National Park from Wildlife Sanctuary recently to protect
Option a is incorrect. Breakwater: It is a wall built out the sanctity of area from coal and oil mining exploration.
into the sea to protect the land from the force of the waves. It is known as Jeypore rainforest having fauna like
Option b is incorrect. Red Tide: It is discoloration of Malayan sun bear and marbled cat. It is also referred as
seawater caused by a bloom of toxic red dinoflagellates. The Amazon of East and is home to Asiatic Elephant.
Option d is incorrect. Tombolo: It is a wave deposition Option b is incorrect. Dibru-Saikhowa is a National Park
landform that connects an island to the mainland. as well as a Biosphere Reserve situated in the south bank
of the river Brahmaputra in the extreme east of Assam
199. Solution: (b) state in India.
Exp) Option b is correct. Option c is incorrect. Nameri National Park is located
Statement a is incorrect. The greater one-horned rhino in the foothills of the Eastern Himalayas in the Sonitpur
is basically solitary except when adult males or rhinos District of Assam. The area is criss-crossed by the river

193 Workbook
GEOGRAPHY

Jia- Bhoroli and its tributaries namely the Diji, Dinai, surviving population of Nilgiri tahr are found in
Doigurung and Nameri. Eravikulam national park.
Option d is incorrect. Orang National park is the oldest Statement b is correct. The Park is of undulating terrain
game reserve of the Assam on the northern bank of river and the highest peak of western ghats Anamudi (2695
Brahmaputra in the Darrang and Sonitpur districts. It is m) is found here.
also known as the mini Kaziranga National Park. Statement c is incorrect. River Kaveri does not pass
203. Solution: (d) through this park. Many tributaries of Periyar and Kaveri
River passes through this park. The Kaveri flows through
Exp) Option d is correct. the states of Karnataka and Tamil Nadu. Eravikulam
Option a is incorrect. Rushikonda beach is widely known National Park is located along the Western Ghats in
for its golden sands and tidy waves of Bay of Bengal. It is Kerala.
located in Visakhapatnam, Andhra Pradesh. It has been Statement d is correct. Three major types of plant
given Blue Flag certification recently. communities found in this park are: Grasslands, Shrub
Option b is incorrect. Radhanagar Beach is one of the Land and Shola Forests. The high plateau and the hills
most famous attractions and is situated at Havelock Island rising above it are primarily covered by Grasslands. Shrub
in Andaman and Nicobar Islands. It has been given Blue Lands are seen along the base of the cliffs. Shola Forests
Flag certification recently. are located in the valleys and folds.
Option c is incorrect. Kappad beach is situated in
Kozhikode, Kerala. It has historical importance also as 207. Solution: (c)
Vasco da Gama first landed on the shores of Kappad. It Exp) Option c is correct.
has been given Blue Flag certification recently. Chilika Lake is situated on the east coast of India, and
Option d is correct. Recently Padubidri beach has connected to the Bay of Bengal through a narrow sea
been accorded ‘Blue Flag’ tag by Foundation for mouth, Chilika is also a lagoon. And like all coastal
Environment Education (FEE) for having grey water lagoons, its waters are rich with life.
treatment plants, solid waste management plants, It is the largest brackish Water Lagoon with estuarine
disabled-friendly equipment to enable them to enter character that sprawls along the east coast. It is the largest
seawater, clean drinking water, bathing facility, disabled- wintering ground for migratory waterfowl found
friendly and general toilets, solar power plant, solar anywhere on the Indian sub-continent.
lighting. It is situated in Karnataka and is famous for
Dakkebali ritual held once in two years. The population of Irrawaddy or Snubfin dolphins
(locally known as “Bhuasuni Magar”) is threatened due
204. Solution: (b) to intensive fishing and plying of mechanized boats in the
Exp) Option b is correct. lake.
Statement 1 is incorrect. The distinction between Arribada are mass nesting beach (rookery) along the
natural landscape and cultural landscape was provided Chilika coast is at Rushikulya which is located at the
by Kasturirangan report on western ghats. southern Odisha coast. It spreads over six km.
Statement 2 is correct. The committee recommended The Nalabana island is part of the Chilika Lake, is home
the constitution of a Western Ghats Ecology Authority to large flocks of migratory birds that make their way to
(WGEA), as a statutory authority under MOEF&CC the lagoon every year during winter. The Island within
under Environmental (protection) Act, 1986. the lagoon is notified as a Bird Sanctuary under Wildlife
(Protection) Act.
205. Solution: (c) The illegal farming dealt a compounding blow to the
Exp) Option c is correct. lake’s ecosystem. The prawn juveniles that were reared
Tropical Dry Evergreen is a unique forest found only grew in size, feeding on most of the phytoplankton,
in the narrow coastal strip of Tamil Nadu and Andhra which are a crucial part of the marine environment.
Pradesh. Most of the rainfall in these types of forests
208. Solution: (d)
occurs in winters due to North-East monsoon winds in
October – December. Mean annual temperature is about Exp) Option d is correct.
28°C. The growth of evergreen forests in areas of such low Western Himalayan agro-climatic region:
rainfall arouses botanical interest. Bamboos are rare and Topography and temperatures show great variation.
grasses not conspicuous. The important species are jamun,
tamarind, neem, etc. Most of the land under these forests Average temperature in July ranges between 5°C and 30
has been cleared for agriculture or casuarina plantations °C, while in January it ranges between 5 °C and -5 °C.
(resembles feathery conifer in general appearance). Mean annual rainfall varies between 75 cm to 150 cm,
with some of the areas experiences a very low rainfall of
206. Solution: (c) 30 cm too.
Exp) Option c is correct. The region is prone to soil erosion and has low land
Statement a is correct. The highest density and largest productivity.

Workbook 194
GEOGRAPHY

This region is famous aromatic crops like Saffron, forest purpose, signing of forest land by the way of lease
medicinal plants (Kuth, Kuroo & Himalayan Yew), or to any private person or any other authority, etc.
Aromatic plants (Dioscorea, Banafsha, Lavender, Brahmi,
Harad, seabuck thorn, cedar wood) and Fruit crops such 212. Solution: (d)
as Apple, cherry, strawberry, peach, plum, kiwi, apricot, Exp) Option d is correct
almond & walnut. Tropical evergreen forests are found in warm and humid
North West Himalayan states of Kashmir and Himachal areas with an annual precipitation of over 200 cm and
contributes the largest share in India’s Apple production. mean annual temperature above 22 degrees Celsius.
Tropical evergreen forests are well stratified, with layers
209. Solution: (c) closer to the ground and are covered with shrubs and
Exp) Option c is correct creepers, with short structured trees followed by tall
Option 1 and 3 are correct whereas option 2 is incorrect. variety of trees. They are found in the western slope of the
Western Ghats, hills of the north-eastern region and the
Tropical Wet Evergreen Forests are found in Assam,
Andaman and Nicobar Islands.
Arunachal Pradesh, Meghalaya, Nagaland, Tripura,
West Bengal and Andaman and Nicobar Island and on 213. Solution: (a)
the eastern and western slopes of the Western Ghats
Exp) Option a is correct
in such states as Tamil Nadu, Karnataka, Kerala and
Maharashtra. They are also found in southern and Statement 1 is correct. Tropical deciduous forests are
western part of Mizoram, while semi-evergreen forests the most widespread forests in India. They are also
occur in its northern, north-western and central part. called the monsoon forests, spread over regions which
receive rainfall between 70-200 cm.
Since Himachal Pradesh lie outside of tropical region it
could easily be eliminated and answer arrived at. Statement 2 is incorrect. The moist deciduous forests
are found in the north-eastern states along the foothills
The vegetation in Himachal Pradesh varies from dry
of Himalayas, eastern slopes of the Western Ghats and
scrub forests at lower altitudes to alpine pastures at
higher altitudes. In between these two extremes, distinct Odisha. Dry deciduous forest covers vast areas of the
vegetation zones of Mixed Deciduous Forests, Bamboo, country - found in rainier areas of the Peninsula and the
Chir Pine, Oak, Deodar, Kail, Fir and Spruce are found. plains of Uttar Pradesh and Bihar.

210. Solution: (a) 214. Solution: (a)


Exp) Option a is correct Exp) Option a is correct
The Indian State Forest Report (ISFR) assesses the forest The Moist deciduous forests are found in the regions
and tree cover, bamboo resources, carbon stock and forest which record rainfall between 100-200 cm. These forests
fires in the country. It is a biennial publication of Forest are found in the north-eastern states along the foothills
Survey of India (FSI), an organization under the Ministry of Himalayas, eastern slopes of the Western Ghats and
of Environment Forest & Climate Change. According to Odisha. Teak, sal, shisham, hurra, mahua, amla, semul,
the ISFR 2019, in terms of forest cover as percentage kusum, and sandalwood etc. are the main species of these
of total geographical area, the top five states Mizoram forests. They are commercially more exploitable as they
(85.41%), Arunachal Pradesh (79.63%), Meghalaya are gregarious implying consisting of pure strands.
(76.33%), Manipur (75.46%) and Nagaland (75.31%), 215. Solution: (d)
which are all from the northeastern region of the
country. Whereas forest cover as percentage of total Exp) Option d is correct
geographical area in the state of Himachal Pradesh and Statement 1 is incorrect. The mountain forests are
Chattisgarh is 27.72% and 41.13% respectively. found not just in Himalayan region but also in south.
The southern mountain forests are found in three
211. Solution: (a) distinct areas of Peninsular India viz; the Western
Exp) Option a is correct. Ghats, the Vindhyas and the Nilgiris. The temperate
Statement 1 is incorrect: Cultivation of tea, coffee, forests are called Sholas in the Nilgiris, Anaimalai and
spices, rubber, mulberry for rearing silkworms, and Palani hills. Such forests are also found in the Satpura and
cash crops are included under non-forestry activities the Maikal ranges.
and are not allowed in reserve forests under the Forest Statement 2 is correct. Tropical thorn forests occur
Conservation Act 1980. in the areas which receive rainfall less than 50 cm.
Statement 2 is correct: Forest Conservation Act, It includes semi-arid areas of south west Punjab,
1980 was enacted by Parliament of India to control the Haryana, Rajasthan, Gujarat, Madhya Pradesh and
deforestation of the forest areas in India. Section 2 of Uttar Pradesh.
this act makes a provision of taking prior approval by Statement 3 is correct. Littoral and swamp forests can
the state government or any other authority from the survive and grow both in fresh as well as brackish water.
central government before issuing any direction for They occur in and around the deltas, estuaries and creeks
de-reservation of the forest, use of forest land for non- prone to tidal influences (delta or tidal forests).

195 Workbook
GEOGRAPHY

216. Solution: (c) of Forest Report 2019, mangrove cover in the country
Exp) Option c is correct has increased by 54 sq km (1.10%) as compared to the
Statement 1 is correct. According to India State of previous assessment.
Forest Report (ISFR), 2019, the total forest cover of 218. Solution: (b)
the country is 7,12,249 sq km which is 21.67% of the
geographical area of the country. The tree cover of the Exp) Option b is correct
country is estimated as 95,027 sq km which is 2.89% of Statements 1, 2 and 3 are correct.
the geographical area.
The forest policy aimed at:
Statement 2 is incorrect. As per ISFR 2019, area wise
• bringing 33 % of the geographical areas under
Madhya Pradesh has the largest forest cover in the
country followed by Arunachal Pradesh, Chhattisgarh, forest cover
Odisha and Maharashtra. • checks soil erosion, extension of the desert lands
Statement 3 is incorrect. Amongst the States, Gujarat and reduction of floods and droughts
has the largest area of wetlands within Recorded Forest • increasing the forest cover through social forestry
Area (RFA) in the country followed by West Bengal. and afforestation on degraded land
The RFA is the area recorded as forest in the Government
records. • creating of a massive peoples movement involving
women to encourage planting of trees, stop felling
217. Solution: (d) of trees and thus, reduce pressure on the existing
Exp) Option d is correct. forest
Option d is correct. Mangroves are diverse group of Statement 4 is incorrect. It is part of India’s INDC
salt-tolerant plant community of tropical and subtropical (Intended Nationally Determined Contribution)
intertidal regions of the world. According to India State promised at Paris Conference (COP 21) in UNFCCC

Workbook 196
GEOGRAPHY

GEOGRAPHY
ECONOMIC AND HUMAN GEOGRAPHY
1. Groundnut
5.1. Previous Years Questions 2. Sesamum
1. Among the following States, which one has 3. Pearl millet
the most suitable climatic conditions for the Which of the above is/are predominantly rainfed
cultivation of a large variety of orchids with crop/crops?
minimum cost of production, and can develop an
(a) 1 and 2 only
export-oriented industry in this field?
(b) 2 and 3 only
[UPSC CSE Pre. 2011]
(c) 3 only
(a) Andhra Pradesh (d) 1, 2 and 3
(b) Arunachal Pradesh
(c) Madhya Pradesh 6. Consider the following crops of India:
(d) Uttar Pradesh [UPSC CSE Pre. 2012]
2. Salinization occurs when the irrigation water 1. Cow pea
accumulated in the soil evaporates, leaving 2. Green gram
behind salts and minerals. What are the effects of 3. Pigeon pea
salinization on the irrigated land? Which of the above is/are used as pulse, fodder and
[UPSC CSE Pre. 2011] green manure?
(a) It greatly increases the crop production (a) 1 and 2 only
(b) It makes some soils impermeable (b) 2 only
(c) It raises the water table (c) 1 and 3 only
(d) It fills the air spaces in the soil with water (d) 1,2 and 3
3. The lower Gangetic plain is characterised by 7. Which of the following is the chief characteristic
humid climate with high temperature throughout of ‘mixed farming’? [UPSC CSE Pre. 2012]
the year. Which one among the following pairs of (a) Cultivation of both cash crops and food crops
crops is most suitable for this region? (b) Cultivation of two or more crops in the same
[UPSC CSE Pre. 2011] field
(a) Paddy and cotton (c) Rearing of animals and cultivation of crops
(b) Wheat and Jute together
(c) Paddy and Jute (d) None of the above.
(d) Wheat and cotton
8. Which one among the following industries is the
4. Despite having large reserves of coal, why does maximum consumer of water in India?
India import millions of tonnes of coal? [UPSC CSE Pre. 2013]
[UPSC CSE Pre. 2012] (a) Engineering
1. It is the policy of India to save its own coal (b) Paper and pulp
reserves for future and import it from other (c) Textiles
countries for the present use. (d) Thermal power
2. Most of the power plants in India are coal based 9. Which of the following is / are the characteristic/
and they are not able to get sufficient supplies of characteristics of Indian coal?
coal from within the country. [UPSC CSE Pre. 2013]
3. Steel companies need large quantity of coking 1. High ash content
coal which has to be imported
2. Low sulphur content
Which of the statements given above is/are correct? 3. Low ash fusion temperature
(a) 1 only
Select the correct answer using the codes given
(b) 2 and 3 only
below.
(c) 1 and 3 only
(d) 1, 2 and 3 (a) 1 and 2 only
(b) 2 only
5. Consider the following crops of India: (c) 1 and3 only
[UPSC CSE Pre. 2012] (d) 1, 2 and 3

197 Workbook
GEOGRAPHY

10. Which of the following statements regarding 3. Kancheepuram


laterite soils of India are correct? 4. Karnal
[UPSC CSE Pre. 2013] Which of the above are famous for the production of
1. They are generally red in colour. traditional sarees/fabric?
2. They are rich in nitrogen and potash. (a) 1 and 2 only
3. They are well-developed in Rajasthan and UP. (b) 2 and 3 only
4. Tapioca and cashew nuts grow well on these (c) 1, 2 and 3 only
soils. (d) 1, 3 and 4 only
Select the correct answer using the codes given 15. Consider the following pairs:
below. [UPSC CSE Pre. 2014]
(a) 1, 2 and 3
National Highway Cities connected
(b) 2, 3 and 4
(c) 1 and 4 1. NH 4 Chennai and
(d) 2 and 3 only Hyderabad
11. Consider the following crops 2. NH 6 Mumbai and Kolkata
[UPSC CSE Pre. 2013] 3. NH 15 Ahmedabad and
1. Cotton Jodhpur
2. Groundnut Which of the above pairs is/are correctly matched?
3. Rice (a) 1 and 2 only
4. Wheat (b) 3 only
Which of these are Kharif crops? (c) 1, 2 and 3
(d) None
(a) 1 and 4 only
(b) 2 and 3 only 16. Consider the following pairs:
(c) 1, 2 and 3 only [UPSC CSE Pre. 2014]
(d) 2, 3 and 4 only
Programme/Project Ministry
12. Consider the following statements:
[UPSC CSE Pre. 2013] 1. Drought-Prone Ministry of
Area Programme Agriculture
1. Natural gas occurs in the Gondwana beds.
2. Mica occurs in abundance in Kodarma. 2. Desert Ministry of
3. Dharwars are famous for petroleum. Development Environment and
Programme Forests
Which of the statements given above is/are correct?
(a) 1 and 2 only 3. National Watershed Ministry of Rural
(b) 2 only Development Development
(c) 2 and 3 only Project for Rainfed
(d) None Areas
Which of the above pairs is/are correctly matched?
13. Consider the following pairs:
(a) 1 and 2 only
[UPSC CSE Pre. 2014] (b) 3 only
(c) 1, 2 and 3
Region Well-known for the
(d) None
production of
1. Kinnaur Areca nut 17. What are the benefits of implementing the
‘Integrated Watershed Development Programme?
2. Mewat Mango [UPSC CSE Pre. 2014]
3. Coromandel Soya bean 1. Prevention of soil runoff
Which of the above pairs is/are correctly matched? 2. Linking the country’s perennial rivers with
seasonal rivers
(a) 1 and 2 only
(b) 3 only 3. Rainwater harvesting and recharge of
(c) 1, 2 and 3 groundwater table
(d) None 4. Regeneration of natural vegetation
Select the correct answer using the code given below:
14. Consider the following towns of India:
(a) 1 and 2 only
[UPSC CSE Pre. 2014] (b) 2, 3 and 4 only
1. Bhadrachalam (c) 1, 3 and 4 only
2. Chanderi (d) 1, 2, 3 and 4

Workbook 198
GEOGRAPHY

18. In India, the steel production industry requires 3. Rajasthan has iron ore mines.
the import of [UPSC CSE Pre. 2015] Which of the statements given above is/are correct?
(a) Saltpetre (a) 1 and 2 only
(b) rock phosphate (b) 2 only
(c) coking coal (c) 1 and 3 only
(d) All of the above (d) 3 only
19. In which of the following activities are Indian
23. With reference to the cultivation of Kharif crops in
Remote Sensing (IRS) satellites used?
[UPSC CSE Pre. 2015] which in the last five years consider the following
statements: [UPSC CSE Pre. 2019]
1. Assessment of crop productivity
1. Area under rice cultivation is the highest.
2. Locating groundwater resources
3. Mineral exploration 2. Area under the cultivation of jowar is more than
that of oilseeds.
4. Telecommunications
3. Area of cotton cultivation is more than that of
5. Traffic studies
sugarcane.
Select the correct answer using the code given below.
4. Area under sugarcane cultivation has steadily
(a) 1, 2 and 3 only decreased.
(b) 4 and 5 only
(c) 1 and 2 only Which of the statements given above is/are correct?
(d) 1, 2, 3, 4 and 5 (a) 1 and 3 only
(b) 2, 3 and 4 only
20. In which of the following regions of India are shale (c) 2 and 4 only
gas resources found? [UPSC CSE Pre. 2016]
(d) 1, 2, 3 and 4
1. Cambay Basin
2. Cauvery Basin 24. With reference to the management of minor
3. Krishna-Godavari Basin minerals in India, consider the following
statements: [UPSC CSE Pre. 2019]
Select the correct answer using the code given below:
1. Sand is a ‘minor mineral’ according to the
(a) 1 and 2 only
prevailing law in the country.
(b) 3 only
(c) 2 and 3 only 2. State Governments have the power to grant
(d) 1, 2 and 3 mining leases of minor minerals, but the powers
regarding the formation of rules related to the
21. With reference to ‘Initiative for Nutritional grant of minor minerals lie with the Central
Security through Intensive Millets Promotion’, Government.
which of the following statements is/are correct? 3. State Government have the power to frame rules
[UPSC CSE Pre. 2016] to prevent illegal mining of minor minerals.
1. This initiative aims to demonstrate the improved
Which of the statements given above is/are correct?
production and post-harvest technologies, and
to demonstrate value addition techniques, in an (a) 1 and 3 only
integrated manner, with cluster approach. (b) 2 and 3 only
2. Poor, small, marginal and tribal farmers have (c) 3 only
larger stake in this scheme. (d) 1, 2 and 3
3. An important objective of the scheme is to 25. Consider the following minerals:
encourage farmers of commercial crops to shift [UPSC CSE Pre. 2020]
to millet cultivation by offering them free kits of 1. Bentonite
critical inputs of nutrients and micro-irrigation
equipment. 2. Chromite
3. Kyanite
Select the correct answer using the code given below.
4. Sillimanite
(a) 1 only
(b) 2 and 3 only In India, which of the above is/are officially
(c) 1 and 2 only designated as major minerals?
(d) 1, 2 and 3 (a) 1 and 2 only
(b) 4 only
22. Consider the following statements:
(c) 1 and 3 only
[UPSC CSE Pre. 2018] (d) 2, 3 and 4 only
1. In India, State Governments do not have the
power to auction non-coal mines. 26. With reference to pulse production in India,
2. Andhra Pradesh and Jharkhand do not have consider the following statements:
gold mines. [UPSC CSE Pre. 2020]

199 Workbook
GEOGRAPHY

1. Black gram can be cultivated as both kharif and 3. Goa has the largest floating solar photovoltaic
rabi crop. project in India.
2. Green-gram alone accounts for nearly half of Which of the statements given above is/are correct?
pulse production. (a) 1 and 2
3. In the last three decades, while the production (b) 2 only
of kharif pulses has increased the production of (c) 1 and 3
rabi pulses has decreased. (d) 3 only
Which of the statements given above is/are correct?
(a) 1 only Practice Questions
(b) 2 and 3 only
(c) 2 only 5.2. World Economic and Human Geography
(d) 1, 2 and 3
31. Which of the following statement is incorrect
27. “The crop is subtropical in nature. A hard frost about uranium pollution in ground water?
is injurious to it. It requires at least 210 frost (a) The primary source of uranium in ground water
-free days and 50 to 100 centimeters of rainfall is geogenic.
for its growth. A light well-drained soil capable (b) Excessive ground water extraction can lead to
of retaining moisture is ideally suited for the uranium enrichment in aquifers.
cultivation of the crop.” (c) World Health Organization has set a limit
Which one of the following is that crop? for safe drinking water at 100 micrograms of
[UPSC CSE Pre. 2020] uranium per liter.
(a) Cotton (d) Uranium contamination causes chronic kidney
(b) Jute diseases.
(c) Sugarcane 32. Which of the following statements is/are correct
(d) Tea with regard to various irrigation techniques?
28. With reference to India, consider the following 1. The productive irrigation provides soil moisture
statements: [UPSC CSE Pre. 2022] to maximum possible area.
1. Monazite is a source of rare earths. 2. The protective irrigation is practiced to obtain
2. Monazite contains thorium. the full potential of high yielding varieties.
3. Monazite occurs naturally in the entire Indian 3. The water input per unit area of cultivated land
coastal sands in India. is higher in productive irrigation than protective
4. In India, Government bodies only can process or irrigation.
export monazite.” Select the correct answer using the code given below:
Which of the statements given above are correct? (a) 1 and 3 only
(a) 1, 2 and 3 only (b) 2 only
(b) 1, 2 and 4 only (c) 3 only
(c) 3 and 4 only (d) 1, 2 and 3
(d) 1, 2, 3 and 4 33. Consider the following about the agro-climatic
29. Consider the following States: conditions required for growth of a crop:
[UPSC CSE Pre. 2022] 1. Annual rainfall of 70-120 cm
1. Andhra Pradesh 2. Cultivated mostly in red laterite soil
2. Kerala 3. Grows well at altitude of 1000-1200 m.
3. Himachal Pradesh 4. Average temperature requirement of 27°C
4. Tripura Which of the following cereal can be grown in the
conditions mentioned above?
How many of the above are generally known as tea-
producing States? (a) Bajra
(b) Maize
(a) Only one State (c) Jowar
(b) Only two States (d) Ragi
(c) Only three States
(d) All four States 34. “The crop requires a temperature of 15 to 40°C;
rainfall of about 50 cm and fertile soil with good
30. Consider the following statements: drainage for cultivation. Frost is harmful for this
[UPSC CSE Pre. 2022] crop and it can’t sustain more than 100 cm of
1. Gujarat has the largest solar park in India. rainfall”. Which one of the following is that crop?
2. Kerala has a fully solar powered International (a) Tea
Airport. (b) Cotton

Workbook 200
GEOGRAPHY

(c) Jute 2. Protection against soil erosion.


(d) Tobacco 3. Practice of aquaculture
35. With reference to ‘Zero Tillage’ consider the Select the correct answer using the code given below?
following statements: (a) 1 and 2 only
1. It would increase the organic matter of soil. (b) 2 and 3 only
2. There would be less surface runoff of water. (c) 1 and 3 only
3. Problem of perennial weeds becomes negligible. (d) 1, 2 and 3
4. There would be decreased loss of moisture from 40. Consider the following statements:
the soil. 1. Inter cropping is growing two or more crops
Select the correct answer using the code given below: simultaneously in definite row patterns.
(a) 1, 2 and 3 only 2. Mixed cropping is combination of crop
(b) 1, 2 and 4 only production and raising of livestock on a
(c) 3 and 4 only particular farm.
(d) 1, 2, 3 and 4 Which of the statements given above is/are correct?
36. With reference to the ‘Monoculture’ consider the (a) 1 only
following statements: (b) 2 only
1. Monoculture allows farms to specialize in a (c) Both 1 and 2
specific crop or livestock. (d) Neither 1 nor 2
2. Monoculture increases the need for water uses. 41. “It is primarily a crop of mid latitude grasslands.
3. Monoculture cropping reduces fertilizer use. It requires moderate rainfall with highest annual
Which of the statements given below are correct? rainfall limit of 100 cm. The ideal climate for
(a) 1 and 3 only this crop has winter temperature 10°C-15°C and
(b) 2 and 3 only summer temperature is 21°C-26°C. A friable
(c) 1 and 2 only loam and clay loams are the best suited for its
(d) 1, 2 and 3 cultivation. It can also be grown in the black soil.”
The crop mentioned here is:
37. With reference to ‘precision farming’ consider the (a) Sugarcane
following statements: (b) Wheat
1. It is an approach where inputs are utilised in (c) Groundnut
precise amounts to get increased average yields (d) Soybean
compared to traditional cultivation techniques.
2. It can easily be applied on small farm holdings. 42. Which of the following crops belongs to the group
3. It uses Information and Communication of fibre crops?
Technologies in farm management. (a) Patsan, gram, Linseed
Which of the statements given above is/are correct? (b) Lentil, oat, Mustard
(c) Cotton, jute, Sunn hemp
(a) 1 and 2 only (d) Jute, Lentil, Oat
(b) 2 and 3 only
(c) 1 and 3 only 43. Which of the following are the factors affecting
(d) 1, 2 and 3 location of Industries?
38. Consider the following statements regarding 1. Raw material
extensive commercial grain cultivation: 2. Access to labour
1. It is practiced in the interior parts of semi-arid 3. Government Policy
lands of the mid-latitudes. 4. Connectivity
2. It has wheat as a principal crop. 5. Links between Industries
3. There is high yield per acre but low yield per Select the correct answer from the codes given
person. below:
4. This is best developed in Eurasian steppes. (a) 1, 2, 4 and 5 only
Which of the statements given above is/are correct? (b) 1, 3, 4 and 5 only
(a) 1, 2 and 4 only (c) 1, 2, 3 and 5 only
(b) 2, 3 and 4 only (d) 1, 2, 3, 4 and 5
(c) 3 only 44. Which of the following is/are resources of geo-
(d) 1, 3 and 4 thermal energy?
39. Which of the following are characteristics of 1. Hydrothermal
‘wetland farming’? 2. Geopressurised brines
1. Excess of soil moisture. 3. Hot dry rocks

201 Workbook
GEOGRAPHY

4. Magma Central India.


Select the correct answer using the code given below: 2. China is the world’s largest Vanadium producing
(a) 1 only country.
(b) 1, 3 and 4 only 3. Vanadium alloys are corrosion-resistant.
(c) 2, 3 and 4 only 4. It increases fuel-efficiency in automotive and
(d) 1, 2, 3 and 4 aviation industries.
45. Which of the following are the indicators used for Which of the statements given above is/are correct?
measuring the Human Development Index? (a) 1, 2 and 3 only
1. Life expectancy at birth (b) 3 and 4 only
2. Expected years of schooling (c) 2, 3 and 4 only
(d) 1, 2, 3 and 4
3. Per capita Gross Domestic Product
4. Child Mortality Rate 50. Consider the following statements:
5. Per capita Gross National Income 1. Human Development Index (HDI) is published
Select the correct answer using the code given below: by the World Economic Forum.
(a) 1, 2 and 4 only 2. Economic Growth is a quantitative term whereas
(b) 2, 3 and 4 only development includes both quantitative and
(c) 1, 2 and 5 only qualitative aspects.
(d) 2, 3 and 5 only 3. The capability approach focuses on building
human capabilities in the areas of health,
46. Consider the following statements: education as the key to increasing human
1. Population density is the ratio between the development.
number of people to the size of land. Which of the statements given above is/are correct?
2. Water, Landforms, climate and soil affect (a) 2 only
population distribution. (b) 2 and 3 only
3. Demographic Transition theory helps in (c) 1 and 3 only
predicting the future population of any area. (d) 1, 2 and 3
Which of the statements given above are correct? 51. Consider the following statements:
(a) 1 and 2 only 1. Cotton plant requires uniformly high
(b) 2 and 3 only temperature during sprouting stage and rainfall
(c) 1 and 3 only at the time of boll opening.
(d) 1, 2 and 3
2. Ratooning process is employed to extract jute
47. The World Population Prospects Report is fibre from its stem.
released by 3. Jute grows well in red soil and laterite soils.
(a) UN Department of Economic and Social Affairs Which of the statements given above is/are Incorrect?
(b) UN Habitat (United Nations Human Settlements (a) 1 only
Programme) (b) 1 and 2 only
(c) United Nations Population Fund (UNFPA) (c) 2 only
(d) World Bank Group (d) 1, 2 and 3
48. The term is applied to an aggregation or 52. Consider the following statements about different
continuous network of urban communities. It is types of farming systems:
a large area of urban development that resulted
from the merging of originally separate towns 1. Subsistence Farming occurs when nearly all of
or cities. It comprises of the main city with their the harvest output is used for sale or trade.
satellite towns or cities. It is also identified by the 2. Mixed farming is the practice of growing of two
absence of rural areas in between main city and or more crops simultaneously in the same field.
satellite towns. 3. Plantation farming is a large scale commercial
Which of the following best describes the above- farming where usually single crop is cultivated.
mentioned type of Urban Settlement? Which of the above statement(s) is/are correct?
(a) Cosmopolitan (a) 1 and 2 only
(b) Megalopolis (b) 2 and 3 only
(c) City (c) 3 only
(d) Conurbation (d) 1, 2 and 3
49. Consider the following statements with reference 53. Consider the following regions:
to Vanadium: 1. High Plains of U.S.A.
1. In India, concentration of Vanadium is found in 2. Foothills of Altai Mountain

Workbook 202
GEOGRAPHY

3. Shores of the Black Sea 58. With reference to the major industrial regions
4. Murray Darling Basin around the world, consider the following pairs:
In which of the above regions would you likely find Major Industry Country/Region
Steppe or temperate grassland Vegetation?
(a) 1 and 4 only 1. Iron and Steel Germany
(b) 2 and 3 only 2. Textiles India
(c) 2, 3 and 4 only
(d) 1, 2, 3 and 4 3. Information California, USA
Technology
54. Which of the following factors are responsible for
Which of the pairs given above is/are correctly
affecting the location of Cotton Textile industries?
matched?
1. Reliable and cheap local investment
(a) 1 and 2 only
2. Development of hydro-electricity (b) 2 only
3. Good relations between industry and trade (c) 2 and 3 only
unions (d) 1, 2 and 3
Select the correct answer using the code given below:
59. Which of the following are the reasons for
(a) 1 and 2 only
Lithium-ion batteries being preferred over other
(b) 2 and 3 only
batteries?
(c) 1 and 3 only
(d) 1, 2 and 3 1. They are not subject to aging even if not in use.
2. They do not require protection circuit to
55. Which of the following activities are included in maintain voltage within safe limits.
quaternary sector of the economy? 3. They have high energy density.
1. Software developers
4. They have lower self-discharge.
2. Government officials
Select the correct answer using the code given below.
3. Tax consultants
(a) 1, 2 and 3 only
4. Lawyers
(b) 3 and 4 only
5. Academic researcher (c) 1, 2 and 4 only
6. Physicians (d) 1, 2, 3 and 4
Select the correct answer using the code given below:
60. With reference to Coal bed methane (CBM),
(a) 1, 2, 5 and 6 only
consider the following statements:
(b) 1, 3 and 5 only
(c) 2, 3, 4 and 5 only 1. It is formed during the natural conversion of
(d) 1, 3, 5 and 6 only plant material into coal.
2. It can be converted into other types of
56. Which of the following factors are responsible for hydrocarbons like diesel and gasoline.
setting up of petroleum refineries? 3. It is a form of natural gas which does not release
1. Access to labour supply Carbon dioxide during combustion.
2. Near to crude oil producing areas 4. The Gondwana sediments host the bulk of India’s
3. Access to large bodies of water current CBM producing blocks.
4. Climate of the region Which of the above statements are correct?
Select the correct answer using the code given below: (a) 1 and 3 only
(a) 1, 2 and 3 only (b) 2, 3 and 4 only
(b) 2 and 4 only (c) 1, 2 and 4 only
(c) 3 and 4 only
(d) 1, 2, 3 and 4
(d) 1, 2, 3 and 4
57. Which of the following is/are the main reason(s) 5.3. Water Resources and Irrigation
for the growth of fishing industry in Japan?
1. Shallow continental shelf along its coast. 61. With respect to ground water utilization in India
2. Presence of Oyashio & Kuroshio Ocean currents which of the following statements is/are incorrect?
3. Around 80 % of the land is not suitable for 1. Major portion of ground water extracted is used
agriculture for industrial purpose.
Select the correct answer using the code given below: 2. Ground water utilization is very high in the state
of Kerala.
(a) 1 and 2 only
(b) 2 and 3 only Select the correct answer using the code given below:
(c) 3 only (a) 1 only
(d) 1, 2 and 3 (b) 2 only

203 Workbook
GEOGRAPHY

(c) Both 1 and 2 2. It aims to transfer surplus water from the Betwa
(d) Neither 1 nor 2 River in Madhya Pradesh to Ken River in Uttar
Pradesh.
62. With respect to tank irrigation in India which of
the following statements is incorrect? 3. It will pass through the Kanha tiger reserve in
Madhya Pradesh.
(a) It has the lowest share of net area under
irrigation. Which of the statements given above is/are correct?
(b) It is developed in regions where the availability (a) 1 and 2 only
of ground water is low. (b) 2 and 3 only
(c) These have a longer life span than canals. (c) 1 only
(d) Due to excessive irrigation, it creates the problem (d) 1, 2 and 3
of Rey or salty crust.
67. Shivasamundram hydropower project is located
63. With respect to traditional rainwater harvesting in:
techniques in India, which of the following pairs (a) Kerala
is/are correctly matched? (b) Tamil Nadu
(c) Andhra Pradesh
Traditional Structure States
(d) Karnataka
1. Zings Mizoram
68. Consider the following pairs:
2. Kere Karnataka
Canal State
3. Phad Rajasthan
1. Jui canal Madhya Pradesh
4. Ahar Pynes Bihar
Select the correct answer using the code given below 2. Wainganga canal Telangana
(a) 1 and 3 only 3. Sirhind Punjab
(b) 2 and 3 only Which of the pairs given above is/are correctly
(c) 2 and 4 only matched?
(d) 1, 3 and 4 only
(a) 1 only
64. Which of the following statements is/are incorrect (b) 2 and 3 only
with regard to the objectives of National Water (c) 3 only
Policy, 2012? (d) 1, 2 and 3
1. It considers drainage basin as a unit for planning
69. With reference to tank irrigation in India, consider
and management of water resources.
the following statements:
2. Inter-basin water transfer is not allowed under
1. Share of tank irrigated area to the total irrigated
this policy.
area in India is around 40%.
3. It follows polluter pays principle.
2. Maharashtra has the largest area with tank
Select the correct answer using the code given below: irrigation in India.
(a) 2 only
Which of the statements given above is/are correct?
(b) 1 and 2 only
(c) 1 only (a) 1 only
(d) 1, 2 and 3 (b) 2 only
(c) Both 1 and 2
65. Which of the following dams/canals are situated (d) Neither 1 nor 2
in the Chambal River?
70. With reference to canal irrigation consider the
1. Gandhi Sagar dam
following statements:
2. Indira Gandhi canal
1. The concentration of canal irrigation is primarily
3. Rana Pratap Sagar dam practiced in Peninsular India.
4. Jawahar Sagar dam
2. Surface of soil under canal irrigation may
Select the correct answer using the code given below: become salty and unfit for agriculture over a
(a) 1, 2 and 3 only period of time.
(b) 1, 2 and 4 only 3. Areas under canal irrigation suffers from
(c) 1, 3 and 4 only extensive leaching.
(d) 2, 3 and 4 only
Which of the statements given above is/are correct?
66. With reference to Ken-Betwa link project consider (a) 1 and 2 only
the following statements: (b) 3 only
1. It is the first project under National Perspective (c) 2 only
Plan for interlinking of rivers. (d) 2 and 3 only

Workbook 204
GEOGRAPHY

71. With reference to well irrigation consider the Which of the statements given above is/are correct?
following statements: (a) 1 only
1. Reht and Dhinghly are methods for lifting water (b) 2 only
from well. (c) Both 1 and 2
2. Uttar Pradesh has the highest area under well (d) Neither 1 nor 2
irrigation. 76. Shompen tribes are the indigenous people of
3. Fishing is carried out in wells in Northern India (a) Nilgiri Hills
at a large scale. (b) Nicobar Islands
Which of the statements given above is/are correct? (c) Spiti Valley
(a) 1 and 2 only (d) Lakshadweep Islands
(b) 2 and 3 only
(c) 1 and 3 only 77. Which of the following statements is/are incorrect
(d) 1, 2 and 3 regarding land use pattern in India?
1. Net sown area represents the total area sown
72. Which of the following statements is/are correct once or more than once in a particular year.
regarding the merits of farm mechanization in 2. Under the calculation of Gross cropped area, the
India? area sowed more than once in the same year is
1. It can provide input savings on seeds, fertilizers counted only once.
and increase the cropping intensity. 3. Cropping intensity is the ratio of net sown area
2. Uncultivable land can be converted to arable to the gross cropped area.
land, thus enhancing agricultural productivity. Select the correct answer using the code given below:
Select the correct answer using the codes given (a) 1 only
below: (b) 3 only
(a) 1 only (c) 1 and 3 only
(b) 2 only (d) 1, 2 and 3
(c) Both 1 and 2
(d) Neither 1 nor 2 78. This is the land available for cultivation but not
used for cultivation for one reason or other for
73. With reference to ‘Hill Area Development more than five years. This land was used in the past
Programme’, consider the following statements: but not in use now due to constraints like salinity
1. This programme was launched under the Tenth water logging etc. The land mentioned here is:
five-year plan. (a) fallow land
2. This programme focuses on sustainable (b) culturable waste land
development of hilly districts of the states of the (c) barren land
North East only. (d) current fallow land
Which of the statements given above is/are correct? 79. With reference to Green Revolution, consider the
(a) 1 only following statements:
(b) 2 only 1. It accelerated the extension of pulses and oilseed
(c) Both 1 and 2 cultivation to non-traditional areas in India.
(d) Neither 1 nor 2 2. It led to deforestation in some parts of India.
74. Which of the following were the objectives of Which of the statements given above is/are correct?
Drought Prone Area Programme introduced by (a) 1 only
the government in 1973-74? (b) 2 only
1. Employment generation. (c) Both 1 and 2
2. Integrated watershed development. (d) Neither 1 nor 2
3. Creation of rural infrastructure like roads and 80. With reference to High yielding variety (HYV) of
market. seeds consider the following statements:
Select the correct answer using the code given below: 1. They provide early maturity period of crops.
(a) 1 and 2 only 2. They provide dwarf stems to resist wind damage.
(b) 2 only 3. They are highly susceptible to insects, pests and
(c) 1 and 3 only diseases.
(d) 1, 2 and 3
4. They provide high responsiveness to fertilizers.
75. Consider the following statements: Which of the statements given above are correct?
1. The term sustainable development was defined (a) 1, 2 and 3 only
by the Brundtland commission of 1987. (b) 1, 2 and 4 only
2. Sustainable growth was one of the themes of (c) 2, 3 and 4 only
eleventh five-year plan in India. (d) 1, 2, 3 and 4

205 Workbook
GEOGRAPHY

81. With reference to Bringing Green Revolution to 85. Consider the following statements:
Eastern India (BGREI) consider the following 1. India is the largest consumer of sugar in the
statements: world.
1. It aims to increase production of rice and wheat 2. The total cost of producing sugar in India is the
by adopting latest technologies. cheapest among its global peers.
2. It aims to promote post-harvest technology and 3. Sugarcane juice cannot be used for production
provide marketing support. of ethanol.
3. It is exclusively implemented in the states of 4. Government of India does not maintain any
North East India. buffer stock for sugar.
Which of the statements given above is/are correct? Which of the following statements given above is/are
(a) 1 and 2 only correct?
(b) 1 only (a) 1 and 2 only
(c) 2 only (b) 1 only
(d) 1, 2 and 3 (c) 2 and 4 only
(d) 3 and 4 only
82. Few of the strategies have been suggested for
second green revolution. One of the strategies 86. In the context of livestock census, consider the
suggested has been explained below: following statements:
“It is related to substantial use of microorganisms 1. It is conducted once in every five years.
and particularly fungi. This concept was originated 2. It covers the livestock population in rural areas
in China in 1986 and is picking up in India lately”. only.
Which strategy is being talked about in the above 3. Total livestock population in the country
paragraph? decreased by 4.6% over the last census.
(a) White agriculture 4. It is conducted by the Ministry of Agriculture
(b) Green agriculture and farmer’s welfare.
(c) Brown agriculture Which of the statements given above are correct?
(d) Red agriculture (a) 1 only
(b) 1 and 2 only
5.4. Major Crops and Cropping Pattern (c) 2 and 3 only
(d) 3 and 4 only
83. Which of the following are the advantages of Zero
Budget Natural Farming (ZBNF): 87. Consider the following statement about a new
1. Eco-friendly hybrid wheat variety “Karan Vandana”:
2. Improves soil fertility 1. It is resistant to yellow rust and wheat blast
3. Less water intensive which causes widespread losses.
4. Reduces input cost 2. It does not require any irrigation.
5. No livestock requirement 3. It has been developed by the Indian Institute of
Wheat and Barley Research (IIWBR).
Select the correct answer from the codes given
below: Which of the statements given above are correct?
(a) 1 and 2only
(a) 2, 3 and 4 only
(b) 1 and 3 only
(b) 1, 2, 3 and 4 only
(c) 2 and 3 only
(c) 1, 3 and 5 only
(d) 1, 2 and 3
(d) 1, 2, 3, 4 and 5
88. Which of the following factors has resulted in
84. Consider the following statements regarding
concentration of cotton industry near Mumbai?
Sugarcane Industry in India:
1. It was very close to the cotton producing areas.
1. It is a footloose industry spread all over India.
2. Development of Mumbai port.
2. Maharashtra is the largest sugarcane producing
state of India. 3. Mumbai’s growth as a financial centre.
3. India is the largest producer of sugar in the Select the correct answer using the code given below.
world. (a) 1 and 2 only
Which of the statements given above is/are incorrect? (b) 1 and 3 only
(c) 2 and 3 only
(a) 1 only
(d) 1, 2 and 3
(b) 1 and 2 only
(c) 2 and 3 only 89. Which of the following statements is/are correct
(d) 1, 2 and 3 regarding the groundnut production in India?

Workbook 206
GEOGRAPHY

1. The kharif season accounts for about 85 percent 93. With reference to the ‘Organic farming’, which of
of its total production in India. the following statements is/are correct?
2. Gujarat is the largest producer of groundnut in 1. In the initial years, the crop yield per area from
India. organic farming is much higher than modern
3. India is the 2nd largest producer of groundnut agriculture.
in the world. 2. Organic farming is often more labour intensive
4. Groundnut is covered under the Price Support than conventional farming.
Scheme (PSS) and Price deficiency payment 3. More options of off-season crops are available in
scheme (PDPS) of PM-AASHA organic farming.
Select the correct answer using the code given below: Select the correct code given below
(a) 2 and 3 only (a) 1 and 3 only
(b) 1 and 3 only (b) 1 and 2 only
(c) 2 and 4 only (c) 2 only
(d) 1, 2, 3 and 4 (d) 2 and 3 only
90. Regarding oilseeds, consider the following 94. Korra, Kodon, Kutki and Hraka are related to
statements: which of the following?
1. Mustard is cultivated as a kharif crop. (a) Variety of Millet crops.
2. Dry lands of Malwa plateau are favourable for (b) Cropping seasons in Southern India.
oilseed cultivation. (c) Water conservation technique in central India.
3. Maharashtra and Madhya Pradesh are the largest (d) Tribal communities in central India.
producer of soyabean in India. 95. “It is a kharif crop which is grown in areas of 40-
Which of the statements given above is/are correct? 50 cm annual rainfall. It is rarely irrigated and
(a) 1 only seldom grown in areas with annual rainfall of
(b) 3 only more than 100 cm. The ideal temperature for it
(c) 2 and 3 only is 25°C -30°C. Bright sunshine after light showers
(d) 1 and 3 only is very useful in the early stages of growth. It can
be grown on poor light sandy soils and upland
91. With reference to role of SFAC (Small farmer gravely soils. It is mainly grown in Rajasthan” In
agri-business consortium), which of the following the above paragraph, which crop of the following
statements is/are correct? is described?
1. SFAC is the implementing agency for e-NAM or (a) Bajra
National Agriculture Market. (b) Cotton
2. It is one of the Central Procurement Agencies (c) Tobacco
for pulses and oilseeds under Price Stabilisation (d) Gram
Fund
3. It has created the Farmer’s Innovation fund to 96. With reference to pulses cultivation in India
propagate the innovations of farmers. consider the following statements:
1. India is the largest producer as well as the
Select the correct answer using the code given below:
consumer of pulses in the world.
(a) 1 only
2. Pulses crops except Arhar can fix nitrogen from
(b) 1 and 2 only
the air.
(c) 2 only
(d) 1 and 3 only Which of the statements given above is/are correct?
(a) 1 only
92. Consider the following events related to (b) 2 only
development of agriculture in India: (c) Both 1 and 2
1. Launch of Accelerated Irrigation benefits (d) Neither 1 nor 2
programme
2. Launch of Command area development 97. With reference to BT cotton in India, consider the
programme. following statements:
3. Setting up of NABARD. 1. It is a transgenic crop designed to combat pink
4. Launch of Svamitva scheme bollworm.
2. Use of insecticides by farmers increased
What is the correct chronological order of the above
drastically after the introduction of BT cotton in
events, starting from the earliest?
India.
(a) 1-2-3-4
3. The Bt cotton seeds cannot be reused.
(b) 2-3-1-4
(c) 3-2-1-4 Which of the statements given above is/are correct?
(d) 2-1-3-4 (a) 1 and 2 only

207 Workbook
GEOGRAPHY

(b) 2 and 3 only (c) 2, 4 and 5 only


(c) 1 only (d) 1, 2, 3, 4 and 5
(d) 1, 2 and 3
102. Consider the following statements regarding
98. Recently U.N. General Assembly have adopted a Agriculture census:
resolution sponsored by India declaring 2023 as 1. It is conducted on quinquennial basis by
the International Year of Millets. Which one of the Ministry of Agriculture.
following statements is incorrect with reference to 2. It gives information on land-use, cropping
Millets in India? pattern and livestock owned by agricultural
(a) India is the largest producer of Millets in the families.
world. 3. Latest census saw an increase in percentage of
(b) Millets are a group of highly variable small- women landholders.
seeded grasses. Which of the statements given above is/are correct?
(c) Millets are very sensitive to climate change. (a) 1 and 2 only
(d) Millets are used as fodder crop for cattle. (b) 2 and 3 only
99. With reference to coffee cultivation in India (c) 1 and 3 only
consider the following statements: (d) 1, 2 and 3
1. It requires cold climate with low rainfall below
100 cm. 5.5. Industry and Mineral and Energy
2. It is generally grown on well drained plains. Resources
3. Laterite soil is ideal for coffee plant. 103. Which of the following are the factors affecting
4. It requires dry weather at the time of ripening location of Industries?
of berries. 1. Raw material
Which of the statements given above is/are incorrect? 2. Access to labour
(a) 1, 2 and 3 only 3. Government Policy
(b) 1, 2 and 4 only 4. Connectivity
(c) 1, 3 and 4 only 5. Links between Industries
(d) 2, 3 and 4 only
Select the correct answer from the codes given
100. Consider the following statements about Pigeon below:
pea cultivation in India: (a) 1, 2, 4 and 5 only
1. Pigeon pea is cultivated over marginal lands in (b) 1, 3, 4 and 5 only
the dry areas in India. (c) 1, 2, 3 and 5 only
2. Punjab contributes more than half of the total (d) 1, 2, 3, 4 and 5
production of Pigeon pea. 104. Consider the following statements:
Which of the following statements given above is/are 1. Bauxite is found mainly in tertiary deposits and
correct? hill ranges of peninsular India.
(a) 1 only 2. Copper deposits are found in Jharkhand and
(b) 2 only Madhya Pradesh region.
(c) Both 1 and 2 3. Mica is produced in the areas of Andhra Pradesh
(d) Neither 1 nor 2 and Tamil Nadu.
101. Consider the following pairs: Which of the following statements given above is/are
correct?
Agricultural Crop/Product (a) 1 and 2 only
Revolution associated (b) 2 and 3 only
1. Silver Revolution Cotton (c) 1 and 3 only
(d) 1, 2 and 3
2. Round Revolution Potato
3. Golden Revolution Jute 105. With reference to the Coal Sector in India in 2020,
consider the following statements:
4. White Revolution Milk 1. India is the world’s largest producer of coal.
5. Grey Revolution Fertilizer 2. India is the 5th largest country in terms of coal
Which of the pairs given above are correctly deposits.
matched? 3. Total quantity of coal import has steadily
(a) 1, 2 and 3 only increased in the past decade.
(b) 1, 3 and 5 only Which of the statements given above is/are correct?

Workbook 208
GEOGRAPHY

(a) 1 and 2 only 110. Consider the following statements:


(b) 2 only 1. Odisha is the largest producer of manganese in
(c) 1 and 3 only India.
(d) 1, 2 and 3 2. Pallakad and Kollam districts in Kerala are rich
106. Consider the following mining regions: in monazite deposits.
1. Kudremukh Which of the statements given above is/are correct?
2. Bellary (a) 1 only
3. Singrauli (b) 2 only
4. Bailadila (c) Both 1 and 2
(d) Neither 1 nor 2
5. Malanjkhand
6. Mayurbhanj 111. With regard to Bauxite, consider the following
Which of the above regions are related to commercial statements:
Iron ore mining? 1. It is mainly used in the manufacturing of
(a) 1, 2, 3 and 5 only aluminum.
(b) 1, 2, 4 and 6 only 2. India has the second largest bauxite reserves in
(c) 2, 3, 4 and 5 only the world.
(d) 1, 4, 5 and 6 only Which of the statements given above is/are correct?
107. Consider the following pairs: (a) 1 only
(b) 2 only
Steel Plants in India Set up in (c) Both 1 and 2
Collaboration with (d) Neither 1 nor 2
1. Rourkela Steel Plant Germany 112. It is the largest dam of India by volume. It is also
2. Bhilai Steel Plant Russia largest artificial lake in India. It is constructed
on one of the tributaries of Ganga. The above
3. Durgapur Steel USA information refers to which dam?
Plant (a) Bhakra Nangal Dam
4. Bokaro Steel Plant United Kingdom (b) Rihand dam
Which of the pairs given above is/are correctly (c) Hirakud dam
matched? (d) Tehri Dam
(a) 1 and 2 only 113. Consider the following statements regarding
(b) 3 and 4 only various resources in India:
(c) 1, 2 and 3 only 1. The state of Uttarakhand has the largest
(d) 1, 2, 3 and 4 unexploited hydropower potential.
108. This belt is rich in ferrous metals and bauxite. 2. The state of Odisha tops the list of India’s coal
It contains high grade iron ore, manganese and reserves and production.
limestone. It lacks in coal deposits except Neyveli 3. The state of Gujarat has the largest wind energy
lignite. potential.
Identify the above mineral belt in India- Which of the statements given above is/are correct?
(a) The North-Eastern Plateau Region (a) 1 and 3 only
(b) The South-Western Plateau Region (b) 2 only
(c) The North-Western Region (c) 3 only
(d) The Himalayan belt (d) 1, 2 and 3
109. With respect to iron ore reserves in India which of 114. With reference to petroleum resource in India,
the following statements is/are correct? consider the following statements:
1. Uttarakhand and Himachal Pradesh have no 1. Crude petroleum occurs in metamorphic rocks
iron reserves. 2. Mumbai High, and Assam are two of the most
2. Odisha is the largest producer of iron ore in productive petroleum deposits in India.
India. 3. India is the third largest consumer of petroleum
3. Cuddapah rock system is rich in iron deposits. products in the world
Select the correct answer using the code given below: Which of the above statements is/are correct?
(a) 1 and 2 only (a) 1 and 2 only
(b) 1 and 3 only (b) 2 only
(c) 2 and 3 only (c) 2 and 3 only
(d) 1, 2 and 3 (d) 1 and 3 only

209 Workbook
GEOGRAPHY

115. Which of the following regions in India have a 1. A minimum population of 10,000.
government declared reserves of Natural Gas? 2. At least 75 per cent of the male main working
1. Tripura population engaged in non-agricultural pursuits.
2. Goa 3. A density of population of at least 500 persons
3. Krishna-Godavari delta per sq. km.
Select the correct answer using the code given below. Select the correct answer using the code given below:
(a) 1 and 2 only (a) 2 only
(b) 2 and 3 only (b) 1 and 2 only
(c) 1 and 3 only (c) 2 and 3 only
(d) 1, 2 and 3 (d) 1, 2 and 3

116. Consider the following statements: 120. Consider the following statements, in the context
1. The Eastern Dedicated Freight Corridor of population distribution in India:
connects Ludhiana in Punjab to Dankuni in 1. Development of irrigation and availability of
West Bengal. mineral and energy resources has resulted in the
increase of population in some regions of India.
2. Dedicated Freight Corridor Corporation of India
is a statutory body that undertakes planning and 2. Deltas and Coastal Plains have higher proportion
development of the Dedicated Freight Corridors. of population than the interior districts in
southern Indian States.
3. The Western Dedicated Freight Corridor is
funded entirely by the World Bank. Which of the statements given above is/are correct?
Which of the statements given above is/are correct? (a) 1 only
(b) 2 only
(a) 1 only (c) Both 1 and 2
(b) 1 and 2 only (d) Neither 1 nor 2
(c) 2 and 3 only
(d) 1 and 3 only 121. Consider the following statements:
1. As per the Census data, the sex ratio in India has
5.6. Population steadily increased since independence.
2. The literacy rate in India was less than 60% per
117. Consider the following statements: cent, as per the Census 2011
1. Statutory Towns are notified under law by the 3. For the purpose of census 2011, a person aged
concerned State Government. seven and above, who can both read and write
2. A town with maximum population of 5000 with understanding in any language, was treated
could be considered as a census town. as literate.
3. Statutory Towns with population of 10,00,000 Which of the statements given above is/are correct?
and above are categorized as cities. (a) 3 only
4. Among states, Himachal Pradesh has the lowest (b) 1 and 2 only
proportion of urban population. (c) 1 and 3 only
Which of the statements given above are correct? (d) 1, 2 and 3
(a) 1 and 3 only 122. With reference to the age-sex pyramid, consider
(b) 1, 2 and 4 only the following statements:
(c) 1 and 4 only 1. The age-sex pyramid which is bell shaped and
(d) 2, 3 and 4 only tapered towards the top indicates a near constant
118. In the context of population pyramid, consider the population.
following statements: 2. The age-sex pyramid with narrow base and a
1. It can be used to compare differences between tapered top indicates low birth and death rates.
male and female population of an area. Which of the statements given above is/are correct?
2. A stationary trend in the graph is represented by (a) 1 only
square or pillar shape rather than a pyramid. (b) 2 only
3. India’s population pyramid is bottom heavy. (c) Both 1 and 2
Which of the statements given below are correct? (d) Neither 1 nor 2
(a) 1 and 3 only 123. With reference to the population growth in India,
(b) 2 and 3 only consider the following statements:
(c) 1 and 2 only 1. India has never seen a negative population
(d) 1, 2 and 3 growth rate in the decadal census since 1901.
119. As per the definition given by Census 2011, which 2. The population growth rate of the country has
of the following is/are the criteria for recognition started slowing down after 1981, as compared to
as a Census Town? the previous census.

Workbook 210
GEOGRAPHY

Which of the statements given above is/are correct? 128. With reference to migration in India, consider the
(a) 1 only following statements:
(b) 2 only 1. The intra-state migration is dominated by female
(c) Both 1 and 2 migrants as compared to male population.
(d) Neither 1 nor 2 2. The rural to urban inter-state migration is
124. According to the 2011 population census, which of dominated by male migrants.
the statements given below is/are incorrect? Which of the statements given above is/are incorrect?
1. No state has registered an increase in population (a) 1 only
growth rate during 2001-11. (b) 2 only
2. Maharashtra has seen the highest percentage fall (c) Both 1 and 2
in the population growth rate when compared (d) Neither 1 nor 2
with the 2001 census.
3. Uttar Pradesh and Bihar have seen an increase in 129. Consider the following states:
the population growth rate as compared to the 1. Uttar Pradesh
previous decade (1991-2001). 2. Gujarat
Select the correct answer using the code given below: 3. Maharashtra
(a) 1 only 4. Kerala
(b) 1 and 3 only With reference to the states mentioned above, in
(c) 3 only terms of net-in-migration from other states, which
(d) 2 and 3 only one of the following is the correct descending order?
125. With reference to the Distribution of Population (a) 3-2-1-4
according to Census of 2011, arrange the following (b) 3-1-2-4
states in increasing order of Population: (c) 2-3-4-1
1. Uttar Pradesh (d) 1-3-2-4
2. Maharashtra
130. With reference to an urban agglomeration, which
3. Bihar of the statements given below is/are correct?
4. West Bengal 1. It must consist of at least one statutory town.
Which of the statements given above is/are correct? 2. Its total population should not be less than
(a) 4-3-2-1 20,000 as per the 2001 Census.
(b) 3-4-2-1
(c) 4-3-1-2 Select the correct answer using the code given below:
(d) 3-4-1-2 (a) 1 only
(b) 2 only
126. Consider the following statements: (c) Both 1 and 2
1. The first complete Census was conducted in (d) Neither 1 nor 2
India 1881.
2. Currently, the Census is conducted by the NITI 131. With reference to the urbanisation in India in
Aayog. recent years, consider the following statements:
3. The census of 2021 will be conducted in all the 1. In India, there has been a consistent growth in
scheduled languages. the urban population in the last few decades.
2. In India, the growth rate of urbanisation has
Which of the statements given above is/are correct?
slowed down during last two decades.
(a) 1 only
(b) 3 only Which of the statements given above is/are correct?
(c) 1 and 3 only (a) 1 only
(d) 1, 2 and 3 (b) 2 only
(c) Both 1 and 2
127. The data collected through the census is used for (d) Neither 1 nor 2
1. Administration
2. Demarcation of Constituencies 132. Consider the following statements:
3. Allocation of representation to the local bodies 1. National Digital Communications Policy aims to
accomplish the objective of ‘Broadband for all’
4. Planning and policy-making
by 2025.
Which of the statements given above is/are correct? 2. The PM-WANI initiative seeks to establish
(a) 1, 2 and 3 only Public Wi-Fi networks to be run and maintained
(b) 2 and 3 only solely by C-DoT.
(c) 1, 3 and 4 only
Which of the statement(s) given above is/are correct?
(d) 1, 2, 3 and 4
(a) 1 only

211 Workbook
GEOGRAPHY

(b) 2 only 2. Most of the world’s uranium reserves are found


(c) Both 1 and 2 in Kazakhstan.
(d) Neither 1 nor 2 3. Both Uranium and Thorium are fissile substance
133. Consider the following statements regarding the and can be used directly in nuclear reactors.
economic reforms of 1990s: Which of the statements given above is/are correct?
1. Balance of payment crisis was the immediate (a) 1 only
reason for introducing these reforms. (b) 2 and 3 only
2. It led to de-reservation of all industries except (c) 1 and 3 only
Railways and Atomic energy. (d) 1, 2 and 3
3. Post 1991 reforms, growth rate of industrial 138. In context of Zero Budget Natural Farming
sectors has continuously increased. (ZBNF), the term ‘Waaphasa’ refers to:
Which of the statements given above is/are correct? (a) Microbial coating of seeds with formulations of
(a) 1 only cow urine and cow dung.
(b) 2 only (b) Gradual reduction of chemical fertilizers by
(c) 1 and 3 only using green manure.
(d) 1, 2 and 3 (c) Building up of soil humus to increase soil
aeration.
5.7. India Human and Economic Geography (d) Adoption of integrated insect and pest
management methods.
134. Which of the following statements is correct about
Manganese in India? 139. Consider the following pairs:
(a) It is found in Dharwar system of rocks.
National Waterway in River/ River System
(b) Jharkhand is the largest producer of manganese. India
(c) The most important manganese ore is Limonite.
(d) It is found as a free element in nature. 1. National Ganga- Teesta–
Waterways-1 Hooghly River system
135. Consider the following pairs:
2. National Brahmaputra
Mineral Mines in India Waterways-2
1. Gold Hutti and Ramagiri 3. National Godavari and Kaveri
Waterways-4 River system
2. Silver Sindeshar Khurd and
Zawwar 4. National Brahmani and Maha-
Waterways-5 nadi delta river system.
3. Diamond Panna
Which of the above pairs is/are correctly matched? Which of the pairs given above are correctly
matched?
(a) 1 and 2 only
(b) 2 only (a) 1 and 2 only
(c) 2 and 3 only (b) 1, 2 and 4 only
(d) 1, 2 and 3 (c) 2 and 4 only
(d) 3 and 4 only
136. Consider the following statements with reference
to rare earth elements in India: 140. Which among the following are principal reasons
1. India imports rare earth metals due to lack of behind the decline in share of railways in total
sufficient reserves in India. freight transport in India?
2. Matsya 6000 is a deep-sea vehicle to be utilized 1. Absence of last mile delivery
for deep sea exploration of rare minerals. 2. Non-competitive tariff structure
3. Rare earth elements are found only in the 3. Low capability to generate operational surplus
western coast of India. 4. No foreign direct investment (FDI) permitted in
Which of the statements given above is/are correct? railway infrastructure sector
(a) 1 and 2 only Select the correct answer using the code given below:
(b) 2 only (a) 1 and 2 only
(c) 2 and 3 only (b) 1, 2 and 3 only
(d) 3 only (c) 3 and 4 only
(d) 1, 2, 3 and 4
137. Consider the following statements regarding
Uranium and Thorium deposits in India: 141. Which of the following is the most appropriate
1. World’s largest thorium reserves are found in reason for India losing out to countries like
India. Bangladesh and Vietnam in textiles sector?

Workbook 212
GEOGRAPHY

(a) High tariffs faced by Indian textile exporters in 145. With reference to urbanization in India and the
foreign markets. world, which of the following statements is/are
(b) No subsidy is provided to the Indian textile correct?
industries to increase employment. 1. The growth rate of urban population in India
(c) Indian domestic policy favors clothing based on is constantly increasing every year since
man-made fibres. independence.
(d) India’s GSP (Generalised system of preferences)
benefits were terminated by US. 2. There has been decline in rural population in
India in the last decade.
142. With reference to migration in India as per Census 3. Among the world capital cities, Delhi has the
2011, consider the following statements: largest population.
1. The rural-to-rural internal migration in India is
Select the correct answer using the code given below:
higher than rural to urban migration.
(a) 1 and 2 only
2. The share of migration from rural to urban areas
is highest among males than females. (b) 2 and 3 only
(c) 1, 2 and 3
3. Tamil Nadu receives the highest number of
(d) None of the above
migrants in India.
4. The largest share of international migrants in 146. With respect to changes in Land-use categories
India is from Nepal. in India between 1951 and 2015, consider the
Which of the statements given above is/are incorrect? following statements:
(a) 1 only 1. Less than 30% of India’s geographical area is
(b) 1and 2 only under cultivation.
(c) 2, 3 and 4 only 2. Area under non-agricultural use has highest rate
(d) 1, 2, 3 and 4 only of increase compared to other categories.
143. With respect to skewed Sex Ratio at birth in India, 3. Pastures and grazing lands have shown
consider the following statements: considerable decline over the said period.
1. According to National Family Health Survey-5, Which of the above-mentioned statements are
Haryana has the lowest sex ratio at birth among correct?
all states. (a) 1 only
2. The Sex ratio at birth is defined as the number (b) 1 and 3 only
of females per thousand males in the age group (c) 2 and 3 only
of 0-6 years. (d) 1, 2 and 3
3. No State or UT in India has Sex Ratio at birth
greater than 1000. 147. Which of the following statement is correct
regarding groundwater utilization in India?
Which of the above statements is/are correct?
(a) All the users of the groundwater have to obtain
(a) 1 only
a No Objection Certificate from Central Ground
(b) 1 and 3 only
(c) 2 and 3 only Water Board (CGWB) at regular interval.
(d) None of the above (b) Every owner of land can collect and dispose
water under the land within his own limits.
144. Consider the following pairs (c) India is the largest exporter of ground water in
the world.
Rural Settlement Commonly found
(d) India has more usable groundwater than surface
Areas
water.
1. Clustered North Eastern States
Settlement 148. Which one of the following is incorrect with
reference to sectoral usage of water resources in
2. Hamleted Chhattisgarh India?
Settlements (a) Agriculture accounts for more than three-fourths
3. Semi-clustered Lower Himalayan of the surface and ground water utilization.
Settlement Valley (b) The share of industrial sector is about 20 per cent
4. Dispersed Alluvial Plains of the surface water utilization.
Settlement (c) The share of domestic sector is higher in surface
water utilization as compared to groundwater.
Which of the above pairs is/are correct? (d) Among the states of India, Uttar Pradesh has
(a) 1 and 2 only highest net annual ground water availability.
(b) 2 and 4 only
(c) 1, 3 and 4 only 149. Consider the following statements with reference
(d) 1, 2 and 4 only to distribution of minerals resources in India:

213 Workbook
GEOGRAPHY

1. Most of the metallic minerals in India occur 4. A downward trend of crude birth rate is held
in the peninsular plateau region in the old responsible for declining population growth
crystalline rocks. post 1981.
2. Most of the coal reserves of India occur in the Which of the above statements is/are correct?
valleys of Narmada. (a) 2 and 4 only
3. Petroleum reserves are located in the sedimentary (b) 1 and 3 only
basins of Assam, Gujarat and Mumbai High. (c) 2, 3 and 4 only
Which of the above statements is/are correct in the (d) 1, 2, 3 and 4
above context? 154. Consider the following statements:
(a) 1 and 2 only 1. The National Education Policy (NEP) 2020 sets
(b) 2 and 3 only the target of 50% gross enrolment ratio (GER) in
(c) 1 and 3 only higher education by 2035.
(d) 1, 2 and 3
2. The literacy rate for Muslim women in India
150. With reference to mines and their major minerals, is higher than women of Scheduled Tribes and
which of the following pairs are correctly matched? Scheduled Caste.
3. As per Census 2011, literacy rate of Scheduled
Mines Minerals they are Tribes was more than that of Scheduled Caste
famous for population.
1. Badampahar Iron Which of the above statements is/are correct in the
2. Lohardaga Bauxite above context?
(a) 2 only
3. Jhunjhunu Copper
(b) 1 and 2 only
Select the correct answer using the code given below: (c) 2 and 3 only
(a) 1 only (d) 1, 2 and 3
(b) 2 and 3 only
(c) 1, 2 and 3 155. With reference to types of Mining and their
(d) None of these features, which of the following pairs is/are
correctly matched?
151. What do you understand by the term ‘Warabandi
system’? Types of Mining feature/description
(a) A land revenue system 1. Open surface Overhead costs such
(b) A rotational water allocation system mining as safety precautions
(c) A system to convert wasteland into cultivated and equipment are
land relatively low
(d) A system to collect Agricultural waste 2. Underground Higher incidents of
152. Which of the following statements are correct mining fatal accidents
regarding footloose industry in India? 3. Rat hole mining Deep vertical shafts
1. These industries are generally less polluting. with narrow horizontal
2. Factors like resources or transport have very tunnels
little effect on costs of these industries. Select the correct answer using the code given below
3. These are highly labour intensive industries. (a) 2 only
4. Cement manufacturing is a foot loose industry. (b) 1 and 3 only
Select the correct answer using the code given below: (c) 3 only
(a) 1 and 2 only (d) 1, 2 and 3
(b) 1, 2 and 3 only 156. Consider the following statements with reference
(c) 2 and 4 only to Fast-moving consumer goods (FMCG) sector in
(d) 1, 3 and 4 only India:
153. Consider the following statements with reference 1. Household and personal care is the major
to population of India: segment of FMCG in India.
1. There was a negative growth rate in India’s 2. The urban segment is the largest contributor
population during 1951-1981. to the overall revenue generated by the FMCG
2. The decades 1921-1951 are referred to as the sector in India.
period of steady population growth. 3. 100% FDI is allowed in multi-brand retail for
3. The decades 1911-1921 are referred to as the FMCG sector.
period of population explosion in India, Which of the above statements is/are correct in the

Workbook 214
GEOGRAPHY

above context? (b) 2 only


(a) 1 and 2 only (c) Both 1 and 2
(b) 2 and 3 only (d) Neither 1 nor 2
(c) 1, 2 and 3 159. Consider the following statements regarding Iron-
(d) 1 and 3 only steel industry in India:
157. Consider the following statements with reference 1. The demand of iron ore for Indian Steel Industry
to Logistics sector in India: is met from domestic supply while the demand
1. The logistics cost in India is considerably lower of coking coal is met mainly through imports.
than that of developed economies. 2. The production cost of steel is relatively lower
2. Road freight accounts for more than half of all than aluminium when used in automotive
the freight transported in India. production.
3. Railways accounts for less than 20 percent of all 3. India is the second largest producer of crude
the freight transported in India. steel in world.
4. The third Logistics Ease Across Different States Which of the statements above is/are correct?
(LEADS) index 2021 is topped by Maharashtra. (a) 1 and 2 only
Which of the above statements is/are incorrect in (b) 2 and 3 only
the above context? (c) 1 and 3 only
(a) 1 and 2 only (d) 1, 2 and 3
(b) 2, 3 and 4 only 160. With reference to the land resources of India,
(c) 1, 2, 3 and 4 consider the following statements:
(d) 1, 3 and 4 only 1. About 30% of India’s surface area is covered by
158. With reference to the population growth in India, hills and mountains.
consider the following statements: 2. India has an estimate of about 25% as a wasteland.
1. India has never seen a negative population 3. The highest incidence of wastelands is recorded
growth rate in the decadal census since 1901. in Rajasthan.
2. The decadal population growth rate of the Which of the statements given above is/are correct?
country has started slowing down after 1981, as (a) 2 and 3 only
compared to the previous census. (b) 1 only
Which of the statements given above is/are correct? (c) 1 and 3 only
(a) 1 only (d) 1, 2 and 3

215 Workbook
GEOGRAPHY

SOLUTIONS

India imports huge quantity of coal out of compulsion


5.1. Previous Years Questions and not as a policy matter.
1. Solution: (b) Statement 2 is correct. Coal meets around 52% of primary
Exp) Option b is correct. commercial energy needs in India and around 66% of
India’s power generation is coal based. The domestic
Orchid is a diverse and widespread family of flowering
production is able to meet only approximately 80% of
plants, with blooms that are often colourful and fragrant.
domestic demand for the rest, coal needs to be imported.
The temperature range suitable for most of those orchids is
65 to 850F. For satisfactory growth of orchids, atmospheric Statement 3 is correct. In India, about 97 percent coal is
humidity should not be less than 30 per cent at night and of Gondwana type and remaining is the Tertiary type. The
70 to 80 per cent during the daytime. It requires cool coal obtained from the Gondwana formations is mainly
and moist climate along with frost free conditions for bituminous and needs to be converted into Coke before
proper flowering. Hence, Arunachal Pradesh has the it can be used in the iron and steel industry. Thus, the
most suitable climatic conditions for the cultivation of a imports compensate for the lack of good quality coal.
large variety of orchids.
5. Solution: (d)
The population of Orchid diminishes as one proceeds
towards poles from tropics. The ecological condition Exp) Option d is correct.
and climate variation in Arunachal Pradesh is so much Option 1 is correct. About 85% of total groundnut in
favourable that the state holds about 60% of the Indian India is sown in kharif season under rainfed conditions.
variety of orchids. Till today about 605 spp.(species) of Option 2 is correct. Sesamum is a draught tolerant short
orchids of Arunachal Pradesh have been identified and duration crop grown predominantly in rainfed regions.
the work is in progress. Another reason for growing sesamum in rainfed region is
Some orchids start flowering from September to that the irrigated sesame crop requires frequent weeding
throughout winter i.e., up to February and some are in operations, whereas rainfed crop needs it only once or twice.
bloom from March to September. So, the orchids in Option 3 is correct. Pearl millet is generally grown as a
Arunachal Pradesh are in bloom throughout the year. kharif crop in rainfed regions of Rajasthan, Maharashtra,
Gujarat. More than 85 % of the total pearl millet in India
2. Solution: (b) is sown under rainfed conditions.
Exp) Option b is correct. Thus, all the above are predominantly rainfed crops,
Salinization leads to intense water logging which means grown in Kharif season and do not require dedicated
filling air spaces in the soil with water, making soils irrigation facilities.
impermeable.
6. Solution: (a)
3. Solution: (c) Exp) Option a is correct.
Exp) Option c is correct. Option 1 is correct. Due to presence of high content
Paddy requires high temperature (above 25°C) and high of protein, Cowpea seed is a nutritious component in
humidity with annual rainfall above 100 cm. Jute needs a the human diet, as well as a nutritious livestock feed.
plain alluvial soil and standing water, these conditions are The stalk and stem of cowpea are harvested and used as
available in the lower Gangetic plain. This is the reason fodder whilst the decaying root residues provide manure
West Bengal tops the country in Jute production. for cultivated fields.
Option a, b and d are incorrect. These conditions are not Option 2 is correct. Greengram is consumed as pulse and
suitable for Cotton and Wheat. Cotton requires rainfall fed to animals as it is an excellent source of high quality
between 50-100 cm, whereas rainfall in lower Gangetic protein (25%) having high digestibility. The husk of the
region is in 100-200 cm, also this region lacks black soil seed can be soaked in water and used as cattle feed.
which is ideal for growing cotton. Green gram is also used as green manure crop for its
Wheat is suited for areas with low humidity and moderate capacity to fix the atmospheric nitrogen (30-40 kg N/
rainfall of 50-75 cm. ha).
Option 3 is incorrect. Pigeon pea is a legume grown in
4. Solution: (b)
semi-arid tropical regions. Young seeds are consumed as a
Exp) Option b is correct. fresh vegetable or can be allowed to mature before drying
Statement 1 is incorrect. The imports are mainly to and eating as a pulse. The leaves and seed husks of the
compensate the lack of good quality coal, especially plant can be used as an animal feed. However, it is not
coking coal from the mining sources in the country. Thus generally preferred as a green manure.

Workbook 216
GEOGRAPHY

7. Solution: (c) 11. Solution: (c)


Exp) Option c is correct. Exp) Option 1, 2 and 3 are correct.
The main characteristic of the mixed farming is that farms The Kharif season differs in every state of the country but
produce both crops and livestock and the two enterprises is generally from June to September.
are interwoven and integrated. These crops are usually sown at the beginning of the
Mixed farming reduces dependence on external sources monsoon season around June and harvested by September
like fertilizers because the crop and animal components or October.
in the farm support each other Rice, maize, bajra, ragi, soybean, groundnut, cotton are
some of the Kharif crops.
8. Solution: (d)
Option 4 is incorrect. Wheat is a rabi crop and requires
Exp) Option d is correct.
a cool growing season and a bright sunshine at the time of
Among industries, thermal power plants are the biggest ripening. It requires 50 to 75 cm of annual rainfall evenly
consumers of water and account for 87.87% of the total distributed over the growing season.
industrial water use, followed by engineering (5.05%)
pulp & paper (2.26%), textiles (2.07%) and Steel industry 12. Solution: (b)
(1.29%). Exp) Option b is correct.
However, irrigation is the largest user of India’s water Statement 1 is incorrect. Gondwana beds are known for
reserve, followed by domestic sector and industrial sector. coal deposits and more recently for shale gas deposits but
not natural gas.
9. Solution: (a)
Statement 2 is correct. Kodarma district in Jharkhand
Exp) Option a is correct. has been famous worldwide for mica mining, especially
Statement 1 is correct. Due to its drift origin, Indian Coal for ruby mica.
has comparatively high ash content (35 – 45%), compared Statement 3 is incorrect. Dharwars located in Karnataka
to 10-20% present in imported coal. Coal seams formed possess minerals such as iron and manganese but does not
due to drift origin contains higher ash as compared to possess any petroleum reserves.
in-situ origin of formation. Higher ash content coal
burn slowly and it is costly to clean. [According to NCERT, the “Dharwar system of rocks”
possess petroleum and uranium reserves. However,
Statement 2 is correct. Indian coal has low sulphur according to UPSC’s official answer key, the answer is “b”,
content(0.5-1%). Low sulphur content means lower SOX thus here we assume that UPSC is asking about Dharwars
emissions. in Karnataka only]
Statement 3 is incorrect. The coal has high ash fusion
temperature (AFT) of about 1,500°C. low-rank coals, such 13. Solution: (d)
as lignite the operating temperature of the gasifier must be Exp) Option d is correct.
lower than the AFT to prevent slagging. For gasification of Pair 1 is incorrectly matched. Kinnaur Valley in
high-rank coals in entrained beds, however, the operating Himachal Pradesh is well known for Apples and
temperature of the gasifier should be greater than the AFT orchards. India is the highest producer of areca nut with
so that the molten ash (slag) could flow down the walls Karnataka and Kerala accounting for nearly 72 per cent of
and drain easily from the gasifier in order to reduce the the total production.
undesirable slag related problems
Pair 2 is incorrectly matched: The major mango-growing
10. Solution: (c) states are Andhra Pradesh, Uttar Pradesh, Karnataka,
Bihar, Gujarat and Tamil Nadu. Whereas, Mewat lies on
Exp) Option c is correct. the border of Haryana and Rajasthan.
Statement 1 is correct. Laterite soil are leached soils Pair 3 is incorrectly matched. Production of soybean
which develops in areas with high temperature and heavy in India is dominated by Maharashtra and Madhya
rainfall. These Soils are acidic in nature and coarse and Pradesh which contribute 89 per cent of the total
crumbly in texture. The high iron oxide content gives it production. Whereas, Coromandel lies in the eastern and
a red-rusty colour. southern part of Tamil Nadu.
Statement 2 is incorrect. Laterite soils lack nitrogen,
potassium and organic elements, which makes these soils 14. Solution: (b)
unsuitable for cultivation. Exp) Option b is correct.
Statement 3 is incorrect. The laterite soils are commonly Option 1 is incorrect. Bhadrachalam in Telangana is a
found in Karnataka, Kerala, Tamil Nadu, Madhya Pradesh hindu pilgrimage site situated in Telangana. A 17th-
and the hilly areas of Odisha and Assam. century Bhadrachalam Temple complex, dedicated to
Statement 4 is correct. With proper irrigation and use Lord Rama is situated here.
of fertilizers make it suitable for growing crops, such Option 2 and 3 are correct. Chanderi Sarees emanating
as cashew, tea, coffee, rubber, cinchona, coconut, and from Chanderi in Madhya Pradesh and Kancheepuram
Tapioca. Sarees produced in Kancheepuram town in Tamil Nadu,

217 Workbook
GEOGRAPHY

are famous varieties of traditional saress. Both these Project (NRLP) formally known as the National
varieties have earned Geographical Indication Tag in Perspective Plan, envisages the transfer of water from
2005. water ‘surplus’ basins to water ‘deficit’ regions. Thus,
Option 4 is correct. Karnal is a town located in Haryana, Inter-Linking the country’s perennial rivers with seasonal
it is known for production of rice and wheat. However, rivers is not a benefit of implementing the ‘Integrated
it is not a place of origin for any traditional saree or fabric. Watershed Development Programme.

15. Solution: (d) 18. Solution: (c)


Exp) Option d is correct. Exp) Option c is correct.
Pair 1 is incorrectly matched. NH-4 links four of the 10 Option a is incorrect. Saltpetre is one of the major
most populous Indian cities - Mumbai, Pune, Bangalore, constituent of gun powder, it is not used in the production
and Chennai. NH 4 is 1,235 km in length and passes of steel. It is mainly used in fertilizers and fireworks.
through the states of Maharashtra, Karnataka and Tamil Option b is incorrect. The reserves of chemical and
Nadu. fertilizer grades rock phosphate in India are very
Pair 2 is incorrectly matched. NH-6 runs over 1,949 km limited. Thus, India imports rock phosphate for the
from Surat to Kolkata fertilizer industry. Only about 10-15% requirement of
Pair 3 is incorrectly matched. NH-15 connects Gujarat raw material for phosphate fertilizer production is met
and Punjab. through indigenous sources. The remaining requirement
is met through import in the form of rock phosphate,
16. Solution: (d) phosphoric acid and direct fertilizers.
Exp) Option d is correct. Option c is correct. Coking coal is a vital ingredient
Pair 1 is incorrectly matched. Drought prone area in the steel making process. The coal found in India is
programme was initiated during the Fourth Five Year mainly of non-coking quality and hence coking coal has
Plan in 1973, with the objectives of providing employment to be imported. 70% of the steel produced today uses coal.
to the people in drought-prone areas and creating 19. Solution: (a)
productive assets. It is an initiative of Ministry of Rural
Development. Exp) Option a is correct.
Pair 2 is incorrectly matched. The basic object of the India’s remote sensing program was developed with the
programme is to minimise the adverse effect of drought idea of applying space technologies for the benefit of
and control desertification through rejuvenation of humankind and the development of the country.
natural resource base of the identified desert areas. It was India’s remote sensing programme under the ISRO
introduced as a centrally sponsored scheme in 1977- 78 started off in 1988 with the IRS-1A, the first of the series
under the Ministry of Rural Development. of indigenous state-of-art operating remote sensing
Pair 3 is incorrectly matched. The scheme of National satellites, which was successfully launched into a polar
Watershed Development Project for Rainfed Areas sun-synchronous orbit on March 17, 1988 from the Soviet
(NWDPRA) was launched in 1990-91. It is under the Cosmodrome at Baikonur.
aegis of Ministry of Agriculture. The main objective of the Applications:
scheme are sustainable management of natural resources, Space Based Inputs for Decentralized Planning (SIS-DP)
enhancement of agricultural production, restoration of
ecological balance, reduction in regional disparity and National Urban Information System (NUIS)
creation of sustained employment opportunities. ISRO Disaster Management Support Programme (ISRO-
DMSP)
17. Solution: (c)
Preharvest crop area and production estimation of major
Exp) Option c is correct. crops.
Statement 1, 3 and 4 are correct.The main aims of this Drought monitoring and assessment based on vegetation
programme are as follows: condition.
To restore the ecological balance by harnessing, conserving Flood risk zone mapping and flood damage assessment.
and developing degraded natural resources such as soil,
vegetative cover and water. Hydro-geomorphological maps for locating underground
water resources for drilling well.
• Prevention of soil run-off, soil loss.
Irrigation command area status monitoring
• Regeneration of natural vegetation.
Snow-melt run-off estimates for planning water use in
• Rain water harvesting and recharging of the ground down stream projects
Water table.
• Land use and land cover mapping
Enabling multi-cropping and the introduction of diverse
agro-based activities, which help to provide sustainable • Urban planning
livelihoods to the people residing in the watershed area. • Forest survey
Statement 2 is incorrect. The National River Linking • Wetland mapping

Workbook 218
GEOGRAPHY

• Environmental impact analysis 22. Solution: (d)


• Mineral Prospecting Exp) Option d is correct.
• Coastal studies Statement 1 is incorrect. As per the new mining law,
Indian Remote Sensing satellite is not used for Mines and Minerals (Development and Regulation)
telecommunications or in traffic studies and for these, Amendment Act, 2015, the non-coal mines have to be
geo-stationary satellite is utilised. auctioned by the respective state governments.
Statement 2 is incorrect. Largest resources in terms of
20. Solution: (d) gold ore (primary) are located in Bihar (44%) followed
Exp) Option d is correct. by Rajasthan (25%), Karnataka (21%),West Bengal (3%),
Oil Shales are usually fine-grained sedimentary rocks Andhra Pradesh (3% ), Jharkhand (2 %). Karnataka was
containing relatively large amounts of organic matter from the leading producer of primary gold accounting for 99%
which significant quantities of shale oil and combustible of the total production. The remaining production was
gas can be extracted by destructive distillation. reported from Jharkhand.
Shale gas can emerge as an important new source of Statement 3 is correct. Hematite and Magnetite are
energy in the country. India has several Shale Formations the most prominent of the iron ores found in India.
which seem to hold shale gas. The Shale Gas Formations Almost all the iron is coming from Hematite reserves,
are spread over several sedimentary basins, such as, Magnetite reserves are not being exploited as they are
Gangetic plain, Gujarat, Rajasthan, Andhra Pradesh and located in fragile zones like Western Ghats. Around 96%
other coastal areas in the country including hydrocarbon- of hematite resources are confined to Odisha, Jharkhand,
bearing ones - Cambay, Assam Arkan & Damodar Basins Chhattisgarh, Karnataka and Goa. The remaining
have large shale deposits. Various developmental activities resources are spread in Maharashtra, Madhya Pradesh,
are going on in Gandhar area of Cambay Basin, Krishna- Rajasthan, Assam etc. Both Hematite and Magnetite
Godavari Basin, Cauvery Basin and Assam & Assam ores occur in Rajasthan.
Arkan Basin.
23. Solution: (a)
ONGC is presently carrying out shale gas and oil
exploration in four onland basins, namely, Cambay, KG, Exp) Option a is correct.
Cauvery, Assam and Assam-Arakan. A total of twelve Statement 1 is correct. In 2017, Area under rice was
exploratory locations for shale gas and oil are available highest at 43.19 million hectares.
for drilling in 2016-17 and subsequent year. In addition, Statement 2 is incorrect. Area under oilseeds was 26.2 million
coring and other data collection are planned in suitable hectares, compared to 5.1 million hectares under jowar.
exploratory wells in identified blocks. ONGC has drilled Statement 3 is correct. India has the largest area under
18 wells so far and one well is under drilling. So far 69 cotton cultivation in the world.
cores have been collected in 17 wells.
Area under cotton cultivation has been consistently high
21. Solution: (c) in past five years. In 2018, 126 lack hectare was under
Exp) Option c is correct. cotton cultivation, compared to 54.35 lakh hectares under
sugarcane cultivation.
Statement 1 is correct. Launched in 2011, Initiative for
Nutritional Security through Intensive Millets Promotion Statement 4 is incorrect. Area under sugarcane cultivation
(INSIMP) is a part of Rashtriya Krishi VikasYojana” was 4.95 million hectares in 2013, 5.066 million hectares
(RKVY). The scheme aims to demonstrate the in 2014-15. This trend does not show a steady decline.
improved production and post-harvest technologies 24. Solution: (a)
in an integrated manner with visible impact to catalyze
increased production of millets in the country. Besides Exp) Option a is correct.
increasing production of millets, the Scheme through Statement 1 is correct. As defined under the Mines
processing and value addition techniques is expected and Minerals (Development and Regulation) Act, 1957
to generate consumer demand for millet based food (MMDR Act) Sand is a minor mineral.
products. Statement 2 is incorrect. The administration of the
Statement 2 is correct. As millets have low productivity mining of minor minerals is a subject that is, completely
in general and are taken by resource poor and tribal within the domain of the respective State Governments.
farmers in remote interior areas, they have larger stake in This act empowers State Government to grant mining
this scheme. leases of minor minerals. Also, it empowers State
Statement 3 is incorrect. A key feature of INSIMP is governments to frame rules for regulating the grant of
giving input kits, comprising urea and pesticides; costing mineral concessions in respect of minor minerals.
Rs 2,000-3,000 depending on the type of crop; and seed Statement 3 is correct. Control of illegal mining is
kits, comprising hybrid seeds to the farmers. However, under the legislative and administrative jurisdiction
there is no provision to provide micro-irrigation of State Governments. Section 23C of the MMDR Act,
equipment. 1957 empowers State Governments to frame rules to

219 Workbook
GEOGRAPHY

prevent illegal mining and the state governments may, by 28. Solution: (b)
notification in the Official Gazette, make such rules for Exp) Option b is the correct answer.
preventing illegal mining, transportation and storage of
minerals (both major minerals and minor minerals). Monazite is a brown crystalline mineral consisting of
cerium, lanthanum, other rare earth elements, and
25. Solution: (d) thorium. Monazite usually occurs in small, isolated
Exp) Option d is correct. crystals or grains that are resistant to weathering and
become concentrated in soils.
In India, the minerals are classified as minor minerals and
major minerals. Statement 1 and 2 are correct.
Major minerals are those specified in the first schedule Monazite is a major source of rare earths and thorium.
appended in the Mines and Minerals (Development It yields a number of rare-earth elements, such as
and Regulation) Act, 1957 (MMDR Act 1957) and the neodymium and praseodymium. Both of these are in
common major minerals are Lignite, Chromite, Kyanite, demand internationally for making high-performance
Sillimanite, Coal, Uranium, iron ore, gold etc. It may be rare-earth magnets (components of power wind turbines,
noted that there is no official definition for “major minerals” electric vehicles and robotics)
in the MMDR Act. Hence, whatever is not declared as a
“minor mineral” may be treated as the major mineral. Statement 3 is incorrect. Though monazite is found in
The list of minerals to be notified as minor minerals most coastal areas of India. It is questionable to be found
include: Agate, Ball Clay, Barytes, Calcareous Sand, along entire coast.
Calcite, Chalk, China Clay, Clay (Others), Diaspore, The main mines are found along the coasts of southern
Dolomite, Felsite, Fireclay, Fuschite Quartzite, Gypsum, India in Kerala, Tamil Nadu and in Orissa.
Jasper, Kaolin, Laterite, Limekankar, Mica, Quartzite,
Statement 4 is correct. Export of monazite without
Sand (Others), Shale, Silica Sand, Slate, and Steatite/Talc/
a license from AERB is a violation of the Atomic
Soapstone, Barytes, Bentonite, Corundum and Sapphire,
Energy (Radiation Protection) Rules 2004. Indian Rare
Dunite & Pyroxinite, Felspar, Fuller’s Earth, Granite
and Selenite, Ball Clay, Other Clays and Shale, Marble, Earths Limited (IREL), a wholly owned Public Sector
Ochre and Other Calcareous Materials, Pyrophyllite, Undertaking of the Government of India (GOI) under
Quartz & Other Silica Minerals, Slate, Sandstone & Other DAE, is the only entity which has been permitted to
Dimension Stones, Talc, Soapstone and Steatite. produce and process monazite, and handle it for domestic
use as well as for export.
26. Solution: (a)
29. Solution: (c)
Exp) Option a is correct.
Exp: Option c is the correct answer.
Statement 1 is correct. Black gram it is cultivated
throughout the country as Kharif crop. Though, It is best As per the Tea Board under the Ministry of Commerce
suited to rice fallows during rabi in southern and south- and Industry, Assam, West Bengal, Tamil Nadu, Himachal
eastern parts of India. Thus, it can be cultivated as both Pradesh, Tripura, Kerela and Karnataka are the states
Kharif and Rabi crop. generally known as tea-producing States. Thus, option c
Statement 2 is incorrect. Green-gram or moong has is correct.
a share of 40% of pulse production in the country.
30. Solution: (b)
Statement 3 is incorrect. In the last three decades the
production of kharif pulses has fluctuated, but overall Exp) Option b is the correct answer.
it has increased. The production of rabi pulses has also Statement 1 is incorrect: Rajasthan tops the list of
increased from 10.32 lakh tonnes in 1992-97 period solar park installations in the country followed by
to 12.29 lakh tonnes in 2012-17 period during the last Karnataka and then Andhra Pradesh. Bhadla Solar Park
three decades. in Rajasthan, with a capacity of 2245 MW, is the world’s
27. Solution: (a) largest solar park.
Exp) Option a is correct. Statement 2 is correct: Cochin International Airport,
India’s first airport built under a public-private-
Cotton is the crop of tropical and sub-tropical areas and
partnership (PPP) model, becomes the first airport in
requires uniformly high temperature varying between
the world that operates completely on solar power. This
21°C and 30°C. The growth of cotton is retarded when the
plant is the first Megawatt scale installation of a Solar PV
temperature falls below 20°C. Frost is enemy number one
system in the State of Kerala
of the cotton plant and it is grown in areas having at least
210 frost free days in a year. The modest requirement of Statement 3 is incorrect: Andhra Pradesh is now home
water can be met by an average annual rainfall of 50- 100 to India’s largest floating solar power plant. State-run
cm. However, it is successfully grown in areas of lesser NTPC started operations at India’s largest floating solar PV
rainfall with the help of irrigation. project at its Simhadri thermal station in Visakhapatnam.

Workbook 220
GEOGRAPHY

34. Solution: (d)


5.2. World Economic and Human Geography Exp) Option d is correct
31. Solution: (c) Agro climatic conditions of Tobacco-
Exp) Option c is correct. 15°C to 40°C temperatures and about 50cm of rainfall
Statement a is correct: Uranium is a radioactive mineral More than 100cm of annual rainfall and frost is harmful
whose primary source is geogenic (naturally occurring). for the crop.
Many of India’s aquifers are composed of clay, silt and
Fertile soils with good drainage
gravel carried down from Himalayan weathering by
streams of uranium-rich granitic rocks. Important Producing Areas: Gujarat, Andhra Pradesh,
Karnataka, Tamil Nadu and Bihar. More than 1/3rd tobac
Statement b is correct: Anthropogenic factors such as of the country is produced by Andhra Pradesh alone.
ground water table decline and nitrate pollution may
further enhance uranium mobilization. When over- 35. Solution: (b)
pumping of these aquifers’ ground water occurs and their Exp) Option b is correct
water levels decline, it induces oxidation conditions
that, in turn, enhance uranium enrichment in the shallow Statement 1 is correct. Organic matter content increases
ground water that remains. due to less mineralization (less use of fertilizers).
Statement c is incorrect: World Health Organisation Statement 2 is correct. Surface runoff is reduced due to
has set a provisional safe drinking water standard of 30 presence of mulch (crop residue).
micrograms of uranium (and not 100 micrograms) per Statement 3 is incorrect. In the zero-tillage farming,
litre for India. the weeds are controlled by the use of herbicides.
Otherwise, the problem of Perennial weeds will rise.
Statement d is correct: Exposure to uranium in
drinking water is linked to chronic kidney diseases. It Statement 4 is correct. Since there no-tilling which
is has become a major issue as groundwater depletion results in slow evaporation, it leads to better absorption
and nitrate pollution may be aggravating the already of rainwater, and better efficiency of irrigation. This would
present natural uranium contamination to dangerous lead to greater yields especially in hot and dry climates.
levels. 36. Solution: (c)
32. Solution: (c) Exp) Option c is correct
Exp) Option c is correct Statement 1 is correct. Monoculture allows the
Statement 1 is incorrect. Productive irrigation is meant organization to specialize in a specific crop or livestock.
to provide sufficient soil moisture in the cropping season This agricultural practice produces larger-than-average
to achieve high productivity. So here the efficiency crop harvest outputs by utilizing fewer resources (like pest
matters not the area. control, for example) than a traditional farm might use.
Whereas, the strategy of Protective irrigation is to Statement 2 is correct. Monocultures are very thirsty
crops and also very inefficient when it comes to water
provide soil moisture to maximum possible area.
utilization. Thus Monoculture increases the need for
Statement 2 is incorrect. If we have to obtain the full water uses.
potential of high yielding variety i.e., to achieve high
Statement 3 is incorrect. In monoculture farmers
productivity from the high yielding crops, the best
focused on high-yielding monoculture crops in which
strategy is to adopt the Productive irrigation and not they need to apply fertilizers in order to get the
the protective irrigation as it only serves the purpose of maximum productivity from their crops. Fertilizer use
protecting crops from dry spell of insufficient moisture. increases. Longer the span of monoculture crops grown
Statement 3 is correct. In Productive irrigation, the on the same land more is the use of fertilizers.
water input per unit area of cultivated land is higher than
protective irrigation. 37. Solution: (c)
Exp) Option c is correct
33. Solution: (d)
Statement 1 is correct. Precision farming is an approach
Exp) Option d is correct
where inputs are utilised in precise amounts to get
increased average yields when compared to traditional
Agro climatic conditions required for growth of Ragi:
cultivation techniques.
It is suited for cultivation in areas with annual rainfall of
Statement 2 is incorrect. One of the challenges in
70-120 cm precision farming is that it cannot easily be applied on
It does not tolerate heavy rainfall and requires a dry spell small farm holdings.
at the time of ripening Statement 3 is correct. It is information and technology-
It is cultivated mostly in red lateritic soil based farm management system that identifies analyses
It grows well in altitude of 1000-1200m with average and manages variability in fields by conducting crop
temperature of 27°C production practices at the right place and time and in

221 Workbook
GEOGRAPHY

the right way, for optimum profitability, sustainability and 41. Solution: (b)
protection of the land resource. Exp) Option b is correct.
38. Solution: (a) Wheat is primarily a crop of mid-latitude grassland
Exp) Option a is correct. and requires a cool climate with moderate rainfall. The
ideal wheat climate has winter temperature 10°C -15°C
Statement 1 is correct. Commercial grain cultivation is and summer temperature varying from 21°C-26°C. The
practiced in the interior parts of semi-arid lands of the temperature should be below at the time of sowing but
mid-latitudes.
as the harvesting time approach higher temperature are
Statement 2 is correct. Wheat is the principal crop, required for proper ripening of the crop.
though other crops like corn, barley, oats and rye are also
grown. Wheat thrives well in areas receiving an annual rainfall of
about 75 cm, annual rainfall of 100 cm is the highest limit
Statement 3 is incorrect. The size of the farm is very of wheat cultivation.
large, therefore the entire operations of cultivation from
ploughing to harvesting are mechanized. There is low Although wheat can be grown in a variety of soils, well
yield per acre but high yield per person. Combine crews drained fertile, friable loams and clay loams are the best
are capable of harvesting grain over many hectares in a suited soils for wheat cultivation. It also grows well in the
single day. black soil of the Deccan plateau.
Statement 4 is correct. This type of agriculture is best 42. Solution: (c)
developed in Eurasian steppes, the Canadian and
American Prairies, the Pampas of Argentina, the Exp) Option c is correct
Velds of South Africa, the Australian Downs and the Cotton, jute and Sunn hemp fall in category of fiber
Canterbury Plains of New Zealand. crop.
Sunn hemp is one of the oldest fiber species grown since
39. Solution: (c)
600 BC. Cotton is most important fiber crop of entire
Exp) Option (c) is the correct answer. world.
On the basis of the main source of moisture for crops, Jute is the second important fiber crop of India. In India
farming can be classified as irrigated and rainfed (barani). cotton, jute, hemp and natural silk are the four major fiber
Rainfed farming is further classified on the basis of crops grown in India. The first three are derived from the
adequacy of soil moisture during cropping season into crops grown in the soil, the latter is obtained from cocoons
dryland and wetland farming. of silkworms fed on green leaves especially mulberry.
Statement 1 is correct. In wetland farming, the rainfall is
in excess of soil moisture requirements of plants during 43. Solution: (d)
the rainy season. Exp) Option d is correct.
Statement 2 is incorrect. Such regions may face flood Location of industries is influenced by several factors like
and soil erosion hazards. These areas grow various water access to raw materials, power, market, capital, transport
intensive crops such as rice, jute and sugarcane. and labour etc.
Statement 3 is correct. It includes practice of aquaculture Statement 1 is correct – Industries using weight-losing
in freshwater bodies. raw materials are located in the regions where raw
40. Solution: (a) materials are located. In iron and steel industries, iron
ore and coal both are weight-losing raw materials.
Exp) Option a is correct
Statement 2 is correct – Labour supply is an important
Statement 1 is correct. Inter-cropping is growing two
factor in the location of industries. Some types of
or more crops simultaneously on the same field in
definite row patterns. A few rows of one crop alternate manufacturing still require skilled labour. Clothing and
with a few rows of a second crop, for example, soyabean apparel industry is a labour intensive industry.
+ maize, or finger millet (bajra) + cowpea (lobia). The Statement 3 is correct – India, being a democratic
crops are selected such that their nutrient requirements country aims at bringing about economic growth with
are different. balanced regional development.At present, government
Statement 2 is incorrect. Mixed cropping is growing of India provides lots of incentives to industries locating
crops involves growing two or more crops concurrently in backward areas. E.g., Special Economic Zones in hilly
on the same land. It involves giving away some insurance states like Himachal Pradesh.
against failure of one of the crops. For example, wheat and Statement 4 is correct – Speedy and efficient transport
gram, wheat and mustard, sunflower and ground nut, etc. facilities to carry raw materials to the factory and to
under mixed farming there is use of animal husbandry move finished goods to the market are essential for the
along with crop production. development of industries. The cost of transport plays
Mixed Farming is a system of farming on a particular an important role in the location of industrial units.
farm which includes crop production, raising of Statement 5 is correct – Links between Industries is an
livestock etc. important factor for location of industries. Refineries,

Workbook 222
GEOGRAPHY

fertilizer plants and power plants form key input to each The World Population Prospects is published by the
other. E.g. Refinery, Fertiliser Plant and Thermal Power Population Division of the UN Department of Economic
Plant in Panipat (Haryana) and Social Affairs. According to its 2019 report, India is
projected to surpass China as the world’s most populous
44. Solution: (d) country by 2027.
Exp) Option d is correct.
48. Solution: (d)
Geothermal energy is the natural heat of the earth. Earth’s
interior heat originated from its fiery consolidation of Exp) Option d is correct.
dust and gas over 4 billion years ago. It is continually The term conurbation was coined by Patrick Geddes in
regenerated by the decay of radioactive elements that 1915 and applied to a large area of urban development
occur in all rocks. that resulted from the merging of originally separate
There are four major types of Geothermal energy towns or cities. Greater London, Manchester, Chicago
resources. and Tokyo are examples.
1. Hydrothermal The conurbations are also identified by distinct absence of
2. Geopressurised brines rural areas in between main city and satellite towns.
3. Hot dry rocks The city of New Delhi itself has transformed substantially
4. Magma into a conurbation since the time of the first Master
Plan for Delhi (1962)- spawning a complex regional
45. Solution: (c) network of towns and cities within the National Capital
Exp) Option c is correct. Region, shedding its earlier unidimensional image as the
The Human Development Index (HDI) was created to administrative capital of the country.
emphasize that people and their capabilities should be 49. Solution: (c)
the ultimate criteria for assessing the development of a
country, not economic growth alone. Exp) Option c is correct.
There are three dimensions to HDI - Statement 1 is incorrect. Promising concentrations
of vanadium in the palaeo-proterozoic carbonaceous
1. The health dimension is assessed by life expectancy
phyllite rocks have been found in the Depo and Tamang
at birth,
areas of Arunachal Pradesh’s Papum Pare district.
2. The education dimension is measured by mean of
years of schooling for adults aged 25 years and more Statement 2 is correct. China was the world’s top
and expected years of schooling for children of vanadium-producing country in 2019 with output of
school entering age. 40,000 MT. China, which produces 57% of the world’s
vanadium, consumed 44% of the metal. The largest
3. The standard of living dimension is measured by
deposits are also in China, followed by Russia and South
Gross National Income per capita.
Africa.
The HDI simplifies and captures only part of what human
development entails. It does not reflect on inequalities, Statement 3 is correct. Vanadium alloys are durable
poverty, human security, empowerment, etc. in extreme temperature and environments, and are
corrosion-resistant. Its addition improves the tensile
46. Solution: (d) strength of steel and of reinforcing bars used for
Exp) Option d is correct. buildings, tunnels and bridges.
Statement 1 is correct – The ratio between the numbers Statement 4 is correct. It increases fuel-efficiency in
of people to the size of the land is the density of automotive and aviation industries due to its high
population. It is usually measured in persons per sq km. strength-to-weight ratio. Vanadium also forms the
For example, the area of Region X is 100 sq km and the integral part of vanadium redox batteries that have the
population is 1,50,000 persons. The density of population least ecological impact in energy storage.
is calculated as: 1,50,000/100 = 1,500 person/sq km 50. Solution: (b)
Statement 2 is correct – Various factors that influence Exp) Option b is correct.
population distribution are –
Statement 1 is Incorrect. HDI is released by the UNDP.
Geographical – Availability of water, Landforms, It is a summary measure of average achievement in key
Climate, Soils. dimensions of human development: a long and healthy
Economic – Minerals, Urbanization, Industrialization. life, being knowledgeable and have a decent standard
Statement 3 is correct – Demographic Transition of living.
theory can be used to describe and predict the future India ranked 131 among 189 countries on the Human
population of any area. Development Index (HDI) for 2019.
47. Solution: (a) Statement 2 is correct.Both growth and development
refer to changes over a period of time. The difference
Exp) Option a is correct
is that growth is quantitative and value neutral. It may

223 Workbook
GEOGRAPHY

have a positive or a negative sign. This means that the on the same piece of land, for example, wheat + gram, or
change may be either positive (showing an increase) or wheat + mustard, or groundnut + sunflower
negative (indicating a decrease). Development means a Statement 3 is correct – Plantation is a type of commercial
qualitative change which is always value positive. This farming. In this type of farming, a single crop is grown on
means that development cannot take place unless there a large area.
is an increment or addition to the existing conditions.
Development occurs when positive growth takes place. 53. Solution: (d)
Statement 3 is correct.Capability Approach is associated Exp) Option d is correct.
with Prof. Amartya Sen. Building human capabilities in Steppes are located in the heart of the continents. Their
the areas of health, education and access to resources is climate is continental with extremes of temperature. With
the key to increasing human development. recorded temperatures as high as 45 °C in summer and in
51. Solution: (d) winter falls to −55 °C. Trees are very scarce in the steppes,
because of the scanty rainfall.
Exp) Option d is correct
There are two main kinds of grasslands: tropical and
Statement 1 is Incorrect – Cotton plant requires bright temperate. Examples of temperate grasslands include
sun-shine, high temperature (21°C to 30°C), light rainfall Eurasian steppes, North American prairies, and Argentine
(50-100 cm) and 210 frost-free days for its growth. High pampas.
amount of rainfall in beginninghelps in sprouting of
seeds. During the flowering and ripening stage, cotton Statement 1 is correct. High Plains of U.S.A. are the
requires sunny and dry weather.Moist weather and regions of prairies or steppes. In the United States
rainfall at the time of boll-opening may damage the Midwest, they’re often called prairies. It is one of the
fibre. It also makes cotton plant vulnerable to pests and major grasslands of the world. It is found in the continent
diseases. of North America where it is distributed between the
United States of America, Canada, and Mexico. It includes
Statement 2 is Incorrect – Retting is the process of the areas of great plains as well as the wet and hilly land
extracting fiber from stem of the plants. The various areas of the east.
ways of Retting are: Mechanical retting (hammering),
chemical retting (boiling & applying chemicals), steam/ Statement 2 is correct. The Altai Mountain Forests,
vapor/dew retting, and water or microbial retting. Grasslands and Desert Steppe bioregion is in the Eastern
ThusRettingprocess is employed to extract jute fibre Eurasia realm. It contains four ecoregions – Altai Montane
from its stem. Forest and Forest Steppe, Altai Steppe and Semi-Desert,
Altai Alpine Meadow and Tundra, Great Lakes Basin
Ratooning or Ratoon cropping is a form of cultivation Desert Steppe.
in which a second crop is allowed to grow from the
remains of one already harvested. Sugarcane, pineapple Statement 3 is correct. The Pontic–Caspian steppe are
and banana are crops in which this method is practiced. formed by the Caspian steppe and the Pontic steppe. It
Ratooning cannot be used endlessly as the yield and is the steppe land stretching from the northern shores
quality decrease after each cycle. of the Black Sea (the Pontus Euxinus of antiquity) to the
northern area around the Caspian Sea.
Statement 3 is Incorrect – Jute grows well on well-
drained fertile soils in the flood plains where soils Statement 4 is correct. In Australia, the temperate
are renewed every year. The grey alluvial soil of good grasslands are better known as Downs and are found in
depth, receiving salt from annual floods, is the best the Murray-Darling Basin of Southern Australia.
for jute. Plenty of standing water is required for growth 54. Solution: (d)
and processing of jute crop. High temperature is required
during the time of growth. Exp) Option d is correct.
The cotton sector in India is considered the second most
52. Solution: (c) developed sector in the textile industry (after man-made
Exp) Option c is correct fibres). At 18% of the global total, India is the world’s
Statement 1 is Incorrect – Subsistence farming is a type largest producer of cotton.
of farming where almost all of the produce is consumed Statement 1 is correct. The availability of reliable and
by the farmer and his family and very little or nothing is cheap financial facilities and services has played an
spared for sale or trade. Commercial farming is a form important part in the gone past in determining the
of farming where nearly all of the harvest output is used location of industries. The Cotton textile mills were set up
for sale or trade at Kanpur based on local investment.
Statement 2 is Incorrect – Mixed farming includes both Statement 2 is correct. The development of hydro-
the cultivation of crops along with rearing of livestock. electricity also favoured the location of the cotton textile
Equal emphasis is laid on crop cultivation and animal mills away from the cotton producing areas. The rapid
husbandry. Animals like cattle, sheep, pigs and poultry development of this industry in Tamil Nadu is the result
provide the main income along with crops. Mixed of the abundant availability of hydel power for the mills.
cropping is growing two or more crops simultaneously As power is the most essential input to run the industries

Workbook 224
GEOGRAPHY

hence, no industry can do without the supply of regular Statement 3 is correct. Refineries must also be near
good-quality energy. large bodies of water or ports, due to the large amount
Statement 3 is correct. Apart from the availability of of water necessary used to process oil, especially in the
cheap low-wage labour, the labour relations is also a purification and cooling processes, and for pipeline and
vital factor for setting up of any industry at any particular cargo deliveries by way of ship. Half of the crude oil that
location and their smooth functioning. The regions, refineries in California receive is imported from Alaska
which are marked by the presence of strong trade unions or foreign countries by way of water, which is why many
refineries in California are located in the Bay Area
and where the governments have been silent spectators
towards the unlawful activities of the trade unions, those Statement 4 is correct. The climate of the region also plays
regions are generally given a low priority for setting up important role in setting up of oil refineries. Plunging
any industry. temperatures can result in many different malfunctions
in a refinery that might require a shutdown, such as
55. Solution: (b) flaring issues, flow control equipment failure, or frozen
Exp) Option b is correct. product within the piping system. Steam and heat are
essential in almost every refinery process. For example,
The quaternary sector or quaternary industry is the crude oil needs to be kept warm or else it will thicken
economic activity based on the intellectual or knowledge- and clog up the piping. When temperatures get too low,
based economy. it may be impossible for the refinery to keep the crude and
Option b is correct. It includes work that conceives, other liquids flowing efficiently.
creates, interprets, organizes, directs and transmits
with the help and support of scientific and technical 57. Solution: (d)
knowledge. For example, mutual fund managers, tax Exp) Option d is correct
consultants, software developers, statisticians, academic Japan is an island nation with a population of 128 million,
researcher, financial analyst, web developer etc. a land area of 377 801 km², a coastline of 29 751 km,
The foundation of this type of activity is creation. It is and an EEZ of approximately 4.05 million km², the sixth
the sector of mind, knowledge, and skill. It creates value largest in the world and about twelve times larger than the
for the society from technology applications or human national land area.
ingenuity. Statements 1, 2 and 3 are correct.
The main difference between the tertiary sector and the A combination of warm (Kuroshio) and cold (Oyashio)
quaternary sector is that, although both provide services, currents flows along the coasts, creating one of the most
the services of quaternary activities are intellectual rather abundant fishing grounds in the world.
than repetitive activities. Teachers, lawyers, physicians,
musicians are the activities included in the Tertiary sector. Other reasons for growth of fishing industry in Japan
include:
Quinary sector involves the highest level of decision
(i) Japan has a large population which provides ready local
makers. These include Highly paid skills of senior
market for fish.
executives, government officials, research scientists,
financial and legal consultants. (ii) Japanese have a long history of sea faring thus are
highly experienced in fishing.
56. Solution: (d) (iii) The shallow continental shelf allows light to the sea
Exp) Option d is correct. bed for growth of micro-organisms which are food for
An oil refinery or petroleum refinery is an industrial fish.
process plant where petroleum (crude oil) is transformed (iv) Japan has developed unique marketing and
and refined into useful products such as gasoline (petrol), distribution system of fish and fishery products to meet
diesel fuel, asphalt base, fuel oils, heating oil, kerosene, domestic demand and earn revenue from fisheries export.
liquefied petroleum gas and petroleum naphtha. (v) Japan is also not endowed with natural resources,
Statement 1 is correct. In petroleum refineries, there is for as much as 80% of the land is not suitable for
a huge demand of skilled labours and other labours for agriculture. Thus, it heavily depends on the sea to
welding, scaffolding, clamping, unplugging of certain survive.
lines, mechanical support for machines and motors and
58. Solution: (d)
lot more. So, access to labour supply is an important factor
responsible for setting up of petroleum refineries. Exp) Option d is correct
Statement 2 is correct. Many oil refineries are located on The world’s major industries are the iron and steel industry,
the oil fields themselves. For refineries based on fields in the textile industry and the information technology
such countries the greatest advantage is the proximity of industry. The iron and steel and textile industry are the
the crude oil. Transport costs are saved and refining can older industries while information technology is an
begin as soon as the oil is brought to the surface. However, emerging industry.
it is not essential that these refineries should be located Pair 1 is correct. The countries in which iron and steel
near oil producing areas. industry is located are Germany, USA, China, Japan and

225 Workbook
GEOGRAPHY

Russia. Other main producers of iron and steel in the One of the advantages of Li-ion batteries is the fact that
world are South Korea, Brazil, Ukraine, India, France, they come in all shapes and sizes, giving the users with
Italy and Great Britain, South Africa, Australia, Austria, many options to choose from according to their needs.
Netherlands, Czech Republic, Romania, Spain, Belgium,
Sweden, etc. Limitations -
Pair 2 is correct. Textile industry is concentrated in India, Requires protection circuit to maintain voltage and
Hong Kong, South Korea, Japan and Taiwan. The leading current within safe limits.
producers of cotton cloth in the world are China, India,
Russia, USA, Japan, Italy, Germany, Hong Kong, Egypt, Subject to aging, even if not in use - storage in a cool
France and Romania. place at 40% charge reduces the aging effect.
Pair 3 is correct. The major hubs of Information Transportation restrictions - shipment of larger
technology industry are the Silicon Valley of Central quantities may be subject to regulatory control. This
California and the Bangalore region of India. Silicon
restriction does not apply to personal carry-on batteries.
Valley is a region in Northern California that serves as a
global center for high technology and innovation. Located Expensive to manufacture - about 40 percent higher in
in the southern part of the San Francisco Bay Area, it cost than nickel-cadmium.
corresponds roughly to the geographical Santa Clara
Valley. San Jose is Silicon Valley’s largest city, Not fully mature - metals and chemicals are changing on
a continuing basis.
59. Solution: (b)
Petrol run vehicles use lead-acid batteries, and not Li-ion
Exp) Option b is correct
batteries, because the latter is sensitive to overheating.
Lithium is one of the lightest elements, and it has the
strongest electrochemical potential of any element. This They also degrade faster in higher temperatures and can
enables a lithium-based battery to pack a lot of energy even explode on overheating.
storage in a small, light battery. As a result, lithium-ion
Dependent on flammable and combustible materials,
batteries have become the battery of choice in many
consumer electronics such as laptops and cell phones. current Li-ion batteries are susceptible to fire and
China is home to majority of the global lithium cell explosion incidents, most of which is without any
manufacturing capacity, followed by the U.S., far behind warning.
in second place. Australia, Chile and China (in that order)
are major producers of Lithium in the world. 60. Solution: (c)
The advantages/disadvantages of Lithium-ion batteries Exp) Option c is correct
over other batteries are:
Coal bed methane (CBM), an important unconventional
Advantages -
gas occurring naturally in coal beds, is Coal bed methane
High energy density - This implies Li-ion batteries can
(CBM) is a form of natural gas which can be recovered
last way longer between charges, while maintaining high
current output; this is because lithium is highly reactive from coal deposits or coal seams (a coal deposit is a
element, and a lot of energy can be stored in its atomic geographical location containing mineable accumulations
bonds. of coal while a coal seam is entrapment of coal in
Do not need prolonged priming when new. One regular underlying rock).
charge is all that’s needed.
Statement 1 is correct. The gas is formed during the
Relatively low self-discharge - Lithium batteries have low natural conversion of plant material into coal, known as
self-discharge, not only whilst being used, but lithium-
ion batteries have a clear advantage when not being coalification. When coalification occurs, the coal becomes
used as well. When not in use, the rate of self-discharge saturated with water and methane gas is trapped within
is extremely low. The leakage of harmful chemicals is a it. CBM can be recovered from coal deposits and seams
common phenomenon in conventional batteries. through drilling and extraction. The composition of CBM
Low Maintenance - no periodic discharge is needed; in a sample of coal varies widely across several locations.
there is no memory. Generally, it consists predominantly of methane gas
Specialty cells can provide very high current to (CH4) although it can contain trace amounts of ethane
applications such as power tools. (C2H6), carbon dioxide (CO2) and water (H2O).
Lithium-ion batteries can serve as a solution for varied
power demands across the spectrum. It can range from Statement 2 is correct. Coal bed methane can be
vast energy storage to portable energy storage solutions. converted into other types of hydrocarbons like diesel and
They can thus be used as large or small batteries. gasoline commonly used as fuel for automobiles.

Workbook 226
GEOGRAPHY

water is used by agriculture sector and remaining for


domestic and industrial purposes.
Statement 2 is incorrect: The groundwater utilization is
very high in the states of Punjab, Haryana, Rajasthan,
and Tamil Nadu. However, the states like Chhattisgarh,
Odisha, Kerala, etc., utilize only a small proportion of
their groundwater potentials.
62. Solution: (d)
Exp) Option d is correct
Statement a is correct: Tank irrigation is restricted only
to hard rock areas of Peninsular India. Due to its low
level of development these account for merely 3% of net
area under irrigation.
Statement 3 is incorrect. CBM is a fossil fuel
which releases CO2, a known greenhouse gas
during combustion. Moreover, its main component,
methane, offers up to 72 times more global warming
potential (GWP) per unit mass than CO2.
Statement 4 is correct. The Gondwana sediments of Statement b is correct: Due to hard rock structure there is
eastern India host the bulk of India’s coal reserves and little percolation of rain water as a result ground water
all the current CBM producing blocks. India has the fifth availability is restricted. So, tanks have been developed
largest proven coal reserves in the world and thus holds in these areas. Also these areas have undulating relief and
significant prospects for exploration and exploitation of hard rock which makes it difficult to dig canal and wells.
CBM. Statement c is correct: Tank irrigation has multiple
advantages over canal and well irrigation as they are
5.3. Water Resources and Irrigation mostly natural and do not require heavy cost for
61. Solution: (c) construction. As they are generally constructed on rocky
bed, they have a longer life span. Against it canals are
Exp) Option c is correct
vulnerable to seepage and leaks.
Statement d is incorrect: Rey is the salty crust which is
formed over the soil due to excessive irrigation in due
to canals (and not tank irrigation). Excessive irrigation
increases ground water levels which then becomes
susceptible to capillary action and bring alkaline salts to
the surface. On the other hand, in tank irrigation water
supply is limited and they dry up during summers when
needed the most.
63. Solution: (c)
Exp) Option c is correct
Pair 1 is incorrectly matched. Zings are water harvesting
structures found in Ladakh. They are small tanks which
collects melted glacier water. A network of guiding
channels is essential to the system that brings the water
from the glacier to the tank.
Pair 2 is correctly matched. These are traditional method
of irrigation in the Central Karnataka Plateau. The
outflow of one tank supplied the next all the way down
the course of the stream; the tanks are built in a series,
usually situated a few kilometers apart.
Pair 3 is incorrectly matched. The community-managed
phad irrigation system, prevalent in north-western
Statement 1 is incorrect: India is the largest extractor of Maharashtra. The system operated on three rivers in the
ground water in the world. This is due to large dependence Tapi basin - Panjhra, Mosam and Aram.
for irrigation on groundwater resources. As per Central Pair 4 is correctly matched. Ahar Pynes is a traditional
Ground Water Board 2006, about 92% of the ground floodwater harvesting system is indigenous to south

227 Workbook
GEOGRAPHY

Bihar. The soil here is sandy and does not retain water. Shivasamundram hydropower project was setup in
Groundwater levels are low. Rivers in this region swell Cauvery River in Karnataka. It is the oldest major power
only during the monsoon, but the water is swiftly carried station in India. It was commissioned in 1902.
away or percolates down into the sand. All these factors
make floodwater harvesting the best option here, to which 68. Solution: (c)
this system is admirably suited. Exp) Option c is correct
Pair 1 is incorrect. Jui canal is situated in Haryana.
64. Solution: (d)
Pair 2 is incorrect. Wainganga canal is situated in
Exp) Option d is correct.
Madhya Pradesh.
Statement 1 is incorrect. National water policy of 2012
Pair 3 is correct. Sirhind canal is situated in Punjab.
considers sub-basin/river basin as a unit for planning,
development and management of water resource. 69. Solution: (d)
Whereas the previous water policy considered drainage
Exp) Option d is correct
basin as a whole as a unit for planning, development and
management of water resources. Statement 1 is incorrect. A tank is a reservoir for
irrigation, a small lake or pool made by damming the
Statement 2 is incorrect. In the previous water policy of
valley of a stream to retain the monsoon rain for later use.
1987 and 2002, inter-basin water transfer was allowed
It accounts for approximately 3% of the net irrigated
after taking into account the requirement of areas and area in India.
basins, but the water policy of 2012 allows inter basin
transfer not merely for increasing production but also Statement 2 is incorrect. Tank Irrigation is popular in
for meeting basic human need and achieving equity the peninsular plateau area where Andhra Pradesh,
and social justice. Telangana and Tamil Nadu are the leading states.
Andhra Pradesh has the largest area (29%) of tank
Statement 3 is incorrect. National Water Policy, 2012 irrigation in India followed by Tamil Nadu (23%).
does not follow polluter pay principle, rather it gives
incentives for effluent treatment. Policies are criticized for 70. Solution: (c)
terming water as an economic good. They do not focus on Exp) Option c is correct
water pollution. So, the government is considering a new
water policy which would follow polluter pays principle. Statement 1 is incorrect. Canal irrigation is almost
absent in peninsular India due to rocky and uneven
65. Solution: (c) terrain. However deltaic areas do have some canals. The
Exp) Option c is correct main concentration of canal irrigation is in the Northern
plains of India especially the areas comprising Uttar
Option 1, 3 and 4 are correct. The Chambal River has Pradesh, Haryana and Punjab.
4 dams situated on it which are Gandhi Sagar dam,
Rana Pratap Sagar dam, Jawahar Sagar Dam and kota Statement 2 is correct. Canal irrigation can lead to soil
barrage. problems like ‘reh’ where surface of soil becomes salty
and unfit for agriculture.
Option 2 is incorrect. The Indira Gandhi Canal is
the longest canal of India which starts from the Harike Statement 3 is incorrect. Canal irrigation is characterized
Barrage at Harike, a few kilometers below the confluence by capillary action (and not leaching) where alkaline
of the Satluj and Beas rivers in Punjab and terminates in salts are brought to the surface.
Thar Desert, Rajasthan. 71. Solution: (a)
66. Solution: (c) Exp) Option a is correct
Exp) Option c is correct Statement 1 is correct. Persian wheel, charas or mot,
dhinghly and reht are some popular methods for lifting
Statement 1 is correct. Ken Betwa Link Project (KBLP),
water from well.
the first project of the National Perspective Plan for
interlinking of rivers. Madhya Pradesh and Uttar Pradesh Statement 2 is correct. Uttar Pradesh has the highest area
signed a memorandum of agreement to implement the under well irrigation (28%) followed by Rajasthan (10%).
Ken Betwa Link Project. Statement 3 is incorrect. Fishing is not carried out in
Statement 2 is incorrect. The Ken-Betwa Link Project wells. Fishing is carried on in tanks which supplements
(KBLP) is the river interlinking project that aims to both the food resources and income of the farmer.
transfer surplus water from the Ken River in MP to
72. Solution: (c)
Betwa in UP to irrigate the drought-prone Bundelkhand
region. Exp) Option c is correct.
Statement 3 is incorrect. It will pass through Panna tiger Statement 1 is correct. Studies have shown a direct
reserve in Madhya Pradesh. relationship between farm mechanization (farm power
availability) and farm yield. Farm mechanization is
67. Solution: (d) said to provide a number of input savings:
Exp) Option d is correct • Seeds (approximately 15-20 percent)

Workbook 228
GEOGRAPHY

• Fertilizers (approximately 15-20 percent) Statement 2 is incorrect: Sustainable growth was one
• Increased cropping intensity (approximately 5-20 of the themes of Twelfth (not eleventh) five-year plan
percent). in India. The duration of this plan was 2012 to 2017.
The main theme this plan was “Faster, More Inclusive
Statement 2 is correct. There are various social
and Sustainable Growth”. Sustainable growth takes
benefits of farm mechanization. It helps in conversion
care of ecological, social and economic aspects of a
of uncultivable land to agricultural land through
development process in a country.
advanced tilling techniques and also in shifting land
used for feed and fodder cultivation by draught animals 76. Solution: (b)
towards food production.
Exp) Option b is correct
73. Solution: (d) The Shompen or Shom Pen are the indigenous people
Exp) Option d is correct. of the interior of Great Nicobar Island, part of the
Statement 1 is incorrect: Hill Area Development Indian union territory of Andaman and Nicobar
Programme (HADP) was a regional development Islands. Geographically, the tribe can be classified into
programme initiated during the Fifth Five Year Plan four different groups depending on their location in the
(1974-79). This programme aimed at harnessing the Great Nicobar Island — namely north-eastern, western,
indigenous resources of the hill areas through development southern and central groups.
of horticulture, plantation, animal husbandry and small- The Shompen are a designated Scheduled Tribe. They
scale industry. are one of the least known particularly vulnerable tribal
Statement 2 is incorrect: Initially, it comprised of all groups (PVTGs) in the Andaman and Nicobar Islands.
hilly districts of Uttar Pradesh, Mikir hill and North They practice a hunter-gatherer subsistence economy.
Cachar hills of Assam, Darjeeling district of West
77. Solution: (d)
Bengal. In 1965, National Development Council (NDC)
recommended the certain areas to be included under Exp) Option d is correct.
the Hill Area Development Programme. Later on, the Statement 1 is incorrect. Net sown area represents the
whole north eastern India was included under this total area sown with crops. But the area sown more than
programme. once in the same year is counted only once.
74. Solution: (d) Statement 2 is incorrect. Gross cropped area represents
the total area sown once and/or more than once in a
Exp) Option d is correct. particular year, i.e., the area is counted as many times as
Introduced in 1973-74, Drought Prone Are Programme there are sowings in a year.
(DPAP) is implemented by the Ministry of Rural
Statement 3 is incorrect. Cropping intensity is the ratio
Development with the overall objective of the programme
of the gross cropped area to net sown area.
is to mitigate the impact of droughts on crop production,
livestock, humans, water resources and productivity of 78. Solution: (b)
land ultimately leading to drought proofing of the target
Exp) Option b is correct
region. This programme was initiated during the fourth
five year plan and it is implementing following measures: Option a is incorrect. Fallow other than Current Fallow
a) Employment generation in drought prone areas is also a cultivable land which is left uncultivated for
more than a year but less than five years. If the land is
b) Focus on labour intensive civil works left uncultivated for more than five years, it would be
c) Irrigation projects categorised as culturable wasteland.
d) Afforestation Option b is correct. Culturable waste land is the land
e) Adoption of Integrated watershed management ap- available for cultivation but not used for cultivation for
proach one reason or other for more than five years. This land
is used in past but not in use now due to constraints
f) Creation of rural infrastructure such as electricity,
like salinity water logging etc. It can be brought under
roads and market.
cultivation after improving it through reclamation
75. Solution: (a) practices.
Exp) Option a is correct. Option c is incorrect. Barren and Wastelands are land
Statement 1 is correct: Brundtland Commission such as barren hilly terrains, desert lands, ravines, etc. and
published its report “Our Common Future” in 1987. It normally cannot be brought under cultivation with the
defined the sustainable development as the development available technology.
which meets the needs of the present without Option d is incorrect. Current Fallow is the land
compromising the ability of future generations to meet which is left without cultivation for one or less than one
their own needs. This commission was established by the agricultural year. Fallowing is a cultural practice adopted
United nation as the world commission on environment for giving the land rest. The land recoups the lost fertility
and development. through natural processes.

229 Workbook
GEOGRAPHY

79. Solution: (b) White agriculture is related to substantial use of


Exp) Option b is correct microorganisms and particularly fungi. The concept of
Statement 1 is incorrect. Green Revolution helped white organism was originated in China in 1986 and is
in diffusion of rice and wheat cultivation to non- picking up in India lately. In this context, white refers to
traditional areas. Rice cultivation was spread over the white-coated scientists and technicians who perform
semi-arid areas of Punjab Haryana and western part of high tech processes to produce food directly from
Uttar Pradesh. Similarly, wheat cultivation was spread to
Rajasthan, Madhya Pradesh and Maharashtra. microorganisms or use them to improve green agriculture.
Statement 2 is correct. Green revolution has left a deep
impact on various ecological aspects in different parts 5.4. Major Crops and Cropping Pattern
of the country. Some of the ecological problems created
by green revolution are soil salinity and alkalinity, water 83. Solution: (b)
logging, desertification, soil erosion and degradation, Exp) Option b is correct.
deforestation, depletion of under groundwater,
environmental pollution and health hazards. It has Option 1 is correct – Zero budget natural farming
increased the deforestation in India as more cultivable (ZBNF) is a method of chemical-free agriculture drawing
land is needed to increase agriculture production. Today from natural methods of farming, which makes it eco-
Punjab and Haryana are almost devoid of forests for this friendly.
particular reason.
Option 2 is correct – ZBNF promotes the application of
80. Solution: (d) jeevamrutha — a mixture of fresh desi cow dung and aged
Exp) Option d is correct desi cow urine, jaggery, pulse flour, water and soil — on
Statement 1 is correct. High yielding variety (HYV) farmland.
of seeds enhances the productivity in agriculture. Seed
Option 3 is correct – ZBNF promotes minimal watering
quality is estimated to account for 20-25% of productivity.
They provide early maturity of crop by shortening the and discourages intensive irrigation.
life cycle of plant. This helps the farmer to practices Option 4 is correct – ZBNF reduces farmers direct cost
multiple cropping patterns. and encourages them to use natural inputs such as cow
Statement 2 is correct. HYV provide dwarf stems to dung and cow urine. Without the need to spend money
resist wind damage and can also carry heavy load of
on these inputs — or take loans to buy them — the cost
crop.
of production is reduced and farming made into a “zero
Statement 3 is correct. Crop based on HYV seed are
highly susceptible to insects, pests and diseases. This budget” exercise, breaking the debt cycle for many small
is due to the fact that new variety needs heavy dose of farmers.
irrigation and fertilizers. These conditions facilitate lead Option 5 is incorrect – Livestock requirement is necessity
to fast growth of pests.
for ZBNF. Application of Jeevamrutha — a mixture of
Statement 4 is correct. HYV seeds gives much higher fresh desi cow dung and aged desi cow urine. This requires
yield as compared to older seed when same amount of
fertilizer is applied. atleast one cow for 30 acres of land with the caveat that it
must be a local Indian breed — not an imported Jersey or
81. Solution: (a) Holstein.
Exp) Option a is correct
Statement 1 is correct. It aims to increase production & 84. Solution: (d)
productivity of rice and wheat by adopting latest crop Exp) Option d is correct.
production technologies.
Statement 1 is incorrect – Sugarcane is a weight-losing
Statement 2 is correct. It will promote post-harvest crop. The ratio of sugar to sugarcane varies between 9 to
technology and marketing support along with creating
water harvesting structures and efficient utilization of 12 percent depending on its variety. Its sucrose content
water potential. begins to dry during haulage after it has been harvested
Statement 3 is incorrect. It is being implemented in from the field. Better recovery of sugar is dependent upon
eastern India comprising seven (7) States namely; its being crushed within 24 hours of its harvesting. Sugar
Assam, Bihar, Chhattisgarh, Jharkhand, Odisha, factories hence are located within the cane-producing
Eastern Uttar Pradesh (Purvanchal) and West Bengal. regions.
82. Solution: (a) Statement 2 is incorrect – Uttar Pradesh is the leading
Exp) Option a is correct Sugarcane producing state, way ahead of Maharashtra.

Workbook 230
GEOGRAPHY

Statement 2 is incorrect. It can be cultivated in areas


with irrigation support which is one of the important
production conditions.
Statement 3 is correct. It has been developed by the Indian
Institute of Wheat and Barley Research (IIWBR) under
the Indian Council of Agricultural Research (ICAR).
88. Solution: (d)
Exp) Option d is correct.
Statement 1 is correct. In 1854, the first modern cotton
mill was established in Mumbai. This city had several
advantages as a cotton textile manufacturing centre. It
was very close to the cotton producing areas of Gujarat
and Maharashtra.
Statement 3 is incorrect – India is the second-largest Statement 2 is correct. Raw cotton used to be brought to
producer of sugar in the world after Brazil. Mumbai port to be transported to England. Therefore,
85. Solution: (b) cotton was available in Mumbai city itself. The machinery
required for a cotton textile mill could be directly
Exp) Option b is correct. imported from England.
Statement 1 is correct. India is the largest consumer of
Statement 3 is correct. Moreover, Mumbai even then
sugar in the world.
was the financial centre and the capital needed to
Statement 2 is incorrect.n Brazil, Thailand and Australia, start an industry was available there. As a large town,
the cane price per ton was USD 25.11, 27.45 and 24.05, providing employment opportunities attracted labour in
respectively, while in India it was USD 42.30 (in 2017–18 large numbers. Hence, cheap and abundant labour too
sugar season). This makes nearly a 65% difference in cost was available locally.
price. As a result, the total cost of producing sugar in
India turns out to be ₹36 per kilo as compared to ₹18.50 89. Solution: (d)
globally.
Exp) Option d is correct.
Statement 3 is incorrect. Government notified the
new National Policy on Biofuels, 2018, under which Statement 1 is correct. There are two peanut growing
sugarcane juice has been allowed for the production of seasons in India: kharif and rabi. The kharif season
ethanol. accounts for about 85 percent of the total. The kharif
crop is typically planted in late June and harvested in
Statement 4 is incorrect. Government of India created late October, and grown primarily in Gujarat, Andhra
buffer stock of 30 LMT in the 2017–18 sugar season. For Pradesh, and Maharashtra. The rabi crop is planted from
2018–19 sugar season, the buffer stock is to be increased
late October through January and harvested in April. Rabi
to 40 LMT at a reimbursement cost of ₹1674 crore.
peanuts are grown in the southern regions of Tamil Nadu,
86. Solution: (a) Karnataka, and Telangana.
Exp) Option a is correct. Statement 2 is correct. Gujarat is the largest producing
Statement 1 is correct – The Livestock Census has been state accounting for 40% of total groundnut produced
conducted in the country periodically since 1919-20. in the country. Other groundnut producing states are
Since then, it has been conducted once every 5 years. Rajasthan, Andhra Pradesh, Tamil Nadu and Karnataka.
Recently, the 20th Livestock Census was conducted in Statement 3 is correct. Currently, India is the 2nd largest
participation with all States and Union Territories. producer of groundnut in the world only after China.
Statement 2 is incorrect – The census covers both rural Statement 4 is correct. Pradhan Mantri Annadata Aay
as well as urban areas. Sanrakshan Abhiyan” (PM-AASHA) is an umbrella
Statement 3 is incorrect – The total Livestock population scheme which will provide Minimum Support Price
is 535.78 million in the country showing an increase of (MSP) assurance for groundnut procurement from
4.6% over Livestock Census-2012. farmers. Components of PM-AASHA are PSS (Price
Statement 4 is incorrect – It is conducted by the Ministry support scheme), PDPS (Price deficiency payment
of Fisheries, Animal Husbandry & Dairying. scheme), Pilot of Private Procurement & Stockist Scheme
(PPPS).
87. Solution: (b)
Exp) Option b is correct. 90. Solution: (c)
Statement 1 is correct. The new hybrid wheat variety Exp) Option c is correct.
‘Karan Vandana’ had a significant yield advantage over Statement 1 is incorrect. Mustard is a subtropical crop
the existing varieties of wheat. It is resistant to both cultivated during rabi season in north-western and
yellow rust and wheat blast diseases. central parts of India.

231 Workbook
GEOGRAPHY

Statement 2 is correct. The oilseeds are produced for launched by Hon’ble Prime Minister of India on
extracting edible oils. Drylands of Malwa plateau, National Panchayat Day i.e., 24th April 2020.
Marathwada, Gujarat, Rajasthan, Telangana and
Rayalseema region of Andhra Pradesh and Karnataka 93. Solution: (c)
plateau are oilseeds growing regions of India. Exp) Option c is correct
Statement 3 is correct. Production of soybean in India Statement 1 is incorrect. It has been observed that the
is dominated by Maharashtra and Madhya Pradesh yields from organic farming are less than modern
which contribute more than 80 per cent of the total agricultural farming. Therefore, small and marginal
production. Rajasthan, Andhra Pradesh, Karnataka, farmers may find it difficult to adapt to large-scale
Chhattisgarh and Gujarat contribute the remaining 11 per production
cent production.
Statement 2 is correct. Organic farming requires more
91. Solution: (b) labour input than conventional farming.
Exp) Option b is correct Statement 3 is incorrect. Choice in production of off-
season crops is quite limited in organic farming.
Statement 1 and 2 are correct. The main functions of
SFAC are: 94. Solution: (a)
1. Promotion of development of small agribusiness Exp) Option a is correct
through VCA (Venture capital assistance) scheme;
Project Development Facility (PDF). Option a is correct. Millets are short duration warm
2. Helping formation and growth of Farmer Producer weather crops. These are coarse grain crops and are used
Organizations (FPOs) / Farmer Producer Companies for both food and fodder. Some of the major millet varities
(FPCs); are Jawar, Bajra, Ragi, Korra, Kodon, Kutki, Hraka, Bauti,
3. Improving availability of working capital and Rajgira. In India, Jawar, Bajra and Ragi are grown on
development of business activities of FPOs/FPCs large areas, but unfortunately area under these crops has
through Equity Grant and Credit Guarantee Fund drastically reduced over the years.
Scheme (EGCGF); 95. Solution: (a)
4. Implementation of National Agriculture Market
Exp) Option a is correct
(e-NAM) Electronic Trading platform.
5. SFAC is one of the Central Procurement Agencies Option a is correct. Bajra is a kharif crop which is grown
for pulses and oilseeds under Price Stabilisation in areas of 40-50 cm annual rainfall. It is rarely irrigated
Fund of Department of Consumer Affairs. and seldom grown in areas with annual rainfall of more
than 100 cm. The ideal temperature for it is 25°C -30°C.
Statement 3 is incorrect. Farmer’s innovation fund is set
It can be grown on poor light sandy soils and upland
up by ICAR (Indian council of agricultural research)
gravely soils. It is mainly grown in Rajasthan (around
not by SFAC. It will be used to scientifically validate,
40%).
upscale and propagate innovations by farmers.
Option b is incorrect. Cotton requires uniformly high
92. Solution: (b) temperature ranging between 21-30- degree temperature.
Exp) Option b is correct The growth of cotton is retarded when the temperature
Statement 2: Command area development programme falls below 20-degree temperature. An annual average
was launched in 1974-75 to bridge / narrow the gap rainfall of 50-100 cm of rainfall could be enough. Cotton
between irrigation potential created and actually utilized is a kharif crop which requires 6-8 months to mature.
in major and medium irrigation schemes. Option c is incorrect. Tobacco can withstand a wide range
Statement 3: NABARD is a statutory body established of temperature varying from 16-35- degree temperature.
in 1982 under National Bank for Agriculture and It normally requires 100 cm of annual average rainfall. It
Rural Development Act, 1981. It is the apex banking requires a well-drained friable sandy loam, not too rich
institution to provide finance for Agriculture and rural in organic matter but rich in mineral salts allowing full
development. development of roots.
Statement 1: Accelerated Irrigation benefits programme Option d is incorrect. Gram is a rabi crop. It can be
(AIBP) was launched in 1996-97 to provide Central grown in a wide range of climatic conditions but it
Assistance to major/medium irrigation projects in the prefers mild cool and comparatively dry climate with
country, with the objective to accelerate implementation 20–25-degree temperature and 40-50 cm rainfall. It is the
of such projects which were beyond resource capability of most important of all the pulses and accounts for 37% of
the States or were in advanced stage of completion. the production.
Statement 4: SVAMITVA (Survey of Villages and Mapping 96. Solution: (a)
with Improvised Technology in Village Areas) scheme
is a collaborative effort of the Ministry of Panchayati Exp) Option a is correct
Raj, State Panchayati Raj Departments, State Revenue Statement 1 is correct. India is the largest producer as
Departments and Survey of India. The scheme was well as the consumer of pulses in the world. India has

Workbook 232
GEOGRAPHY

now almost achieved self-reliance in pulses. India’s pulses 99. Solution: (a)
productivity is 23.62% of world’s total productivity. Exp) Option a is correct
Statement 2 is incorrect. All pulses crop including arhar Statement 1 is incorrect. Coffee plant requires hot and
help in restoring soil fertility by fixing nitrogen from humid climate. It needs rainfall from 150 cm – 250 cm.
the air. Therefore, these are mostly grown in rotation with
other crops. Major pulse producing states in India are Statement 2 is incorrect. This crop is grown on hill
Madhya Pradesh, Uttar Pradesh, Rajasthan, Maharashtra slopes because stagnant water is harmful for coffee plant.
and Karnataka. Statement 3 is incorrect. Loams (not Laterite soil)
containing iron and calcium are ideal for coffee plant.
97. Solution: (c)
Statement 4 is correct. Dry weather is necessary at the
Exp) Option c is correct time of ripening of the berries.
Statement 1 is correct. Bt cotton is an insect-resistant
transgenic crop designed to combat the bollworm.
Bt cotton was created by genetically altering the cotton
genome to express a microbial protein from the bacterium
Bacillus thuringiensis.
Statement 2 is incorrect. There was a decrease in the
use of insecticides after introduction of BT cotton.
The introduction of hybrid BT cotton led to an initial
reduction in insecticide usage but by 2012, insecticide use
was at pre-2002 level.
Statement 3 is incorrect. When using BT cotton seeds
cannot be reused and farmers need to buy new stock for
every growing season. In 2017, the Punjab Agricultural
University (PAU) has developed the first genetically-
modified (Bt) cotton seeds that can be reused, resulting
in saving of input cost to farmers.
98. Solution: (c)
Exp) Option c is correct.
Statement a is correct. India is the world’s leading
producer of millet. In 2016, global production of millet
was 28.4 million tonnes, led by India with 36% of the
world total. India, Niger, and China are the largest 100. Solution: (a)
producers of millet in the world, accounting for more
than 55% of global production. Exp) Option a is correct.
Statement b is correct. Millets are a group of highly Statement 1 is correct. Tur is the second important
variable small-seeded grasses, widely grown around the pulse crop in the country. It is also known as red gram
world as cereal crops or grains for human food and as or pigeon pea. It is cultivated over marginal lands and
fodder. under rainfed conditions in the dry areas of central and
southern states of the country.
Statement c is incorrect. Millets are significantly less
sensitive to climate variability and generally experienced Statement 2 is Incorrect. Maharashtra alone contributes
smaller decline in yields under climate extremes. Millets about one-third of the total production of Pigeon
can grow in areas with less than 350 mm of rainfall and pea. Other leading producer states are Uttar Pradesh,
the cultivation cycle completed within 70-100 days has Karnataka, Gujarat and Madhya Pradesh.
resulted in farmers in dry regions witnessing a millet
revival. Increasingly erratic rainfall patterns and crop 101. Solution: (c)
losses due to climate change factors has forced farmers Exp) Option c is correct
to return to sorghum, little millet and foxtail millet Pair 1 is Incorrect –Silver revolution is related to Egg/
cultivation. It has been realised that this crop is much Poultry production. The exponential increase in Cotton
more resilient to environmental stress, gives an assured fibre is known as Silver Fibre revolution.
yield in both low and excess rainfall conditions, while
keeping input costs low. Pair 2 is correct – Round revolution was started to
increase the production and yield of the Potato.
Statement d is correct. Millets are the cereal crops
generally small-seeded and known for high nutritive Pair 3 is Incorrect – Golden Revolution is related to
value. They provide food for the poor people and fodder Honey/Horticulture production. The Golden Fibre
for cattle. Millets are important crops in the semiarid revolution is linked to Jute fibre production.
tropics of Asia and Africa. Pair 4 is correct – White revolution is famous for

233 Workbook
GEOGRAPHY

revolutionizing milk & dairy farming in India. The credit of India provides lots of incentives to industries locating
for this goes to VergheseKurien. in backward areas. E.g., Special Economic Zones in hilly
Pair 5 is correct – Grey Revolution is related to increase states like Himachal Pradesh.
and growth in the use of fertilizers. It is also linked to the Statement 4 is correct – Speedy and efficient transport
production of wool. facilities to carry raw materials to the factory and to
102. Solution: (c) move finished goods to the market are essential for the
Exp) Option c is correct. development of industries. The cost of transport plays
an important role in the location of industrial units.
Statement 1 is correct – In India, Agriculture Census
is being conducted every 5 years. It is conducted by Statement 5 is correct – Links between Industries is an
Department of Agriculture, Cooperation & Farmers important factor for location of industries. Refineries,
Welfare under Ministry of Agriculture. The latestsurvey fertilizer plants and power plants form key input to each
which is 10th edition was conducted in 2015-16. other. E.g. Refinery, Fertiliser Plant and Thermal Power
Statement 2 is Incorrect – The basic unit of data collection Plant in Panipat (Haryana)
in Agriculture Census is the operational holding. Periodic
Agriculture Census is the main source of information 104. Solution: (d)
in the country on basic characteristics of operational Exp) Option d is correct.
holdings such as land use, cropping pattern, irrigation Statement 1 is correct. Bauxite is the ore which is used
status, and tenancy. Information regarding livestock is
in manufacturing of aluminium. Bauxite is found mainly
provided by livestock census(not agricultural census).
in tertiary deposits and is associated with laterite rocks
Statement 3 is correct –The percentage of land holders
occurring extensively either on the plateau or hill ranges
who are women has increased from 12.79% in 2010-11 to
13.87% in 2015-16, with a corresponding increase of 1.2 of peninsular India and also in the coastal tracts of the
percentage points in the operated area. country.
Statement 2 is correct. Copper is an indispensable
5.5. Industry and Mineral and Energy metal in the electrical industry for making wires, electric
Resources motors, transformers and generators. It is alloyable,
malleable and ductile. It is also mixed with gold to provide
103. Solution: (d) strength to jewellery. The Copper deposits mainly occur
Exp) Option d is correct. in Singhbhum district in Jharkhand, Balaghat district in
Location of industries is influenced by several factors like Madhya Pradesh and Jhunjhunu and Alwar districts in
access to raw materials, power, market, capital, transport Rajasthan.
and labour etc.
Statement 3 is correct. Mica is mainly used in the
electrical and electronic industries. It can be split into very
thin sheets which are tough and flexible. Mica in India
is produced in Jharkhand, Andhra Pradesh, Telangana
and Rajasthan followed by Tamil Nadu, West Bengal
and Madhya Pradesh. In Jharkhand high quality mica
is obtained in a belt extending over a distance of about
150 km, in length and about 22 km, in width in lower
Hazaribagh plateau.

105. Solution: (b)


Exp) Option b is correct.
Statement 1 is incorrect: India is the second largest
Statement 1 is correct – Industries using weight-losing producer of coal in the world, after China. India is
raw materials are located in the regions where raw currently producing about 729 million tonnes of coal.
materials are located. In iron and steel industries, iron However, the domestic production is not able to meet the
ore and coal both are weight-losing raw materials. demand of coal in the country.
Statement 2 is correct – Labour supply is an important Statement 2 is correct: India is world’s 5th largest country
factor in the location of industries. Some types of
manufacturing still require skilled labour. Clothing and in terms of coal deposits, with coal reserves which may
apparel industry is a labour intensive industry. last at least 100 years more.
Statement 3 is correct – India, being a democratic Statement 3 is incorrect: Total quantity of coal imported
country aims at bringing about economic growth with by India has not steadily increased in the past decade. It
balanced regional development. At present, government witnessed a dip in 2014-15.

Workbook 234
GEOGRAPHY

and Kerala. This belt is rich in ferrous metals and bauxite.


It also contains high grade iron ore, manganese and
limestone. This belt lacks in coal deposits except Neyveli
lignite.
This belt does not have as diversified mineral deposits as
the north-eastern belt.
109. Solution: (c)
Exp) Option c is correct.
Statement 1 is incorrect: Garhwal in Uttarakhand and
106. Solution: (b) Kangra valley in Himachal Pradesh have a reserve
Exp) Option b is correct. of limonite iron ore. It has 40-60% iron content and is
India has the largest reserve of iron ore in Asia. In India, yellowish in color.
about 95 per cent of total reserves of iron ore is located Statement 2 is correct: 50% iron ore is produced by
in the States of Orissa, Jharkhand, Chhattisgarh, Odisha alone which makes it the largest producing state.
Karnataka, Goa, Andhra Pradesh and Tamil Nadu. Apart from it Goa, Chhattisgarh and Jharkhand are also
Option 1 and 2 are correct. Iron ore deposits are found in dominating contributors.
following areas of Karnataka state Statement 3 is correct: Dharwad and Cuddapah rock
1. Sandur-Hospet area of Bellary district systems are the metallic mineral resources in India.
2. Baba Budan hills and Kudremukh in Chikmagalur Collectively these accounts for than 90% of iron ore
district reserves in the country.
3. Parts of Shimoga, Chitradurg and Tumkur districts.
110. Solution: (c)
Option 4 is correct. In Chhattisgarh, iron ore deposits
are found in Durg, Dantewada and Bailadila area. Exp) Option c is correct.
Bailadila is one of the largest mechanized mine in Asia. Statement 1 is correct.Odisha is the leading producer
Option 6 is correct. of Manganese. Major mines in Odisha are located in
the central part of the iron ore belt of India, particularly
1) In Orissa, iron ore occurs in a series of hill ranges in in Bonai, Kendujhar, Sundergarh, Gangpur, Koraput,
Sundergarh, Mayurbhanj and Jhar. Kalahandi and Bolangir. Karnataka is another major
2) The important mines are Gurumahisani, Sulaipet, producer and here the mines are located in Dharwad,
Badampahar (Mayurbhanj), Kiruburu (Kendujhar) Ballari, Belagavi, North Canara, Chikkmagaluru,
and Bonai (Sundergarh). Shivamogga, Chitradurg and Tumkur.
Option 3 is incorrect. Singrauli is the largest coal field of Statement 2 is correct.World’s richest monazite deposits
Madhya Pradesh. occur in Palakkad and Kollam districts of Kerala, they
Option 5 is incorrect. Malanjkhand is the largest copper also occur near Vishakhapatnam in Andhra Pradesh
mine of India, located in Madhya Pradesh. It contains and in the Mahanadi river delta in Odisha.
nearly 70% of the India’s reserve.
111. Solution: (a)
107. Solution: (a) Exp) Option a is correct.
Exp) Option a is correct. Statement 1 is correct. Bauxite is a sedimentary rock
Pair 1 is correctly matched. The Rourkela Steel plant with relatively high aluminium content. It is the world’s
was set up in 1959 in the Sundargarh district of Orissa in main source of aluminium and gallium. Bauxite is found
collaboration with Germany. mainly in tertiary deposits and is associated with laterite
Pair 2 is correctly matched. The Bhilai Steel Plant was rocks occurring extensively either on the plateau or hill
established with Russian collaboration in Durg District ranges of peninsular India and also in the coastal tracts
of Chhattisgarh and started production in 1959. of the country.
Pair 3 is incorrectly matched. Durgapur Steel Plant, Statement 2 is incorrect. Guinea has the largest bauxite
in West Bengal, was set up in collaboration with the reserves in the world followed by Australia, Vietnam,
government of the United Kingdom and started Brazil, etc. India has 8th largest bauxite reserves in the
production in 1962. world as of 2020.
Pair 4 is incorrectly matched. Bokaro Steel Plant was set 112. Solution: (b)
up in 1964 at Bokaro with Russian collaboration.
Exp) Option b is correct.
108. Solution: (b) Rihand Dam
Exp) Option b is correct. Rihand Dam also known as Govind Ballabh Pant Sagar,
The South-Western Plateau Region: - This belt extends is the largest dam of India by volume. The reservoir of
over Karnataka, Goa and contiguous Tamil Nadu uplands Rihand Dam is called Govind Ballabh Pant Sagar and is

235 Workbook
GEOGRAPHY

India’s largest artificial lake. It is situated on Son River, a for goods trains.In the first phase, Dedicated Freight
tributary of Ganga. Rihand Dam is a concrete gravity dam Corridor Corporation of India (DFCCIL) is constructing
located at Pipri in Sonbhadra District in Uttar Pradesh, the Western Dedicated Freight Corridor (1504 Route km)
India. and Eastern Dedicated Freight Corridor (1856 route km).
113. Solution: (c) Statement 1 is correct. The Eastern Dedicated Freight
Corridor starts from Sahnewal near Ludhiana (Punjab)
Exp) Option c is correct. and passes through the states of Punjab, Haryana, Uttar
Statement 1 is incorrect – Hydropower potential is located Pradesh, Bihar and Jharkhand to terminate at Dankuni
mainly in northern and north-eastern regions. Arunachal in West Bengal. The World Bank is providing the majority
Pradesh has the largest unexploited hydropower of the funding for the project.
potential of 47 GW, followed by Uttarakhand with 12 Statement 2 is Incorrect.Dedicated Freight Corridor
GW. Corporation of India (DFCCIL) is a public sector
Statement 2 is incorrect – The state of Jharkhand tops undertaking under the Ministry of Railways that
the list of India’s coal reserves — at more than 26% — undertakes planning and development of the Dedicated
and production. The Jharia coalfield, which is located Freight Corridors.
south of Dhanbad, is India’s oldest and richest coalfield Statement 3 is Incorrect. The Western Dedicated Freight
with the best bituminous coal in its reserves. The total coal Corridor connects Dadri in Uttar Pradesh to Jawaharlal
reserves in Jharkhand are estimated to be 83.15 billion Nehru Port (JNPT) in Mumbai. It is being funded by the
tonnes. Japan International Cooperation Agency.
Second, on the list for coal reserves is the state of
Odisha. Situated on the east coast of India, it has more 5.6. Population
than 24% of the country’s total reserves and is responsible
for about 15% of India’s total coal production. 117. Solution: (c)
Statement 3 is correct – The state of Gujarat has the Exp) Option c is correct.
largest wind energy potential of 84431.33 MW, while Statement 1 is correct. Statutory Towns are notified under
the state of Tamil Nadu has the highest installed capacity law by the concerned State Government. All places with a
of wind power. municipality, corporation, cantonment board or notified
114. Solution: (c) town area committee, etc. are called as Statutory Towns.
Exp) Option c is correct. Statement 2 is incorrect. Census Towns is a town with
minimum population of 5,000. Besides this there are
Statement 1 is incorrect – Crude petroleum occurs other conditions that needs to be fulfilled.
in sedimentary rocks of the tertiary period, and not
metamorphic rocks. Statement 3 is incorrect. For the 2021 Census of India, all
Statutory Towns with population of 1,00,000 and above
Statement 2 is correct – Mumbai High, Gulf of Cambay are categorized as cities.
and the Assam are the most productive petroleum
deposits in India. Statement 4 is correct. Among states, the proportion of
urban population is lowest in Himachal Pradesh with
Statement 3 is correct – As per Invest India, India is 10.0 per cent followed by Bihar with 11.3 percent, Assam
currently the third largest consumer of oil and petroleum (14.1 percent) and Orissa (16.7 percent). Goa is the most
products in the world after China and the US. urbanised State with 62.2 percent urban population. In
115. Solution: (c) the North-Eastern States, Mizoram is most urbanised
with 51.5 per cent urban population.
Exp) Option c is correct.
Statements 1 and 3 correct: In India, natural gas has 118. Solution: (d)
been found in Tripura, Rajasthan, Maharashtra and in Exp) Option d is correct.
the Krishna Godavari delta. Petroleum oil is found in Statement 1 is correct. A population pyramid is a way
Assam, Gujarat, Mumbai High and in the river basins of to visualize two variables: age and sex. They are used
Godavari and Krishna. by demographers, who study populations. A population
Statement 2 is incorrect: The reserves of Natural Gas pyramid is a graph that shows the distribution of ages
have not been found in Goa. across a population divided down the center between
The Gas Authority of India Limited was set up in 1984 as a male and female members of the population.
public sector undertaking to transport and market natural A population pyramid can be used to compare
gas. differences between male and female populations of an
area.
116. Solution: (a)
Statement 2 is correct. The “stationary” trend which is
Exp) Option a is correct. a population with low mortality and low fertility rates.
Indian Railways is developing the dedicated freight These graphs have a square or “pillar” shape rather than
corridors to provide exclusive movement on fast speeds a pyramid one. These population pyramids represent

Workbook 236
GEOGRAPHY

a stable population that will not change significantly literacy rate.


barring any sudden changes to fertility or mortality rates. Statement 3 is correct. For the purpose of census 2011,
Statement 3 is correct. As shown in the below figure, a person aged seven and above, who can both read and
India’s population pyramid is bottom heavy. write with understanding in any language, was treated as
literate.
122. Solution: (c)
Exp) Option c is correct.
Statement 1 is correct. The age-sex pyramid which is
bell shaped and tapered towards the top represents that
the birth and death rates are almost equal leading to a near
constant population.
Statement 2 is correct. The age-sex pyramid that
has a narrow base and a tapered top shows low birth
and death rates.e.g., Japan The population growth in
developed countries is usually zero or negative.
123. Solution: (b)
Exp) Option b is correct.
119. Solution: (a) Statement 1 is incorrect. India had a negative population
growth rate in 1921 decadal census. The country’s
Exp) Option a is correct. population growth rate was negative by 0.31 percent.
As per 2011 Census, all places which satisfy the following Statement 2 is correct. In 1961 and 1971, the population
criteria are called as Census Towns: growth rate was 21.51 percent and 24.80 percent
1. A minimum population of 5,000; respectively. In 1981, it became 24.66 percent. After that,
2. At least 75 per cent of the male main working the growth rate of country’s population though remained
population engaged in non-agricultural pursuits; and high, has started slowing down gradually. It was 23.85
3. A density of population of at least 400 persons per sq. percent in 1991, 21.54 percent in 2001 and 17.64 percent
km. in 2011.
120. Solution: (c) 124. Solution: (b)
Exp) Option c is correct. Exp) Option b is correct.
Statement 1 is correct. In India, development of irrigation Statement 1 is incorrect. According to the 2011 census,
in states like Rajasthan and availability of mineral and some states and union territories have seen the increase
energy resources in states like Jharkhand have resulted in the decadal growth rate. For example, Tamil Nadu
in moderate to high concentration of population in areas (3.9 percentage points) and Puducherry (7.1 percentage
which were previously very thinly populated. This is also points) have registered some increase during 2001-2011
true regarding the development of transport network over the previous decade.
Peninsular States. Statement 2 is correct. Many major states have seen the
Statement 2 is correct. Physical factors also affect fall in decadal growth rate during the 2001-2011 period.
the distribution of population. Climate, terrain and This was highest for Maharashtra (6.7 percentage
availability of water largely determines the pattern of the points), the lowest for Andhra Pradesh (3.5% percentage
population distribution. Hence, we observe that the North points).
Indian Plains, deltas and Coastal Plains have higher Statement 3 is incorrect. During 2001-2011, the growth
proportion of population than the interior districts of rates of almost all States and Union Territories have
southern and central Indian States, Himalayas and some registered a lower figure compared to the previous
of the north eastern and the western states. decade, namely, 1991-2001. The percentage decadal
121. Solution: (a) growth rates of the six most populous States, namely,
Uttar Pradesh, Maharashtra, Bihar, West Bengal, Andhra
Exp) Option a is correct. Pradesh and Madhya Pradesh have all fallen during 2001-
Statement 1 is Incorrect. As per the Census 2011, sex 2011 compared to 1991-2001.
ratio has increased from 933 females per thousand males
in 2001 to 943 females per thousand males in 2011. 125. Solution: (a)
However, there is no steady increase since independence. Exp) Option a is correct
Statement 2 is Incorrect. The literacy rate in the country The percentage shares of population of the states and
is more than 74.01% per cent as per the Census 2011. It Union Territories in the country show that Uttar Pradesh
stood at 82.14% for males and 65.46% for females. Kerala has the highest population followed by Maharashtra,
retained its position by being on top with a 93.91 percent Bihar and West Bengal.

237 Workbook
GEOGRAPHY

128. Solution: (d)


Exp) Option d is correct.
Statement 1 is correct. The intra-state migration is
categorized into four types - rural to rural, rural to urban,
urban to urban and urban to rural. In India, out of the
total number of persons registered as “migrants” in the
2011 Census, only 11.91% (5.43 crore) had moved to one
state from another, while nearly 39.57 crore had moved
within their states. The stream was dominated by female
migrants in all four categories. Most of these were
migrants related to marriage.
Statement 2 is correct. In the context of inter-state
migration, men predominate the rural to urban stream
of inter-state migration due to economic reasons. But
at the same time, females predominate the streams of
short distance rural to rural inter-state migration.
129. Solution: (b)
Exp) Option b is correct.

The correct decreasing order of the States given above, on


the basis of net in-migration from other states is as follows
1. Maharashtra – 90.87 lakhs
2. Uttar Pradesh – 40.62 lakhs
3. Gujarat – 39.16 lakhs
126. Solution: (a) 4. Kerala – 6.54 lakhs
Exp) Option a is correct
130. Solution: (c)
Statement 1 is correct. The first population Census
in India was conducted in 1872 but its first complete Exp) Option c is correct.
Census was conducted only in 1881. Population data are Statement 1 is correct. An urban agglomeration is a
collected through Census operation held every 10 years continuous urban spread constituting a town and its
in our country. adjoining outgrowths (OGs), or two or more physically
Statement 2 is incorrect. The responsibility of contiguous towns together with or without outgrowths of
conducting the decennial Census rests with the Office such towns. An Urban Agglomeration must consist of at
of the Registrar General and Census Commissioner, least a statutory town.
India under Ministry of Home Affairs, Government of Statement 2 is correct. Its total population, including all
India. the constituents, should not be less than 20,000 as per
the 2001 Census. On the basis of local conditions, many
Statement 3 is incorrect The Census 2021 will be
other areas have been treated as urban agglomerations if
conducted in 18 languages out of the 22 scheduled
they satisfied the basic condition of contiguity. Examples:
languages, while Census 2011 was in sixteen of the Greater Mumbai Urban Agglomeration, Delhi Urban
eighteen scheduled languages declared at that time. It also Agglomeration, etc.
will introduce a code directory to streamline the process.
131. Solution: (c)
127. Solution: (d)
Exp) Option c is correct.
Exp) Option d is correct.
Statement 1 is correct. The level of urbanisation is
The data collected through the census is used for measured in terms of percentage of urban population
1. Administration, to total population. In India, there has been a consistent
2. Planning and policy-making as well as management growth in the urban population in last few decades.
and evaluation of various programmes run or to be Percentage of urban population in the country -
introduced by the government, NGOs, researchers, In 1981 (23.34 percent), in 1991 (25.71 percent), in 2001
commercial and private enterprises, etc. (27.78 percent), in 2011 (31.16 percent), people of the
3. Demarcation of constituencies total population live in urban areas.
4. Allocation of representation to the Parliament Statement 2 is correct. The growth rate of urbanisation
5. Allocation of representation to State legislative has slowed down during last two decades.
assemblies In 1991, the decennial growth rate was 36.47 percent, in
6. Allocation of representation to Local bodies. 2001, it was 31.13 percent, in 2011, it was 31.08 percent.

Workbook 238
GEOGRAPHY

132. Solution: (d)


Exp) Option d is correct.
Statement 1 is incorrect: The National Digital
Communications Policy, 2018 seeks to unlock the
transformative power of digital communications
networks and to achieve the goal of digital empowerment
and improved well-being of the people of India.
The Policy aims to accomplish the following Strategic
Objectives by 2022:
1. Provisioning of Broadband for All
2. Creating 4 Million additional jobs in the Digital
Communications sector 5.6. India Human and Economic Geography
3. Enhancing the contribution of the Digital 134. Solution: (a)
Communications sector to 8% of India’s GDP from ~
Exp) Option a is correct
6% in 2017
Manganese is a silver metallic element. It occurs in
4. Propelling India to the Top 50 Nations in the ICT many minerals such as manganite, sugilite, rhodonite,
Development Index of ITU from 134 in 2017 rhodochrosite, and pyrolusite.
5. Enhancing India’s contribution to Global Value Indian manganese ore deposits occur mainly as
Chains metamorphosed bedded sedimentary deposits associated
6. Ensuring Digital Sovereignty with Gondite Series (Archaeans) of Madhya Pradesh
(Balaghat, Chhindwara & Jhabua districts), Maharashtra
Statement 2 is incorrect: (Bhandara & Nagpur districts), Gujarat (Panchmahal
PM-WANI (Public Wi-Fi Access Network Interface) district), Odisha (Sundergarh district).
initiative will promote the growth of Public Wi-Fi Statement a is correct.The Dharwar systems of rocks
Networks in the country and, in turn, will help in are highly metalliferous. They bear out gold, iron ore,
proliferation of Broadband Internet, enhancement of manganese mica, cobalt, chromium, copper, tungsten,
lead, nickel, precious stones and budding stones. The
income and employment and empowerment of people. Dharwar system of rocks occurs in scattered patches
133. Solution: (a) in: (i) southern Deccan, (ii) central and eastern parts of
the Peninsula, (iii) north- western region, and (iv) the
Exp) Option a is correct. Himalayan region.
Statement 1 is correct: Economic reforms of 1991 were Statement b is incorrect. Orissa is the largest producer
introduced in the wake of balance of payment crisis and produces over 37 per cent manganese ore of India. It
and increased inflation rate. Foreign exchange reserves is obtained from Gondite deposits in Sundargarh district
and Kodurite and Khondolite deposits in Kalahandi and
were dropped to levels that were not sufficient for even
Koraput Districts.
a fortnight. The government was not able to repay its
Statement c is incorrect. The most important manganese
borrowings from abroad. All these issues lead to adopting ore is pyrolusite. Limonite is an ore of iron. Other
the measures to reform the economy. important manganese ores are the oxides pyrolusite,
Statement 2 is incorrect: Economic reforms of 1991 romanechite, manganite, and hausmannite and the
leads to dismantling of license raj and deregulation of carbonate ore rhodochrosite. Rhodonite and braunite,
both silicate ores, are frequently found with the oxides.
industries so as to promote growth of a more efficient
and competitive industrial economy. It allowed the entry Statement d is incorrect. On Earth, manganese is
never found as a free metal, but it is found in a number
of private companies in previously reserved sectors i.e.
of minerals. The most important of these minerals
de-reservation of all industries except three industries is pyrolusite, the main ore mineral for manganese.
like Defence, Railways and Atomic energy. Manganese is essential to iron and steel production
Statement 3 is incorrect: Post economic reforms of 1991, 135. Solution: (d)
the growth of agriculture sector has declined. While
Exp) Option d is correct.
industrial sector reported fluctuation in growth rate.
Pair 1 is correctly matched. The famous gold mines
Whereas the growth rate of service sector has gone up. in India are Kolar (Karnataka), Hutti (Karnataka),
Industrial Production Growth rate Ramagiri( Andhra Pradesh), and Lawa (Jharkhand).

239 Workbook
GEOGRAPHY

The largest reserves of gold ores are located in Bihar (44 Statement 2 is incorrect. Australia possesses around
per cent), followed by Rajasthan (25 per cent). 30% of the world’s known recoverable uranium
Pair 2 is correctly matched. The five largest silver mines reserves. Kazakhstan contains about 13% of the world’s
by production in India are Sindeshar Khurd (Rajasthan), recoverable uranium, with 50 known deposits and around
Rampura Agucha Mine (Rajasthan), Rajpura Dariba Mine 20 operating uranium mines.
(Rajasthan) Zawwar (Rajasthan), Hutti (Karnataka). Statement 3 is incorrect. Much like uranium, thorium
Pair 3 is correctly matched. The famous diamond mines is also a fertile substance, but not a fissile substance
in India are Panna in Madhya Pradesh. In India, the by itself. It requires work to make it usable in a nuclear
diamond resources are concentrated mostly in three states reactor. The process through which Thorium can be made
- Madhya Pradesh, Andhra Pradesh and Chhattisgarh. usable in the reactor is a three-stage process. It starts
with using Pressurized Heavy Water Reactors (PHWR)
136. Solution: (b) and light water reactors to convert natural uranium to
Exp) Option b is correct. plutonium. Next, the neutrons from plutonium breed
U-233 from Thorium. The final stage, Advanced Heavy
The rare-earth elements (REE), also called the rare-earth Water reactors burn U-233 with Thorium, and about 66
metals or the lanthanides (though yttrium and scandium per cent of power is generated from Thorium fission.
are usually included as rare-earths) are a set of 17 nearly-
indistinguishable lustrous silvery-white soft heavy metals. 138. Solution: (c)
Statement 1 is incorrect. India imports rare earth Exp) Option c is correct
metals not due to lack of its reserves. In fact, India Zero-budget natural farming is a holistic agroecological
is the fifth largest reservoir of rare earth elements alternative based on modern and traditional science that
in the world. But the real issue is that the Indian rare mitigates the consequences of climate change, reduces
earth industry keeps on operating on the downstream input costs and creates sustainable agricultural livelihoods.
production process only. The upstream process involves
higher environmental risk and advanced technology, Option c is correct.
which is lacking. Hence, the imports are high. Farmers use four treatments to enhance productivity at
Statement 2 is correct. Matsya 6000 is an Indian manned low cost:
deep-submergence vehicle under deep ocean mission Bijamrita (Seed treatments using local cow dung and
intended to be utilised for deep sea exploration of rare urine);
minerals. Currently under development, the vehicle Jiwamrita (soil inoculant made of cow dung and urine);
would consist a titanium-alloy sphere of 80mm thickness
along with diameter of 2.1m which can withstand the Mulching (ensure favourable microclimate soil); and
pressure of 600 bar. Waaphasa (soil aeration).
Statement 3 is incorrect. Rare earth elements not These treatments are chemical-free agriculture at the
limited to the western coast of India. For example, the lowest/zero-cost and enhance the incomes of farmers.
states having resources of in situ monazite are Odisha, Thus, “Four wheels of ZBNF”are Bijamrit, Jivamrit,
Andhra Pradesh, Tamil Nadu, Kerala, West Bengal, Mulching and Waaphasa.
Jharkhand.
Bijamrit is the microbial coating of seeds with formulations
The entire landmass and rocks around Indian Ocean of cow urine and cow dung.
Region (IOR) contain the REEs as the coastline is
Jivamrit is the enhancement of soil microbes using an
enriched with ‘Mineral Sands’, an easily recognizable
inoculum of cow dung, cow urine, and jaggery. Mulching
black colour sand used as alloy, paint, filtration, and
is the covering of soil with crops or crop residues.
sandblasting. These mineral sands contain rare earth
elements. Waaphasa is the building up of soil humus to increase
soil aeration.
137. Solution: (a) In addition, ZBNF includes three methods of insect and
Exp) Option a is correct. pest management: Agniastra, Brahmastra and Neemastra
Uranium and thorium are naturally occurring, (all different preparations using cow urine, cow dung,
radioactive heavy metals with unusual properties. The tobacco, fruits, green chilli, garlic and neem).
energy generated by the natural breakdown of radioactive
139. Solution: (c)
elements is immense and can be used in nuclear reactors.
Exp) Option c is correct
Statement 1 is correct. World’s largest thorium reserves
in the form of monazite are found in India on the coast To promote Inland Water Transport in the country 111
of Kerala. The country’s thorium reserves make up 25 waterways (including 5 existing and 106 new) have
per cent of the global reserves. India’s thorium deposits, been declared as National Waterways (NWs) under the
estimated at 360,000 tonnes, far outweigh its natural National Waterways Act, 2016.
uranium deposits at 70,000 tonnes. It can easily be used Pair 1 is incorrect. National Waterways-1 - Ganga-
as a fuel to cut down on the import of Uranium from Bhagirathi-Hooghly River system from Allahabad
different countries to Haldia was declared as National Waterway No.1

Workbook 240
GEOGRAPHY

vide National Waterway (Allahabad-Haldia stretch of and attracting private and foreign direct investment. The
the Ganga Bhagirathi-Hooghly River) Act 1982. The FDI Policy permits 100% FDI in the railway’s infrastructure
waterway extends from Haldia to Allahabad for a distance sector. FDI is permitted in the construction, operation
of 1620 kms. and maintenance of the railway transport sector.
Pair 2 is correct. National Waterways-2 - Brahmaputra
from Dhubri to Sadiyawas in Assam declared as National
Waterway no. 2 vide National Waterway (Sadiya-Dhubri
stretch of the Brahmaputra River) Act 1988. From Dhubri
to Sadiya, the waterway extends for a distance of 891Km.
The river Brahmaputra flows down the centre of Assam
Valley. It receives a number of tributaries like Subansiri,
Jia Bharali, Dihing, Burhi Dihing, Disang, Dhansiri and
Kopili.
Pair 3 is incorrect. National Waterways-4 - Kakinada-
Puducherry canal stretch (767km) along with Godavari
River stretch (171km) between (Bhadrachalam and
Rajhamundry) and Krishna River stretch(157km) 141. Solution: (a)
between (Wazirabad and Vijaywada) is termed as NW-4. Exp) Option a is correct.
Total length of NW-4 is 1095 km. India is among the world’s largest producers of Textiles
Pair 4 is correct. National Waterways-5 - The East Coast and Apparel. The domestic apparel & textile industry in
Canal stretch along with Brahmani and Mahanadi delta India contributes 5% to the country’s GDP, 7% of industry
river system is defined as NW-5. It includes the canal output in value terms, and 12% of the country’s export
stretch between Geonkhali and Charbatia (217 km), part earnings. India is the 6th largest exporter of textiles and
of Matai River between Charbatia and Dhamra (40 km), apparel in the world. However, in recent times India is
part of Brahmani, Kharsua and Dhamra river system losing out to countries like Bangladesh and Vietnam in
between Talcher and Dhamra (265 km) and Mahanadi textiles sector.
delta river system between Mangalgadi and Paradeep (101
Option a is correct. The most appropriate reason for India
km). Total length of NW-5 is 623 km.
losing out to Bangladesh and Vietnam in textiles sector
140. Solution: (b) is high tariffs faced by Indian exporters as compared to
Exp) Option b is correct Bangladesh and Vietnam exporters in foreign markets.
For example, in the EU, Bangladesh’s exports enter mostly
Indian Railways is a government-owned-railway system duty free while Indian exports face an average tariff of
under the ownership of Ministry of Railways, Government
9.2 per cent. As India withdrew from RCEP, India will be
of India that operates India’s national railway system. It
disadvantaged in the market. India industry’s dependence
manages the Fourth-largest railway network in the world
on duty drawbacks at the expense of innovation are
by size, with a route length of 67,956 km as of 31 March
2020. causing the textile industry to lose out to Bangladesh.
Statement 1 is correct. Railways has been losing out Option b is incorrect. It is not the case that no subsidy
on the FMCG and automobile markets (which are is provided to the Indian textile industries to increase
preferring roads) due to absence of last mile linkages. employment. Under, Pradhan Mantri Rojgar Protsahan
Yojana (PMRPY), Government is paying the 8.33%
Statement 2 is correct. The share of Indian Railways in contribution of employers to the Employee Pension
freight movement has been witnessing a drop over a
Scheme (EPS) in respect of new employees having a
period of time, mainly on account of non-competitive
UAN. For the textile (apparel) sector, the Government
tariff structure. Railways’ freight rates are already higher
than other modes of transport for several commodities. will also be paying the 3.67% Employees Provident Fund
(EPF) contribution of the eligible employer for these new
Statement 3 is correct. Railways’ operating ratio has employees.
consistently been higher than 90% in the past several
years, which indicates that its capability to generate Option c is incorrect. The world demand is increasingly
operational surplus is low. Further, its expenditure on shifting toward clothing based on man-made fibres
staff and pension has been increasing. Consequently, while Indian domestic tax policy favours cotton-based
capacity growth is increasingly being funded through production and the tariff policy shields an inefficient
borrowings. man-made fibre sector.
Statements 4 is incorrect. Facing stiff competition Option d is incorrect. India’s GSP (Generalised system
from other modes of transportation, the government is of preferences) benefits were terminated by US in 2019
introducing various transformative measures in order but this is not the appropriate reason for losing out
to keep railways on track. These measures include new to Bangladesh and Vietnam in textiles sector as the
modernised wagons, dedicated freight corridors, high Generalised System of Preferences (GSP) facility for
speed rail, high-capacity rolling stock, port connectivity, Bangladesh was suspended in June 2013.

241 Workbook
GEOGRAPHY

142. Solution: (c) Statement 3 is incorrect - As per NFHS-5, the sex ratio at
Exp) Option c is correct birth has improved from 919 in NFHS-4 to 929 in NFHS-
5, an increase of 10 points. Except for Tripura, Ladakh,
Migration is the physical movement of people from one
place to another. and Lakshadweep, all states and UTs have a child sex
ratio of less than 1000.
Statement 1 is correct. As per the 2011 census, there
were 21 crore rural-rural migrants which formed 54% 144. Solution: (a)
of classifiable internal migration (the Census did not
Exp) Option a is correct
classify 5.3 crore people as originating from either rural or
urban areas). Rural-urban and urban-urban movement Settlements vary in size and type. They range from a
accounted for around 8 crore migrants each. There were hamlet to metropolitan cities. With size, the economic
around 3 crore urban-rural migrants (7% of classifiable character and social structure of settlements changes and
internal migration). so do its ecology and technology.
Statement 2 is incorrect. The share of rural-to-urban Pair 1 is correct – Clustered settlements are generally
migrants in the population rose from 5.06% in 2001 to found in fertile alluvial plains and in the north-eastern
6.5% in 2011. Women outnumber men in making the states. Sometimes, people live in compact village for
rural-to-urban shift. Of the 78 million, 55% were females security or defence reasons, such as in the Bundelkhand
and they outnumbered males in 554 of the 640 districts. region of central India and in Nagaland. In Rajasthan,
The share of male rural-to-urban migrants in the total scarcity of water has necessitated compact settlement
male population rose from 4.6% to 5.7%; for females, the for maximum utilisation of available water resources. is
share rose from 5.5% to 7.4%. a compact or closely built-up area of houses. In this type
Statement 3 is incorrect. As per Census 2011 data on of village, the general living area is distinct and separated
migration, Maharashtra has highest number of migrants from the surrounding farms, barns and pastures.
in India. Of the 5.74 crore migrants in Maharashtra, 27.55
lakh reported their last place of residence to be Uttar Pair 2 is correct - Hamleted settlements are fragmented
Pradesh; 5.68 lakh said Bihar. Internal migration from into several units physically separated from each other
within Maharashtra had the lion’s share of migrants: 4.79 bearing a common name. These units are locally called
crore. panna, para, palli, nagla, dhani, etc. in various parts of
the country. This segmentation of a large village is often
Statement 4 is incorrect. In India five million people
from other countries are residing legally. The largest share motivated by social and ethnic factors. Such villages are
of migrants in India is from Bangladesh followed by more frequently found in the middle and lower Ganga
Pakistan and Nepal. plain, Chhattisgarh and lower valleys of the Himalayas.
Pair 3 is incorrect – Hamlets comprises most of the
143. Solution: (d) settlements in Lower Himalayan Valley. Semi-clustered
Exp) Option d is correct. or fragmented settlements can be seen in Gujarat plain
The sex ratio is the ratio of males to females in a population. and some parts of Rajasthan. In this case, one or more
Statement 1 is incorrect – The sex ratio at birth increased sections of the village society choose or is forced to live a
from 819 to 834 between 2001 and 2011 Census for little away from the main cluster or village.
Haryana. According to NFHS V, it is 893. We can observe Pair 4 is incorrect – In Alluvial plains, mostly clustered
the improvement in Haryana. However, it is not the state settlements are found. Dispersed or isolated settlement
with lowest Sex ratio at birth. Rajasthan and Himachal pattern in India appears in the form of isolated huts or
Pradesh have 891 and 875 respectively, lower than that hamlets of few huts in remote jungles, or on small hills
of Haryana. with farms or pasture on the slopes. Extreme dispersion of
Statement 2 is incorrect - It is Child Sex Ratio, which is settlement is often caused by extremely fragmented nature
defined as the number of females per thousand males of the terrain and land resource base of habitable areas.
in the age group 0–6 years in a human population. Sex Many areas of Meghalaya, Uttarakhand, Himachal
ratio (also known as Gender Ratio) is the ratio of males Pradesh and Kerala have this type of settlement
to females in a population. That is, it is the proportion of
males in a population. 145. Solution: (d)
The child sex ratio in the country declined from 927 Exp) Option d is correct.
in 2001 to 919 in 2011 census. The sex ratio at birth is Urbanization is the process through which cities grow,
calculated as the number of females per 1,000 males
and higher and higher percentages of the population
born in the last five years. This is a very vital indicator
comes to live in the urban areas.
to understand the underlying abnormalities which are
leading to an imbalanced sex ratio. If the sex ratio at birth Statement 1 is incorrect: Although the absolute number
improves, then it would surely lead to an increase in the of urban populations is increasing constantly, the
overall sex ratio in each age group growth rate has been variable.

Workbook 242
GEOGRAPHY

Statement 2 is correct: The rate of increase is the


highest in case of area under non-agricultural uses.
This is due to the changing structure of Indian economy,
which is increasingly depending on the contribution
from industrial and services sectors and expansion of
related infrastructural facilities. Also, an expansion of
area under both urban and rural settlements has added to
the increase. Thus, the area under non-agricultural uses is
increasing at the expense of wastelands and agricultural
land.
Statement 3 is correct: The four categories that
have registered a decline are barren and wasteland,
culturable wasteland, area under pastures and tree
Statement 2 is incorrect: Both the urban and rural
population has been consistently increasing although crops and fallow lands. The decline in land under
the rate of increase has declined. pastures and grazing lands can be explained by pressure
from agricultural land. Illegal encroachment due to
expansion of cultivation on common pasture lands is
largely responsible for this decline.

147. Solution: (b)


Exp) Option b is correct
The overall contribution of rainfall to the country’s
annual ground water resource is 68% and the share
of other resources, such as canal seepage, return flow
from irrigation, recharge from tanks, ponds and water
conservation structures taken together is 32%
Statement 3 is incorrect: Japan’s capital Tokyo has the
largest population of the world’s capital cities at over Option a is incorrect - The new guidelines prohibit
37 million people. This was followed by Delhi (India) at new industry and mining projects in over-exploited
over 28 million; Mexico City (Mexico) at 21 million; and zones and makes it mandatory for existing industries,
Cairo (Egypt) with 20 million. Across the world the most commercial units and big housing societies to take ‘no
common capital size was in the range of 1 to 5 million objection certificate’ (NOC). The domestic consumers,
people. rural drinking water schemes, armed forces, farmers
and micro & small enterprises (that withdraw up to
146. Solution: (c) 10 m3 water per day), however, are exempted from
Exp) Option c is correct. the requirement of a no objection certificate from the
Land use change is a process which transforms the Central Ground Water Board (CGWB).
landscape by direct human-induced land use such as Option b is correct - Section 7(g) of the Easement
settlements, commercial and economic uses and forestry Act, 1882 states that every owner of land has the right
activities. to collect and dispose within his own limits all water
Statement 1 is incorrect: Around 51% of India s under the land and on its surface which does not pass
geographical area is already under cultivation as in a defined channel. The legal consequence of this law
compared to 11% of the world average. The present is that the owner of the land can dig wells in his land and
cropping intensity of 136% has registered an increase of extract water based on availability and his discretion.
only 25% since independence. Further, rainfed drylands 24 Additionally, landowners are not legally liable for
constitute 65% of the total net sown area. 2/3rd of Area is any damage caused to the water resources as a result of
under Food Grains and 54% under Rice and Wheat. overextraction.

243 Workbook
GEOGRAPHY

Option c is incorrect - India is the larger extractor Statement 1 is correct. Most of the metallic minerals in
of ground water but is not the largest exporter. It is India occur in the peninsular plateau region in the old
third largest exporter of ground water – 12% of world crystalline rocks.
total. Pakistan is the largest exporter of ground water. Statement 2 is incorrect. Over 97 per cent of coal reserves
However, looking at the total export of virtual water, India occur in the valleys of Damodar, Sone, Mahanadi and
is at the top position. Godavari.
Option d is incorrect - India accounts for 16 per cent of Statement 3 is correct Petroleum reserves are located in
the world’s population living in less than 2.5 per cent of the sedimentary basins of Assam, Gujarat and Mumbai
the global area, and has just 4 per cent of the global water High i.e., off-shore region in the Arabian Sea. New
resources. According to the Central Water Commission reserves have been located in the Krishna-Godavari and
(CWC), the estimated water resources potential of the Kaveri basins.
country, which occurs as natural runoff in the rivers, Most of the major mineral resources occur to the east
is 1,999 billion cubic metres. Of this, the estimated of a line linking Mangaluru and Kanpur. Minerals are
utilisable resources are 1,122 billion cubic metres per generally concentrated in three broad belts in India.
year — 690 BCM per year surface water and 432 BCM
per year replenishable groundwater. 150. Solution: (c)
148. Solution: (b) Exp) Option c is correct
Exp) Option b is correct. About 95 per cent of total reserves of iron ore is located in
the States of Odisha, Jharkhand, Chhattisgarh, Karnataka,
India accounts for about 2.45 per cent of world’s surface Goa, Telangana, Andhra Pradesh and Tamil Nadu.
area, and only 4 per cent of the world’s water resources.
The total water available from precipitation in the country Pair 1 is correct. In Odisha, iron ore occurs in a series
in a year is about 4,000 cubic km. of hill ranges in Sundergarh, Mayurbhanj and Jhar.
The important mines are Gurumahisani, Sulaipet,
Option a is correct. Agriculture accounts for most of the Badampahar (Mayurbhaj), Kiruburu (Kendujhar) and
surface and ground water utilization. It accounts for Bonai (Sundergarh).
89 per cent of the surface water and 92 per cent of the
Pair 2 is correct. Bauxite is found mainly in tertiary
groundwater utilization.
deposits and is associated with laterite rocks occurring
Option b is incorrect. The share of industrial sector is extensively either on the plateau or hill ranges of
limited to 2 per cent of the surface water utilization and peninsular India and also in the coastal tracts of the
5 per cent of the ground-water, country. Odisha happens to be the largest producer of
Option c is correct. The share of domestic sector is higher Bauxite. Lohardaga in Jharkhand have rich Bauxite
(9 per cent) in surface water utilization as compared to deposits. Gujarat, Chhattisgarh, Madhya Pradesh and
groundwater. Maharashtra are other major producers.
The share of agricultural sector in total water utilization is Bauxite is the ore, which is used in manufacturing of
much higher than other sectors. aluminium.
Pair 3 is correct. The Copper deposits mainly occur in
Singhbhum district in Jharkhand, Balaghat district in
Madhya Pradesh and Jhunjhunu and Alwar districts in
Rajasthan. Minor producers of copper are Agnigundala
in Guntur District (Andhra Pradesh), Chitradurg and
Hasan districts (Karnataka) and South Arcot district
(Tamil Nadu).
Copper is an indispensable metal in the electrical
industry for making wires, electric motors, transformers
and generators. It is alloyable, malleable and ductile. It
is also mixed with gold to provide strength to jewellery.
Option d is correct. Among the states of India Uttar
151. Solution: (b)
Pradesh has highest net annual ground water
availability (about 72 bcm) while Delhi has the least (0.29 Exp) Option b is correct
bcm). Warabandi is system of rotation of supply of water
according to a predetermine schedule as per area and
149. Solution: (c) crop needs specifying the day, time and duration of
Exp) Option c is correct supply to each holding to ensure equitable water
India is one of the richest countries in minerals resources distribution among farmers of an outlet command.
in the world. Since India’s internal structure of earth is the Warabandi system of water distribution is followed for
product of ancient hard rock’s therefore almost all kinds canal irrigation system at many parts in India including
of minerals are found here especially in Gondwana rocks. Madhya Pradesh. It is observed that tail end fields receive

Workbook 244
GEOGRAPHY

less amount of water compared to fields situated at or of people at large. Consequently, there was a high natural
near the head of water courses. This is because of seepage increase and higher growth rate.
losses in unlined water courses. Statement 2 is correct. The decades 1921-1951 are
Option b is correct. Warabandi system is a rotational referred to as the period of steady population growth.
water allocation system which can fulfil the requirement An overall improvement in health and sanitation
of equality. Warabandi is an Urdu word that combines throughout the country brought down the mortality rate.
wahr (“turn”) and bandi (“fixed)”. So “warabandi” means At the same time better transport and communication
“rotation of water supply according to a fixed schedule”. system improved distribution system. The crude birth
Warabandi system helping farmer can be allocated the rate remained high in this period leading to higher
same volume of water after the systematic rotation of water growth rate than the previous phase. This is impressive at
as the system will equalize the water allocation according the backdrop of Great Economic Depression, 1920s and
to the regular and approved time schedule including the World War II.
day supply will start, and for how long. Statement 3 is incorrect. The period from 1901-1921 is
referred to as a period of stagnant or stationary phase of
152. Solution: (a)
growth of India’s population, since in this period growth
Exp) Option a is correct. rate was very low, even recording a negative growth rate
Footloose industry is a general term for an industry that during 1911-1921. Both the birth rate and death rate
can be placed and located at any location without effect were high keeping the rate of increase low. Poor health
from factors of production such as resources, land, labour, and medical services, illiteracy of people at large and
and capital. Following are some of the characteristics of inefficient distribution system of food and other basic
the industry - necessities were largely responsible for a high birth and
Statement 1 is correct. Footloose industries produce in death rates in this period.
relatively small quantities, employing smaller workforces Statement 4 is correct. In the post 1981 till present, the
and are considered to be more efficient from an growth rate of country’s population though remained
ecological point of view. These are generally not polluting high, has started slowing down gradually. A downward
industries. trend of crude birth rate is held responsible for such
a population growth. This was, in turn, affected by an
Statement 2 is correct. These industries can be placed
increase in the mean age at marriage, improved quality
and located at any location without effect from factors
of life particularly education of females in the country.
such as resources or transport. These industries often
The growth rate of population is, however, still high in the
have spatially fixed costs, which means that the costs of
country, and it has been projected by World Development
the products do not change despite where the product is
Report that population of India will touch 1,350 million
assembled.
by 2025.
Statement 3 is incorrect. These industries produce their
products in small numbers and they do not require a large 154. Solution: (b)
labour force. Exp) Option b is correct
Statement 4 is incorrect. Cement manufacturing is not The New Education Policy announced by Government
a foot loose industry as this is a raw material specific of India is based on 5 pillars of access, equity, quality,
industry. Footloose industries can be located at a wide affordability and accountability is aimed for sustainable
variety of places, as these are not weight-losing nor raw- development.
material-specific Statement 1 is correct. The new education policy 2020
Diamonds, computer chips, and mobile manufacturing aims at improving the Gross Enrollment Rate (GER)
are some examples of footloose industries. in higher education and Government of India has set a
target of achieving 50 per cent GER by 2035. To achieve
153. Solution: (a) this target, the government has decided to add 3.5 crore
Exp) Option a is correct new seats in Higher Education Institutions of India by
The history of growth in India’s population can be divided 2035.
into four distinct phases – the points of division being Statement 2 is correct. The literacy rate of 69% among
1911, 1921, 1951 and 1981. Muslim Women was higher than the rate among women
Statement 1 is incorrect. The decades 1951-1981 are of ST (61%) and SC (64%).
referred to as the period of population explosion in Statement 3 is incorrect. In India, scheduled caste and
India, which was caused by a rapid fall in the mortality scheduled tribe and other backward class are officially
rate but a high fertility rate of population in the country. recognized by the constitution as groups of disadvantaged
The average annual growth rate was as high as 2.2 per indigenous people. The literacy rate of the total
cent. It is in this period, after the Independence, that population in the country was about 73 percent in 2011,
developmental activities were introduced through a in comparison to about 66 percent among the scheduled
centralised planning process and economy started caste population. As per Census 2011, literacy rate of
showing up ensuring the improvement of living condition Scheduled Tribes (STs) was 59%.

245 Workbook
GEOGRAPHY

stems from the fact that FMCG products usually have a


short shelf life and are non-durable.
Fast moving consumer goods (FMCG) are the 4th largest
sector in the Indian economy.
Statement 1 is correct. There are three main segments in
the sector – food and beverages (19%), healthcare (31%)
and household & personal care (remaining 50%).
Statement 2 is correct. The urban segment is the largest
contributor (accounts for a revenue share of around 55%)
to the overall revenue generated by the FMCG sector in
India.
However, in the last few years, the FMCG market has
grown at a faster pace in rural India compared with urban
India. Semi-urban and rural segments are growing at a
rapid pace and FMCG products account for 50% of the
total rural spending.
Statement 3 is incorrect. Government gave approval of
155. Solution: (d) 100% Foreign Direct Investment (FDI) in the cash and
Exp) Option d is correct carry segment and in single-brand retail along with 51%
FDI (and not 100%) in multi-brand retail for FMCG
Depending on the mode of occurrence and the nature of sector.
the ore, mining is of two types: surface and underground
mining. 100% FDI allowed in food processing segment.
Pair 1 is correct. The surface mining also known as 157. Solution: (d)
open-cast mining is the easiest and the cheapest way of Exp) Option d is correct
mining minerals that occur close to the surface. Open-
cast mining is a surface mining technique of extracting Logistics is management of flow of goods from origin to
rock or minerals from the earth by their removal from an destination to meet customer requirements in minimum
open pit or borrow. time at minimum cost.
Overhead costs such as safety precautions and Logistics include packaging, warehousing, transport and
equipment are relatively low in this method. The output delivery of raw materials and final goods.
is both large and rapid. Logistics and transportation sector is considered as
Pair 2 is correct. When the ore lies deep below the surface, backbone of the economy because it provides flow of
underground mining method (shaft method) has to be goods efficiently on which success of other commercial
used. In this method, vertical shafts have to be sunk, sectors depend. Therefore, the logistics infrastructure has
from where underground galleries radiate to reach the gained a lot of attention both from business industry as
minerals. Minerals are extracted and transported to the well as policy makers.
surface through these passages. Statement 1 is incorrect. India’s logistics cost is estimated
This method is risky. Poisonous gases, fires, floods and to account somewhere between 13-14 per cent of the
caving in lead to fatal accidents. GDP. This compares poorly with the cost of 7-8 per cent
for developed economies. One of the key reasons for poor
When it comes to coal mining accidents, India has a
productivity is the lack of automation in logistics.
higher proportion of deaths resulting from strata fall (or
fall of the roof and sides of underground mines) than Statement 2 is correct and Statement 3 is incorrect.
from the use of explosives, which account for the bulk of While road freight accounts for 54% and railways 33%,
the accidents in countries such as China and the US. just 6% of freight transported in India is carried by coastal
Pair 3 is correct. Since the 1980s, rat-hole mining is the shipping and inland waterways.
primarily practiced mining technique in Meghalaya Statement 4 is incorrect. The Logistics Ease Across
in which deep vertical shafts with narrow horizontal Different States (LEADS) 2021 index is a composite
tunnels of 3 to 4 feet diameter are dug and miners are indicator to assess international trade logistics across
sent down to extract coal till 100 to 150 metre and in some states and Union territories. It is based on a stakeholders’
case even more than that. survey conducted by Deloitte for the Ministry of
Commerce and Industry. The LEADS report also aims to
156. Solution: (a) enhance focus on improving logistics performance across
Exp) Option a is correct states which is key to improving the country’s trade and
Products that are classified under the FMCG banner reducing transaction cost.
include food, beverages, personal hygiene and 2021 is the third year in a row that Gujarat (and not
household cleaning utensils. The term “fast- moving” Maharashtra) remained on top of the rankings.

Workbook 246
GEOGRAPHY

Gujarat was followed by Haryana, Punjab, Tamil Nadu, recyclability, safety advantages and mass-reducing
and Maharashtra. The report said that it ranked the possibilities of steel is what makes it much better than
states on the basis of their logistics ecosystem, while other materials. In comparison to steel, aluminium
highlighting the key logistics-related challenges faced costs as much as 60% more when used in automotive
by the stakeholders and also includes suggestive production. In raw materials costs, aluminium is about
recommendations. three times more expensive than steel, while in terms of
conversion costs it is about twice as expensive.
158. Solution: (b)
Statement 3 is correct. India has second highest
Exp) Option b is correct. crude steel production in world with 111.2mnT of steel
Population growth is the increase in the number of people production after China.
in a population. India’s current annual population growth
rate is 1 per cent. 160. Solution: (b)
Statement 1 is incorrect. India had a negative population Exp) Option b is correct.
growth rate in 1921 decadal census. The country’s In India, Land resources includes 43% of plains, 30 % of
population growth rate was negative by 0.31 percent. mountains and 27% of plateaus.
Statement 2 is correct. In 1961 and 1971, the population Statement 1 is correct. The physical features in India are
growth rate was 21.51 percent and 24.80 percent diverse and complex. There are mountains, hills, plateaus
respectively. In 1981, it became 24.66 percent. After that, and plains which produce varied human response to the
the decadal growth rate of country’s population though use of land resources. About 30% of India’s surface area
remained high, has started slowing down gradually. is covered by hills and mountains. These are either too
It was 23.85 percent in 1991, 21.54 percent in 2001 and steep or too cold for cultivation. About 25% of this land
17.64 percent in 2011. is topographically usable which is scattered across the
country. Plateaus constitute 28% of the total surface area
159. Solution: (d) but only a quarter of this is fit for cultivation. The plains
Exp) Option d is correct. cover 43% of the total area and nearly 95% of it is suitable
The Indian steel industry was delicensed and de-controlled for cultivation.
in 1991 and 1992 respectively. Statement 2 is incorrect. The National Remote Sensing
Statement 1 is correct. In 2019, India became the 2nd Agency has estimated 53 million hectares (16%) as
largest steel producer in the world after China and the wasteland in the country.
largest producer of sponge iron in the world. The demand Statement 3 is incorrect. Among the states, the highest
of raw material for Iron-steel industry in India, i.e., iron incidence of wastelands is recorded in Jammu and
ore for Indian Steel Industry is met from the domestic Kashmir (60%) followed by Rajasthan (38%), Sikkim
supply in the country while the demand of coking coal is and Himachal Pradesh (37% each) and Gujarat (17%).
met mainly through imports. The Government of India constituted the National
Statement 2 is correct. Steel compared to other materials Wasteland Development Board in 1985 with a view to
of its type has low production costs. The cost efficiency, enhancing productivity of wastelands.

247 Workbook
Workbooks are a series of question banks of MCQs aimed at covering the
UPSC Civil Services Prelims Examination syllabus holistically. The workbook
of each subject is divided into units and each unit consists of two parts:
Previous Year Questions and Practise Questions. The combination of both
these sections will help students check the questions asked by the
commission as well as practise similar questions.

FORUM LEARNING CENTRE


DELHI - 2nd Floor, IAPL House, PATNA - 2nd Floor, AG Palace, HYDERABAD - 1st Floor,
19, Pusa Road, Opp. Metro East Boring SM Plaza, RTC X Rd, Indira Park Road,
Pillar 95-96, Karol Bagh - 110005 Canal Road - 800001 Jawahar Nagar - 500020

You might also like